Скачать как pdf или txt
Скачать как pdf или txt
Вы находитесь на странице: 1из 497

Читателю книги

Настоящий сборник содержит четыреста задач, предла-


гавшихся школьникам –– участникам физической олимпи-
ады в Санкт-Петербурге в 1996––2006 годах. Для Вашего
удобства мы разделили эти задачи по уровню сложности
(для районного и городского туров олимпиады), затем –– по
классам, а внутри классов –– по темам: кинематике и общим
вопросам, механике, теплоте и молекулярной физике, элек-
тричеству и магнетизму, оптике. Разбивка задач по клас-
сам соответствовала требованиям школьной программы на
момент проведения олимпиад. Некоторые темы (например,
поверхностное натяжение в жидкостях) в настоящее время
исключены из школьной программы.
Предложенная классификация, во многом, условна: во-пер-
вых, многие олимпиадные задачи затрагивают сразу несколь-
ко тем школьной программы по физике, во-вторых, некото-
рые трудные и интересные задачи предлагались уже на пер-
вом туре олимпиады, в то время как каждый вариант второ-
го тура обязательно содержал несложные, «утешительные»
задачи. Размещение задач в книге не соответствует хроно-
логическому порядку их появления на олимпиадах.
Отдельно Вашему вниманию предлагаются эксперимен-
тальные задачи. В этих задачах победителям теоретических
туров олимпиады предлагалось самостоятельно разработать
и поставить физический эксперимент, провести элементар-
ную статистическую обработку полученных данных и сде-
лать вывод о наблюдаемом явлении. Эти навыки, к сожа-
лению, лежат за рамками традиции преподавания физики,
сложившейся в средней школе, поэтому представляют осо-
бый интерес. Список физических констант, плотностей раз-
личных материалов и других величин, значения которых
считаются в задачнике известными, приведён в конце сбор-
ника.
Все задачи в книге снабжены подробными решениями.
Решение олимпиадных задач редко сводится к получению
формального ответа; всегда требуется установить, когда по-
лученный результат имеет физический смысл. Такие задачи
правильнее называть этюдами, самостоятельными произве-
дениями для изучения. Работая над книгой, мы уделяли
4

большое внимание наглядности представления решений и


их результатов, что позволило сделать анализ ряда задач
более интересным и глубоким. Мы надеемся, что настоя-
щий сборник станет полезным учебным пособием, которое
не только поможет Вам в усвоении школьной программы и
приёмов решения задач, но также познакомит Вас с мыш-
лением людей, профессионально занимающихся физикой.
На протяжении пятнадцати лет работой Жюри петер-
бургской физической олимпиады руководит доктор физи-
ко-математических наук, профессор Санкт-Петербургского
Государственного университета А. А. Курдюмов. Вместе с
ним, в разные годы, в Жюри сотрудничали и продолжа-
ют сотрудничать Барыгин И., Богословский Н., Бубли-
ков С., Волчёнков Д., Волчёнкова Л., Звонарев М., Кова-
левский Д., Кожемяк А., Колалис Р., Колеватов Р., Ко-
марова М., Компаниец М., Компаниец Т., Ланцов А., Ла-
рионова Н., Молеваник С., Налимов М., Павловский К.,
Путров П., Савельев А., Сашов С., Тараканов Н., Чепель В.,
Чирцов А., Шапиро А., Шендерович И., Эпштейн Ю. Они
являются авторами задач, предлагаемых в книге.
Своим появлением эта книга также обязана школьни-
кам, более 20 тысяч из которых приняли участие в физи-
ческой олимпиаде в Санкт-Петербурге за последние десять
лет. Они неутомимо решали наши задачи и предлагали свои
оригинальные, изумлявшие нас своей красотой.
Книга увидела свет благодаря поддержке петербургско-
го представительства международной компании Моторола
и особенно В. С. Полутина, М. Ю. Округиной, С. Н. Барано-
ва, А. В. Степанова, М. Н. Варзар и Д. О. Вавилова. Жюри
Санкт-Петербургской городской физической олимпиады и
составитель сборника лично выражают им свою искреннюю
признательность за финансирование настоящего издания и
предоставление призов победителям олимпиады.

Д. Ю. Волчёнков, Составитель сборника


УСЛОВИЯ ЗАДАЧ
Задачи районных туров
7––8 к л а с с ы
Качественные задачи
1. От катера, идущего по каналу,
расходится волна, имеющая форму,
показанную на рис. 1. В какую сто-
рону течёт вода по каналу, и почему
волна изогнута?
2. Почему при резком тормо-
жении переднего колеса велосипе-
да есть опасность перелететь через Рис. 1
руль?
3. Предложите способ нахождения объёма пустот в ведре
сухого песка с помощью ведра воды и мензурки.
4. Почему быстро идущий катер выталкивается из воды,
а при выключении двигателя –– снова погружается в воду?
5. В пасмурный день за окном автомобиля эксперимен-
татор на палке укрепил термометр. Затем автомобиль тро-
нулся и быстро набрал большую скорость. Через несколько
минут экспериментатор посмотрел на термометр, который
он всё это время держал за окном. В какую сторону изме-
нились его показания? Ответ поясните.
6. Дачник Родион, вскрывая банку с консервированны-
ми фруктами, заметил, что сразу после открытия крышки
фрукты начинают тонуть. Объясните это явление.
7. В кастрюле в большом количестве кипящей воды ва-
рится картошка. Что следует сделать, чтобы картошка сва-
рилась быстрее: плотно закрыть кастрюлю крышкой или от-
лить часть воды? Ответ пояснить.
8. Известно, что доберманы идут по следу, ориентируясь
на запах на высоте приблизительно 0,5 метра от земли, а
болонки –– по запаху следов, оставленных на земле. На тур-
нире доберману и болонке дали задание найти человека, ко-
торый прошёл мимо несколько часов назад. Какая из собак
успешнее справится с заданием?
9. После дождя заднее стекло автомобиля покрыто ка-
пельками воды. В каком случае они исчезнут быстрее: если
автомобиль движется или стоит? Ответ обосновать.
8 Условия задач

10. Медицинский и уличный ртутные термометры име-


ют почти одинаковые размеры (около 10––15 см в длину).
Почему же уличным термометром можно измерять темпе-
ратуры от −30 ◦ C до +50 ◦ C, а медицинским –– только от
35 ◦ C до 42 ◦ C?

Кинематика и начальные физические понятия


11. Во время археологических раскопок была
найдена старинная бутылка, нижняя часть кото-
рой имеет форму параллелепипеда и по объёму со-
ставляет более половины от всей бутылки. Верх-
няя часть бутылки имеет неправильную форму
(см. рис. 2). Имея в распоряжении линейку, проб-
ку к этой бутылке и неограниченные запасы воды,
определите её объём.
Рис. 2
12. На соревнованиях по плаванию два пловца
стартуют одновременно. Первый проплывает бас-
сейн за 1,5 мин, а второй –– за 70 сек. Достигнув противопо-
ложного края бассейна, каждый пловец разворачивается и
плывёт в обратную сторону. Через какое время после старта
второй пловец поравняется с первым, обойдя его на круг?
13. Сосуд объёмом V = 1 л заполнен на три четверти во-
дой. Когда в него погрузили кусок меди, уровень воды под-
нялся, и часть её объёмом V0 = 100 мл вылилась через край.
Найдите массу куска меди. Плотность меди известна.
14. Велосипедист едет по дороге и через каждые 6 сек
проезжает мимо столба линии электропередачи. Увеличив
скорость на некоторую величину Dv, велосипедист стал про-
езжать мимо столбов через каждые 4 сек. Как часто он будет
проезжать мимо столбов, если увеличит скорость ещё на та-
кую же величину?
15. Пешеход за первые 20 сек прошёл 30 м, за следую-
щие 40 сек –– 58 м, и за последние 30 сек –– 45 м. Опреде-
лить скорость движения на каждом участке и найти сред-
нюю скорость за всё время движения.
16. Семиклассник ходит в школу из дома с постоянной
скоростью 2 м/с. Расстояние от дома до школы 103 м. Од-
нажды он решает вернуться с полпути домой, чтобы выклю-
чить забытый электроприбор. Успеет ли мальчик в школу к
Задачи районных туров 9

началу урока, если с этого момента будет бежать со скоро-


стью 14,4 км/ч.
17. Дельфин плывёт со скоростью 18 км/ч вдоль сте-
нок квадратного бассейна, описывая квадрат на постоянном
расстоянии от прямолинейных участков стенок. За 1 мин он
огибает бассейн 3 раза. Найти расстояние между дельфином
и стенкой. Длина каждой стенки 30 м.
18. Содержимое пакета гречневой крупы массой 1 кг за-
лили 3 л воды и сварили. Сколько воды выкипело при при-
готовлении каши? Считать, что вода либо выкипает, ли-
бо впитывается, целиком расходуясь на увеличение объёма
зерна. Плотности сухой и варёной гречи известны.
19. Двое часовых, двигаясь прямолинейно, охраняют с
противоположных сторон один небольшой объект. Графики
зависимости координат часовых от времени даны на рис. 3.
Постройте: 1) графики зависимости скорости vx часовых от
времени, 2) график зависимости скорости ux первого часо-
вого относительно второго от времени.
20

10
t, сек 10 20 30 40
x, м

−10

−20

Рис. 3

20. Продавец Василий работает в магазине, расположен-


ном рядом со станцией метро «Ломоносовская». Известно,
что он бегает со скоростью 10 км/ч. Чтобы прийти в мага-
зин точно в 9 часов утра, Василий каждый день бежит по
эскалатору. Иногда он в спешке путает эскалатор, работа-
ющий на подъём, с эскалатором, работающим на спуск, и
тогда он опаздывает на 12 мин. Однажды эскалатор стоял,
и Василий опоздал на 3 мин. Какова скорость эскалатора?
21. На уроке физкультуры Петя и Маша бежали вместе
по прямой дорожке, стартовав от школы. Затем Петя побе-
10 Условия задач

жал быстрее, а Маша пошла. Через некоторое время ребята


одновременно повернули обратно и достигли школы одно-
временно. Графики зависимости проекции скорости ребят
на направлении дорожки от времени даны на рис. 4. По-
строить графики зависимости расстояния между Петей и
Машей от времени.

Vx , м/с
5

2,5
1,25
1 t,
t,
t, мин
t, мин
3 t, мин
мин
мин 7

−1,25

−3,125

Рис. 4

22. У угла прямоугольного стола для чаепития стоит


Алиса (см. рис. 5). На расстоянии 48 м от неё стоят Мартов-
ский Заяц и Болванщик. В поисках чистого чайного при-
бора они начали разбегаться в разные стороны вдоль сто-
ла, причём скорость Зайца равна 6 м/с, а скорость Болван-
щика –– 1,5 м/с. На каком расстоянии от Алисы они снова
встретятся? Длина стола 100 м, ширина –– 8 м.

76
6 м/с 1,5 м/с
8 Алиса

100

Рис. 5

23. Первую половину пути Баба-Яга летела со скоростью


20 км/ч. Затем погода испортилась, и половину времени
Яга пролетела со скоростью 10 км/ч. В довершение бед у
неё сломалась метла, и пришлось оставшееся время идти
Задачи районных туров 11

пешком со скоростью 5 км/ч. Найти среднюю скорость ба-


бушки.
24. Туристы наполнили котелок до краёв снегом и вы-
топили из него 0,75 л воды. Найдите объём котелка, если
известно, что вода в четыре раза плотнее снега, собранного
в котелок туристами.
25. Два друга, Пётр и Павел, поехали на поезде. У Петра
был билет в первый вагон, а у Павла –– в последний (ваго-
ны нумеруются от локомотива). На одной из промежуточ-
ных остановок локомотив перецепили к хвосту поезда, так,
что Пётр приехал в конечный пункт в последнем вагоне, а
Павел –– в первом. Сравните пути вагонов, в которых ехали
Пётр и Павел.
26. Изготовленный из железа кубик имеет плотность
7800 кг/м3 . Кубик нагрели, и из-за теплового расширения
длины его рёбер увеличились на 0,5%. На сколько измени-
лась плотность кубика?
27. Проволоку производят вытягиванием из цельной за-
готовки меди. На изготовление медной проволоки прямо-
угольного сечения в течение суток израсходовано 8640 кг
меди. Скорость протягивания проволоки постоянна в тече-
ние суток и равна 1 м/с. Найдите площадь поперечного се-
чения проволоки, если за истекшие сутки производилась
проволока сечением только одного размера. Плотность меди
известна.
28. Имеются чугунный шар и шар из лёгкого неизвест-
ного сплава. Масса чугунного шара в 1,25 раза больше мас-
сы шара из неизвестного сплава. Объём чугунного шара в
2 раза меньше объёма второго шара. Плотность чугуна из-
вестна. Какова плотность неизвестного сплава?
29. В безветренную погоду на некоторой высоте над зем-
лёй неподвижно завис воздушный шар. Парашютист вы-
прыгнул из гондолы шара вертикально вниз. График ско-
рости парашютиста приведён на рис. 6. С какой высоты над
поверхностью Земли парашютист начал опускаться с посто-
янной скоростью?
30. Два мальчика проходят с постоянными скоростями
прямолинейный участок дороги. Первый мальчик идёт на
ходулях со скоростью 7,5 км/ч, в 1,5 раза большей, чем
скорость второго мальчика идущего без ходулей. 1) Како-
12 Условия задач

V, м/c

t, сек
10 210

Рис. 6

ва средняя скорость ботинка второго мальчика (идущего без


ходулей) за один шаг движущейся ноги? 2) Какова средняя
скорость ботинка второго мальчика за всё время движения
по этому участку дороги? Ответы обоснуйте.
31. Для определения постоянной скорости судна относи-
тельно воды производят пробег судна по прямолинейному
участку реки между пристанями, расположенными на од-
ном берегу на расстоянии L = 4,2 км. Время пробега по те-
чению реки t1 = 300 cек, а против течения –– t2 = 420 cек. Ка-
кова скорость судна относительно воды? Скорость течения
реки постоянна на всём участке испытаний.
32. Железный дровосек, стоящий на берегу реки в точ-
ке B (см. рис. 7), роняет в воду топор, и от него по воде
начинают расходиться волны. Через какое время волны на
воде от топора достигнут людоеда, находящегося на проти-
воположном берегу, в точке A, расположенной напротив B?
Скорость течения реки равна u, скорость распространения
волн относительно воды v, ширина реки d.

u
u
u
u
u
u

Рис. 7
Задачи районных туров 13

Механика и гидростатика
33. В одном из солёных озёр Канады плотность воды за-
висит от глубины так, как показано на рис. 8. На какой глу-
бине и в каком положении будет плавать тонкая однородная
палка? Плотность материала, из которого она сделана, рав-
на 1020 кг/м3 .

, кг/м3

1100
?
1000

0 100 h, м

Рис. 8 Рис. 9

34. Цилиндр имеет радиус 1 см, площадь сечения


3,14 см2 и длину 6,6 см. На половине длины цилиндра впа-
яли вертикальную тонкую трубку. Правая часть сосуда со-
общается с атмосферой (см. рис. 9). К правой стенке ци-
линдра с помощью лёгкой пружины жёсткостью 6 Н/м при-
крепили массивный тонкий поршень. Длина нерастянутой
пружины равна 3 см. Систему установили на упор, находя-
щийся по центру цилиндра. Затем в цилиндр, придерживая
его рукой, начали наливать воду. В тот момент, когда масса
воды стала равна массе поршня, система оказалась в равно-
весии. Поршень плотно, но без трения прилегает к стенкам
цилиндра. На какой высоте находится вода в трубке?
35. Имеется система сообщающихся сосудов. Левый со-
суд плотно закрыт сверху лёгким поршнем, на поршне ле-
жит груз массой 3 кг. В правом сосуде на воде лежит очень
лёгкий куб. Длина стороны куба –– 10 см. Площади сечений
левого и правого сосудов равны соответственно S1 = 0,03 м2
и S2 = 0,05 м2 . Поршень связан верёвкой с кубом через си-
стему блоков под водой (см. рис. 10). Первоначально верёв-
14 Условия задач

ка не провисает. Груз убирают с поршня. На сколько под-


нимется поршень?

k
m
m
m
m
m
S 2S

Рис. 10 Рис. 11

36. Имеется система сообщающихся сосудов (см. рис. 11).


К левому поршню на кронштейне прикреплена пружина
жёсткостью k, которая другим своим концом соединена
с правым поршнем. Когда поршни находятся на одном
уровне, пружина не растянута. На сколько растянется пру-
жина, если на правый поршень поместить груз массой m?
Площади поршней равны S и 2S. Плотность жидкости ––
r, ускорение свободного падения g. Поршни, кронштейн и
пружину считать невесомыми. Считать, что кронштейн мо-
жет двигаться только вертикально.
37. В водоёме на глубине h = 10 м на краю плоского усту-
па лежит доска длиной L = 2 м, шириной a = 10 см и толщи-
ной b = 1 см. Масса доски –– 4 кг. Половина доски плотно
прижата к поверхности уступа, так что между доской и по-
верхностью отсутствует вода и воздух (см. рис. 12). Мини-
мальная сила, которую нужно приложить к середине доски
для того, чтобы приподнять прижатую часть, равна F1 . Ес-
ли прикладывать силу к правому краю доски, то для того
же потребуется сила F2 . Найти численное значение отноше-
ния F1 /F2 .
38. На дно аквариума, заполненного водой, кладут ка-
мень (при этом вода через край аквариума не переливает-
ся). Как изменится потенциальная энергия воды в аквари-
уме?
Задачи районных туров 15

39. Невесомая нерастяжимая нить привязана к неболь-


шому кубику. Затем она пропущена ещё сквозь 3 таких же
кубика, как показано на рисунке. Все кубики стоят на сто-
ле. Чтобы сдвинуть один кубик, к нему требуется прило-
жить силу, равную 10 Н. Расстояние между соседними ку-
биками равно 5 см. Систему начинают медленно тянуть за
нитку (см. рис. 13). Нарисуйте график зависимости силы
F, которую нужно для этого прикладывать, от пройденного
левым кубиком расстояния. Трения между ниткой и куби-
ками нет.
40. На динамометре висит гирька массой 100 г. Снизу к
ней прикреплён второй динамометр, другой конец которого
жёстко закреплён. Показания нижнего динамометра –– 5 Н.
Какая сила приложена к верхнему динамометру?
41. К трём одинаковым динамометрам (см. рис. 14) под-
вешен груз. Показания верхнего и нижнего динамометров
90 Н и 30 Н соответственно. Определите показания средне-
го динамометра и его массу.
42. На столе стоит кубик, площадь грани которого со-
ставляет S1 = 25 см2 . Его масса равна m1 = 90 г. На него ста-
вят тело неправильной формы. Площадь его контакта с ку-
биком составляет S2 = 16 см2 . Сверху ставят ещё один кубик
со стороной a = 3 см (см. рис. 15). Площадь контакта этого
16 Условия задач
Задачи районных туров 17

кубика с телом неправильной формы составляет 9 см2 . Из-


вестно, что все давления в местах соприкосновения тел (и со
столом) равны. Найдите массу тела неправильной формы и
верхнего кубика.
43. Левое плечо лёгкого рычага имеет длину L1 = 8 см, а
правое –– L2 = 4 см. К левому плечу подвешен алюминиевый
куб, а к правому –– гиря массой M2 = 300 г (см. рис. 16). Ко-
гда куб погрузили в воду на 2/3 его объёма, оказалось, что
рычаг уравновешен. Найдите объём куба. Плотности алю-
миния и воды, а также ускорение свободного падения из-
вестны.
44. К потолку подвешена лёгкая пружина с маленьким
шариком, имеющим массу 100 г. К последнему прикрепле-
на вторая лёгкая пружина с ещё одним таким же шариком.
Длины недеформированных пружин равны 10 см и 20 см, а
жёсткости пружин равны 200 Н/м и 100 Н/м соответствен-
но. Найдите расстояние от потолка до нижнего шарика.
45. Невесомая жидкость находится между двумя порш-
нями, неподвижно скреплёнными друг с другом твёрдым
стержнем (см. рис. 17). К малому поршню прикреплён ди-
намометр, к которому приложили силу F. Найдите давле-
ние в жидкости. Площадь малого поршня –– S1 , площадь
18 Условия задач

большого поршня –– S2 . Атмосферное давление не учиты-


вать.
46. Однородное цилиндрическое бревно радиуса R плава-
ет в воде, причём над поверхностью воды выступает 1/4 его
объёма. Из 10 таких же брёвен связали плот (см. рис. 18).
На какую высоту выступает над водой плавающий плот?
47. Два кубика весом P1 и P2 и рёбрами и b соответ-
ственно соединены невесомым тонким стержнем длиной L.
Система погружена в жидкость плотности r. Где надо поме-
стить точку опоры, чтобы стержень был в равновесии?
48. В прямоугольный сосуд с площадью внутреннего
поперечного сечения S0 = 100 см2 налит столб воды высо-
Задачи районных туров 19

той H = 50 см (см. рис. 19). На поверхности воды плавает


невесомый поршень с прямоугольным отверстием, в кото-
рое плотно вставлена невесомая трубка прямоугольной фор-
мы. Поршень вплотную прилегает к стенкам сосуда. Высота
трубки h = 1 cм, площадь внутреннего поперечного сечения
трубки S = 10 см2 . Поршень начинают опускать вниз с по-
стоянной скоростью V = 0,1 м/с. 1) С какой скоростью от-
носительно земли движется верхний уровень воды в труб-
ке? 2) Через какой промежуток времени после приведения
поршня в движение вода начнёт переливаться через верх-
ний край трубки?
49. Горизонтальный желоб расположен почти вплотную
с невесомой чашей точных весов. Желоб поддерживают так,
20 Условия задач

что он всегда находится на одном уровне с чашей весов.


Невесомая нерастяжимая нить, на которую нанизаны пять
хрустальных одинаковых бусинок, лежит на желобе. Нить
расположена прямолинейно вдоль желоба так, что первая
бусинка находится на расстоянии 3 см от чаши весов. Бу-
синки приклеены к нити на расстоянии 3 см друг от друга
так, что крайние бусинки приклеены к краям нити. Нить
приводят в движение со скоростью 1,5 см/с так, что бусин-
ки с желоба попадают на чашу и останавливаются на ней.
После опускания всей нити с бусинками на чашу весы по-
казали 15 г. Бусинки считать точками, имеющими массу,
но лишёнными объёма. 1) Какое значение покажут весы к
концу 10 сек от начала движения нити? 2) Какова масса од-
ной бусинки? 3) Каков реальный объём одной хрустальной
бусинки? Плотность хрусталя равна 3 г/см3 .
50. Два свинцовых цилиндра массой 0,2 кг каждый,
плотно прижатые друг к другу свежими срезами, слиплись.
Верхний цилиндр подвешивают торцом к прочной верти-
кальной и нерастяжимой проволоке, верхний конец кото-
рой закреплён. К торцу нижнего цилиндра подвешивают
невесомый мешок. В мешок в течение 60 с тонкой струй-
кой с пренебрежимо малой высоты насыпают песок. К кон-
цу 60-ой секунды, когда в мешке оказалось 9,8 кг песка,
нижний цилиндр оторвался от верхнего. Постройте график
зависимости силы натяжения проволоки от времени. Объ-
ясните график и подтвердите его числовыми расчётами.
51. По свежему снегу прошёл снегоход массой 375 кг с
площадью опорной поверхности гусениц 0,75 м2 . По следу
снегохода идёт человек массой 80 кг. Площадь одной по-
дошвы валенок 250 см2 . Будет ли человек проваливаться в
снег?
52. На концы лёгкого стержня длиной 40 см нанизаны
два шарика, первый сделан из чугуна, второй –– из магния.
Стержень погружают в воду и уравновешивают в горизон-
тальном положении. Точечная опора, на которой уравнове-
шивается система, располагается при этом точно по сере-
дине стержня. На сколько нужно передвинуть вдоль стерж-
ня второй шарик, чтобы система сохраняла равновесие в
воздухе? Плотности чугуна и магния известны.
Задачи районных туров 21

53. U-образный сосуд с одной стороны закрыт плотно


прилегающим к его краям поршнем и заполнен водой, как
показано на рис. 20. В трубку B начинают медленно зали-
вать воду. Как только высота уровня воды в нём достигает
h = 5 м, поршень приподнимается. В этот момент воду за-
ливать прекращают. На какой высоте h установится вода в
трубке B? Площади сечения трубки B и узкой части трубки
A равны. Площади сечения широкой и узкой частей трубки
A равны соответственно S1 = 4 м2 и S2 = 3 м2 . Высота узкой
части трубки A x = 1 м.
54. На трёх пружинах, прикреплённых к потолку, гори-
зонтально висит однородный тонкий стержень (см. рис. 21).
Расстояние между первой и второй пружинами L12 = 10 см,
расстояние между второй и третьей пружинами L23 = 30 см.
Коэффициенты жёсткости пружин известны: k1 = 1 кH/м,
k2 = 2 кH/м, k3 = 1 кH/м. Растяжения первой и третьей пру-
жин равны Dx1 = 5 см и Dx3 = 10 см. Найти растяжение вто-
рой пружины Dx2 .
55. Тело поднимают с помощью наклонной плоскости и
системы блоков (см. рис. 22). Какую минимальную силу F
22 Условия задач

нужно приложить, чтобы поднять тело массы m? Высота


наклонной плоскости равна H, длина L. Блоки невесомые.
Трением пренебречь.
56. К потолку прикреплена конструкция, состоящая из
двух пружин и двух маленьких чашек A и B (см. рис. 23).
Расстояние от пола до потолка равно 2 м. Жёсткости пру-
жин равны k1 = 15 Н/м и k2 = 30 Н/м. Длины нерастянутых
пружин одинаковы и равны 30 см. Масса чашки A равна
m = 100 г, чашка B невесома. Груз какой массы надо поло-
жить в чашку A, чтобы чашка B достала до пола? Какой
груз надо положить в чашку B, чтобы она достала до пола
(чашка A при этом пуста)?

Работа и теплота
57. В результате измерения КПД двигателя получился
равным 20%. Впоследствии оказалось, что за время измере-
ния 5% топлива вытекло через трещину в топливном шлан-
ге. Какой результат измерения КПД получится после устра-
нения неисправности?
58. Автомобиль двигался по прямому участку шоссе 7
часов. Первые два часа он ехал со скоростью 30 км/ч, ещё
час –– со скоростью 50 км/ч, а оставшееся время –– со скоро-
стью 70 км/ч. Расход топлива зависит от скорости автомо-
биля так, как показано на рис. 24. Начальный запас топ-
Задачи районных туров 23

лива в бензобаке автомобиля равен 50 л. Какое количество


топлива осталось в бензобаке в конце пути?
П р и м е ч а н и е. Расходом топлива называется коли-
чество топлива (измеренное в литрах), которое необходимо
для преодоления пути в 100 км.
59. В бассейн по трубе, в которой установлен нагреватель
мощностью P = 1 МВт, подаётся вода из резервуара. Темпе-
ратура воды в резервуаре tР = 5 ◦ C. В первый раз пустой бас-
сейн заполняется за время t = 21 мин, при этом температу-
ра воды после заполнения t1 = 20 ◦ C. Во второй раз в бас-
сейне было изначально некоторое количество воды при тем-
пературе t0 = 15 ◦ C. Оставшуюся часть заполняли за время
t = 21 мин. Температура воды после заполнения оказалась
t2 = 25 ◦ C. Сколько воды первоначально было в бассейне во
втором случае? Остыванием воды в бассейне пренебречь.
Теплоёмкость воды известна.
60. В закрытом металлическом бидоне находится керо-
син. Предложите способ, позволяющий определить пример-
ный уровень керосина в бидоне, не пользуясь никакими из-
мерительными приборами и не опрокидывая бидон.
61. Свинцовая пуля массой 9 г, летящая со скоростью
825 м/с, разогревается из-за трения о воздух до температу-
ры 150 ◦ C. Пуля попадает в сугроб с температурой 0 ◦ C и
24 Условия задач

застревает в нём. Какое наибольшее количество снега мо-


жет при этом растаять? Удельные теплота плавления льда
и теплоёмкость свинца известны.
62. На 1 этаже дома барометр показывал давление
p1 = 101080 Па. Определить, на каком этаже прибор по-
казывает p2 = 100681 Па, если расстояние между этажами
равно 3,5 м. Считать, что во всём диапазоне высот плот-
ность воздуха постоянна и известна.
63. В закрытом сосуде в воде, температура которой рав-
на 0 ◦ C, плавает кусок льда массой m = 0,1 кг, в который
вмёрзла свинцовая дробинка. Когда льдинке передали теп-
лоту 32 кДж, льдинка начала тонуть. Какова была масса
дробинки? Плотности льда, свинца, воды, а также удельная
теплота плавления льда известны.
64. Вода массой m = 3,6 кг, оставленная в пустом холо-
дильнике, за T = 1 ч охладилась от температуры t1 = 10 ◦ C
до температуры t2 = 0 ◦ C. При этом холодильник отдавал в
окружающее пространство тепло с мощностью P = 300 Вт.
Какую мощность потребляет холодильник от сети? Удель-
ная теплоёмкость воды известна.
Задачи районных туров 25

65. В сосуде находится вода при температуре t0 = 0 ◦ C.


От этого сосуда отводят тепло с помощью двух металли-
ческих стержней, торцы которых находятся в дне сосуда.
Сначала тепло отводят через один стержень с мощностью
P1 = 1 кДж/с, а спустя T = 1 мин начинают одновременно
отводить и через второй стержень, с такой же по величине
мощностью P2 = 1 кДж/с. Дно сосуда покрыто составом,
препятствующим образованию ледяной корки, поэтому весь
образовавшийся лёд сразу всплывает на поверхность сосуда
(см. рис. 25). Постройте график зависимости массы обра-
зовавшегося льда от времени. Удельная теплота плавления
льда известна. Теплообменом воды с окружающей средой и
теплоёмкостью сосуда пренебречь.

9 класс
Кинематика
66. Велосипед с колёсами, имеющими форму равносто-
роннего треугольника (см. рис. 26), за время t прошёл по
дороге достаточно большое расстояние s. Найдите среднее
значение модуля скорости точки, расположенной в вершине
колеса. Колёса не проскальзывают по дороге, велосипед не
отрывается от земли.
26 Условия задач

67. На станции метро есть два эскалатора, движущиеся


в противоположных направлениях с одной скоростью. На
каждом эскалаторе стоит по пассажиру. В момент, когда
пассажиры поравнялись, каждый из них побежал вниз по
эскалатору с ускорением a. Один пассажир добежал до осно-
вания эскалатора на время Dt раньше, чем другой. Найдите
скорость эскалаторов.
68. Кузьма бежал по кругу с постоянной скоростью. В
точке A он встретил Матвея, который двигался с постоян-
ным ускорением по диаметру AB (см. рис. 27). Скорость
Матвея в момент встречи была равна скорости Кузьмы.
Кузьма, не изменяя скорости, пробежал полкруга и встре-
тился с Матвеем в точке B, куда тот как раз успел добежать.
Определите отношение ускорений Кузьмы и Матвея.
69. Бомбардировщик летит горизонтально на высоте
H = 500 м с постоянным ускорением = 2 м/с2 и через рав-
ные промежутки времени t = 0,5 сек производит бомбомета-
ние. Найти расстояние между точкам разрыва 9-ой и 11-ой
бомб, если первая бомба была сброшена при скорости само-
лёта V0 = 100 м/с. Сопротивлением воздуха пренебречь.
70. С территории военной части X, расположенной вбли-
зи города Y, одновременно выехали три танка. Ехали они по
одной дороге, и скорость каждого из них была постоянна.
Скорость первого танка равнялась 30 км/ч, скорость второ-
го –– 20 км/ч. Первый танк въехал в город Y в 19:00, второй
Задачи районных туров 27

танк –– в 20:00, а третий –– в 21:00. Найдите скорость третье-


го танка.
71. На перегоне в 500 м встречаются два трамвая, рав-
номерно движущиеся навстречу друг другу по параллель-
ным путям со скоростями 9 км/ч и 10 км/ч соответственно.
Трамваи вышли на перегон одновременно, и кондукторы в
обоих трамваях находились в тот момент у кабин вагоново-
жатых (таким образом, расстояние между кондукторами в
начальный момент времени равно 500 м). Кондукторы пе-
ремещаются по вагону со скоростью 3 км/ч. Пройдя вагон,
длина которого составляет 25 м, кондуктор сразу же раз-
ворачивается и идёт с той же скоростью обратно. Поравняв-
шись, кондукторы обоих трамваев приветствовали друг дру-
га. Через какое время после начала движения произошла их
встреча?
72. Из города A в город B выехала машина. Через 2 ч
вдогонку за ней выехала другая машина со скоростью в 3
раза большей, чем у первой. Через какое время T вторая
машина догонит первую?
73. Электричка двигается с постоянным ускорением. Её
скорость равна V0 , когда первый вагон начинает въезжать в
туннель. Длина туннеля равна длине электрички. Известно,
что головная кабина машиниста пробыла в туннеле в 2 раза
дольше, чем хвостовая. Какую скорость имела электричка,
когда целиком выехала из туннеля?
28 Условия задач

74. Хоккеист движется вдоль борта хоккейной коробки


со скоростью V . В некоторый момент он посылает шайбу
к противоположному борту со скоростью u. Каков должен
быть угол между векторами V и u, чтобы шайба вернулась
к хоккеисту, упруго отразившись от противоположного бор-
та? Трением шайбы о лёд и воздух пренебречь.
75. К краю лестницы, каждая ступенька которой имеет
длину a = 40 см и высоту h = 15 см, подъезжает шайба, име-
ющая скорость u = 10 м/с. Найдите номер ступеньки, на ко-
торую упадёт шайба (изначально находившаяся на нулевой
ступеньке).
76. По прямому участку железнодорожного пути движет-
ся с постоянной скоростью платформа. На ней расположе-
но орудие, ствол которого направлен вертикально вверх. В
некоторый момент времени орудие производит выстрел, и,
одновременно с ним, платформа начинает тормозиться с по-
стоянным ускорением a. К моменту, когда снаряд касается
земли, скорость платформы составляет половину первона-
чальной. Найдите расстояние от точки выстрела до точки
падения снаряда, если начальная скорость снаряда относи-
тельно орудия равна v0 , а ускорение свободного падения ––
g.
77. «Летающая тарелка» движется со скоростью V .
Представим себе, что такая «тарелка» движется с такой ско-
ростью, улетая от земли под углом a к горизонту. Лучи
p
солнечного света падают под углом − a к горизонту (см.
2
рис. 28). С какой скоростью движется по земле её тень, ес-
ли скорость света равна c?
78. По прямому участку железнодорожного пути движет-
ся вагон со скоростью 36 км/ч. В вагоне мальчик пуска-
ет игрушечный состав по рельсам расположенным поперёк
вагона. Скорость игрушки относительно пола вагона равна
1 м/с. Найти скорость игрушечного состава относительно
Земли.
79. Две одинаковые лодки, связанные между собой лёг-
ким канатом, покоились на поверхности воды на расстоя-
нии L друг от друга. В некоторый момент времени матро-
сы на одной из лодок стали тянуть канат, и лодка начала
Задачи районных туров 29

двигаться с постоянным ускорением a. Через какое время


лодки столкнутся?
80. Мальчик раскручивает верёвку с привязанным к её
концу камнем. В момент, когда траектория камня в гори-
зонтальной плоскости представляет собой окружность ради-
уса R, а высота камня над землёй –– h, он отрывается от ве-
рёвки. Найти расстояние от точки на земле, где стоит маль-
чик, до точки падения камня. Скорость камня в момент от-
рыва равна v. Сопротивлением воздуха пренебречь.
81. На рис. 29 изображены две прямые полосы обла-
ков AB и CD (вид сверху), находящихся на разных высотах.
Векторы скорости ветра на этих высотах V AB и VCD также
показаны на рис. 29. Объясните, как построить вектор V0
скорости движения по поверхности земли точки пересече-
ния теней облаков O.
82. (Ср. с задачей 78.) По круговому участку железно-
дорожного пути радиусом 500 м движется вагон с постоян-
ной скоростью 10 м/с. На полу вагона мальчик пускает за-
водную игрушку точно по ходу движения поезда. Игрушка
вначале ускоряется, увеличивая свою скорость на 0,7 м/с
за каждую секунду, а затем начинает замедляться. К кон-
цу движения её ускорение относительно пола вагона рав-
но 0,5 м/с2 . Найдите абсолютные величины ускорений иг-
рушки относительно земли в начале и в конце её движения.
30 Условия задач

Укажите направления этих ускорений по отношению к на-


правлению движения поезда.
Механика и гидростатика
83. B системе, изображённой на рис. 30, пружины име-
ют жёсткости k1 = 100 Н/м и k2 = 200 Н/м. К нижнему бло-
ку подвешивают груз массой M = 8 кг. Система приходит в
равновесие. На сколько сместился нижний блок? Пружины,
нити и блоки невесомы. Нити нерастяжимы.
84. Труба, сечение которой является квадратом со сто-
роной a = 20 см, закрыта поршнем (см. рис. 31). К трубе
присоединена трубка. Часть трубы, находящаяся справа от
поршня, полностью заполнена водой, уровень воды в трубке
равен h = 15 см. Силу какой величины и направления надо
прикладывать к поршню, чтобы удерживать его в равнове-
сии? Трение отсутствует.
85. Нити, привязанные к трём грузам, связаны в од-
ной точке (см. рис. 32). Две нити переброшены через непо-
движные невесомые блоки. Система находится в равнове-
сии, причём нити образуют с вертикалью углы 45◦ и 30◦ .
Масса левого груза равна 1 кг. Определите массы осталь-
ных грузов. Нить невесома, трения в блоках нет.
86. Какова минимальная площадь присоски, с помо-
щью которой можно прикрепить пробковый брусок массы
Задачи районных туров 31
32 Условия задач

m к горизонтальной поверхности на глубине h под водой


(рис. 33)? Плотность пробки r, плотность воды r0 , атмо-
сферное давление pa .
87. В один из двух одинаковых сосудов налито в два раза
больше воды, чем в другой. Сосуды закрыты одинаковыми
поршнями, плотно прилегающими к стенкам и поверхности
воды, и подвешены за эти поршни к плечам рычага. Трение
между поршнями и стенками отсутствует. Система находит-
ся в равновесии, а поршни –– на одном уровне (см. рис. 34).
В стенках сосудов на одном уровне проделывают маленькие
отверстия и соединяют сосуды лёгкой тонкой трубкой. В ка-
кую сторону потечёт вода по трубке?
88. Прямоугольная пластинка (см. рис. 35), изготовлен-
ная из материала с плотностью r, прикреплена к горизон-
тальной оси. Трение в оси отсутствует. По разные стороны
пластинки поддерживают разность давлений воздуха Dp, и
Задачи районных туров 33

в положении равновесия она составляет угол a с вертика-


лью. Определите толщину пластинки.
89. Между краями пропасти шириной H = 38 м висит
практически нерастяжимая верёвка длиной L = 40 м. Аль-
пинист массой m = 80 кг хочет перебраться по ней через про-
пасть. Сможет ли он это сделать, если верёвка рвётся при
силе натяжения T = 1200 H?
90. Грузы, имеющие массы M и m (M > m) при помощи
невесомой нерастяжимой нити подвешены на блоке, как по-
казано на рис. 36. С каким минимальным ускорением нуж-
но двигать блок в вертикальном направлении, чтобы уско-
рения грузов (относительно земли) были направлены в одну
сторону? Грузы двигаются по вертикали. Ускорение свобод-
ного падения равно g. Сопротивлением воздуха пренебречь.
91. По горизонтальной плоскости скользит без трения
точечная шайба массы m (см. рис. 37). Скорость шайбы v.
Перпендикулярно направлению движения шайбы движется
лента транспортёра с такой же по модулю скоростью V . Си-
ла трения между поверхностями шайбы и транспортёра F.
Какой должна быть ширина транспортёра H для того, что-
бы шайба переехала через него?
92. Две невесомых пружины с коэффициентами жёстко-
сти k1 = 24 Н/м и k2 = 12 Н/м и лёгкая шайба, скользящая
вдоль стержня без трения, соединены вместе, как показа-
но на рис. 38. К свободному концу пружины прикладыва-
ют такую силу F(t), что он движется вправо с постоянной
скоростью V = 0,2 м/с. Найдите скорость шайбы. Постройте
график зависимости силы F(t) от времени.
34 Условия задач
Задачи районных туров 35

93. Объём лёгкой оболочки гелиевого зонда равен 64 м3 .


Чтобы удержать зонд, его прикрепляют невесомым нерас-
тяжимым тросом к лёгкой пластине, притёртой к такой же,
неподвижной горизонтальной пластине (см. рис. 39). Най-
дите минимальную площадь таких пластин. Плотности воз-
духа и гелия известны.
94. Лыжник на водных лыжах движется по прямой с по-
стоянной скоростью. Сила натяжения каната, за который
держится лыжник равна T, а масса лыжника равна m. Най-
ти угол под которым наклонены лыжи к зеркалу воды.
95. К концам невесомой нерастяжимой нити, перекину-
той через блок (см. рис. 40), прикреплены грузы с массами
m1 и m2 , причём m1 > m2 . В начальный момент грузы непо-
движны, и расстояние между ними равно h. Через какое
время грузы окажутся на одинаковом расстоянии от блока?
96. Сосуд с водой уравновешен через блок грузом массы
M. Система устроена так (см. рис. 41), что нити, проходя-
щие через блок, вертикальны. Определить силу натяжения
нити. Считать, что груз лежит на дне сосуда. Масса сосуда
вместе с водой –– m, ускорение свободного падения –– g.
36 Условия задач
Задачи районных туров 37

97. Сосуд имеет снизу сливное отверстие, герметично за-


ткнутое пробкой. К пробке привязана нерастяжимая нить
длиной L = 10 см. Другой конец нити привязан к бруску ку-
бической формы, изготовленного из материала, имеющего
плотность r1 = 102 кг/м3 . В сосуд медленно наливают воду.
Минимальный размер бруска, при котором пробка вытаски-
вается из отверстия, составляет a = 10 см. Чему равна пло-
щадь пробки? Массой пробки пренебречь. Трением между
пробкой и стенками сосуда пренебречь, пробка над краями
отверстия не выступает.
98. На гладком закреплённом шаре удерживается тело
массой m (см. рис. 42). В некоторый момент тело освобож-
дают и прикладывают к нему силу F, меньшую mg. Найти
ускорение тела. Угол a = 30◦ , а угол b = 75◦ .
99. В межзвёздном пространстве находится массивная
труба длиной 10 м и внутренним радиусом 2 м. Внутрь тру-
бы, по касательной к внутренней её поверхности, залетает
малое тело массой 5 г и начинает скользить вдоль поверх-
ности без трения под углом 30◦ к продольной оси. Скорость
тела относительно трубы составляет 8000 м/с. Определите,
через какое время тело покинет трубу, и с какой силой оно
действует на стенку трубы в процессе своего движения.
100. Доску массой 10 кг можно равномерно переме-
щать по деревянному полу, приложив горизонтальную си-
лу F = 50 Н. На доску поставили деревянный ящик массой
40 кг. Оба тела соединили невесомой нерастяжимой нитью,
перекинутой через неподвижный блок (см. рис. 43). Опре-
делите, какую горизонтальную силу надо приложить к дос-
38 Условия задач

ке, чтобы она начала равномерно скользить вдоль пола по


направлению от блока. Считать, что коэффициенты трения
между всеми поверхностями одинаковы.
101. Жонглёр держит за концы невесомую, нерастяжи-
мую нить, на которую нанизаны два шарика массой m каж-
дый, которые могут без трения скользить по ней. Край-
ние участки нити всегда составляют угол a с вертикалью
(рис. 44), а сила натяжения нити постоянна и равна T. Че-
рез какое время шарики столкнутся, если в начальный мо-
мент они неподвижны, находятся на одинаковой высоте на
расстоянии L друг от друга?
Задачи районных туров 39

Теплота

102. В вертикальном цилиндре под поршнем площа-


дью S = 300 см2 находится лёд при температуре t = 0 ◦ C. В
поршне находится нагреватель мощностью P = 1 кВт. После
включения нагревателя поршень начал равномерно опус-
каться. Определите его скорость. Плотность воды и льда,
а также удельная теплота плавления льда известны. Тепло-
обменом с окружающей средой пренебречь.
103. Сосуд наполнен до краёв водой массой M = 20 кг с
температурой t1 = 10 ◦ C. В него аккуратно опускают кусок
льда массой m = 2,1 кг, имеющий температуру t0 = 0 ◦ C. Ка-
кая температура установится в сосуде? Удельная теплоём-
кость воды и удельная теплота плавления льда известны.
Тепловыми потерями пренебречь.
104. Из четырёх нихромовых проволок с удельным со-
противлением r, площадью сечения s и длиной L, выполне-
на фигура, представляющая собой крест. Крест подключают
к источнику постоянного тока с напряжением U следующим
образом: положительный полюс –– к точке пересечения про-
волок, отрицательный полюс –– к концам креста. Фигуру по-
мещают в термос, наполненный дистиллированной водой, и
замыкают ключ. Через какое время вода в термосе закипит?
Масса воды –– m, удельная теплоёмкость –– c, начальная тем-
пература –– T. Потерями тепла пренебречь.
105. Имеются две трубы, подсоединённых к смесителю.
На каждой из труб имеется кран, которым можно регули-
ровать поток воды по трубе, изменяя его от нуля до макси-
мального значения J0 = 1 л/с. В трубах течёт вода с темпера-
турой t1 = 10 ◦ C в одной трубе и t2 = 50 ◦ C в другой. Построй-
те график зависимости максимально возможного потока во-
ды с данной температурой, вытекающего из смесителя, от
температуры этой воды. Тепловыми потерями пренебречь.
106. В верхнем колене длинного вертикального стоя-
ка теплоизолированной трубы образовалась ледяная пробка
весом 8 кг при 0 ◦ C. Когда в трубу налили 6,26 л кипятка,
пробка оторвалась и упала на дно стояка. Чему равна мини-
мально возможная высота стояка, если после падения весь
лёд растаял?
40 Условия задач

107. Два тела, имеющих одинаковые удельные теплоём-


кости, помещены в пустой калориметр. Графики зависимо-
сти температуры тел от времени приведены на рис. 45. Най-
дите отношение масс этих тел. Теплоёмкостью калориметра
пренебречь.
108. Имеются одинаковые резисторы, имеющие форму
правильного цилиндра. Боковая поверхность каждого рези-
стора хорошо теплоизолирована, и при его нагревании теп-
лоотдача происходит только через торцы. Один из резисто-
ров подсоединили к идеальной батарее. При этом он нагрел-
ся до температуры t1 = 38 ◦ C. Потом к этой батарее последо-
вательно подключили два таких резистора, плотно совме-
стив их торцы и обеспечив хороший электрический и теп-
ловой контакт (см. рис. 46). До какой температуры нагре-
ются резисторы? Комнатная температура t0 = 20 ◦ C. Мощ-
ность теплоотдачи пропорциональна разности температур
резистора и окружающей среды. Сопротивление резисторов
при нагревании не изменяется.
Задачи районных туров 41

Электричество
109. Две бусинки, массой m каждая, подвешены к одной
точке на нитях длиной L (см. рис. 47). Заряды бусинок –– од-
ного знака, и сила взаимного отталкивания между бусинка-
ми в состоянии равновесия равна F. Определите расстояние
между бусинками.
110. К источнику постоянного напряжения 200 В под-
ключена схема из четырёх резисторов (см. рис. 48). На двух
резисторах выделяется мощность 50 Вт, на двух других ––
100 Вт. Как изменятся эти мощности, если замкнуть ключ
К?
111. В кабине космического корабля высотой H = 3 м со-
здано однородное электрическое поле, направленное от по-
толка к полу, напряжённостью E = 103 В/м. От пола к по-
42 Условия задач

толку с разными начальными скоростями кидают малень-


кий упругий шарик с зарядом q = 10−5 Кл и массой m = 1 г.
Постройте график зависимости времени, за которое он упа-
дёт на пол, от начальной скорости, с которой его кидают.
Сопротивлением воздуха пренебречь. Шарик могут бросать
со скоростями от 1 м/с до 10 м/с.
112. Сопротивление элемента X меняется в зависимости
от напряжения на нём. Если напряжение U < Uкр , то сопро-
тивление X равно R, а при U > Uкр сопротивление равно
2R. Из трёх элементов X собирают схему, показанную на
рис. 49. Найдите зависимость тока, текущего через схему,
от напряжения на ней.
Задачи районных туров 43

113. Когда в настольную лампу вставили лампочку, на


которой рассеивается мощность W1 = 60 Вт, то оказалось,
что на соединительных проводах лампы рассеивается мощ-
ность W2 = 10 мВт. Какая мощность будет рассеиваться на
соединительных проводах, если поставить лампочку мощ-
ностью W3 = 40 Вт? Напряжение в сети в обоих случаях счи-
тать равным U = 220 В.
114. Пустотелый металлический шар имеет радиус r = 10 см
и толщину стенок d = 1 мм. Он изготовлен из меди, за ис-
ключением полоски на «экваторе» шириной a = 2 мм, ко-
торая выполнена из алюминия (см. рис. 50). Когда на «по-
люса» шара было подано напряжение U = 0,1 мВ, через него
пошёл ток I = 5,12 А. Опыт повторили с другим шаром, у ко-
торого вместо алюминиевой полоски была железная. Какой
ток пойдёт через этот шар? Удельные сопротивление алюми-
ния и железа известны. Контактной разностью потенциалов
пренебречь.
115. Схема на рис. 51 собрана из одинаковых лампочек
и подключена к источнику постоянного напряжения. Рас-
положите лампочки в порядке возрастания яркости.

10 к л а с с
Кинематика, единицы измерения и качественные задачи
116. Птица вырвалась из рук птицелова и полетела от
него по прямой с постоянной скоростью. Птицелов тут же
бросил в неё камень со скоростью v под углом a к горизонту.
Получилось так, что камень попал в птицу в верхней точке
своей траектории. Какова была скорость птицы?
44 Условия задач

117. В специальной системе единиц ССЕ используются


те же формулы, что и в системе единиц СИ. Заряд электрона
в этой системе равен 4,8 · 10−10 езссе. Единица напряжения
ССЕ в 300 раз больше, чем в СИ. Единицы времени в обеих
системах совпадают. Чему равен 1 Джоуль в ССЕ?
П р и м е ч а н и е. Заряд электрона равен 1,6 · 10−19 Кл.
118. Материальная точка может двигаться вдоль прямой
OX, причём проекция ускорения на эту ось зависит от коор-
динаты x так, как указано на графике (см. рис. 52). Если в
начале движения, при x = 0, проекция скорости материаль-
ной точки оказалась равна v0x = 1 м/с. Через какое время
координата и проекция скорости материальной точки снова
примут те же значения?
119. В два одинаковых стакана налили горячий чай. В
одном из стаканов чай помешивают ложкой. В каком из
стаканов чай будет остывать быстрее и почему? (Укажите
несколько причин, на Ваш взгляд наиболее значимых.)
120. Физики в стране Х пользуются теми же формула-
ми, что и мы, но единицы измерения величин у них другие.
Температура измеряется в градусах Фаренгейта (◦ F), мощ-
ность в лошадиных силах (hp), ускорение в единицах g (g),
длина в футах (ft). Чему равна удельная теплоёмкость воды
в системе единиц страны Х, если в системе СИ она равна
c = 4,2 кДж/(кг · ◦ C)?
Задачи районных туров 45

П р и м е ч а н и е: 1 ◦ F = (5/9) ◦ C, 1 hp = 0,73 кВт, 1 g = 9,8 м/с2 ,


1 ft = 0,3 м.
121. Из центра пола цилиндрической комнаты под углом
a к горизонту с некоторой скоростью бросают мячик. Через
время T, после трёх упругих ударов о стенки и потолок, он
опять оказывается в центре пола. Определите скорость мя-
чика при броске. Радиус комнаты равен r. Сопротивлением
воздуха пренебречь. Ускорение свободного падения равно g.
122. Горит башня, причём возгорание произошло в двух
местах: на 1/10 высоты башни и на L = 220 м выше. Пла-
мя распространяется вверх в s = 7 раз быстрее, чем вниз.
Башня сгорела дотла за t1 = 60 ч. Если бы L было в 2 раза
больше, башня сгорела бы целиком за t2 = 61 ч, а если бы
в 2 раза меньше, то время бы не изменилось (60 ч). Чему
была равна высота башни?
123. Вдоль бесконечно длинной дороги стоят светофоры,
отделённые друг от друга одинаковыми расстояниями L. На
каждом светофоре периодически включается на некоторое
время красный свет, а затем, на то же самое время, вклю-
чается зелёный, причём, на каждом следующем светофоре
красный свет включается в тот момент, когда на преды-
46 Условия задач

дущем включается зелёный. Найти все возможные перио-


ды работы светофоров (время от одного включения крас-
ного света до следующего на данном светофоре), при кото-
рых возможно равномерное безостановочное движение ма-
шин со скоростью V .
124. Вдоль бесконечно длинной дороги стоят шлагбау-
мы, отделённые друг от друга одинаковыми расстояния-
ми. Каждый шлагбаум периодически открывается на пери-
од T1 = 1,1t, а затем закрывается на время T2 = 0,9t, при-
чём, каждый последующий шлагбаум открывается через
некоторое, одинаковое для всех шлагбаумов, время после
предыдущего. При каких расстояниях между шлагбаума-
ми возможно равномерное безостановочное движение ма-
шин со скоростью V в обе стороны? Как изменится ответ,
если T2 = 1,1t, а T1 = 0,9t?
Механика и гидростатика
125. В мензурке достаточной высоты упруго скачет боль-
шое количество шариков. Каждый из них долетает ровно
до отметки 750 мл. В некоторый момент мензурку начина-
ют опускать вниз с ускорением g/2. Какая часть шариков
после этого будет долетать до отметки 1 литр? Шарики не
сталкиваются друг с другом и ударяются о дно мензурки в
случайные моменты времени. Мензурка имеет цилиндриче-
скую форму.
126. Маленькая монета находится внутри тазика, сече-
ние которого показано на рис. 53. На какой высоте относи-
тельно дна тазика будет находиться монета, если тазик не
наклоняя двигать с горизонтальным ускорением 20 м/с2 ?
Сухим трением монеты о тазик можно пренебречь.
127. В комнате висит герметичный пакет, наполненный
лёгким газом и прикреплённый к полу верёвкой. Как из-
менится высота пакета над полом, если воздух в комнате
нагреть? Ответ обоснуйте.
128. Машина массы m стоит на горизонтальном покры-
тии с коэффициентом трения m. Двигатель машины разви-
вает постоянную мощность P, передаваемую на задние ко-
лёса машины. Какой путь пройдёт машина при старте с
места до окончания пробуксовки колёс? Центр масс маши-
ны всегда расположен посередине между передней и задней
Задачи районных туров 47
48 Условия задач

осью на высоте h от поверхности дороги. Расстояние между


осями L. Автомобиль разгоняется по прямой с постоянным
ускорением, его передние колёса не отрываются от поверх-
ности.
129. На длинный горизонтальный стержень надеты N
одинаковых неупругих бусинок (см. рис. 54). С краю нахо-
дится ещё одна бусинка в 2 раза большей массы. Ей прида-
ют некоторую скорость в направлении остальных бусинок,
сообщая кинетическую энергию E. Найдите выделившееся
в результате соударений тепло. Трением пренебречь.
130. Было решено построить глубоководную лаборато-
рию. Для этого под воду опускают лёгкие модули объёма V
с помощью прикреплённого к ним балласта массы M. По-
сле этого модули закрепляют на дне, а балласт медленно
поднимают на борт судна (см. рис. 55). Каков КПД описан-
ного способа погружения? Плотность вещества балласта r,
плотность воды r0 , ускорение свободного падения g. Разме-
ры модуля и балласта малы по сравнению с глубиной их
погружения.
Задачи районных туров 49

131. На краю доски длиной L лежит кусочек резины. Ко-


эффициент трения покоя между резиной и доской равен m1 ,
коэффициент трения скольжения равен m2 . Конец доски,
на котором лежит резинка, начинают медленно поднимать,
и в некоторый момент она соскальзывает. После начала её
движения угол между доской и горизонтом не меняется. За
какое время резинка достигнет другого края доски? Ускоре-
ние свободного падения равно g.
132. Автомобиль двигается вниз по склону с постоян-
ным ускорением a = 1 м/с2 . Каким должен быть коэффици-
ент трения, чтобы это было возможно? Уклон горы a = 45◦ .
133. Известно, что разгоняя поезд из состояния покоя
с ускорением a, электровоз совершает работу A. Чему ока-
жется равна эта работа, если за время разгона грузчики пе-
ренесут из заднего вагона в передний груз массой m? Массу
грузчиков не учитывать. Расстояние между первым и по-
следним вагоном равно L.
134. Небольшое тело массы m устанавливают в верхней
точке наклонной плоскости высоты h и сообщают ему на-
чальную скорость V0 , направленную вдоль наклонной плос-
кости. Поверхность плоскости неоднородна, поэтому ско-
рость тела изменяется произвольным образом. В нижней
точке плоскости скорость тела также равна V0 . Найти ра-
боту силы трения.
135. Для тренировки космонавтов используется имита-
тор невесомости. Космонавт крепится к легкому тонкому
тросу, намотанному на лёгкий барабан радиуса r1 . На бара-
бане имеется тормозное устройство в виде цилиндра радиу-
са r2 , к которому могут с силой F прижиматься тормозные
колодки. Коэффициент трения между колодками и бараба-
ном m. В процессе тренировки космонавт свободно падает
с нулевой начальной скоростью время T, затем включается
тормоз, и космонавт достигает земли с нулевой скоростью.
Какова должна быть минимальная высота начальной точки
полёта? Ускорение свободного падения равно g.
136. На горизонтальном шероховатом столе лежит кусок
мела массой m0 . Ему придают скорость V в горизонтальной
плоскости. Какое время t он будет двигаться, если известно,
что при движении по столу он истирается, теряя массу h на
50 Условия задач

каждом метре пути? Ускорение свободного падения равно


g, коэффициент трения равен m.
137. В вертикальной трубе находятся поршень массой M
и шарик массой m, лежащий на поршне. Шарик поднима-
ют на высоту h и отпускают без начальной скорости. Уда-
ры шарика о поршень абсолютно упругие, сила сухого тре-
ния между поршнем и трубой равна F. Насколько опустится
поршень через достаточно большое время? Ускорение сво-
бодного падения равно g. Сопротивлением воздуха можно
пренебречь.
138. Лёгкий жёсткий стержень с шариком массы m на
конце свободно вращается в вертикальной плоскости вокруг
точки O. Известно, что в верхней точке траектории модуль
силы натяжения стержня равен T, что в два раза меньше,
чем в нижней точке. Найдите отношение скоростей шарика
в верхней и нижней точках траектории. Ускорение свобод-
ного падения равно g.
139. На упругом шнуре, подчиняющемся закону Гука, в
поле силы тяжести подвешен точечный магнит массы m1 .
К нему снизу с большого расстояния медленно подносят де-
ревянную подставку, на которой точно под первым лежит
второй магнит. Магниты притягиваются. В некоторый мо-
мент времени, когда удлинение шнура стало в 3 раза боль-
ше первоначального, нижний магнит оторвался от подстав-
ки. Найдите массу нижнего магнита. Ускорение свободного
падения равно g.
140. Найти расстояние от центра планеты до её непо-
движного (всё время находящегося над одной и той же точ-
кой планеты) спутника. Ускорение свободного падения на
поверхности планеты –– g, радиус планеты –– r, период её
вращения вокруг оси –– T.
141. Мальчик везёт на санках дырявое ведро с водой. В
каждую секунду масса ведра с санками уменьшается на 1/n
от своей первоначальной величины m0 . Какую работу про-
извёл мальчик, если скорость санок V0 и угол a, под кото-
рым он держал верёвку, оставались всё время постоянны-
ми, а масса груза уменьшилась за это время наполовину?
Коэффициент трения санок о снег равен k, а ускорение сво-
бодного падения –– g.
Задачи районных туров 51

142. Два невесомых блока соединены невесомой нерас-


тяжимой нитью (см. рис. 56). Левый груз представляет со-
бой гайку, которая может с трением скользить по нити. В
начальный момент оба груза закреплены и находятся в рав-
новесии. Затем, гайка начинает скользить по нити с посто-
янным ускорением a относительно земли. Чему равно уско-
рение второго груза?
143. Две частицы движутся вдоль одной прямой навстре-
чу друг другу со скоростями V1 и V2 соответственно. По-
сле их абсолютно неупругого столкновения скорости частиц
оказались равны V . Найти отношение масс частиц.
144. Математический маятник длиной L, совершая ма-
лые колебания, может абсолютно упруго соударяться со
стенкой (см. рис. 57). Угол a, который стенка образует с
вертикалью, мал. В начальный момент маятнику, находя-
щемуся в положении равновесия, сообщают некоторую ско-
рость V . Найти зависимость периода колебаний маятника
от величины его начальной скорости. Время соударений ма-
ятника со стенкой мало.
145. На гладком конусе с углом при вершине 120◦ шар-
нирно закреплён невесомый нерастяжимый стержень дли-
52 Условия задач

ной L (см. рис. 58). К концу стержня прикреплён груз. Вся


система вращается вокруг вертикальной оси. При какой ча-
стоте вращения груз разорвёт стержень, если стержень вы-
держивает утроенный вес груза?
146. Имеется длинная труба, внутри которой находится
поршень площадью S, который может двигаться с трением
внутри трубы. Слева от поршня –– вакуум, а в правую часть
трубы направляется поток частиц, летящих со скоростью V
параллельно стенкам. Трение таково, что поршень медлен-
но движется с постоянной скоростью U. Найти мощность
силы трения, если плотность частиц в потоке мала и равна
r. Удар частиц о поршень считать абсолютно упругим.
Задачи районных туров 53

Теплота и молекулярная физика


147. В сосуде объёмом V0 находится смесь водорода и ге-
лия. Когда при изохорном нагреве смеси к ней подвели теп-
ло Q, давление в сосуде возросло на DP. Найти отношение
масс водорода и гелия в смеси. Внутренняя энергия одно-
5 3
го моля гелия равна RT, водорода RT, а их молярные
2 2
массы равны 4 г/моль и 2 г/моль соответственно.
148. Цилиндр, заполненный газом, закрыт невесомым
поршнем площади S (см. рис. 59). Внутри цилиндра на-
ходится спираль сопротивления r, по которой течёт ток I.
Поршень равномерно движется со скоростью V . Определите
теплоёмкость одного моля газа в этом процессе. Атмосфер-
ное давление –– p. Газовая постоянная –– R. В решении ис-
пользовать уравнение состояния для идеального газа.
149. С некоторой массой идеального газа был совершены
процессы 2––3––4––2 и 2––1––3––2, изображённые на рис. 60.
В каком из этих циклов газ совершил бо́льшую работу?
150. В герметичном кубе объёма V находится гелий. При
низкой температуре силы давления на все стенки, кроме
нижней, оказались равными нулю, а сила давления на по-
следнюю –– равной F0 . Найдите силу, действующую на стен-
ки куба при высокой температуре T, если известно, что эта
сила одинакова для всех стенок. Ускорение свободного па-
дения –– g, молярная масса гелия –– m.
151. С помощью кипятильника, рассчитанного на напря-
жение 110 В, можно вскипятить воду в чайнике за время t.
Известно, что превышение мощности кипятильника на 20%
приводит к выходу его из строя. Как с помощью двух кипя-
тильников на 110 В вскипятить такое же количество воды
в чайнике, если напряжение в розетке 220 В? Какое время
потребуется для этого? Потерями тепла пренебречь.
54 Условия задач

152. Стакан массой m, площадью основания S, и высо-


той H плавает вверх дном, наполовину погрузившись в во-
ду. Атмосферное давление равно P0 , температура воздуха
постоянна. На сколько изменится глубина погружения ста-
кана (при условии, что стакан не вынимали из воды), если
атмосферное давление уменьшится на k%?
153. В левой части теплоизолированного сосуда (см.
рис. 61) находится один моль одноатомного газа, а в пра-
вой –– вакуум. Система пребывает в состоянии теплового
равновесия при температуре T0 . Когда клапан в перегород-
ке открывается, газ быстро распределяется по всему сосуду,
вращая при этом вертушку, которая поднимает груз до вы-
соты H, где он затем фиксируется. Масса груза равна m, его
теплоёмкость –– C. Теплоёмкостью вертушки и сосуда можно
пренебречь. Найдите конечную температуру, установившу-
юся в системе.

Электричество
154. Схема, представленная на рис. 62, состоит из 3 иде-
альных батареек и 2 резисторов. Напряжения на батарей-
ках равны U1 = U2 = 2 В, U3 = 6 В. Сопротивления резисто-
ров равны R1 = R2 = 4 Ом. Определите токи, текущие через
каждый из элементов схемы. Батарейки 1 и 2 одинаковы.
Задачи районных туров 55
56 Условия задач

155. Отрицательно заряженная частица с пренебрежимо


малой начальной скоростью вылетает из пластины A (см.
рис. 63), имеющей потенциал −f, и ускоряется в направле-
нии плоской заземлённой сетки B. Затем, частица влетает
в плоский конденсатор на расстоянии d/4 от отрицательно
заряженной пластины, причём скорость частицы в этот мо-
мент параллельна пластинам конденсатора. При какой мак-
симальной разности потенциалов между пластинами кон-
денсатора частица сможет его покинуть, если расстояние
между пластинами равно d, а длина пластин –– L. Краевыми
эффектами, силой тяжести, а также полем между сеткой B
и конденсатором пренебречь.
156. Имеются два куска проволоки, сделанные из оди-
накового материала. Теплоёмкости кусков одинаковы. Если
их подвесить, то удлинение, вызванное собственной тяже-
стью, у первого куска будет в 2 раза больше, чем у второго.
Как относятся электрические сопротивления кусков? Пол-
ное электрическое сопротивление проволоки зависит от её
Задачи районных туров 57

длины, площади сечения и удельного электрического сопро-


тивления материала, из которого она сделана.
157. Зависимость тока от напряжения на элементе X
приведена на рис. 64. Постройте график зависимости тока
от напряжения для схемы, изображённой на рис. 65. Схема
состоит из резисторов R и элементов X.
158. В пространстве параллельно друг другу, и перпен-
дикулярно вектору g находятся 3 плоские, бесконечно про-
тяжённые равномерно заряженные диэлектрические пла-
стины. Расстояния между пластинами равны H12 = 200 см и
H23 = 50 см. Потенциал средней пластины равен Ф = −2 В,
а потенциалы крайних пластин равны 0. Из некоторой точ-
ки нижней пластины вылетает заряженная частица массы
m = 100 г с зарядом Q = 1 Кл со скоростью v = 5 м/с, направ-
ленной под углом a = 30◦ к пластине. Частица может свобод-
но проходить сквозь среднюю пластину, но мгновенно по-
глощается при столкновении с крайними пластинами. Ка-
ково горизонтальное смещение частицы от начального мо-
мента до момента поглощения частицы?
58 Условия задач

159. Схема, изображённая на рис. 66, собрана из оди-


наковых лампочек и подключена к источнику постоянного
напряжения. Расположите лампочки в порядке яркости све-
чения.

11 к л а с с
Задания тестов
Необходимо выбрать из предложенных ответов толь-
ко один правильный.
160. Одна калория равна:
a) 4,2 кг · м3 /сек; b) 6,2 кг/(м2 · сек);
c) 1,16 · 10−3 Вт · час; d) 2,4 м2 /сек; e) 42 кг2 · м/сек;
f) теплоте, содержащейся в одном калориметре весом
1 кг при нуле градусов Цельсия.
161. Схема, изображённая на рис. 67, подключена к ис-
точнику постоянного напряжения 220 В. Схема состоит из
двух резисторов, одного реостата R, двух идеальных ампер-
метров и идеального вольтметра. Как изменятся показания
приборов, если движок реостата сдвигают так, что его со-
противление увеличивается?
a) I1 и I2 уменьшаются, V увеличивается;
Задачи районных туров 59
60 Условия задач

b) I1 уменьшается, I2 , V увеличиваются;
c) I1 , I2 и V увеличиваются;
d) V уменьшается, I1 , I2 увеличиваются;
e) I2 уменьшается, I1 , V увеличиваются;
f) V , I2 уменьшаются, I1 увеличивается.
162. Мячик отпустили без начальной скорости с высо-
ты H. Мячик скачет, упруго ударяясь о пол. График зави-
симости проекции скорости мячика на вертикальную ось,
направленную вверх, изображается ломаной, соединяющей
следующие пронумерованные точки (см. рис. 68). Сопротив-
лением воздуха пренебречь.
a) 1––8––5––9––7; b) 1––2––5––9––7;
c) 1––2––5––3––7; d) 0––4––3––6––7;
e) 1––8––2––5––9––3––7; f) 0––4––5––9––7.
163. Каждая из трёх тепловых машин использует в каче-
стве рабочего тела один моль идеального одноатомного га-
за. В первой машине газ участвует в циклическом процес-
се 1––2––4––1 (см. рис. 69), во второй машине –– в процессе
1––2––3––1, в третьей машине используется цикл 1––3––4––1.
У какой машины самый маленький к.п.д.?
a) у первой машины; b) у второй машины;
c) у третьей машины; d) у всех машин к.п.д. одинаков;
Задачи районных туров 61

e) у первой и третьей машин; f) у второй и третьей ма-


шин.
164. С вертолёта, находящегося на большой высоте,
сбросили теннисный мячик. В конце падения мячик абсо-
лютно упруго ударился о наклонную плиту. Известно, что
мячик отскочил от плиты под углом 45◦ к вертикали (см.
рис. 70). Модуль ускорения мячика сразу после отскока ра-
вен √  √  √
a) g; b) 2g; c) 2g; d) 2 + 2g; e) 2 − 2g; f) 0.
165. Три трубы одинакового размера расположены вер-
тикально. Первая труба сделана из меди, вторая из резины,
а третья –– из меди со вставкой в неё резиновой вертикаль-
62 Условия задач

ной полосы (заштрихована на рис. 71). В трубы одновремен-


но бросили одинаковые магниты. Из какой трубы магнит
выпадет раньше остальных?
П р и м е ч а н и е. Считайте известным, что магнит не
прилипает к меди.
а) из первой; b) из второй;
c) из третьей; d) из первой и третьей;
e) из второй и третьей; f) из всех одновременно;
166. Качающийся маятник сначала сильно охладили,
так, что на него из воздуха начал намерзать лёд. Затем ма-
ятник сильно нагрели, так что лёд с него испарился. Как
будет меняться амплитуда колебаний маятника? Затухани-
ем маятника из-за сопротивления воздуха пренебречь.
a) сначала увеличится, а затем уменьшится;
b) сначала уменьшится, а затем увеличится;
c) сначала уменьшится, а затем не изменится;
d) сначала не изменится, а затем увеличится;
e) не изменится;
f) будет всё время увеличиваться.
167. Над идеальным газом совершили процесс 1––2––3 и
построили различные графики зависимостей температуры
T, давления P и объёма V друг от друга (см. рис. 72). Позже
оказалось, что некоторые оси координат забыли подписать,
а на втором графике забыли пронумеровать точки. Замени-
те символы A, B на верные физические величины и прону-
меруйте точки X, Y, Z на втором графике.
Задачи районных туров 63

a) A = V , B = T, X = 1, Y = 2, Z = 3;
b) A = T, B = P, X = 1, Y = 2, Z = 3;
c) A = V , B = T, X = 3, Y = 2, Z = 1;
d) A = T, B = P, X = 3, Y = 2, Z = 1;
e) A = T, B = T, X = 1, Y = 2, Z = 3;
f) A = T, B = T, X = 3, Y = 2, Z = 1.
168. Диэлектрический шар радиуса r с зарядом q поме-
стили рядом с бесконечной металлической плоскостью, так
что центр шара находится на расстоянии R от плоскости.
Заряд распределён по шару равномерно. Когда шар отпу-
стили, он испытал неупругое столкновение с плоскостью.
Сколько при этом выделилось тепла?
a) kq2 /(R + r); b) 0; c) kq2 /r;
d) kq2 /R − kq2 /r; e) kq2 /(2R) − kq2 /(2r);
f) kq2 /(4r) − kq2 /(4R).
169. Тело съезжает без начальной скорости с горки и по-
падает в «мёртвую петлю» (см. рис. 73). Трения нет. Гра-
фик зависимости от времени для проекции ускорения тела
на ось OX изображается линией, соединяющий следующие
точки на рис. 74:
a) 1, 2, 4, 7, 8, 9, 11; b) 1, 2, 4, 5, 6, 8, 9, 10, 11;
c) 1, 2, 4, 5, 6, 8, 9, 11; d) 1, 2, 3, 4, 7, 8, 9, 11;
e) 1, 2, 3, 4, 7, 8, 9, 10, 11; f) 1, 3, 4, 5, 6, 8, 9, 11.
170. На тележке закреплён контейнер, к стенкам кото-
рого растянутыми одинаковыми пружинами прикреплены
грузы с массами m и 2m, связанные нитью (рис. 75). Нить
перерезают. Куда поедет тележка в первый момент и где от-
64 Условия задач
Задачи районных туров 65

носительно первоначального положения она остановится?


Трение грузов о контейнер слабое, тележка ездит без тре-
ния.
a) сначала –– направо, остановится слева;
b) сначала –– направо, остановится справа;
c) сначала –– налево, остановится слева;
d) сначала –– налево, остановится справа;
e) сначала –– налево, остановится там же, откуда начала
движение;
f) сначала –– направо, остановится там же, откуда начала
движение.
171. Изогнутая проволока расположена в вертикальной
плоскости, как изображено на рис. 76. С края проволоки
отпустили без начальной скорости бусинку; движение бу-
синки в поле силы тяжести периодическое. Как изменится
период, если все размеры системы увеличить в 4 раза? Тре-
нием пренебречь.
66 Условия задач

a) увеличится в 16 раз; b) увеличится в 4 раза;


c) увеличится в 2 раза; d) уменьшится в 16 раз;
e) уменьшится в 4 раза; f) уменьшится в 2 раза.
172. Идеальный газ участвует в некотором процессе. На
графике (см. рис. 77) представлена зависимость внутренней
энергии газа U от его давления P. Какая из линий, нари-
сованных на рис. 78, правильно показывает, как давление
газа зависит от его объёма V ?
a) ABE; b) ABD; c) CBD; d) CBE; e) EBC; f)
EBD.
173. Две диэлектрические большие пластины (см. рис. 79)
равномерно заряжены с плотностью заряда +s и −s. Между
пластинами вдоль одной прямой летает маленький шарик
с зарядом +q. Соударения шарика с пластинами абсолютно
упругие. Какой из графиков (см. рис. 80) изображает зави-
Задачи районных туров 67

симость проекции скорости шарика на ось OX от времени?


Силой тяжести пренебречь. Заряд шарика не меняется при
ударе.
a) 7––8––9––10––11––12; b) 7––4––11;
c) 2––7––8––9––4––5––6––11––12;
d) 1––2––7––8––3––4––9––10––5––6––11––12;
e) 2––7––4––11––6;
f) 1––2––7––3––4––9––5––6––11.
174. Квадратная рамка состоит из провода, по которо-
му течёт постоянный ток. Рамку размещают так, что она
частично оказывается в области, где действует однородное
магнитное поле индукции B (заштриховано на рис. 81).
Рамку отпускают. Куда начнёт двигаться и как станет пово-
рачиваться рамка? Направление тока указано на рисунке.
Силой тяжести пренебречь.
a) будет сдвигаться вправо, поворачиваться по часовой
стрелке;
b) вниз, по часовой; c) вправо, против часовой;
d) влево, против часовой;
e) вверх, не будет поворачиваться;
f) вправо, не будет поворачиваться.

Кинематика, механика и гидростатика


175. Плоская Земля неподвижно покоится в Центре Ми-
роздания на спинах трёх слонов (см. рис. 82). Вокруг центра
Земли по круговой орбите радиуса R двигается Луна мас-
сой M. На поверхность Земли около центра положили бру-
68 Условия задач
Задачи районных туров 69

сок. При каком коэффициенте трения бруска о землю он не


сдвинется с места? Плоскость Земли и плоскость вращения
Луны взаимно перпендикулярны. Ускорение свободного па-
дения на Земле около бруска считать равным g, гравитаци-
онное взаимодействие тел подчиняется закону Ньютона с
гравитационной постоянной G.
176. Два маятника с нитями длиной l1 и l2 с одинаковы-
ми массами m подвешены так (см. рис. 83), что шарики на-
ходятся в непосредственной близости друг от друга. Первый
шарик отводят на небольшой угол и отпускают. Определите
период колебаний системы, если удары шариков абсолютно
упругие.
177. Плотно закрытый железнодорожный вагон удаля-
ется с постоянной скоростью V . Привязанный к его полу
воздушный шарик неподвижен относительно вагона, а удер-
живающая его нить отклонена в перпендикулярном скоро-
сти направлении и составляет угол a с вертикалью. В какую
сторону поворачивает железнодорожный путь? Чему равен
радиус закругления?
178. Небольшой шарик массой m, способный скользить
без трения по горизонтальному стрежню, прикреплён к од-
70 Условия задач

ному из его концов невесомой пружиной жёсткости k. Ша-


рик покоится. После резкого поворота стержня на угол a
в вертикальной плоскости вокруг точки нахождения шари-
ка (пружина поворачивается вместе со стержнем) послед-
ний начинает совершать малые гармонические колебания.
Определить амплитуду и период этих колебаний.
179. Протон (ядро атома водорода) и a-частица (ядро ато-
ма гелия, состоящее из двух протонов и двух нейтронов)
разгоняются одинаковой ускоряющей разностью потенци-
алов и влетают в однородное магнитное поле перпендику-
лярно его линиям. Определить отношение радиусов орбит и
нарисовать траектории движения частиц в магнитном поле.
Массы протона и нейтрона равны.
180. На одном из островов Бермудского треугольника
ускорение свободного падения отклонено на юг и составля-
ет угол a с вертикалью. На каком расстоянии от туземца
упадёт стрела, выпущенная им вертикально вверх с началь-
ной скоростью V0 ? В каком направлении следует выпустить
стрелу для того, чтобы она вернулась обратно?
181. Мальчик сидит на расстоянии R от центра диска,
равномерно раскручивающегося из состояния покоя до уг-
ловой скорости w0 за время T. Какое число оборотов сделает
Задачи районных туров 71

мальчик прежде, чем он начнёт скользить относительно ко-


леса, если коэффициент трения о его поверхность равен m?
182. Тележка массой M, двигающаяся со скоростью V
прямолинейно, наталкивается на лёгкую пружину длиной
L, прикреплённую к стене (см. рис. 84). На тележке закреп-
лён хрупкий предмет массы m, который разбивается, если
его перемещать с ускорением бо́льшим, чем a0 . Какой долж-
на быть жёсткость пружины, чтобы в процессе столкнове-
ния хрупкий предмет не разбился? Трением тележки о пол
пренебречь. Пружину считать идеальной при любой длине
от 0 до L.
183. Тонкостенный перевёрнутый стакан высотой L
быстро погрузили в воду на глубину H, при этом в первый
момент после погружения вода в стакане поднялась до уров-
ня l0 (см. рис. 85). Стакан удерживают под водой с помо-
щью динамометра. На сколько и почему изменится показа-
ние динамометра с течением времени по сравнению с пер-
воначальным? Атмосферное давление равно p0 , плотность
72 Условия задач

воды r0 , площадь дна стакана S. Считать, что температуры


атмосферы и воды совпадают.
184. С наклонной плоскости с углом наклона a съезжает
массивный клин, имеющий угол наклона f (см. рис. 86). На
клин положили лёгкий брусок. С каким ускорением отно-
сительно клина он поедет? Считать, что брусок много легче
клина. Силой трения между плоскостью и клином, а также
между клином и бруском пренебречь.
185. Пловец, многократно преодолевая дистанцию дли-
ной L из A в B, (см. рис. 87) обнаружил, что если выбегать
на берег и пробегать часть пути по земле, то можно добрать-
ся из A в B быстрее, чем если плыть напрямую. Скорости
Задачи районных туров 73

движения пловца по воде и по земле равны V1 и V2 соответ-


ственно. Отрезок AB параллелен берегу. На каком расстоя-
нии от берега могут находиться точки A и B?
186. Стакан c достаточным количеством воды поставили
в микроволновую печь, на расстоянии R от центра враща-
ющейся подставки. Какую форму примет поверхность жид-
кости в стакане, если подставка вращается в печи с угловой
скоростью W?
187. В горизонтально расположенной стеклянной труб-
ке длиной L находится капля ртути массой m (см. рис. 88).
Один из концов трубки герметично закрывают и плавно рас-
кручивают систему до угловой скорости w. Ось вращения
вертикальна. Найти, в каком месте трубки расположится
капля, если первоначально она находилась на расстоянии
x от закрытого конца. Атмосферное давление p A , внутрен-
ний радиус трубки R. Считать, что размеры капли много
меньше x, трением пренебречь.
188. Электропоезд, составленный из одинаковых вагонов
длиной l, начинает торможение в тот момент, когда первый
вагон состава заходит в туннель длиной L. Двигаясь равно-
замедленно, поезд останавливается в тот момент, когда его
последний вагон выходит из туннеля. Известно, что пасса-
жир первого вагона находился в туннеле в течение време-
ни T1 , а последнего –– TN . Чему равно количество вагонов
в составе электропоезда? Расстоянием между вагонами пре-
небречь.
74 Условия задач

189. Камень брошен из точки, расположенной на гори-


зонтальной плоскости, с начальной скоростью V0 под углом
a к горизонту. Определить полную работу, совершённую си-
лой тяжести. Построить графики зависимостей от времени
y-координаты камня и проекций его скорости и ускорения
на ось Oy, направленную вертикально вверх. Сопротивле-
ния воздуха нет.
190. Какую горизонтальную скорость необходимо сооб-
щить маленькому мячику, лежащему на краю верхней сту-
пеньки лестницы для того, чтобы первый отскок мяча про-
изошёл от ступеньки с номером N. Длины и высоты ступе-
нек лестницы равны b и h.
191. В вертикально направленной струе «сверхтекучего
гелия» на высоте h1 неподвижно висит небольшой конус
массой m (см. рис. 89). При ударе о его поверхность жид-
кость отскакивает во все стороны точно горизонтально. Ес-
ли массу конуса удвоить, то он зависнет на высоте h2 . Ка-
кова будет высота фонтана, если удалить конус?
192. Для проведения взрывных работ изготовлен ком-
плект из 10 запальных шнуров, толщины и скорости горе-
ния каждого из которых случайно изменяются по длине, но
полное время сгорания одинаково и равно 1 минуте. Как из
Задачи районных туров 75

имеющегося набора изготовить шнуры, время горения кото-


рых равно 45 секундам? Сколько таких шнуров можно из-
готовить? Шнуры можно тушить, отрезая от них горящую
часть.
193. Две одинаковые доски образуют угол a = 25◦ . На од-
ну из досок, рядом с её краем, налетает маленькая шайба,
скорость которой параллельна второй доске (см. рис. 90).
Считая соударения абсолютно упругими, сосчитать, сколь-
ко раз шайба ударится о доски. Трением и силой тяжести
пренебречь.
194. В центре круглого стола поставили блюдце. Коэф-
фициент трения блюдца о стол равен m. Стол двигали пря-
молинейно с ускорением a в течение промежутка времени
T, а затем равномерно затормозили за то же время. Каков
должен быть минимальный радиус стола L, чтобы блюдце
не упало с него?

Теплота и молекулярная физика


195. Тепловоз «Карно» ездит из пункта A в пункт B по
маршруту длиной L = 20 км. Двигатель тепловоза работает
по циклу Карно, нагревая воду от температуры атмосферы
76 Условия задач

до 100 ◦ C. Обычно тепловоз выезжал из A ночью, когда тем-


пература воздуха была равна Tн = 5 ◦ C, и в ту же ночь воз-
вращался; запас топлива в тепловозе рассчитан строго для
поездки ночью до пункта B и обратно. Однажды чиновни-
ки задержали тепловоз в пункте B, так что обратно он вы-
ехал днём, когда температура воздуха составляла Tд = 25 ◦ C.
На сколько «Карно» не доедет до пункта A из-за нехватки
топлива? Считать силу трения и силу сопротивления воз-
духа постоянными, тепловые потери при нагревании воды
составляют долю h от энергии сгорающего топлива.
196. К резиновому воздушному шару, содержащему мас-
су m гелия, привязана очень длинная верёвка, масса еди-
ницы длины верёвки l. При атмосферном давлении p0 шар
зависает над полом на высоте H (см. рис. 91). На какой вы-
соте окажется шар, если атмосферное давление увеличит-
ся и станет равным p1 ? Считать, что давление газа в шаре
всегда на Dp больше, чем давление окружающего воздуха
в данный момент. Молярные массы воздуха и гелия равны
Mв и Mг соответственно.
197. Сосуд разделён на две половины герметичной пере-
городкой. В левой половине находится в равновесии смесь
гелия и ксенона (см. рис. 92); масса содержащегося гелия
m1 , масса ксенона m2 . В правой половине сосуда первона-
чально –– вакуум. В перегородке на короткое время открыли
Задачи районных туров 77

небольшое отверстие. Найти отношение концентраций ге-


лия и ксенона в правой части сосуда после того, как отвер-
стие закрыли. Молярные массы гелия и ксенона равны M1
и M2 соответственно.
198. Груз поднимают при помощи невесомого поршня,
скользящего без трения в вертикальном теплоизолирован-
ном цилиндре. Под поршнем находится идеальный одно-
атомный газ, медленно нагреваемый при помощи электро-
нагревателя с КПД h = 1/2. Определить КПД подъёмного
устройства, если атмосферное давление отсутствует.
199. Правый конец металлического стержня длиной 1 м
погружен в ацетон, левый погружают в кипящую воду. На
стержне, на расстоянии 47 см от его левого конца, лежит
маленький кристалл нафталина. Сколько ацетона выкипит,
пока расплавится весь нафталин? Считайте, что вся тепло-
передача происходит только через стержень, а поток тепло-
вой энергии через тонкий слой прямо пропорционален раз-
78 Условия задач

ности температур на торцах слоя. Количество кипящей во-


ды в сосуде очень велико, кипение поддерживается. Темпе-
ратура кипения ацетона и температура плавления нафтали-
на заданы.
200. Теплоизолированный сосуд разделён на две части
теплоизолирующим поршнем (см. рис. 93). С одной сторо-
ны от поршня, занимая объём V1 , находится масса m1 водо-
рода, с другой стороны, в объёме V2 , –– масса m2 гелия. Га-
зы в сосуде нагревают, подключая к источнику постоянно-
го напряжения два последовательно соединённых нагрева-
тельных элемента. Первый из этих элементов нагревает во-
дород; известна зависимость его электрического сопротив-
ления от температуры R1 (T). Второй элемент находится в
отделении с гелием. При включении нагревательных эле-
ментов поршень не сдвинулся. Какова зависимость сопро-
тивления второго элемента от температуры?
201. Горизонтально расположенный теплоизолирован-
ный сосуд разделён на N частей V1 , V2 , . . . , VN закреп-
лёнными поршнями, между которыми находятся различ-
ные массы идеального одноатомного газа при различных
начальных температурах и давлениях P1 , P2 , . . . , PN . Опре-
делить давление в секции сосуда с номером i после того, как
поршни получили возможность свободно перемещаться, а в
сосуде установилось термодинамическое равновесие. Тепло-
ёмкостью поршней пренебречь.
Задачи районных туров 79

202. Горизонтально расположенный теплоизолирующий


цилиндр разделён перегородкой на два равных объёма V , в
которых находится по одинаковому числу молей n идеаль-
ного одноатомного газа при температурах T1 и T2 (T1 > T2 ).
Около небольшого отверстия в перегородке со стороны более
нагретого газа расположен небольшой пропеллер, приводи-
мый в движение струёй перетекающего через отверстие га-
за (см. рис. 94). Приводимый им в движение генератор под-
ключён к расположенному в том же объёме нагревателю. До
того, как отверстие было открыто, температура помещён-
ных внутрь сосуда приборов равнялась температуре окру-
жающего газа. Определить давление, которое установится в
системе после достижения равновесия, если суммарная теп-
лоёмкость устройств внутри цилиндра равна C.
Электричество и магнетизм
203. На нерастяжимой нити длиной 2L закреплены на
расстоянии L/2 друг от друга два маленьких шарика с заря-
дами +q и −2q. Концы нити связали, образовавшееся коль-
цо натянули на два тонких цилиндра, расположенных на
расстоянии L друг от друга. Радиусы цилиндров очень ма-
лы, так что обе половины нити лежат практически на одной
прямой (см. рис. 95). Систему поместили в однородное элек-
трическое поле E. Первоначально нить вместе с шариками
имела небольшую скорость, так что она скользила по ци-
80 Условия задач

линдрам. Затем, из-за небольшого трения о воздух, система


остановилась. Где могут расположиться шарики? Силой тя-
жести и трением нити о цилиндры пренебречь.
204. Жёсткий металлический изолированный провод
изогнут в форме пятиконечной звезды; по нему течёт элек-
трический ток I. Провод лежит на столе, в магнитном по-
ле, как изображено на рис. 96. Магнитное поле индукции
B перпендикулярно поверхности стола; слева от линии AA
оно направлено вниз, а справа –– вверх. Длина одной сторо-
ны клеточки на рисунке равна d. Определить силу, действу-
ющую со стороны поля на провод.
205. T-образный каркас вращается с частотой w вокруг
точки O в сильном магнитном поле индукции B. На пере-
Задачи районных туров 81

кладину каркаса насажены две маленькие заряженные бу-


синки массы m, имеющие заряды +q каждая (см. рис. 97);
бусинки могут скользить по проволоке без трения. Найди-
те положение равновесия бусинок. Исследуйте его устойчи-
вость. Что будет, если изменить знак заряда одной из бу-
синок? Бусинки не могут слететь с перекладины, «ножка»
T-образного каркаса имеет длину l. Направление поля B
указано на рисунке, рамка вращается в плоскости рисунка.
206. Лента транспортёра массой M длиной L натянута
вокруг двух неподвижных цилиндров радиуса R. В середине
транспортёр пересекает большую металлическую заряжен-
ную плоскость. На противоположных концах ленты закреп-
лены два легких шарика (см. рис. 98). Лента разгоняется за
счёт того, что каждый раз, когда один из шариков проходит
точку A, ему сообщают заряд −q. Затем, в точке B, сразу
за плоскостью, шарику сообщают заряд +q, а в точке C ша-
рик заряд теряет. Первоначально лента покоится, один из
шариков находится в точке A. Какой станет скорость транс-
портёра через 10 полных оборотов? Трением ленты о цилин-
дры пренебречь, плотность заряда плоскости +s.
207. В катушке индуктивности, в вакууме, расположены
на одной прямой положительно заряженные частицы (см.
рис. 99). Сначала ток через катушку не течёт, частицы по-
коятся (электростатическое взаимодействие между частица-
82 Условия задач

ми можно считать пренебрежимо малым). Через катушку


начинают пропускать ток, так что магнитное поле B внутри
катушки за малый промежуток времени Dt увеличивается
до значения B0 (график B(t) см. на рис. 99). Опишите тра-
ектории частиц и их взаимное расположение через время
t после включения поля. Считайте, что взаимодействие ча-
стиц друг с другом пренебрежимо мало по сравнению с вза-
имодействием с катушкой на протяжении всего движения.
Поле направлено «за рисунок», силой тяжести пренебречь.
208. На железный сердечник (см. рис. 100) намотаны
две катушки. Магнитный поток, создаваемый каждой ка-
тушкой, не выходит из железного сердечника и делится по-
ровну между разветвлениями. При включении катушки 1
в цепь переменного тока с напряжением U1 = 40 B напря-
жение на катушке 2 равно U. Какое напряжение будет на
зажимах катушки 1, если катушку 2 включить в цепь пере-
менного тока с напряжением U?
209. Известны потенциалы f1 , f2 , f3 , f4 на концах
идеальных проводников, входящих в чёрный ящик C (см.
рис. 101). Известны также токи I1 , I2 , I3 , втекающие и вы-
текающие через концы этих проводов (направления токов
Задачи районных туров 83
84 Условия задач

указаны на рисунке стрелками). Сначала заряд ящика C ра-


вен нулю. Известны также сопротивления R1 и R2 . Каким
будет его заряд через время t?
210. Две частицы, имеющие заряды q1 и q2 и рав-
ные массы, могут скользить без трения вдоль двух па-
раллельных прямых, расположенных на расстоянии L (см.
рис. 102). В начальный момент частица q1 движется со ско-
ростью V0 из бесконечности, приближаясь к покоящейся ча-
стице q2 . Определить установившиеся скорости частиц.
211. Заряженная частица (q, m) может скользить без тре-
ния по проволочному кольцу радиусом R, расположенному
вертикально (см. рис. 103). Какое вертикальное электриче-
Задачи районных туров 85

ское поле нужно приложить, чтобы частота малых колеба-


ний частицы уменьшилась в два раза?
212. Для описания электрических свойств нервного во-
локна (аксона) можно пользоваться следующей моделью:
аксон представляет собой тонкостенную проводящую труб-
ку с внутренним радиусом r и толщиной h, причём r  h.
Удельное сопротивление вещества трубки равно r. Какой
ток протекает через аксон, если удельное сопротивление
раствора в трубке –– r0 , а разность потенциалов между рас-
твором в трубке и внешней средой есть U0 ?
213. В однородном магнитном поле, направленном вер-
тикально вверх, под углом a к горизонту со скоростью V
запущена частица массы m с зарядом q. На некоторой вы-
соте над точкой старта частицы расположена горизонталь-
ная плоскость (см. рис. 104). В результате упругого столк-
новения частицы с плоскостью её заряд уменьшился вдвое.
Траектория частицы после соударения пересекает её траек-
торию частицы до соударения N раз, причём одно из пере-
сечений происходит в точке старта. Найти, на какой высоте
расположена плоскость. Силой тяжести пренебречь. Индук-
ция магнитного поля равна B.
П р и м е ч а н и е. На рисунке оси Ox, Oy, Oz взаимно
перпендикулярны.
214. Два бесконечно длинных тонких стержня располо-
жены в горизонтальной плоскости параллельно друг другу
на расстоянии 2R и заряжены положительно с постоянной
плотностью заряда на единицу длины s. Вдоль вертикаль-
ной нити, туго натянутой посередине между стержнями,
86 Условия задач

может без трения скользить отрицательно заряженная ча-


стица массой m. При каком заряде частицы её можно под-
нять вверх, медленно поднимая вверх стержни? Ускорение
свободного падения –– g. Электрическое поле заряженного
стержня зависит от расстояния до него: E(r) = s/(2pe0 r).
215. Три концентрические металлические сферы с ради-
усами R, 2R, 3R заряжены зарядами q, −q и q соответствен-
но. Внутреннюю сферу (см. рис. 105) соединили очень тон-
ким проводником с наружной сквозь маленькое отверстие в
средней. Сколько тепла при этом выделилось?
216. На горизонтальную диэлектрическую спицу нани-
заны две маленькие положительно заряженные бусинки с
зарядом q каждая и с массами m1 и m2 . Спица вращается
с некоторой частотой w вокруг проходящей через её край
вертикальной оси в однородном магнитном поле B, направ-
ленном вниз (см. рис. 106). Определите, где расположатся
заряды, если масса ближней к центру частицы m1 > m2 ? Си-
лу трения считать пренебрежимо малой.
217. Известно, что спутник Юпитера Ио имеет ионосфе-
ру, проводящую электрический ток. Оцените разность по-
тенциалов, которая наводится поперёк Ио при его движе-
нии по круговой орбите в магнитном поле Юпитера. Орби-
тальная скорость Ио равна VK = 17,3 км/с, а силовые ли-
нии поля Юпитера вращаются вместе с планетой так, что
их линейная скорость вдоль орбиты равна VR = 75 км/с и
совпадает по направлению с VK . Радиус Ио равен 1820 км.
Напряжённость магнитного поля Юпитера равна 2 · 10−6 Тл.
Задачи районных туров 87
88 Условия задач

218. Маленький шарик массы m, имеющий заряд q, вле-


тает в точке A в цилиндр радиуса R перпендикулярно к его
поверхности. Внутри цилиндра имеется магнитное поле на-
пряжённостью B, направленное вдоль оси цилиндра. Все со-
ударения шарика с цилиндром упруги (см. рис. 107). Най-
дите возможные значения скорости шарика, если известно,
что после некоторого числа соударений он возвращается в
точку A.
219. Две диэлектрические пластины, равномерно заря-
женные разноимёнными поверхностными зарядами, распо-
ложены вертикально. Небольшой стальной шарик, перво-
начально имеющий малый заряд +Q, начинает падать без
начальной скорости из точки, расположенной на уровне
верхнего края пластин, посередине между ними. При каж-
дом ударе о пластину он приобретает дополнительный за-
ряд DQ = ±2Q, знак которого определяется её зарядом. Из-
вестно, что за время падения шарик успевает удариться о
пластины N раз. Сколько соударений с пластинами испыта-
ет маленькая капля ртути, выпущенная между пластинами
при тех же начальных условиях?
220. Заряженной частице с массой m, помещённой в ва-
кууме на границе двух областей, в которых созданы одно-
родные магнитные поля B1 и B2 (B2 > B1 ), сообщают на-
чальную скорость V0 , направленную перпендикулярно гра-
Задачи районных туров 89

нице раздела (см. рис. 108). При каких значениях заряда


частицы её траектория пройдёт через точку M, расположен-
ную на расстоянии L от точки старта?
221. Колебательный контур состоит из катушки с ин-
дуктивностью L, конденсатора с ёмкостью C и резистора с
сопротивлением R. В начальный момент на конденсаторе
имелся заряд q и протекал ток I. Какое полное количество
тепловой энергии выделится на резисторе за время затуха-
ния колебаний в контуре, если суммарное сопротивление
проводов равно r? (Лиц, знающих о возможности излуче-
ния электромагнитных волн ускоренно движущимися заря-
дами, просят не беспокоиться –– в данной ситуации роль это-
го эффекта мала).

Оптика
222. Два плоских квадратных зеркала со сторонами a и
2a образуют прямой угол. На расстоянии a от маленького
зеркала и на расстоянии 2a от большого расположен источ-
ник света. Найти область в плоскости рис. 109, в которой
можно наблюдать ровно 2 изображения источника в зерка-
лах.
223. Девочка разглядывает себя в витрине, находясь от
неё на расстоянии 1 м. Витрина состоит из двух стёкол. Де-
вочка видит 2 своих отражения, причём ей кажется, что
90 Условия задач

размер одного изображения составляет 3/4 размера друго-


го. Найти расстояние между стёклами витрины.
224. Собирающую тонкую линзу с фокусным расстояни-
ем f освещают точечным источником, расположенным на
оптической оси на расстоянии a = 2f от линзы (см. рис. 110).
За линзой в фокальной плоскости расположен экран. Опре-
делить площадь тени от линзы на экране. Линза представ-
ляет собой круг радиуса R.
Задачи городских туров
7––8 к л а с с ы
Кинематика, механика и гидростатика
225. Путешественник попал в страну, в которой единицы
измерения расстояния в a раз, времени в b раз, массы в g
раз превосходят соответствующие единицы на его родине.
Как соотносятся единицы измерения силы в этой стране и
на родине путешественника? Формулы силы в этих странах
имеют одинаковый вид.
226. Поверхность образца породы начинает разрушать-
ся, если на неё воздействует давление большее некоторой
определённой величины. Модель лунохода в земных испы-
таниях имела наибольшую массу, при которой она двигаясь
по образцу данной породы, ещё не разрушала её. Во сколько
раз размеры оригинала лунохода могут отличаться от зем-
ной его модели, чтобы луноход не разрушал эту породу на
Луне? Луноход и его модель изготовлены из одних и тех
же материалов. Значения ускорения свободного падения на
поверхностях Земли и Луны известны.
227. Младший Брат отдыхал со своим конем у дороги.
Вдруг он увидел, что мимо него бежит Лиса, держащая в зу-
бах Петушка. Младший Брат тут же бросился седлать коня.
Оседлав его, он поскакал за Лисой, догнал её и освободил
Петушка. На следующий день та же история повторилась со
Старшим Братом. Старший Брат седлал коня вдвое дольше,
чем Младший, но и скакал его конь вдвое быстрее. Кто из
братьев провёл больше времени на коне в погоне за Лисой?
Лиса всё время бежит с постоянной скоростью.
228. Два спортсмена по команде тренера начинают бе-
жать с линии старта по прямой дорожке в одном направле-
нии. В определённые моменты времени они по свистку ме-
няют свою скорость. Графики зависимости скорости каждо-
го спортсмена указаны на рис. 111. Через какое время после
старта они снова поравняются?
229. На столе лежал восковой куб. Давление, оказыва-
емое им на стол, составляло 100 Па. Сверху на воск поло-
жили стальной куб, ребро которого в 3 раза больше ребра
воскового куба. Воск расплющился, и площадь его контак-
92 Условия задач

та со столом увеличилась вдвое. Чему стало равно давление


на стол? Плотности воска и стали известны.
230. При подготовке полёта на Марс Незнайка сконстру-
ировал прибор для измерения ускорения свободного паде-
ния. Прибор представляет собой сосуд, в который помеща-
ют тело цилиндрической формы, привязанное ко дну сосуда
ниткой. Плотность тела меньше плотности воды. При про-
ведении измерений сосуд постепенно заполняют водой и из-
меряют уровень погружения цилиндра в воду в тот момент,
когда нить рвётся. В процессе градуировки прибора выяс-
нилось, что на Земле нить порвалась при погружении на
20 см, а на Луне –– при погружении на 71 см. При какой
глубине погружения порвётся нить при проведении измере-
ний на Марсе? Нить рвётся при одной и той же силе натя-
жения. Значения ускорения свободного падения на поверх-
ностях Земли, Марса и Луны известны.
231. Две дрессированные белки лезут по дереву. Одна
лезет вертикально вверх со скоростью v1 = 10 см/с, другая ––
вертикально вниз со скоростью v2 = 8 см/с. Белки тянут лёг-
кую чашку за прикреплённые к ней одинаковые лёгкие пру-
жины с жёсткостью 10 Н/м каждая. Идёт дождь, из-за кото-
Задачи городских туров 93

рого в чашку каждую секунду набирается 1 г воды. С какой


скоростью движется чашка?
232. Вася гуляет со своим любимым бульдогом Закуса-
ем вокруг ёлки по круговой дорожке радиуса 28 м. Заку-
сай бегает вокруг Васи на натянутом поводке длиной 7 м.
Скорость движения Васи постоянна по модулю и в каждый
момент времени равна скорости Закусая относительно Ва-
си. Когда скорость бульдога относительно земли становится
равной нулю, он гавкает. В тот момент, когда Вася начал
движение по дорожке, Закусай гавкнул. Сколько раз буль-
дог гавкнет, пока Вася пройдёт полный круг по дорожке?
233. Пластмассовый кубик со стороной 10 см привязан к
невесомой нерастяжимой нити, которая намотана на катуш-
ку (см. рис. 112). Разматывая катушку, кубик погружают в
бассейн с жидкостью. Плотность жидкости зависит от глу-
бины. График этой зависимости представлен на рис. 113.
В самом начале погружения нижняя грань кубика касается
жидкости. Постройте график зависимости силы натяжения
нити от длины её размотанной части. Плотность пластмас-
сы, из которой сделан куб, известна.
234. Система состоит из подвижных и неподвижных
невесомых блоков, грузов и лёгкой нерастяжимой нити (см.
94 Условия задач

рис. 114). Трение в системе отсутствует. Масса крайнего


груза m = 10 кг. Найдите массы остальных грузов, если си-
стема находится в равновесии.
235. В цилиндрическом сосуде есть два крана. Один рас-
положен у самого дна, другой –– на 10 см выше. Если в сосуд
налить воды до уровня 40 см и открыть верхний кран, то
вода начинает вытекать, а высота столба воды при этом за-
висит от времени так, как показано на графике 1 рис. 115.
Если же открыть только нижний кран, то уровень воды бу-
дет меняться со временем так, как показано на графике 2
того же рисунка. За какое время уровень воды опустится от
20 см до 15 см, если открыть оба крана? Считайте, что ско-
Задачи городских туров 95

рость вытекания воды из любого крана не зависит от того,


открыт или закрыт другой кран.
236. Весы, изображённые на рис. 116, состоят из тонкого
невесомого стержня, подвешенного в точке A, гири, жёстко
закреплённой на стержне в точке B, и крючка. Для опре-
деления массы груза весы надо уравновесить, меняя поло-
жение крючка, и затем по нанесённой на стержень шкале
(см. рис. 117) определить массу груза в килограммах. Эти-
ми весами решили взвесить 10 кг груш под водой. Отметьте
на шкале положение крючка, при котором весы находятся в
равновесии. Плотность груш r1 = 800 кг/м3 , плотность ма-
териала гири r2 = 2000 кг/м3 , плотность воды известна.
237. Мальчик поднимается в гору со скоростью 1 м/c.
Когда до вершины остаётся идти 100 м, мальчик отпускает
96 Условия задач

собаку, и она начинает бегать между мальчиком и верши-


ной горы. Собака бежит к вершине со скоростью 3 м/c, а
возвращается к мальчику со скоростью 5 м/c. Какой путь
успеет пробежать собака до того, как мальчик достигнет
вершины? Собака и мальчик двигаются прямолинейно.
238. На рис. 118 представлена система из трёх неве-
сомых блоков и трёх грузов. Крайние грузы сделаны из
алюминия, а средний –– из пластилина. Масса левого гру-
за m = 2 кг. Система находится в равновесии. Какой объ-
ём пластилина следует прилепить или отлепить от средне-
го груза, чтобы система находилась в равновесии, когда все
грузы опущены в воду? Плотности воды (r0 ), алюминия (r1 )
и пластилина (r2 ) известны.
Задачи городских туров 97

239. Капли дождя оставляют на боковом стекле автобу-


са, движущегося со скоростью V1 , след, изображённый на
рис. 119 а, а на боковом стекле грузовика, движущегося
со скоростью V2 , след, изображённый на рис. 119, б. Ка-
кой след будет оставаться на боковом стекле легкового авто-
мобиля, движущегося с скоростью (V1 + V2 )/2? Все машины
движутся в одном направлении.
240. Парашютисты при установлении рекордов скорости
в затяжных прыжках используют тяжёлые предметы, на-
пример, гири. Выпрыгнув с гирей из самолёта, спортсмен
разгоняется до рекордной скорости. Затем на некоторой вы-
соте он отпускает гирю и опускается на землю. Для чего на-
до брать с собой гирю? Когда имеет смысл отпускать гирю:
до раскрытия парашюта или после раскрытия парашюта?
Предположим, разные парашютисты, прыгая с одной вы-
соты, отпускают гирю на разном расстоянии от поверхно-
сти земли. Постройте: 1) график качественной зависимости
времени полёта гири (от момента прыжка парашютиста до
приземления гири) от скорости, с которой гиря падает на
землю; 2) график качественной зависимости времени полё-
та парашютиста от скорости, с которой гиря падает на зем-
лю. Ответы обоснуйте.
241. Два шара одинакового объёма V соединены длинной
нитью. Плотность первого шара r1 , плотность второго шара
r2 . Систему сбрасывают с вертолёта. Через некоторое вре-
мя скорость падения системы становится постоянной, так
98 Условия задач

как сила тяжести уравновешивается силой сопротивления


воздуха. Найдите силу натяжения нити в установившемся
режиме падения.
242. Мячик бросают с земли вертикально вверх с началь-
ной скоростью V0 . Постройте график зависимости проекции
скорости мяча на ось, направленную вертикально вверх, от
времени. Удары о землю абсолютно упругие. Сопротивлени-
ем воздуха пренебречь.
243. В комнате высотой H к потолку одним концом при-
креплена лёгкая пружина жёсткости k, имеющая свобод-
ную длину l0 (l0 < H) (см. рис. 120). На полу под пружиной
размещают брусок высотой x с площадью основания S, из-
готовленный из материала плотностью r. Построить график
зависимости давления бруска на пол от высоты бруска.
244. На наклонной плоскости покоится брусок. Чтобы
сдвинуть его вверх по наклонной плоскости, к нему следу-
ет приложить силу F1 , направленную вверх, вдоль наклон-
ной плоскости, Чтобы сдвинуть тот же брусок по той же
наклонной плоскости вниз, прикладывают к нему силу F2 ,
направленную вниз, вдоль наклонной плоскости. Найдите
силу трения при скольжении бруска, если без приложения
сил F1 или F2 брусок покоится.
Задачи городских туров 99

245. В решётчатом контейнере (см. рис. 121) находят-


ся хлопчато-бумажные мешки с поваренной солью массой
M = 490 кг. К контейнеру вплотную привязан пробковый
поплавок объёмом V = 0,1 м3 . Контейнер поместили в резер-
вуар и залили одним кубометром воды. Известно, что при
данной температуре в одном литре воды может растворить-
ся не более 350 г соли. Всплывёт ли контейнер с поплавком?
Считать, что в процессе растворения соли в воде уровень
жидкости в резервуаре практически не меняется. Массами
и объёмами контейнера, верёвки и мешков пренебречь.
246. C помощью верёвок, перекинутых через систему
блоков, спасатели перемещают равномерно и прямолиней-
но массивную плиту так, как показано на рис. 122. С какой
результирующей силой верёвки действуют на плиту? Спаса-
тели тянут свой конец верёвки с силой F. Массами верёвок
и блоков пренебречь.
247. Невесомый рычаг AC установлен на упоре так, что
плечо BC в 2 раза длиннее AB. К плечам рычага под-
вешены невесомый блок и массивное неоднородное тело
(см. рис. 123). Слева к блоку подвешивают такой груз, что
вся система оказывается в равновесии. Найдите отношение
масс груза и тела.
248. В аквариуме, наполненном водой, плавают два ша-
ра одинакового радиуса (см. рис. 124), сделанные из мате-
риалов разных плотностей –– r1 = 800 кг/м3 и r2 = 900 кг/м3 .
100 Условия задач
Задачи городских туров 101

Аквариум уравновешивают на упоре, рядом с которым име-


ется небольшое отверстие, закрытое пробкой. Пробку акку-
ратно вынимают. В какую сторону наклонится аквариум,
когда один из грузов достигнет дна?
249. К невесомому блоку A, прикреплённому к потол-
ку, подвешен груз неизвестной массы M с одной стороны и
блок B с другой. Через последний блок перекинута нить, на
которой удерживаются два груза m1 = 50 г и m2 = 30 г. Ко
второму грузу прикреплена пружина жёсткости k = 10 Н/м,
которая своим вторым концом вмонтирована в пол (см.
рис. 125). Найдите массу M, при которой система оказы-
вается уравновешена.
250. Два массивных поршня закрывают оба конца жёст-
кого неподвижного S-образного колена (см. рис. 126), за-
полненного водой. К правому поршню прикреплена пружи-
на жёсткости k = 1000 Н/м, другой конец которой вмонтиро-
ван в пол. В равновесии правый поршень находится на рас-
стоянии L = 20 см от конца трубы. На левый поршень акку-
ратно положили тяжёлый груз. Чему равна максимальная
масса груза, при которой вода не будет выливаться из ко-
лена? Площадь левого поршня равна S1 = 100 cм2 , площадь
правого –– S2 = 500 cм2 . Постоянная свободного падения из-
вестна.
251. Из озера A в озеро B течёт река. Скорость течения
реки u = 5 км/ч. Из озера A в озеро B и из озера B в озе-
102 Условия задач
Задачи городских туров 103

ро A в начале каждой минуты отправляются в путешествие


водомерки. Скорость насекомых относительно воды посто-
янна и равна v = 10 км/ч. Расстояние между озёрами равно
s = 10 км. Сколько товарок встретит на своём пути водомер-
ка, отправившаяся из A в B?
252. Имеются два скреплённых блока, радиус у одного в
два раза больше, чем у второго (см. рис. 127). Радиус мень-
шего блока равен r = 10 см. К блокам подвешивают тонкую
однородную палку длины L = 1 м так, что вся конструкция
оказывается уравновешенной. Каково расстояние от левого
крюка до правого конца палки?
253. Имеется цилиндрический сосуд, заполненный неиз-
вестной жидкостью, линейка и два поплавка из разных ма-
териалов. Плотность одного из поплавков известна. Приду-
майте и опишите способ измерения плотности второго по-
плавка.
254. На дне бассейна установлена П-образная конструк-
ция, размеры которой показаны на рис. 128. Плотность
материала конструкции r0 = 900 кг/м3 . Конструкция плот-
но прилегает к полу. К её середине прикреплена пружина
жёсткости k = 1000 Н/м, вмонтированная в пол другим сво-
им концом. Длина недеформированной пружины l = 0,5a,
где a = 1 м. В бассейн медленно наливают воду. Построй-
104 Условия задач

те график зависимости силы давления конструкции на пол


бассейна от уровня воды h. Плотность воды и ускорение
свободного падения известны.
255. В U-образную трубку налили ртуть. Затем в ле-
вое колено долили воду, а в правое колено –– масло (см.
рис. 129). Высота столбика воды составляет h1 = 0,9 м, вы-
сота столбика масла составляет h2 = 1 м. Найдите разность
уровней жидкости в коленах трубки. Плотности воды, ртути
и масла, а также ускорение свободного падения известны.
256. Рабочий поднимает груз массой m = 5 кг с помо-
щью ворота. Верёвка, которой привязан груз, имеет толщи-
ну d = 0,5 см. При наматывании верёвки на ворот витки ло-
жатся один на другой (см. рис. 130). Рабочий крутит ворот
за ручку длиной L = 50 cм. График зависимости периода T
вращения ручки от силы F, которую приходится приклады-
вать рабочему для её вращения, изображён на рис. 131. За
какое время рабочий поднимет груз на высоту H = 19 м, ес-
ли изначально радиус ворота с намотанной верёвкой равен
R0 = 10 см?
П р и м е ч а н и е. Площадь круга радиуса R равна
S = pR2 .
Задачи городских туров 105
106 Условия задач

Теплота
257. Медный кубик со стороной a, нагретый до темпера-
туры t > 0 ◦ C, был помещён в лёгкий, тонкий, плотно при-
легающий к стенкам теплоизолирующий колпачок так, что
открытой осталась только одна грань (см. рис. 132). Кубик
положили открытой гранью вниз на ледяной куб массы M,
плавающий в воде при температуре 0 ◦ C. Кубик начал про-
плавлять во льду углубление квадратного сечения, всё глуб-
же погружаясь в него. После установления в системе тепло-
вого равновесия измерили глубину, на которую погрузил-
ся в воду ледяной куб. Построить график зависимости этой
глубины от первоначальной температуры меди. Известны
плотности воды, льда и меди, а также удельная теплоём-
кость меди и удельная теплота плавления льда. Считать,
что вода, образующаяся при плавлении льда, имеет темпе-
ратуру 0 ◦ C и та её часть, которая вытекает из углубления,
сразу удаляется –– стекает с поверхности ледяного куба.
258. Два литра воды нагревают на плитке мощностью
500 Вт. Часть тепла теряется в окружающую среду. Зависи-
мость мощности тепловых потерь от времени приведена на
рис. 133. Начальная температура воды равна 20 ◦ C. За ка-
Задачи городских туров 107

кое время вода нагреется до 30 ◦ C? Удельная теплоёмкость


воды с известна.
259. Имеются три цилиндрических сосуда, отличающие-
ся только по высоте. Ёмкости сосудов равны 1 л, 2 л и 4 л.
Все сосуды заполнены водой до краёв. Воду в сосудах греют
с помощью кипятильника. Из-за потерь тепла в атмосферу
мощности кипятильника не хватает для того, чтобы вскипя-
тить воду. В первом сосуде воду можно нагреть до t1 = 80 ◦ C,
во втором –– до t2 = 60 ◦ C. До какой температуры можно на-
греть воду в третьем сосуде, если комнатная температура
t = 20 ◦ C? Считайте, что теплоотдача в атмосферу с единицы
площади поверхности пропорциональна разности темпера-
тур воды и окружающей среды. Вода в сосуде прогревается
равномерно.
260. В лаборатории провели исследование 1 г новой
жидкости X. На рис. 134 дан график зависимости объё-
ма синтезированной жидкости от температуры. Оказалось,
что теплоёмкость полученного количества жидкости равна
c = 3 Дж/◦ C и не зависит от температуры, что температура
кипения –– TК = 80 ◦ C, а теплота парообразования –– QП = 240
Дж. Также выяснилось, что жидкость не смешивается с во-
дой. В воду объёмом VВ = 5 мл при температуре TВ = 90 ◦ C
налили VX = 1 мл жидкости X при некоторой температуре
TX . Найти минимальную TX , при которой вся добавленная
108 Условия задач

в воду жидкость выкипит. Удельная теплоёмкость воды из-


вестна.
261. На сильном морозе лыжники дышат через специ-
альную «грелку», внутри пластмассового корпуса которой
находится система проволочных решёток. Решётки нагре-
ваются воздухом, который лыжник выдыхает, и нагревают
вдыхаемый воздух. При температуре на улице T0 = −20 ◦ C
температура грелки, которую использовал лыжник, была
равна T1 = −6 ◦ C. Во время разминки лыжник стал дышать
вдвое чаще. До какой температуры T2 нагрелась грелка?
П р и м е ч а н и е. Температура воздуха, выдыхаемая
лыжником, равна TЛ = 36 ◦ C. Считать, что температура воз-
духа, проходящего через грелку, успевает сравняться с её
температурой. Мощность теплоотдачи от грелки в окружа-
ющую среду (через боковую поверхность) пропорциональна
разности температур грелки и окружающей среды. Тепло-
ёмкость грелки достаточно большая, так что за время вдо-
ха/выдоха её температура практически не меняется.
262. Система, изображённая на рисунке, состоит из
неподвижного блока A, через который перекинута верёвка,
соединяющая кусок льда B при температуре 0 ◦ C и невесо-
мый блок C. Через блок C также перекинута верёвка, на
одном конце которой висит груз массой m = 10 г, а другой
конец которой соединён с полом через пружину жёсткостью
Задачи городских туров 109

k = 100 Н/м. Вначале кусок льда погружен наполовину в во-


ду при температуре tк = 20 ◦ C, находящуюся в стакане. Объ-
ём воды в стакане V = 200 мл. В процессе таяния льда си-
стема приходит в движение, и лёд поднимается из воды. Ка-
кая температура будет у воды, когда лёд полностью выйдет
из неё? Считать, что теплообмен происходит только между
льдом и водой в стакане. Плотности воды и льда, удельные
теплоёмкость воды и теплота плавления льда, а также уско-
рение свободного падения известны.

9 класс
Механика
263. В результате взрыва космический объект превра-
тился в облако мелкой пыли. Сразу после взрыва облако од-
нородно и шарообразно, имеет радиус R и плотность r0 . На-
чальная скорость v каждой пылинки направлена от центра
облака и пропорциональна расстоянию r до центра: v = Hr
(H –– известный коэффициент). Определите плотность пы-
ли на расстоянии x от центра облака через время t после
взрыва. Гравитационным взаимодействием пылинок прене-
бречь.
4
П р и м е ч а н и е. Объём шара радиуса R равен pR3 .
3
264. Кубик массы M стоит на горизонтальной поверх-
ности. Его касается кубик массы m, висящий на невесо-
110 Условия задач

мой нерастяжимой нити. Нить составляет угол a с верти-


калью. В начальный момент кубики неподвижны. Опреде-
лите ускорения кубиков в начальный момент. Трением пре-
небречь. Считайте, что кубики не поворачиваются вокруг
своей оси. Ускорение свободного падения равно g.
265. Старый пятиэтажный дом обветшал, и его балконы
готовы оторваться от фасада при малейшем толчке. Случи-
лось так, что первым рухнул балкон верхнего этажа. Рас-
стояние между балконами на всех этажах одинаково и рав-
но h, так же как и расстояние между балконом 2-го этажа и
землёй. Ускорение свободного падения равно g. Массы бал-
конов одинаковы, удары их друг о друга неупруги. Опреде-
лите скорость балконов в момент удара о землю. Толщиной
балконов и сопротивлением воздуха пренебречь.
266. Робот Вася спроектирован так, что может взбирать-
ся по лестницам. После некоторого времени работы t у Васи
садятся батарейки. Это время зависит от скорости v, с кото-
рой Вася движется по лестнице. На рис. 137 приведён гра-
фик зависимости 1/t(v). Какова максимальная длина лест-
ницы, на которую может взобраться Вася? Пусть теперь Ва-
ся пытается взобраться вверх по движущемуся вниз эскала-
тору. Постройте график зависимости максимальной длины
эскалатора, на который может взобраться Вася, от скорости
этого эскалатора.
Задачи городских туров 111

267. На наклонных рельсах, расположенных под углом


a к горизонту, находится клин массы M. Одна грань клина
вертикальна, другая –– горизонтальна (см. рис. 138). Коэф-
фициент трения между клином и рельсами равен m. Парал-
лельно рельсам, посередине между ними, расположен глад-
кий стержень, на который надето невесомое кольцо. Трения
между кольцом и стержнем нет. К кольцу привязана нить,
проходящая сквозь отверстие в клине (не касаясь его), пе-
рекинутая через блок, закреплённый на углу клина, и при-
вязанная к бруску массы m. Трение между бруском и кли-
ном отсутствует. Найдите ускорение бруска и клина. Блок
и нить невесомы, а нить нерастяжима.
268. Длинный отрезок трубы согнут посередине под пря-
мым углом так, что получился уголок (см. рис. 139). Уголок
закреплён за один из концов на горизонтальной оси. Уго-
лок находится в плоскости, перпендикулярной оси, и мо-
жет вращаться без трения. По гибкому шлангу в уголок по-
даётся вода, при этом в состоянии равновесия струя бьет
из уголка под углом a = 30◦ к горизонту. Под каким углом
112 Условия задач

к горизонту будет направлена струя, если вдвое увеличить


скорость воды, подаваемой по трубе?
269. Если на проволоку, изогнутую в форме спирали (см.
рис. 140, а), насадить бусинку и отпустить её без начальной
скорости, то некоторое вертикальное расстояние она прой-
дёт за время T1 . Если бусинку насадить на прямую прово-
локу и отпустить, то такое же расстояние она пройдёт за
время T2 (см. рис. 140, б). Прямую проволоку располагают
на оси спирали. На неё и на спираль надевают две одина-
ковые бусинки. Бусинки соединены лёгким твёрдым длин-
ным стержнем, составляющим малый угол с вертикалью
(см. рис. 140, в). За какое время скреплённые друг с другом
бусинки преодолеют это же вертикальное расстояние, если
их отпустить без начальной скорости? Трением пренебречь.
270. На две спицы, жёстко соединённые под углом
a = 60◦ , надели резиновое кольцо, длина которого в неде-
формированном состоянии равна 2L (см. рис. 141). К коль-
цу подвесили груз массой m, в результате чего, оно де-
формировалось и приняло в состоянии равновесия форму
треугольника ACD (BD –– ось симметрии рисунка). Трения
между кольцом и спицами нет. На каком расстоянии от точ-
ки находится груз в равновесии? Известно, что если кольцо
разрезать на две одинаковые полоски длины L, то коэффи-
циент упругости каждой такой полоски будет равен k. Мас-
сой резинового кольца пренебречь.
271. На открытой горизонтальной площадке вкопан
столб высотой H = 7,5 м и радиусом r = 7 см. Начертите гра-
Задачи городских туров 113
114 Условия задач

фик зависимости длины тени столба от угла подъёма Солн-


ца над горизонтом. Радиус Солнца R = 700 тыс. км, и сред-
нее расстояние от Земли до Солнца L = 150 млн. км. Макси-
мальная высота Солнца над горизонтом на широте Петер-
бурга a = 60◦ .
272. Два груза с неизвестными массами соединены неве-
сомой нерастяжимой нитью (см. рис. 142). В первом опыте
их размещают, как показано на левом рисунке, а во вто-
ром –– как на правом. Обе системы в начальный момент вре-
мени отпускают, и они начинают скользить без трения. Из-
мерив через одинаковое время пути, пройденные грузами,
выяснили, что в первом случае он в два раза больше, чем
во втором. Найдите отношение масс грузов. Угол наклона
плоскости равен a.
273. Два шарика, имеющих одинаковые массы (см.
рис. 143), лежат на гладком горизонтальном столе, на рас-
стоянии L друг от друга. √ Они соединены невесомой нерас-
тяжимой нитью длиной L 2. Одному из шариков придают
скорость V перпендикулярно линии, проходящей через их
центры. Опишите дальнейшую траекторию движения ша-
риков. В момент натяжения нить обеспечивает абсолютно
упругое взаимодействие шариков.
Задачи городских туров 115
116 Условия задач

274. По горизонтальному полу может ездить без трения


тяжёлый куб. Справа, к грани куба, прислонили массивную
однородную доску (см. рис. 144). Доска находится в равно-
весии; куб при этом необходимо удерживать. Известно, что
коэффициенты трения доски о пол –– m1 , о куб –– m2 . Доска
образует с полом угол a. При каком соотношении коэффи-
циентов трения m1 и m2 возможно такое равновесие? Пусть
теперь m2 = 1/m1 = tg a. Возможно ли при таких условиях
сдвинуть куб вправо, прикладывая к нему горизонтальную
силу?
275. Над обрывом установлено орудие (см. рис. 145),
позволяющее вести огонь в любом направлении. Снаряды
имеют начальную скорость v0 . На расстоянии l от орудия,
под углом f к горизонту, завис воздушный шар. Известно,
что шар находится достаточно далеко от орудия, так что
снаряды в него не попадают. Обстрел стали производить
снарядами, которые взрываются в воздухе через время T
после выстрела. Под каким углом к горизонту следует стре-
лять, чтобы снаряды взрывались как можно ближе к шару?
Ускорение свободного падения g.
276. На клин с углом при вершине 45◦ , стоящий на сто-
ле, кладут брусок такой же массы. Коэффициент трения
между бруском и клином, а также между клином и столом
равен m. При каких значениях m брусок будет скользить по
клину? При каких значениях m клин станет скользить по
столу?
277. Тяжёлый однородный канат (см. рис. 146) свобод-
но подвешен за концы. Силы натяжения каната в точках
Задачи городских туров 117

подвеса равны T1 и T2 , а в самой нижней точке каната T3 .


Найти массу каната. Ускорение свободного падения g.
278. Со стартовой площадки у гладкого озера взлетает
ракета. Ускорение ракеты постоянно, равно a и направле-
но вертикально вверх (см. рис. 147). На противоположном
берегу озера на расстоянии L от точки старта стоит чело-
век и смотрит на отражение ракеты в озере. В некоторый
момент ракета оказалась на высоте H над озером. С какой
скоростью должен двигаться в этот момент по воде катер,
чтобы всё время закрывать человеку изображение ракеты в
озере? Считать, что глаза человека находятся на высоте h
над поверхностью воды.
118 Условия задач

279. Маленькая шайба находится на дне вертикаль-


но расположенного цилиндрического сосуда. Рёбра сосуда
скруглены, так что стенки плавно переходят в дно, образуя
закругления очень малого радиуса. Сосуд имеет высоту h
и радиус основания R. Шайба в начальный момент време-
ни находится на расстоянии L = R/2 от центра дна сосуда,
и её скорость перпендикулярна диаметру, проходящему че-
рез точку, в которой она находится (см. рис. 148, вид сосуда
сверху). С какой скоростью должна двигаться шайба, чтобы
вернуться в ту же точку, совершив M оборотов вокруг цен-
тра и заехав N раз на стенку? Ускорение свободного паде-
ния g. Дно сосуда расположено горизонтально. Размерами
шайбы и её трением о дно и стенки сосуда пренебречь.
280. Две одинаковые очень массивные шайбы радиуса R
каждая двигаются по скользкой горизонтальной плоскости
навстречу друг другу со скоростями V по одной прямой. По-
середине между ними лежит шайба очень маленькой массы,
радиуса r. Её центр находится на расстоянии d от прямой
(см. рис. 149), соединяющей центры тяжёлых шайб. Какую
скорость приобретёт лёгкая шайба после того, как шайбы
разлетятся? Все шайбы жёсткие (недеформируемые), соуда-
рения абсолютно неупругие.
281. Дачнику надо сдвинуть бетонную плиту, имеющую
коэффициент трения о грунт m. Для этого он использует ве-
рёвку длиной L с коэффициентом жёсткости k –– привязы-
вает её (не натягивая) к дереву на расстоянии L от плиты,
Задачи городских туров 119
120 Условия задач

и прикладывает затем к середине верёвки силу, перпенди-


кулярную самой верёвке. Какова максимальная масса пли-
ты, которую он может сдвинуть таким способом, если его
собственная максимальная сила равна F? Размером узлов и
провисанием верёвки в ненатянутом состоянии можно пре-
небречь, считать, что верёвка растягивается слабо. Есть ли
дополнительный выигрыш, если верёвку вначале натянуть
с силой?
282. Массивная бусинка нанизана на невесомую нерас-
тяжимую нить длиной L, по которой может скользить без
трения. Концы нити прикреплены к невесомым кольцам,
которые могут свободно скользить по горизонтальному и
вертикальному стержням (см. рис. 150). В начальный мо-
мент бусинку удерживают в таком положении, чтобы нить и
стержни составляли квадрат. Бусинку отпускают. Найдите
её ускорение вначале движения и время, за которое она до-
стигнет вертикального стержня. Ускорение свободного па-
дения g.
283. Наклонная плоскость имеет угол a с горизонталью
(см. рис. 151). По ней запускают вверх под углом b к го-
ризонтали две цилиндрические шайбы, массой m каждая,
лежащие точно одна на другой (по центру). Коэффициент
трения между шайбами m, а между нижней шайбой и плос-
костью m0 . Какова сила, с которой действует верхняя шайба
на нижнюю шайбу в верхней точке их траектории, если m
достаточно, чтобы шайбы не проскальзывали друг по дру-
Задачи городских туров 121

гу? Может ли начаться такое проскальзывание, если его не


было сначала?

Теплота
284. В чайнике нагревают воду кипятильником, подклю-
чённым к источнику постоянного напряжения U. Масса во-
ды равна m, а её удельная теплоёмкость c. Начальная тем-
пература воды T0 . Через какое время вода закипит? Всеми
потерями тепла и неоднородностью нагревания воды прене-
бречь. Электрическое сопротивление кипятильника зависит
от температуры линейно: R = R0 + aT, где a и R0 –– постоян-
ные величины.
285. Для заполнения проточного бассейна можно ис-
пользовать два крана, дающих одинаковый поток воды:
с горячей и тёплой водой. Температура горячей воды
T1 = 70 ◦ C, температура тёплой воды T2 = 40 ◦ C. При испы-
таниях бассейна заметили, что если открыть только кран
с горячей водой, установившаяся температура воды в бас-
сейне будет равняться T1 = 50 ◦ C. Если же открыть только
кран с тёплой водой, установившаяся температура воды в
бассейне будет равняться T2 = 30 ◦ C. Определите, какая тем-
пература установится в бассейне, если открыть оба крана.
Считайте, что поток тепла от воды прямо пропорционален
разности температур воды и окружающей среды, а устано-
вившийся уровень воды в бассейне одинаков во всех трёх
случаях.
122 Условия задач

286. Утюг устроен следующим образом: его нагреватель


выключается, если температура утюга становится больше
некоторой температуры t2 , и включается, как только его
температура падает ниже t1 (эти температуры неизвестны).
Если включённый утюг стоит с открытой металлической по-
верхностью, его нагреватель работает в среднем k = 1/4 все-
го времени. При этом мощность теплоотдачи можно считать
постоянной. Если утюгом начинают гладить, то промежу-
ток времени между последовательными моментами включе-
ния нагревателя становится в n = 4/3 раза меньше. В этом
случае мощность теплоотдачи также остаётся постоянной.
Какую часть времени он работает в среднем во втором слу-
чае?
287. Экспериментатор взял 4 одинаковых металлических
стержня и собрал из них Y-образную фигуру. К концам
фигуры экспериментатор присоединил 3 одинаковых боль-
ших металлических шара, имеющих температуру t1 = 0 ◦ C,
t2 = 50 ◦ C и t3 = 100 ◦ C (см. рис. 152). Экспериментатор обес-
печил хороший тепловой контакт стержней с шарами и дру-
гими стержнями. Через некоторое время он обнаружил, что
первый шар нагрелся на 0,4 ◦ C. Какую температуру имели
в этот момент два других шара? Считайте, что теплоёмкость
стержней пренебрежимо мала, а теплообмен с окружающей
средой отсутствует. Мощность теплопередачи по стержню
пропорциональна разности температур на его концах.
288. Тело массой m = 70 кг сброшено с большой высо-
ты. По графику зависимости скорости тела от времени (см.
рис. 153, ось скорости на графике не проградуирована)
определите тепловую мощность, выделявшуюся из-за тре-
ния о воздух в моменты времени t1 = 5 сек и t2 = 18 сек.
289. К тряпичному мешку с солью массой M = 490 кг
привязан пробковый поплавок объёмом V = 0,1 м3 . Мешок
поместили в резервуар и залили V1 = 1 м3 воды. Известно,
что при фиксированной температуре в одном литре воды
может раствориться не более k грамм соли. График зависи-
мости k от температуры приведён на рис. 154. Какое коли-
чество теплоты необходимо сообщить системе, чтобы мешок
всплыл, если её начальная температура t0 = 15 ◦ C? Считать,
что в процессе растворения соли в воде уровень жидкости в
резервуаре не меняется. Тепловыделением при растворении
Задачи городских туров 123
124 Условия задач

соли пренебречь. Плотности и теплоёмкости воды, пробки и


соли считать известными.

Электричество
290. В вашем распоряжении имеется неограниченное ко-
личество резисторов произвольного сопротивления и ди-
одов. Диоды пропускают ток только в одном направле-
нии, при этом падение напряжения на них равно 1 В (см.
рис. 155, а). Какую схему нужно собрать, чтобы она име-
ла такую зависимость тока от напряжения, как показано на
рис. 155, б? Постарайтесь использовать как можно меньше
элементов.
291. Знаток физики собрал схему из трёх одинаковых ре-
зисторов, подключил её к источнику постоянного напряже-
ния (который можно считать идеальным) и измерил вольт-
метром напряжение сначала между точками A и D, а потом
между точками A и B –– получилось U1 = 3 В и U2 = 0,9 В
соответственно (см. рис. 156). Тогда знаток физики соеди-
нил точки A и C проводом (сопротивлением которого мож-
но пренебречь) и измерил напряжение между точками B и
D. Что он получил?
292. Два одинаковых цилиндрических проводника, из-
готовлены так, что удельное сопротивление материала ли-
Задачи городских туров 125
126 Условия задач

нейно изменяется вдоль цилиндров (r = kL, где r –– удельное


сопротивление проводника в данной точке, k –– известный
постоянный коэффициент, L –– расстояние от начала про-
водника до данной точки). Проводники соединены парал-
лельно так, что у одного удельное сопротивление возрастает
справа налево, а у другого –– наоборот –– слева направо (см.
рис. 157). Эта схема подключена к источнику постоянного
напряжения U0 . Что показывает идеальный вольтметр, со-
единяющий середины этих проводников?
293. Схема собрана из некоторого количества различаю-
щихся между собой резисторов, реостата, идеальных бата-
рейки, вольтметра и амперметра (см. рис. 158). Ползунок
реостата сдвигают, немного увеличивая его сопротивление.
Как изменятся показания вольтметра и амперметра –– уве-
личатся или уменьшаться?
294. Сопротивление схемы, изображённой на рис. 159,
равно 10 Ом. Если поменять местами резисторы 1 и 3, со-
противление схемы возрастёт в 100 раз. Если же в исходной
схеме поменять резисторы 2 и 3, её сопротивление возрас-
тёт на 0,2%. Найдите сопротивления всех резисторов.
295. Сопротивление Очень Термостойкого Прибора ли-
нейно растёт с температурой, а мощность теплоотдачи с его
поверхности прямо пропорциональна разности температур
прибора и окружающей среды. Если пропускать через при-
бор очень малый ток, его сопротивление равно R0 . Когда
величина тока, протекающего через прибор, приближается
Задачи городских туров 127

к I0 , прибор быстро разогревается и всё же плавится. Какое


напряжение будет на приборе, если через него пропустить
ток I0 /2? Все эксперименты проводятся при одной и той же
комнатной температуре.

10 к л а с с
Механика
296. Две звезды одинаковой массы вращаются вокруг
общего центра масс по круговой орбите. В некоторый мо-
мент времени одна из звёзд вспыхивает как сверхновая и
сбрасывает оболочку массы DM. При каком DM звёздная
система распадётся (звёзды смогут удалиться друг от друга
на бесконечное расстояние)? Считайте, что оболочка сбра-
сывается сферически симметрично, и сброшенная масса мо-
ментально покидает систему.
297. Внутри куба вырезана сферическая полость таким
образом, что центр сферы находится над центром ниж-
128 Условия задач

ней грани куба. Полость наполовину заполнена жидкостью


плотности r2 . Куб очень медленно наклоняют через ребро
AA (см. рис. 160). При каком угле наклона a куб опроки-
нется? Длина ребра куба в n раз больше радиуса полости r,
а центр полости расположен на высоте kr над основанием
куба, причём k > n/2. Плотность вещества куба r1 . Объём
4
полости равен pr3 .
3
298. Внутри широкой трубки сечением S1 и высотой L1
может без трения двигаться плотно прилегающая к стенкам
пробка высотой h, сделанная из материала плотности r2 . К
широкой трубке с двух концов присоединена узкая трубка с
сечением S2 и длиной L2 . Вся система (см. рис. 161) полно-
стью заполнена жидкостью плотности r1 > r2 . В начальный
момент пробка и вода неподвижны. Найти начальное уско-
рение пробки. Вязкостью и сжимаемостью жидкости прене-
бречь. Все изгибы и переходы трубок считать плавными.
299. Гружёный вагон массой M, имеющий скорость V ,
сталкивается с двумя пустыми неподвижно стоящими оди-
наковыми вагонами, соединёнными пружиной жёсткости k.
Длина недеформированной пружины равна l. Чему будет
равно расстояние между груженым и ближайшим к нему
пустым вагоном через время t после столкновения? Масса
пустого вагона в два раза меньше массы груженого, удар
считать кратковременным и абсолютно упругим, трением в
системе и массой пружины пренебречь.
Задачи городских туров 129

300. Из цилиндрической бочки с водой медленно вытя-


гивают цилиндрическое тонкостенное ведро с водой. Нари-
совать график зависимости показаний динамометра от рас-
стояния между дном ведра и дном бочки. Масса ведра с во-
дой m, высота ведра h, высота воды в бочке H, площадь дна
ведра s в 2 раза меньше площади дна бочки. В начальный
момент дно ведро находится на расстоянии L от дна бочки;
плотность воды r0 . Поверхностным натяжением воды пре-
небречь.
301. С гладкого горизонтально расположенного цилин-
дра радиуса R начинает соскальзывать тонкая однородная
верёвка длины pR/2 (см. рис. 162). Найдите в начальный
момент времени силу натяжения верёвки в точке A, распо-
ложенной под углом a к горизонтали (угол a лежит в интер-
вале от 0 до p/2). Масса верёвки m, ускорение свободного
падения g.
302. У идеальной пружины нулевой начальной длины
один конец закреплён, а к другому концу подвешен точеч-
ный груз массы M. Пружину растягивают до длины L, от-
водя груз в сторону, так, что угол пружины с горизонталью
составляет a. Затем груз отпускают. Определить форму и
130 Условия задач

длину траектории груза. Жёсткость пружины k, ускорение


свободного падения g.
303. В вертикальную бесконечно длинную трубу радиуса
R поместили цилиндрическую капсулу (см. рис. 163). Мас-
са капсулы распределена по её боковой поверхности. Сила
трения между капсулой и стенками трубы прямо пропорци-
ональна относительной скорости соприкасающихся поверх-
ностей. Капсуле придали поступательную скорость, направ-
ленную вверх, и, одновременно, закрутили с некоторой уг-
ловой скоростью. Когда через некоторое время капсула вер-
нулась в исходное положение, модуль линейной скорости
изменился на V по сравнению с первоначальным, а угловая
скорость стала равна W . Время подъёма с исходной высоты
до наивысшей точки отличалось от времени спуска до на-
чального положения на T. За всё время дальнейшего спус-
ка по бесконечной трубе капсула повернулась на конечный
угол a. Какова была максимальная высота подъёма капсу-
лы?
304. На столе закреплён вертикально длинный стер-
жень, по которому могут двигаться вертикально 9 одина-
ковых маленьких колец. Кольца не могут проходить друг
сквозь друга. Первоначально кольца покоятся у стола друг
на друге. Начиная с момента времени t = 0 всем кольцам по
очереди, начиная с верхнего, с промежутком в 1 cек сооб-
щают скорость 50 м/с, направленную вверх. Столкновения
колец друг с другом и нижнего кольца со столом абсолютно
Задачи городских туров 131

упругие. Постройте график зависимости координаты от вре-


мени для верхнего и нижнего колец. Трением пренебречь.
305. Система, состоящая из двух невесомых жёстких
стержней, соединённых шарниром A, подвешена к потол-
ку на шарнире O. Массы шарнира A и груза B равны m.
Первоначально угол между стержнями –– прямой. Угол, ко-
торый верхний стержень образует с вертикалью a = 45◦ (см.
рис. 164). Первоначально система покоится, затем её отпус-
кают и она начинает свободно двигаться. Найдите ускоре-
ние точки B в начальный момент движения и силу, дей-
ствующую в этот момент на верхний стержень. Трения в
шарнирах нет.
306. На горизонтально расположенном шероховатом сто-
ле стоит клин массы M с углом a. Проскальзывание меж-
ду клином и столом невозможно. На клин рядом с верши-
ной кладут маленькую шайбу. Коэффициент трения между
шайбой и клином равен m. Какой должна быть масса шай-
бы, чтобы клин оторвался от стола?
П р и м е ч а н и е. Центр масс треугольника находится в
точке пересечения его медиан.
307. Кольца могут скользить без трения вдоль закреп-
лённого длинного вертикального стержня. В начальный мо-
мент времени все кольца лежат на земле. Затем кольцам по
132 Условия задач

очереди, начиная с верхнего, с интервалом времени t со-


общают скорость v  gt вверх. Встречаясь в полёте, кольца
слипаются. Кольца бросают верх до тех пор, пока вся стопка
колец не упадёт на Землю. Когда это произойдёт? Толщиной
колец можно пренебречь.
308. По массивной подвешенной однородной леске сколь-
зит без трения лёгкая бусинка (рис. 165). В начальный мо-
мент бусинка покоится вблизи точки подвеса. Прогиб лески
равен h, длина L, угол лески к горизонту в точке крепле-
ния a. Найдите ускорение бусинки в нижней точке. Ускоре-
ние свободного падения g, деформацией лески под действи-
ем бусинки можно пренебречь.
Задачи городских туров 133

309. Мотоциклист едет по внутренней поверхности вер-


тикального полого цилиндра радиуса R со скоростью v. Тра-
ектория мотоциклиста представляет собой винтовую линию
с шагом h (см. рис. 166). Каким может быть коэффициент
трения покоя между покрышками мотоцикла и поверхно-
стью цилиндра? Размеры мотоцикла много меньше R.

Теплота и молекулярная физика


310. Оцените с возможно большей точностью массу воды
в чайнике. Опишите опыт, который вы будете проводить.
Имеющееся оборудование: термометр, секундомер, нихро-
мовая проволока, источник напряжения (220 В), изолиро-
ванные провода, амперметр. Все необходимые константы
считать известными, чайник считать лёгким.
311. В сосуде под невесомым подвижным поршнем нахо-
дится n молей жидкости и n молей насыщенного пара этой
жидкости при температуре 0 . Смесь медленно нагрели, за-
тратив на это теплоту Q. Температура в сосуде увеличилась
при этом на D. Найти изменение внутренней энергии содер-
жимого сосуда. Объёмом жидкости по сравнению с объёмом
сосуда пренебречь.
312. Газовая смесь состоит из двух изотопов некоторо-
го вещества. Массы молекул лёгкого и тяжёлого изотопа
равны mЛ и mТ . Через патрубок A смесь попадает в обо-
гатитель, состоящий из двух отсеков, разделённых перего-
родкой П с небольшими отверстиями (см. рис. 167). Малая
часть смеси просачивается через отверстия в перегородке,
134 Условия задач

при этом оказывается, что относительное содержание лёг-


кого изотопа увеличивается. Обогащённую таким образом
лёгким изотопом смесь откачивают насосом через патрубок
B. Остальная часть отводится через патрубок D. Какова бы-
ла доля концентрации лёгкого изотопа в исходной смеси,
если его концентрация после обогащения стала составлять
долю C1 от общей концентрации в правом отсеке? Темпе-
ратура газа в отсеках одинакова и постоянна, давления в
отсеках P1 и P2 считать известными и установившимися,
причём P2 < P1 . Газ считайте идеальным.
313. КПД цикла ABCGA, изображённого на рис. 168,
равен h. Найдите КПД цикла AGCDEFCBA. В качестве
рабочего тела в цикле используется идеальный газ. На
рис. 168 все точки соединены прямыми отрезками. Давле-
ния в точках B и D, в точках G и F и в точках A, C и E ––
одинаковы, но неизвестны. Все объёмы показаны на диа-
грамме в единицах V0 .
314. Имеются два одинаковых сверхпрочных воздушных
шарика. Давление p, создаваемое тонкой оболочкой шари-
ка, монотонно растёт при увеличении его объёма V . Перво-
начально в одном из шариков находился гелий при давле-
нии p0 = 106 Па, а другой был пустой. Шарики соединяют
так, что газ может свободно перетекать из одного в другой.
Теплообмена между шариками и окружающей средой нет,
внутренняя энергия оболочек не зависит от температуры.
Как изменятся давление и объём всего газа (возрастут или
уменьшатся) по сравнению с первоначальными значениями
Задачи городских туров 135

после установления равновесия? Найдите конечные давле-


ние и объём газа в случае зависимости p(V ), приведённой
на графике (см. рис. 169). Атмосферным давлением прене-
бречь.
315. Воздушный шар имеет тонкую мягкую нерастяжи-
мую оболочку объёма V0 . Масса груза, прикреплённого к
шару вместе с оболочкой равна m0 . На земле оболочку ша-
ра наполняют до некоторого объёма V1 газом плотности r1
при атмосферном давлении. Найдите, до какой высоты под-
нимется шар. С высотой плотность воздуха меняется по за-
кону r(h) = r0 exp(−ah), где r0 и a –– известные постоянные.
До какого объёма нужно заполнить оболочку шара, чтобы
высота его подъёма была максимальной? Какова эта высо-
та?
316. В сосуде под поршнем находится n = 0,1 молей пара.
При данной температуре T пар начинает конденсироваться
в жидкость, когда объём под поршнем становится меньше
некоторого критического объёма VК (T). Над газом соверша-
ют циклический процесс ABCD (см. рис. 170). Постройте
график этого процесса в координатах p––V , найдите давле-
136 Условия задач
Задачи городских туров 137

ние в угловых точках. Внутренняя энергия одного моля па-


5
ра при температуре T равна RT, плотность жидкости мно-
2
го больше плотности пара. Насыщенный пар считайте иде-
альным газом.
Электричество и магнетизм
317. Если по квадратной диэлектрической пластине рав-
номерно распределить заряд q, то потенциал в её центре бу-
дет равен f1 . Если из шести таких пластин составить полый
куб, то потенциал в его центре будет равен f2 . Определите
потенциал в вершине такого куба. Потенциал на бесконеч-
ности примите равным нулю.
318. Потенциал в центре квадратной диэлектрической
пластины равен f. Вся пластина равномерно заряжена с
одинаковой плотностью. Найти потенциал в углу пластины.
319. Имеется положительно заряженная полубесконеч-
ная нить, начинающаяся из точки O. Плотность заряда
вдоль нити постоянна. Найти направление напряжённости
электрического поля в точке B на прямой линии AOA (см.
рис. 171), перпендикулярной к нити.
320. Два шарика, имеющие заряд +q и массу m каждый
нанизаны на жёстко закреплённые спицы длины L, угол
138 Условия задач

между которыми равен 2a (см. рис. 172, спицы расположе-


ны симметрично относительно вертикальной линии, прохо-
дящей через вершину угла). Первоначально шарики покоят-
ся, соединённые невесомой туго натянутой нитью длины S.
Нить пережигают. Найти скорость шариков в момент, когда
они слетают со спиц. Радиус шариков много меньше длины
спиц. Ускорение свободного падения и электрическую по-
стоянную считать известными.
321. В сосуде из непроводящего материала находятся
равные объёмы двух несмешивающихся жидкостей, удель-
ные электрические сопротивления которых r1 и r2 , соответ-
ственно (см. рис. 173). В сосуд вмонтировано два электрода
Э1 и Э2 , к ним подсоединён источник тока, который, незави-
симо от сопротивления цепи, даёт ток I. Верхний электрод
представляет из себя подвижный поршень. В нижней части
сосуда имеется клапан, через который ежесекундно проте-
кает k капель жидкости объёмом v. После того, как первая
жидкость полностью вытекла из сосуда, клапан закрывают.
Какое количество тепла выделится за время, равное удвоен-
ному времени вытекания первой жидкости? Площадь дна
сосуда и электродов равна S, первоначальный объём каж-
дой жидкости V0 .
Задачи городских туров 139
140 Условия задач

322. Полный заряд параллелепипеда, равномерно заря-


женного по всему объёму, равен Q1 . После того, как по всем
граням параллелепипеда, кроме грани ABCD, равномерно
распределили ещё и поверхностный заряд Q2 , поле в точ-
ке F оказывается равным нулю. Определите величину от-
ношения Q2 /Q1 . Длины рёбер параллелепипеда указаны на
рис. 174.
323. Сопротивление проволоки изменяется с температу-
рой по линейному закону R = R0 (1 + at), где R0 –– сопротив-
ление при температуре 0 ◦ C. Как должно изменяться со вре-
менем подводимое к проволоке напряжение для того, чтобы
температура проволоки линейно возрастала со временем?
Теплоёмкость проволоки равна C, отвода тепла нет.
324. Однородная резинка пренебрежимо малой началь-
ной длины при растяжении подчиняется закону Гука и име-
ет жёсткость K. Материал резинки равномерно заряжен, её
полный заряд Q > 0. Резинку поместили в поле двух заря-
дов +q и −q, расположенных на расстоянии 2L друг от дру-
га (см. рис. 175). Один из концов резинки удерживают по-
середине между зарядами, прикладывая при этом силу F1 .
Какая сила F2 должна быть приложена для удержания вто-
рого конца, если он находится на расстоянии L справа от
заряда q? Взаимодействием зарядов на резинке между со-
бой пренебречь.
325. Плоский диск радиуса R из диэлектрического ма-
териала равномерно заряжен зарядом Q. К середине дис-
Задачи городских туров 141

ка прикреплена заземлённая металлическая сфера радиу-


сом r  R, так что она касается диска. Найдите электроста-
тическую силу, с которой диск действует на сферу.
326. По поверхности диэлектрической фигуры в фор-
ме телефонной трубки равномерно распределён заряд Q > 0.
Фигура помещена в электрическое поле напряжённостью E
так, что она может свободно вращаться вокруг точки A (см.
рис. 176). В положении равновесия угол между отрезком
AB и направлением электрического поля равен a. Какую
работу надо совершить, чтобы медленно повернуть фигуру
в положение, когда отрезок AB направлен вдоль поля? Дли-
на отрезка AB = L. Силой тяжести пренебречь.
327. В далёком космосе находятся два массивных метал-
лических шара радиусами R1 = 10 см и R2 = 20 см, соеди-
нённых тонкой проводящей проволокой. В некоторый мо-
142 Условия задач

мент времени их скорости были равны нулю, расстояние


между их центрами было L = 1 м, а суммарный заряд равен
Q = 10−4 Кл. Найдите кинетическую энергию шаров, когда
расстояние между ними станет очень велико.
Оптика
328. Цилиндрическая часть стеклянной бутылки имеет
внутренний радиус r и внешний радиус R. Бутылку запол-
няют молоком. Показатель преломления стекла равен n1 ,
молока n2 , причём n2 > n1 . При каком соотношении между
R и r при взгляде сбоку будет казаться, что толщина стенок
бутылки равна нулю?
329. Рассеивающая линза c фокусным расстоянием F
подвешена к потолку. На расстоянии 100F от неё, на опти-
ческой оси, расположен неподвижный точечный источник
света. Линзу выводят из равновесия, и она начинает совер-
шать малые колебания. Оптическая ось линзы остаётся при
этом в той же плоскости, а прямая, проходящая через центр
линзы и точку подвеса, перпендикулярна оптической оси.
Максимальный угол наклона к вертикали равен a  1, рас-
стояние от центра линзы до точки подвеса равно F. Най-
дите длину траектории, по которой движется изображение
источника и максимальное ускорение этого изображения.
П р и м е ч а н и е. Если угол a  1 выражен в радианах
(как в этой задаче), то справедливы формулы sin a ≈ a и
tg a ≈ a.
330. Маленькая муха летает по кругу радиусом r = 1 см
с постоянной скоростью u = 10 см/с. Центр круга находит-
ся на оптической оси собирающей линзы на расстоянии
L = 30 см от неё (см. рис. 177). Фокусное расстояние линзы
равно f = 10 см. Найдите минимальную и максимальную ве-
личины скорости изображения мухи, создаваемого линзой.
При каком положении изображения они достигаются?

11 к л а с с
Механика
331. Тонкая массивная шайба надета на длинный стер-
жень радиуса R (см. рис. 178). Когда шайбу закрутили во-
круг стержня с угловой скоростью w, оказалось, что она
Задачи городских туров 143

останавливается через время t0 . В другой раз шайбу за-


крутили с той же угловой скоростью и одновременно при-
дали ей скорость V0 вдоль стержня. Какой путь пройдёт
по стержню шайба до остановки? Зазора между шайбой и
стержнем нет.
332. Тонкий обруч, имеющий массу M, которая сосре-
доточена в оси, на которую он насажен, и радиус R (см.
рис. 179), поставлен на горизонтальную плоскость. По глад-
кому каналу внутри обруча соскальзывает из верхней точки
без начальной скорости шайба массой m. Определить ско-
рость центра обруча, когда шайба находится в точке A (под
углом f от вертикали). Трения нет.
333. Человек встряхивает капилляр длиной l и радиусом
r, полностью заполненный жидкостью плотностью r. При
этом капилляр движется по дуге окружности в вертикаль-
ной плоскости (см. рис. 180) вокруг одного из своих концов
144 Условия задач

с угловой скоростью w. При каком угле f жидкость начнёт


вытряхиваться из капилляра? Коэффициент поверхностно-
го натяжения жидкости равен s.
334. Оцените радиус атомного ядра, если известно, что
помещённый в него мезон (элементарная частица с зарядом
e и массой, превосходящей массу электрона в 200 раз) со-
вершает в нём гармонические колебания с частотой w0 , а
энергия, необходимая для выбивания покоящегося мезона
из центра ядра, равна W0 . Считать, что мезон взаимодей-
ствует с ядром только электростатически, и для описания
его движения применимы законы классической физики.
335. На гладкий бесконечно длинный горизонтальный
стержень нанизаны три бусинки (см. рис. 181). В началь-
ный момент времени бусинки с массой M покоятся, а бу-
синка с массой m движется со скоростью V0 . Оценить, какое
количество соударений произойдёт у маленькой бусинки с
большими бусинками. Найти максимально возможное ко-
личество соударений и установившуюся скорость бусинки.
Трения нет, удары абсолютно упругие, M  m.
336. Какой размер должен иметь наполненный водоро-
дом шар для того, чтобы неподвижно висеть вблизи поверх-
ности Земли под градом, падающим с высоко расположен-
ной тучи. Плотности воздуха, водорода и льда равны r, rВ и
rЛ соответственно. Градинки являются небольшими шари-
ками радиуса r, упруго отскакивающими от оболочки ша-
Задачи городских туров 145
146 Условия задач

ра. Количество градинок в единице объёма воздуха равно n.


Считать, что на сферу, движущуюся в воздухе со скоростью
V , действует сила сопротивления F = −ar2 V . Масса оболоч-
ки шара пренебрежимо мала.
337. Если подуть в графин с длинным узким горлыш-
ком, то раздастся звук частоты n. Найдите объём графина,
если площадь сечения горла равна S, а длина горла –– l. Ско-
рость звука в воздухе равна c.
338. Изогнутая проволока состоит из большого количе-
ства полуокружностей радиуса R. По проволоке может без
трения скользить маленькая бусинка (см. рис. 182). Про-
волока наклонена под углом a = 1/120 радиана к горизон-
ту. Первоначально бусинка находится на верхнем участке
проволоки. Бусинку отпускают. Известно, что к тому мо-
менту, когда бусинка достигла нижнего конца проволоки,
она 150 раз испытала состояние невесомости. Найти длину
проволоки. Ускорение свободного падения равно g.
339. На железнодорожной платформе, способной дви-
гаться по горизонтали без трения, находится рота абсолют-
но одинаковых солдат, каждый из которых может бежать
по платформе с одной и той же скоростью относительно её
поверхности. Солдаты начинают бежать вдоль платформы и
соскакивают с неё. В каком случае скорость платформы бу-
дет больше: если все солдаты начинают бежать и покидают
её одновременно или по очереди?
Задачи городских туров 147

340. Конец однородного стержня длиной L и массой M


закреплён на шарнире так, что стержень может вращаться в
любом направлении без трения. На расстоянии l < L от точ-
ки закрепления к стержню прикреплена пружина жёстко-
сти k, длина которой в недеформированном состоянии пре-
небрежимо мала. В какой точке пространства следует за-
крепить другой конец пружины для того, чтобы стержень
находился в положении безразличного равновесия?
341. На наклонной плоскости (см. рис. 183), составляю-
щей угол 29◦ с горизонтом, неподвижно лежит стопка из 15
одинаковых по форме клиньев с углом при вершине в 1◦ .
По верхней поверхности верхнего клина без трения сколь-
зит шайба массой m. Найти силу, действующую на наклон-
ную плоскость со стороны стопки клиньев, если известно,
что все они покоятся, а трение между их поверхностями от-
сутствует.
342. После очередного прыжка через гиперпространство
космический корабль оказался вблизи очень необычной
планеты Кулогауссия, представляющей собой очень длин-
ный однородный круговой цилиндр, вращающийся вокруг
своей оси с постоянной угловой скоростью w. Чему рав-
на плотность вещества планеты, если высадившиеся на её
боковую поверхность исследователи оказались в состоянии
невесомости?
343. N одинаковых небольших шариков подвешены к
одной точке на невесомых нерастяжимых нитях длиной
L. В начальный момент все маятники находятся в од-
ной плоскости, содержащей вертикаль, проходящую че-
рез точку подвеса (см. рис. 184), и отклонены на углы
148 Условия задач

0 < f1 < f2 < · · · < fn  p/2. Начиная с первого, маятники по-


следовательно отпускают без начальной скорости в момен-
ты времени t1 , t2 , . . . , tn−1 соответственно. В какие момен-
ты времени последний маятник будет находиться в точке
своего начального положения? Все удары считать абсолют-
но упругими.
344. Лёгкая нерастяжимая нить привязана в точке A к
потолку (см. рис. 185), затем пропущена сквозь маленькую
массивную бусинку, зацеплена за два блока B и C, и при-
вязана к бусинке вторым концом. Первоначально бусинку
удерживают так, что углы, которые нить образует с гори-
зонталью, равны a, b и 90◦ . Затем систему отпускают. Най-
дите вектор ускорения бусинки в начальный момент време-
ни. Ускорение свободного падения равно g.
345. Тяжёлый теплоизолированный контейнер массой
M = 10 кг, содержащий m = 1 кг газа, отпустили без началь-
ной скорости с высоты H = 15 м. На какую высоту под-
скочит контейнер, если его соударение с землёй абсолютно
упругое и происходит мгновенно? Сопротивлением воздуха
пренебречь. Считать, что колебания в газе быстро затухают.
346. На стержень длиной L = 1,5 м, закреплённый по
диаметру колеса, нанизана маленькая бусинка массой m = 10 г.
Бусинка прикреплена к ободу колеса (см. рис. 186) идеаль-
ной пружиной жёсткости k = 1 Н/м, длина недеформиро-
ванной пружины 75 см. Первоначально колесо вращается
Задачи городских туров 149

вокруг своей оси с очень большой частотой. В момент, ко-


гда стержень был горизонтален, колесо быстро затормозили
до угловой скорости w = 10 рад/с. Через какое минимальное
количество оборотов колеса система снова вернётся в поло-
жение изображённое на рисунке? Трением между стержнем
и бусинкой пренебречь. Соударения между бусинкой и обо-
дом колеса абсолютно неупругие.
347. На горе над выездом из тоннеля стоит пушка, ко-
торая может стрелять под любым углом к горизонту. Ско-
рость вылета снаряда может меняться. Из тоннеля выез-
жают машины, двигаясь со скоростью V0 . Пушка стреляет
по машине в тот момент, когда она появляется из тоннеля.
Подбитые машины находятся на промежутке от x до y, счи-
тая от тоннеля (см. рис. 187). Определите, на какой высоте
150 Условия задач

расположена пушка над дорогой, и с какой максимальной


скоростью могут вылетать снаряды. Считайте, что подбитые
машины мгновенно останавливаются, и что снаряды попа-
дают именно в те машины, в которые целилась пушка.
Теплота и молекулярная физика
348. В длинной открытой трубе (см. рис. 188) между
двух поршней находятся n молей идеального одноатомного
газа при температуре T. Поршни могут двигаться без тре-
ния только вверх (при попытке движения вниз их закли-
нивает); масса верхнего поршня M, масса нижнего прене-
брежимо мала. К газу подвели некоторую теплоту, так что
он нагрелся до температуры T . При этом верхний поршень
поднялся вверх. Затем газу позволяют остыть до исходной
температуры. Процесс повторяют многократно. Определите
КПД такого подъёмника. Газ считайте лёгким, атмосферное
давление p0 , ускорение свободного падения g, тепловые по-
тери при нагревании газа составляют l%. Площадь каждого
из поршней равна S.
349. Воздушный шарик представляет собой тонкую ре-
зиновую оболочку, имеющую в нерастянутом состоянии ра-
диус r0 и толщину d0 . Материал шарика характеризуется
модулем Юнга E и при растяжении подчиняется следую-
щему закону: произведение относительного удлинения на
модуль Юнга равно механическому напряжению. Кроме то-
го, при растяжении объём материала не изменяется. Ша-
рик помещают в вакуум и начинают надувать идеальным
Задачи городских туров 151

газом с молярной массой m и температурой T. Определите


зависимость радиуса шарика и давления в нём от массы m
закачанного газа. Постройте графики этих зависимостей.
П р и м е ч а н и е. Механическое напряжение s равно от-
ношению растягивающей силы к площади поперечного се-
чения образца.
350. На планете Олимптурия-2 температура воды в
море Победителей составляет зимой t1 = 20 ◦ C, а летом ––
t2 = 1000 ◦ C. В качестве высшей меры поощрения победите-
лей второго тура олимпиады по физике в феврале сажают в
водолазный колокол (сосуд без дна) и опускают на дно моря
Победителей. Многолетние наблюдения показали, что при
подъёме победителей через шесть месяцев после погруже-
ния на глубине H = 14 м из-под колокола начинают выхо-
дить пузыри. Чему равно атмосферное давление на Олимп-
турии-2, если известно, что в течение года оно практически
не меняется? Считать, что дыхание победителя не изменяет
состав газовой смеси под колоколом.
351. Баллон со сжатым воздухом плотно накрыли тяжё-
лым колоколом массы M, объёма V так, что края колокола
152 Условия задач

плотно прилегают к полу. Из баллона начинают медленно


выпускать воздух, и давление в нём падает в два раза. При
этом в некоторый момент колокол отрывается от поверхно-
сти пола и опускается обратно. К моменту, когда все про-
цессы завершились, под колоколом установилось давление
в n = 1,01 раз большее, чем атмосферное. При этом масса ве-
щества, находящегося под колоколом, уменьшилась на m по
сравнению с первоначальным значением. Каким было дав-
ление в баллоне в начальный момент? Диаметр основания
колокола равен D. Температура воздуха –– T, молярная мас-
са –– m, атмосферное давление –– P0 .
352. Теплоизолированный сосуд длиной L разделён по-
полам тонким теплопроводящим поршнем массой M. При
движении поршня на него действует со стороны сосуда (см.
рис. 189) постоянная сила трения Fтр . Слева и справа от
поршня находится по n молей гелия при температуре T0 .
Сосуд повернули на 90◦ , дождались установления в систе-
ме равновесия, а затем повернули обратно. Через некоторое
время в системе установилась температура T. Найдите рав-
новесное положение поршня в сосуде после того, как сосуд
повернули в первый раз. Считать, что во время поворота
поршень практически не сдвигается. Ускорение свободного
падения g. Теплоёмкостью поршня, а так же массой газа по
сравнению с массой поршня пренебречь.
353. В герметичный контейнер объёмом 3 л, заполнен-
ный гелием при давлении 1 атм, поместили 9 г льда. На-
чальная температура системы 0 ◦ C. До какой минимальной
температуры надо нагреть контейнер, чтобы вода: 1) испа-
рилась? 2) выкипела? График зависимости температуры ки-
пения воды от атмосферного давления показан на рис. 190.
Молярная масса воды известна.
354. В сосуде, имеющем форму бублика, расположено
два тонких поршня площадью S, соединённых пружиной
жёсткостью k (см. рис. 191). Первоначально газ в обоих
отсеках сосуда имеет давление p и температуру T, пру-
жина не растянута, а объём отсека с пружиной составляет
a = 1/10 часть всего объёма сосуда V . До какой температуры
следует нагреть отсек с пружиной, чтобы его объём увели-
чился вдвое? Температура остальной части сосуда не меня-
ется, трением пренебречь.
Задачи городских туров 153
154 Условия задач

Электричество и магнетизм
355. C наклонной плоскости, расположенной под углом
a к горизонту, в вертикальном магнитном поле индукции B
скатывается без проскальзывания тонкостенная труба, из-
готовленная из диэлектрического материала. В трубе сдела-
на тонкая канавка, заполненная металлом, так, что обра-
зуется прямоугольный токопроводящий замкнутый контур
сопротивления R (см. рис. 192). Определите среднюю уста-
новившуюся скорость скатывания трубы. Длина трубы L,
диаметр D, масса M, ускорение свободного падения g. Са-
моиндукцией пренебречь.
356. На столе лежит тонкий диэлектрический квадрат
со стороной a. Квадрат равномерно заряжен до заряда√ Q.
На продолжении диагонали квадрата, на расстоянии 2a
от его угла, расположен невесомый равноплечий рычаг дли-
ной r (r  a), который может вращаться вокруг вертикаль-
ной неподвижной оси, проходящей через центр рычага (см.
рис. 193). На концах рычага закреплены два одноимённых
точечных заряда q0 массами m. Первоначально рычаг удер-
живали параллельно стороне квадрата. Определите ускоре-
ние зарядов в момент, когда рычаг отпустили.
357. Катушка, показанная на рис. 194, состоит из ци-
линдра радиуса r1 и двух цилиндров радиуса r2 . Провод
намотали несколько раз вокруг одного из крайних цилин-
дров, а затем –– вокруг среднего, сохраняя направление об-
мотки. Катушку с проводом положили на шероховатый стол
и включили однородное магнитное поле B (см. рис. 195).
Задачи городских туров 155

Левый конец провода, лежащий на столе, подсоединён к од-


ной клемме вольтметра, а правый конец, с помощью сколь-
зящего контакта, –– к другой клемме. Провод тянут вправо
со скоростью V . Найдите показания вольтметра. Считайте,
что катушка катится по столу без проскальзывания, вольт-
метр вместе с контактами неподвижен.
358. Тонкая равномерно заряженная трубка массы m мо-
жет скользить без трения вдоль нити. Заряд трубки Q, дли-
на l. Систему помещают в электрическое поле, изображён-
ное на рис. 196: в заштрихованных областях поле однород-
но и имеет напряжённость E, в незаштрихованной области
шириной L > l поле отсутствует. Найдите период колебаний
трубки вдоль нити как функцию амплитуды A колебаний.
359. Незаряженный проволочный каркас имеет форму
пирамиды (ABCD –– квадрат, точка лежит над его середи-
156 Условия задач
Задачи городских туров 157

ной, см. рис. 197). Каркас поместили в однородное электри-


ческое поле напряжённостью E, направленное вдоль AD.
При этом на ребре DC индуцировался заряд q1 , а на ребре
OC –– заряд q2 . Затем каркас развернули так что, поле ста-
ло направлено по диагонали квадрата AC. Найдите заряды,
индуцированные на каждом ребре каркаса в этом случае.
360. Пластины плоского конденсатора, присоединённого
к источнику с напряжением U, расположены на расстоянии
d друг от друга. С отрицательно заряженной пластины вы-
летают без начальной скорости электроны, которые, притя-
гиваясь к противоположной пластине, создают очень малый
ток через конденсатор. Конденсатор помещают в однород-
ное магнитное поле. Каково наименьшее значение магнит-
ной индукции, запирающее ток через конденсатор? Масса
электрона m, заряд e, краевыми эффектами в конденсаторе
пренебречь. Направление магнитного поля перпендикуляр-
но плоскости рисунка.
361. Над гладкой деревянной поверхностью на высоте H
закреплён положительный точечный заряд Q. На поверхно-
сти разместили три маленьких отрицательно заряженных
тела с зарядами −q. Одно тело удерживают непосредствен-
но под зарядом Q, а два других –– на одинаковом расстоя-
нии от него с противоположных сторон (см. рис. 198). Тела
на поверхности отпускают, и они приходят в движение. Че-
рез некоторое время, вследствие незначительного трения о
воздух, тела останавливаются. Найдите взаимное располо-
жение тел после остановки в случае, если а) Q  q; б) Q  q;
в) q = 0,75Q. Трением о поверхность пренебречь. Считать,
158 Условия задач

что тела настолько тяжелы, что они не отрываются от по-


верхности.
362. На рис. 199 представлена схема экономичного на-
гревателя: ключ K осуществляет переключение контактов
(a) и (b) с частотой n. Во сколько раз КПД такой схемы
превышает КПД нагревателя в случае непосредственного
подключения нагревательного элемента к батарее? Считать,
что конденсатор за период переключения ключа заряжается
и разряжается полностью.
363. Заряженная дробинка массой m движется вблизи
поверхности Земли во взаимно перпендикулярных посто-
янных магнитном (B) и электрическом (E) полях, направ-
Задачи городских туров 159

ленных горизонтально и вертикально вверх соответствен-


но. Укажите точки траектории (трохоиды), соответствую-
щие минимальной и максимальной скорости дробинки, и
найдите её заряд, если 2Vmin = Vmax = V0 . Сопротивлением
воздуха и потерями энергии на излучение пренебречь.
364. Плоское заряженное тело, расположенное горизон-
тально, равномерно вращается с частотой w вокруг верти-
кальной оси (см. рис. 200). Точка x лежит в плоскости тела
и принадлежит оси вращения. В точке x тело создаёт элек-
трический потенциал f. Чему равна индукция магнитного
поля, создаваемого телом в точке x? (Индукция магнитного
поля, создаваемого отрезком проводника с током I длиной
Dl на расстоянии r от проводника под углом a к направле-
I · Dl · sin a
нию тока, равна DB = mm0 ).
4pr2
365. Пластины плоского конденсатора площадью S и
массой m соединены пружиной жёсткостью k, изготовлен-
ной из изолятора (см. рис. 201). Длина недеформированной
пружины L. Конденсатор зарядили до заряда q. Затем его
разряжают через сопротивление R, замыкая ключ. Какая
теплота выделится на сопротивлении в случае, когда раз-
ряд происходит быстро? Какая теплота выделится в случае,
когда разряд происходит медленно?
366. Гальванометр имеет рамку с площадью S, на кото-
рую намотано N витков проволоки. Рамка подвешена на ни-
ти, в которой возникает противодействующий момент M0
при закручивании нити на угол a0 . Магнитное поле B пер-
пендикулярно оси вращения рамки при всех её положени-
160 Условия задач

ях. На какой угол повернётся рамка, если по её обмотке


пропустить ток I?
367. Математический маятник массой m, имеющий за-
ряд q, с длиной нити l, помещён в сильное однородное маг-
нитное поле, вектор индукции которого равен B и направ-
лен вертикально вверх. Найдите круговые частоты этого ма-
ятника. За какое время плоскость качаний маятника сдела-
ет полный оборот?
368. Однородный шар массой M, равномерно заряжен-
ный по объёму электрическим зарядом Q, закреплён на
невесомом нерастяжимом канате и вращается в плоскости,
перпендикулярной линиям магнитного поля B с постоян-
ной угловой скоростью w. Известно, что при удлинении ка-
ната в 3 раза сила его натяжения увеличивается в 2 раза.
Чему равна величина вектора магнитной индукции, если
известно, что после выключения магнитного поля сила на-
тяжения нити стала такой же, как до увеличения длины ни-
ти? Сила тяжести отсутствует.
369. В центре верхней грани прямоугольного паралле-
лепипеда a × a × (a/2), равномерно заряженного по объё-
му электрическим зарядом с объёмной плотностью r, век-
тор напряжённости электрического поля имеет величину E.
Определить напряжённость электрического поля, создавае-
мого коробкой, имеющей форму прямоугольного параллеле-
пипеда b × b × (b/2) без верхней крышки, все стенки которой
равномерно заряжены поверхностным зарядом одинаковой
плотности s.
370. Математический маятник массы m, заряда q, с дли-
ной нити l совершает гармонические колебания амплитуды
Задачи городских туров 161

A на расстоянии r от горизонтальной идеальной металли-


ческой плоскости. Определить период колебаний маятника
и магнитное поле H в точке подвеса. Электромагнитным
излучением и конечностью скорости распространения элек-
тромагнитного поля пренебречь.
371. Изображённая на рис. 202 схема состоит из N оди-
наковых идеальных катушек индуктивности и N идеальных
диодов. Какой ток потечёт через амперметр A в установив-
шемся режиме, если схема питается током I = I0 sin(wt)?
П р и м е ч а н и е. Идеальный диод проводит ток только
в одном направлении: его сопротивление равно нулю, когда
ток через него течёт «по стрелке», и обращается в бесконеч-
ность для токов, текущих «против стрелки».
372. В сферический сосуд большого радиуса налита
несжимаемая, равномерно заряженная жидкость плотности
r с диэлектрической проницаемостью e. Заряд единицы объ-
ёма жидкости s. В сосуд поместили 2 одинаковых незаря-
женных маленьких шарика радиуса r плотностью r0 , из-
готовленных из диэлектрика. Где расположатся шарики?
Ускорение свободного падения g. Поляризацией шариков
пренебречь.
373. Радиальное магнитное поле убывает по закону
A
H = 20 , где A0 –– некоторая константа, а r –– расстояние до
r
некоторой точки O. Проводящее кольцо (см. рис. 203) ради-
уса a с электрическим сопротивлением R перемещают с по-
стоянной скоростью V вдоль прямой OO . Плоскость кольца
перпендикулярна OO , его центр лежит на этой прямой. Ка-
кую силу нужно прикладывать к кольцу в каждый момент
162 Условия задач

времени, чтобы перемещать его описанным образом? Силой


тяжести пренебречь.
П р и м е ч а н и е. Описанное в задаче магнитное поле
принадлежит так называемому магнитному монополю (маг-
нитному заряду), который не существует в макромире и не
обнаружен в микромире. Однако отдельные области неко-
торой системы токов или магнитов можно моделировать с
помощью подобного поля.
Оптика
374. Круглая комната застеклена большим количеством
маленьких плоских зеркал. В комнате находится точечный
источник света. На рис. 204 показаны некоторые из набора
его изображений. Проведя графическое построение, найди-
те положение источника внутри комнаты.
375. Оптическая система состоит из собирающей линзы
с фокусным расстоянием f и плоского зеркала, расположен-
ного на расстоянии a от линзы (a > f) перпендикулярно её
главной оптической оси (см. рис. 205). Постройте все дей-
ствительные изображения полупрямой AB, расположенной
перед линзой на расстоянии H от главной оптической оси.
Задачи городских туров 163
164 Условия задач

376. Построить изображение треугольника ABC (см.


рис. 206) в собирающей линзе с фокусным расстоянием f.
377. В жидкости (см. рис. 207) с оптическим коэффици-
ентом преломления n со скоростью V всплывает сфериче-
ский пузырек воздуха постоянного радиуса r (коэффициент
преломления воздуха n0 = 1). На своём пути он пересекает
своей серединой тонкий луч лазера. На расстоянии L (L  r)
за пузырьком находится вертикальный экран. Определите
зависимость положения световой точки на экране от време-
ни.
378. Изображение квадрата, построенное в некоторой со-
бирающей линзе, представляет из себя две ломаные, уходя-
щие на бесконечность (см. рис. 208). Пользуясь рисунком,
Задачи городских туров 165

восстановите размер квадрата, фокусное расстояние линзы


и её положение.
379. На рисунке изображён четырёхугольник (см. рис. 209).
Укажите, где надо располагать собирающую линзу, и чему
должно быть равно её фокусное расстояние, чтобы его изоб-
ражение имело форму
1) параллелограмма;
2) прямоугольника;
3) квадрата?
Задачу решать графически.
380. На рис. 210 приведено изображение окружности в
собирающей линзе. AA –– главная оптическая ось этой лин-
зы. Пользуясь рисунком, восстановите расположение лин-
зы, её фокусное расстояние, положение и радиус окружно-
сти. Окружность, её изображение и ось AA лежат в плоско-
166 Условия задач

сти рисунка. При построении изображения использовалось


приближение параксиальных лучей.
Задачи городских туров 167
Задачи экспериментального тура
Лучшие участники городского тура физической олим-
пиады приглашаются на экспериментальный тур. Умение
решать экспериментальные задачи требует особых навы-
ков, которые не затрагиваются в школьном курсе физики,
вероятно поэтому «эксперимент» традиционно является
слабым местом даже для самых подготовленных участ-
ников олимпиады. В ходе решения экспериментальной за-
дачи участник должен самостоятельно разработать и де-
тально спланировать эксперимент, дающий максимально
точный ответ на поставленный в условии вопрос, исполь-
зуя только предоставленные приборы и материалы. Ес-
ли имеется несколько способов измерения, то лучшим яв-
ляется тот, который обеспечивает большую точность.
Все измерения необходимо проводить неоднократно (не
менее трёх раз). Это позволяет обнаружить промах. По
разбросу результатов можно найти погрешность измере-
ний. Для всех измерений и для конечного результата обя-
зательно должна быть указана погрешность. Важно по-
нимать, что результат, без указания погрешности прове-
дённых измерений и самого экспериментального метода,
не имеет смысла.

7––8 к л а с с ы
381. «Бутылка». Пластиковая бутылка с небольшим
отверстием в стенке наполнена водой. Бутылка закрыта
крышкой, через которую проходит трубка. Трубка может
двигаться, но герметичность соединения её с крышкой при
этом сохраняется. Если нижнее отверстие трубки находится
выше отверстия в стенке бутылки, то вода из бутылки вы-
текает. Когда трубка опускается ниже отверстия, вода пе-
рестаёт течь. Дайте качественное объяснение наблюдаемым
явлениям.
382. «Хроматография». Даны два стаканчика с синими
красками: тушью и гуашью. Кроме того, имеются три жид-
кости зелёного цвета. Нужно наблюдать, как жидкости впи-
тываются в фильтровальную бумагу и в белую ткань. Опре-
делите, из чего состоят жидкости зелёного цвета. (Одна из
жидкостей была тушь, вторая –– гуашь, а третья –– смесь си-
Задачи экспериментального тура 169

ней туши с жёлтой гуашью.) Дайте качественное объясне-


ние наблюдаемым явлениям.

9 класс
383. Определить силу, которую нужно приложить для
отрыва постоянного магнита от железной пластинки.
Д а н о: магнит с приклеенной к нему ниткой, железная
пластинка, гирька, вес которой слишком мал для отрыва
магнита от пластинки, линейка, штатив.
384. (Задача предлагалась для всех классов). Измерить
зависимость удлинения резинки от приложенной силы.
Д а н о: тонкая, длинная резинка; лист белой бумаги, на
которой нужно построить график; карандаш для построе-
ния графика.
385. Определить, во сколько раз сила трения скольже-
ния больше силы трения качения.
Д а н о: металлическая пластина с ровной поверхностью;
металлический цилиндр; металлический шарик с прикреп-
лённой к нему ниткой; линейка; штатив.
386. Определить плотность растворов соли и сахара в во-
де.
Д а н о: стаканчики; стеклянная капиллярная трубка;
линейка; подкрашенные растворы соли и сахара, вода.
387. Измерить теплоёмкость физического тела.
Д а н о: исследуемое тело, газовый термометр (смещение
капельки воды прямо пропорционально температуре), ста-
канчик, нитка, линейка или стеклянная трубка с деления-
ми, показывающими её объём в миллилитрах. В лаборато-
рии имеется достаточное количество воды со льдом и воды
при комнатной температуре.
П р и м е ч а н и е. Измерения нужно провести двумя
способами, полагая, что вам дана либо только линейка, ли-
бо только стеклянная трубка.

10––11 к л а с с ы
388. Измерить заряд свежепричёсанной девушки.
Д а н о: В лаборатории всегда имеются девушки, две шо-
коладные конфеты в алюминиевой фольге, а также расчёс-
ка, линейка, нитки, аналитические весы.
170 Условия задач

389. Определить скорость распространения деформации


в резине.
Д а н о: большая стирательная резинка «Архитектор»;
контактная пластинка с проводом; металлический шарик
с прикреплёнными ниткой и контактным проводом; шта-
тив; линейка; источник постоянного напряжения; конден-
сатор известной ёмкости; магазин сопротивлений; цифро-
вой вольтметр; соединительные провода и ключи.
390. Определить температурный коэффициент сопротив-
ления вольфрама.
Д а н о: вольфрамовая проволока диаметром 40 мкм и
длиной около 20 см с контактами на концах; стаканчик с
водой и тающим льдом; источник напряжения; вольтметр;
магазин сопротивлений; ключи и соединительные провода.
391. Измерить диэлектрическую постоянную стекла.
Д а н о: стеклянная пластинка с металлическими об-
кладками и присоединёнными к ним контактами; баллисти-
ческий гальванометр; сообщается, что отброс зайчика галь-
ванометра пропорционален протекшему через него заряду;
конденсатор известной ёмкости; источник постоянного на-
пряжения; соединительные провода, ключи, в том числе
двойной, линейка.
392. Измерить величину сопротивления.
Д а н о: Остеклованное сопротивление большого номина-
ла, величину которого нужно измерить; источник напряже-
ния; вольтметр; соединительные провода; стаканчик; тер-
мометр; секундомер; линейка. В лаборатории есть вода с та-
ющим льдом.
393. Изготовить омметр.
Д а н о: железный гвоздь; тонкий изолированный про-
вод; источник напряжения; магазин сопротивлений; скреп-
ка; соединительные провода, ключи.
394. Определить теплоту растворения соли (нитрата ам-
мония) в воде.
Д а н о: стаканчик, тонкая медная проволока с контакта-
ми, цифровой омметр, линейка. В лаборатории имеется во-
да при комнатной температуре и вода с кубиками льда. Соль
даётся порциями известной массы. Сообщается, что сопро-
тивление меди в широком диапазоне температур линейно
зависит от температуры.
Задачи экспериментального тура 171

395. Измерить зависимость сопротивления вольфрамо-


вой проволоки от натяжения.
Д а н о: тонкая вольфрамовая проволока, прикреплён-
ная к контактам на планке; два источника напряжения;
цифровой вольтметр; два магазина сопротивлений; линей-
ка.
396. Определить относительную влажность воздуха в
комнате.
Д а н о: Пластиковая бутылка с герметически закрываю-
щейся крышкой (сквозь крышку проходит тонкая стеклян-
ная трубка), линейка, стаканчик с водой. Давление насы-
щенного пара при комнатной температуре считается извест-
ным.
397. Измерить давление насыщенного пара воды при
комнатной температуре.
Д а н о: То же оборудование как и в задаче 396. Кроме
того, имеется достаточное количество воды с тающим льдом
и пустой стаканчик. Можно попросить дополнительное обо-
рудование. Температура в комнате и давление паров воды
при 0 ◦ C известны.
398. Определить полярность источника тока.
Д а н о: Источник постоянного тока, полярность которо-
го неизвестна, катушка с железным сердечником, иголка,
кусочек поролона, стаканчик с водой и тонкая, гибкая про-
волока с контактами.
399. Проградуировать термометр и измерить температу-
ру комнаты и температуру Вашей ладони.
Д а н о: Газовый термометр, в трубке подкрашенная во-
да, два стаканчика, линейка. В лаборатории имеются весы
и достаточное количество воды со льдом. Значения физиче-
ских констант можно узнать у членов жюри.
400. «Эффект Пелтье» (11 класс). Измерить омическое
сопротивление элемента Пелтье.
Д а н о: Элемент Пелтье, регулируемый источник посто-
янного тока, вольтметр, известное сопротивление, соедини-
тельные провода и ключи.
П р и м е ч а н и я. Эти условия были оглашены за месяц
до эксперимента. Задачу можно решать многими способа-
ми.
172 Условия задач

Для первого способа дополнительно давались: массив-


ное металлическое тело с плоской поверхностью, поролон,
скотч.
Для второго способа дополнительно давались: секундо-
мер и алюминиевый цилиндр известной массы. В лабора-
тории имеется термометр. Разрешалось использовать теп-
ло собственной руки. Значения физических констант можно
было узнать у жюри.
РЕШЕНИЯ ЗАДАЧ
Решения задач районных туров

1. Если бы скорость течения всюду была одинакова,


то волна имела бы форму угла, и изгибов, показанных на
рис. 1 не было бы видно. Поскольку скорость течения в цен-
тре канала больше, чем у берегов, то за катером остаётся
изогнутый след. Из рисунка видно, что вода по краям кана-
ла движется влево относительно воды в центре.
2. На переднее колесо велосипеда резко начинает дей-
ствовать большая тормозящая сила. Велосипед с велоси-
педистом стремятся сохранить свою скорость по инерции.
Поэтому при резком торможении можно перелететь через
руль.
3. Одним из важнейших механических свойств материа-
ла является его пористость, т. е. среднее отношение объёма
пустот в образце к его полному объёму. Простейший способ
оценки объёмов пустот в сыпучих нерастворимых материа-
лах таков: воду осторожно наливают в ведро сухого песка,
пока она не заполнит все полости между песчинками и не
появится на его поверхности. Тогда объём пустот в сухом
песке будет приблизительно равен объёму заполняющей их
воды. Воду можно считать несжимаемой, её объём мож-
но измерять мензуркой. Указанный метод, однако, неточен,
поскольку он не учитывает вклада капиллярных явлений, а
погрешность измерения с трудом поддаётся оценке.
4. На катер, быстро движущийся в воде, действует подъ-
ёмная сила, перпендикулярная встречному потоку жидко-
сти и поднимающая нос катера. При выключении двигате-
лей, катер быстро теряет скорость, подъёмная сила исчеза-
ет, и корпус судна «падает» в воду.
5. В процессе движения автомобиля термометр нагрева-
ется за счёт трения о воздух, и его показания возрастают.
6. При консервировании в банке с фруктами создаётся
давление меньше атмосферного. При вскрытии банки дав-
ление выравнивается, и фрукты начинают быстро сжимать-
ся, так как внутри них содержится некоторое количество
воздуха. Следовательно, объём фруктов уменьшается. На
фрукты действуют сила Архимеда и сила тяжести. Масса
фруктов не меняется, значит, сила тяжести остаётся посто-
176 Решения задач

янной, а сила Архимеда уменьшается с уменьшением объё-


ма, поэтому фрукты тонут.
7. Время, за которое сварится картошка, зависит от ско-
рости поступления в неё теплоты. А это в свою очередь за-
висит только от температуры воды, в которой она находит-
ся. Если мы отольём часть воды, это не изменит её тем-
пературу, она останется равной температуре кипения при
данных условиях. Если же мы закроем кастрюлю крышкой,
давление в кастрюле увеличится из-за интенсивного образо-
вания водяного пара. Что, в свою очередь, приведёт к уве-
личению температуре кипения (так как давление насыщен-
ных паров растёт с температурой). Следовательно, темпера-
тура кипящей воды повысится. Поэтому выгоднее закрыть
кастрюлю крышкой.
8. Собака сможет успешно взять след, если среди моле-
кул воздуха будет достаточное количество молекул пахучих
веществ, которые оставляет после себя человек. В воздухе,
на достаточном расстоянии от земли, из-за ветра и диффу-
зии эти молекулы достаточно быстро рассеиваются, и кон-
центрация запаха стремительно уменьшается. У самой зем-
ли влияние ветра и диффузии меньше и, следовательно, ве-
роятность успешного взятия следа больше.
9. На заднем стекле автомобиля капли исчезают со вре-
менем из-за испарения. Если автомобиль движется, то пары
воды уносятся ветром. Если автомобиль стоит, то насыщен-
ный пар, образующийся у поверхности капель, препятству-
ет их испарению. Таким образом, капли исчезнут быстрее,
если автомобиль движется.
10. Оба термометра состоят из пузырька с ртутью,
соединённого со столбиком, на который нанесена шкала.
Представим, что объём пузырька с ртутью у обоих термо-
метров одинаков, и что оба термометра показывают свою
минимальную температуру. Увеличим температуру каждо-
го на одинаковую величину (допустим на 5 ◦ C). Ртуть при
этом одинаково расширится. Если у одного из них (меди-
цинского) диаметр столбика — меньше, то по нему ртуть
поднимется на большую высоту, а значит, этот термометр
окажется более чувствительным и будет иметь меньший
диапазон температур при одинаковой длине шкал. Таким
образом, температурный диапазон медицинского термомет-
Решения задач районных туров 177

К решению задачи 10

ра меньше либо из-за меньшего сечения столбика термомет-


ра, либо из-за большего объёма пузырька с ртутью.
11. Обозначим площадь основания бутылки S = a · b, где
a, b — длины сторон прямоугольного основания (их можно
измерить линейкой). Нальём в бутылку воду объёмом боль-
шим половины объёма бутылки так, чтобы она помещалась
в её нижнюю часть, имеющую форму параллелепипеда, и
закроем бутылку пробкой. Объём бутылки складывается из
объёма воздуха в ней V1 и воды — V2 . Последняя величина
измеряется в прямом положении: V2 = S · h2 , h2 — расстоя-
ние от дна до уровня воды. V1 измеряется в перевёрнутом
положении: V1 = S · h1 , где h1 — расстояние от дна до уровня
воды.
О т в е т. Объём бутылки равен V = S(h1 + h2 ), где h1 и
h2 измеряются по уровню воды в вертикально стоящей и
перевёрнутой бутылке.
12. Чтобы обойти первого пловца на круг, второму плов-
цу нужно сначала его догнать. Скорость сближения пловцов
равна V = V2 − V1 , где V1 = L/t1 — скорость первого плов-
ца, а V2 = L/t2 — второго, причём, t1 = 90 сек, t2 = 70 сек,
а L — вся длина бассейна. Первый пловец обгоняет второго
на расстояние 2L. Тогда искомое время t = 2L/V = 630 сек.
О т в е т. Второй пловец обгонит первого на один круг
через 630 сек.
13. Объём куска меди Vм равен объёму вытесненной
им воды, которая складывается из объёма V0 вылившей-
ся воды и объёма V1 воды, заполнившей четверть сосуда:
V1 = V /4 = 1000 мл/4 = 250 мл. Значит, объём куска меди
равен Vм = V0 + V1 = 100 мл + 250 мл = 350 мл = 350 м3 , а его
масса — m = rм · Vм = 8,9 г/см3 · 350 см3 = 3115 г.
О т в е т. Масса куска меди равна 3115 г.
178 Решения задач

14. Расстояние между всеми столбами одинаково, обо-


значим его S. Сначала велосипедист проезжал мимо стол-
бов через промежутки времени t1 = S/v. Затем он уве-
личил свою скорость, и промежутки времени сократи-
лись: t2 = S/(v + Dv). Очевидно, что если он увеличит ско-
рость ещё на такую же величину, то t3 = S/(v + 2 · Dv). От-
сюда получаем систему уравнений: v = S/t1 ; v + Dv = S/t2 ;
v + 2 · Dv = S/t3 . Отсюда t3 = t1 · t2 /(2t1 − t2 ) = 3 сек.
О т в е т. После повторного увеличения скорости велоси-
педист будет миновать столбы каждые 3 сек.
15. Скорости пешехода на трёх участках равны V1 = 1,5 м/с,
V2 = 1,45 м/с, V3 = 1,5 м/с. Всего пешеход прошёл 133 м за
90 сек. Таким образом, средняя скорость пешехода равна
133/90 м/с ∼ = 1,47 м/с.
О т в е т. Средняя скорость пешехода примерно равна
1,47 м/с.
16. Мальчик решает вернуться домой с полпути L/2,
уже затратив на дорогу время t0 = L/2Vш , где Vш — его ско-
рость шагом. Расстояние, которое нужно пробежать до дома
и обратно в школу, равно 3L/2. Значит, отношение скоро-
стей бегом и шагом должно быть Vб /Vш > 3, иначе мальчик
опоздает на урок. Чтобы успеть вовремя, мальчик должен
был бы бежать со скоростью 6 м/с, которая соответствует
21,6 км/ч.
О т в е т. Мальчик не успеет к началу урока.
17. Путь, пройденный дельфином вдоль одного прямо-
линейного участка стенки бассейна, равен a − 2x, где a —
длина всего прямолинейного участка бассейна, а x — иско-
мое расстояние. Путь, пройденный дельфином вдоль стенок
бассейна за один оборот, равен 4(a − 2x). Путь, пройденный
дельфином вдоль стенок бассейна за 3 полных оборота, ра-
вен 12(a − 2x) = Vt.
О т в е т. Расстояние между дельфином и стенкой бассей-
на равно (a − Vt/12)/2 = 2,5 м.
18. Объём V варёной крупы, равен сумме объёмов V1
сухой крупы и V2 впитавшейся в крупу воды: V = V2 + V1 .
Согласно определению плотности, объём V равен отноше-
нию массы варёной крупы, складывающейся из массы m1
сухих зёрен и массы mх впитавшейся воды, к плотности r2
Решения задач районных туров 179

К решению задачи 19

варёной крупы, V = (m1 + mх )/r2 . Объём V1 = m1 /r1 . Объём


V2 = mх /r0 . Масса воды, впитавшейся в крупу mх = m1 r1 (r0 − r2 )/(r2 − r1 )/r0 .
О т в е т. Масса выкипевшей воды 1,5 кг.
19. Графики проекции скорости V1x (t), V2x (t) часовых
относительно охраняемого объекта и зависимости проекции
скорости первого часового относительно второго представ-
лены на рисунках. Когда второй часовой неподвижен, зем-
ля движется относительно него со скоростью V3 = −V2 . При-
чём, проекция скорости первого часового относительно вто-
рого равна V1−2 = V1 + V3 = −V2 + V1 .
О т в е т. Графики представлены на рисунках.
20. Рассмотрим случай, когда Василий бежит вверх по
попутному эскалатору. Его скорость относительно Земли
складывается из собственной скорости бега (10 км/ч) и ско-
рости эскалатора v. Обозначим за t то время, за которое
он пробегает эскалатор, t = L/(10 + v), где L длина эскала-
тора. В случае, когда Василий бежит по встречному эска-
латору, его скорость равна разности скорости бега и скоро-
сти эскалатора. Тогда t + 12 мин = L/(10 − v) = t + 0,2 ч. На-
конец, когда эскалатор стоит, скорость Василия равна про-
сто 10 км/ч. В этом случае t + 0,05 ч = L/10. Выразим из
первого уравнения L = (10 + v)t. Подставляя в третье урав-
нение, получим: vt = 0,5 км, откуда v = 5 км/ч.
О т в е т. Скорость эскалатора равна 5 км/ч.
21. Анализ исходного графика Vх (t): Петя и Маша бежа-
ли вместе 1 мин и пробежали Dx1М = Dx1П = 2,5 м/сек · 60 сек = 150 м.
Потом Маша прошла Dx2М = 1,25 м/сек · 120 сек = 150 м, а
Петя побежал быстрее, Dx2П = 5,0 м/сек · 120 сек = 600 м.
180 Решения задач

К решению задачи 21

Затем дети развернулись и достигли школы одновременно,


Dx3М = −1,25 м/сек · 240 сек = −300 м, Dx3П = −3,125 м/сек · 240 сек = −750 м.
Координаты Пети и Маши x1М = 150 м, x2М = 300 м, x1П = 150 м,
x2П = 750 м, x3П = x3М = 0 м. Расстояние между Петей и Ма-
шей l(0 мин) = 0 м, l(1 мин) = 0 м, l(3 мин) = 450 м, l(7 мин) = 0 м.
О т в е т. Требуемый график приведён на рисунке.
22. Периметр стола равен 216 м. Заяц и Болван-
щик движутся навстречу друг другу и встретятся через
t = 216 м/(6 + 1,5) м/сек = 28,8 сек. Болванщик за это вре-
мя пройдёт расстояние s = vt = 1,5 м/сек · 28,8 сек = 43,2 м.
Учтём, что изначально Болванщик находился на расстоя-
нии 48 м от Алисы, следовательно, он встретится с Зайцем
на расстоянии 48 м − 43,2 м = 4,8 м от Алисы (на той сто-
роне стола, на которой они изначально находились).
23. Обозначим пути, которые преодолела Баба Яга со
скоростями 20 км/ч, 10 км/ч и 5 км/ч, как L1 , L2 и L3 со-
ответственно . Полный путь её будем обозначать L. Пусть
время движения на втором участке равно t, такое же время
затрачено и на третий участок. Найдём среднюю скорость
на участке 2 − 3:
L2 + L3 V2 t + V3 t V +V
V2−3 = = = 2 3.
t+t 2t 2

Тогда вторая половина пути (равная L/2) была пройдена


со средней скоростью (10 + 5)/2 = 7,5 км/ч. Время прохож-
дения второй половины пути равно (L/2)/V2−3 = L/15 (оно
же равно 2t). Время прохождения первой половины пути
равно: (L/2)/20 = L/40. Тогда полное время в пути равно:
Решения задач районных туров 181

L/40 + L/15 = (11/120)L. Средняя скорость равна полному


пути, делённому на полное время:
L
Vср = 11 = 120/11 ≈ 11 км/ч.
L
120

О т в е т. Средняя скорость Бабы Яги Vср = 11 км/ч.


24. Поскольку масса содержимого котелка не измени-
лась, то m = 0,75 л · rВ = V rС , где V — искомый объём снега,
rВ — плотность воды, rС — плотность снега. Вода в 4 раза
плотнее снега, поэтому пропорция легко разрешается, отку-
да следует, что в котелке было собрано 3 л снега.
О т в е т. Объём котелка равен 3 л.
25. В процессе перецепления локомотива расстояние
между любыми двумя вагонами, очевидно, не менялось, так
как вагоны не расцеплялись. Таким образом, независимо от
способа перецепления локомотива, в процессе перецепле-
ния вагоны прошли одинаковое расстояние. Также можно
заметить, что и до промежуточной остановки, и после неё
вагоны также проходили одинаковый путь.
О т в е т. Пути вагонов, в которых ехали Пётр и Павел,
одинаковы.
26. Обозначим начальную плотность кубика за r0 . По
m
определению r0 = 3 , где m — масса кубика, a — начальная
a
длина ребра. После теплового расширения длина ребра ста-
ла равной a + 0,005a. Масса кубика не изменилась, поэтому
его плотность стала равна r = m/(a + 0,005a)3 . Интересую-
щая нас величина равна r − r0 = m/(a + 0,005a) − m/a3 ∼ = −116 кг/м3 .
О т в е т. Плотность уменьшилась на 116 кг/м3 .
27. Объём проволоки, вытянутой за время t, равен
V = S · L = S · vt, а её плотность — r = m/V . Тогда площадь се-
чения проволоки равна S = m/r · vt.
О т в е т. Сечение проволоки равно S = 1,12 мм2 .
28. Запишем условие того, масса чугунного шара
в 1,25 раза больше массы шара из неизвестного спла-
ва: mч = 1,25mсп . Записывая каждую массу через объ-
ём и плотность, получим: rчуг Vчуг = 1,25rспл Vспл . Учитывая
соотношение между объёмами Vспл = 2Vчуг , получим, что
rспл = rчуг /2,5 = 2880 кг/м3 .
182 Решения задач

О т в е т. Плотность материала шара равна r1 = 2880 кг/м3 .


29. Очевидно, что горизонтальная прямая на графи-
ке характеризует тот участок пути, на котором парашю-
тист опускался с постоянной скоростью. Парашютист опус-
кался на землю последние 200 секунд пути со скоро-
стью 5 м/сек. Таким образом, начальная высота, с кото-
рой он начал опускаться с постоянной скоростью, равна
h = 5 м/сек(210 − 10) сек = 1 км.
О т в е т. Парашютист начал опускаться с постоянной
скоростью с высоты 1 км.
30. Мальчик двигается с постоянной скоростью, поэто-
му ботинок за один шаг ноги движется со скоростью 2 ра-
за большей скорости самого мальчика, так как вторая но-
га в это время стоит на земле неподвижно: 2v2 = v2 . Сред-
няя скорость движения ботинка второго мальчика на всём
участке дороги равна постоянной скорости мальчика, так
как ботинок проходит путь, равный пути мальчика, за вре-
мя, равное времени движения этого мальчика. Постоянная
же скорость мальчика равна 7,5 км/ч/1,5 = 5 км/ч.
О т в е т. В течение одного шага средняя скорость ботин-
ка будет в 2 раза больше скорости самого мальчика, а сред-
няя скорость движения ботинка на всём участке пути равна
v2 = 5 км/ч.
31. Пробег судна по течению реки L = (v + u)t1 , а про-
тив течения L = (v − u)t2 . Скорость судна относительно воды
v = L(t1 + t2 )/(2t2 t1 ).
О т в е т. Скорость судна относительно реки равна v = 12 м/c.
32. Перейдём в систему отсчёта связанную с водой в ре-
ке, т. е. движущуюся со скоростью u вправо. В этой системе
волны на воде начинают расходиться от места, где упал то-
пор, во все стороны с одинаковой скоростью v. Точка A же
будет двигаться со скоростью u влево. Пусть волна дошла до
точки A через время t. Тогда по теореме Пифагора (см. ри-
сунок)
d
(vt)2 = (ut)2 + d2 ⇒ t = √ .
v2 − u2
Выражение имеет смысл только в случае v > u. Иначе волны
на воде от топора никогда не достигнут точки A.
d
О т в е т. t = √ 2 2 , при v > u.
v −u
Решения задач районных туров 183

К решению задачи 32

К решению задачи 33

33. Выясним сначала, в каком положении будет плавать


палка. Рассмотрим ситуацию, показанную на рисунке. В
этом случае на верхнюю половину палки будет действовать
меньшая сила Архимеда, так как плотность воды, в которой
плавает верхняя половина палки, меньше. Соответственно,
в таком положении палка плавать не может, так как силы
Архимеда будут стремиться развернуть её в горизонтальное
положение. Следовательно, палка может плавать только го-
ризонтально, т. е. все её точки должны лежать на одной глу-
бине. Найдём эту глубину. Для того, чтобы палка плавала,
необходимо, чтобы разность силы тяжести gV rп и силы Ар-
химеда gV rж , была равна нулю (V — объём палки). Отсюда
получим, что rп = rж . Палка будет плавать на такой глубине,
где r = 1020 кг/м3 . По графику находим, что эта глубина —
20 м.
О т в е т. Палка будет плавать на глубине 20 м.
34. Сначала найдём, какую часть цилиндра зани-
мает вода. Это проще всего сделать, используя прави-
184 Решения задач

ло рычага. Сила тяжести воды приложена к точке A


(см. рисунок), а сила тяжести поршня к точке B. Центр
отрезка OC находится в точке P. По правилу рычага,
|AP|mg = |PB|mg, т. е. |AP| = |PB|, где m — масса воды
(и, соответственно, поршня). Но |OA| = |OB|S, а также
|AP| = |OP| − |OA| = |OC|/2 − |OB|/2 и |PB| = |OB| − |OP| = |OB| − |OC|/2.
Следовательно, правило рычага можно переписать в виде,
|OC|/2 − |OB|/2 = |OB| − |OC|/2. Таким образом, |OB| = 2/3 · |OC| = 2/3 · 6,6 см = 4,4
Значит, пружина имеет длину l = 6,6 см − 4,4 см = 2,2 см.
По закону Гука сила упругости пружины составляет
F = k(l − l0 ) = 6 Н/м(2,2 см − 3 см) = −0,048 Н,
где l0 = 3 см — изначальная длина пружины. Знак «−» у
F означает, что пружина сжата, а не растянута. В даль-
нейшем будем писать |F|. Давление со стороны поршня
на воду, составляет p = |F|/S = 0,048 Н/3,14 см2 = 153 Па.
Давление со стороны жидкости на поршень производит-
ся со стороны столба воды искомой высоты h. Сосчитать
его можно так. Давление водяного столба на уровне верх-
ней точки поршня равно p1 = rgh, на уровне нижней точ-
ки p2 = rg(h + 2r), где r — плотность воды, r — радиус ци-
линдра. Так как давление возрастает линейно с высотой
(p = rgx), то давление, оказываемое водой, на поршень рав-
но p = (p1 + p2 )/2, т. е. среднему давлению в верхней и ниж-
ней точках. Условие равновесия запишется тогда в виде
rgh/2 + (rgh + 2rgr)/2 = rg(h + r) = 153 Па. Решая это уравне-
ние, находим h = 0,53 см.
О т в е т. Вода в трубке находится на высоте 0,53 см.
35. Пусть, когда на поршне лежал груз, разность уров-
ней воды равнялась H. Тогда mg = rgHS1 . Пусть поршень
поднялся на высоту h1 , а уровень воды в правом сосуде
опустился на h2 . Предположим, что куб погрузился в во-
ду не полностью, а на глубину h1 . Суммарный объём жид-
кости не изменился, поэтому h1 S1 = h2 S2 + (h1 − h2 )S, где
S = a2 = 0,01 м3 — площадь основания куба. Объём погру-
женной части куба равен V = S(h1 − h2 ) = Sh1 (S1 − S)/(S2 − S).
На поршень действуют силы давления сверху и снизу, а
также сила со стороны верёвки, которая равна силе Архиме-
да, действующей на куб. Давление на поршень снизу боль-
ше атмосферного давления на DP = rg(H − h1 − h2 ). Условие
Решения задач районных туров 185

К решению задачи 34

К решению задачи 35

равенства сил, действующих на поршень вверх и вниз, за-


писывается как
DPS1 = rVg ⇔ rgS1 (H − h1 − h2 ) = rgS(h1 − h2 ),
   
S1 − S S2 − S
mg = rg(S1 (h1 + h2 ) + S(h1 − h2 )) = rgh1 S1 1 + +S .
S2 − S S2 − S
Откуда несложно выразить
   
m S1 − S S2 − S −1
h1 = S1 1 + +S = 6 см.
r S2 − S S2 − S
Видно, что получилось h1 < a = 10 см, значит верно наше
предположение о том, что куб погрузился не полностью.
О т в е т. Высота подъёма поршня равна 6 см.
36. Пусть пружина растянулась на Dx. На эту же ве-
личину левый поршень поднимется относительно правого.
Пусть давление воды под левым поршнем равно P, атмо-
сферное давление P0 . На левый поршень действуют сле-
дующие силы: сила атмосферного давления сверху P0 S1 ,
186 Решения задач

К решению задачи 36

сила давления воды снизу PS1 и сила со стороны крон-


штейна, равная силе с которой пружина действует на крон-
штейн, т. е. она равна kDx и направлена вниз. В итоге по-
лучаем P0 S1 + kDx = PS1 . Давление воды под правым порш-
нем равно P + rgDx. На правый поршень, помимо сил дав-
ления, действуют сила mg со стороны груза и сила kDx со
стороны пружины, (P + rgDx)S2 + kDx = P0 S2 + mg. Объеди-
нив оба уравнения, получим P − P0 = kDx/S1 . Используем
то, что S1 = S, S2 = 2S,

(P − P0 )S2 + rgDxS2 + kDx = mg ⇒ 3kDx + rgDx2S = mg.

О т в е т. Если на правый поршень поместить груз мас-


сой m, то пружина растянется на величину
mg
Dx = .
2rgS + 3k

37. Пусть V — объём доски, m — её масса, L — её


длина, S = 2 м · 0,1 м — площадь верхней поверхности, r —
плотность воды. Рассмотрим силы, которые действуют на
доску. Это сила тяжести mg = 400 Н, приложенная к центру
доски, сила Архимеда

1
F A = V rg = 1/2 · 2 м · 0,1 м · 0,01 м · 1000 кг/м3 · 10 м/с2 = 10 Н,
2

действующая на левую половину доски и приложенная на


расстояние L/4 от левого края, а также сила давления воды
Решения задач районных туров 187

К решению задачи 37

сверху на правую половину. Эта сила равна

1
FP = S(P0 + rgh) =
2
= 1/2 · 0,2 м2 · (100000 Па + 1000 кг/м3 · 10 м/с2 · 10 м) = 20000 Н,

(h = 10 м — глубина, P0 — атмосферное давление) и можно


считать, что она приложена к середине правой половины
доски. В момент отрыва под действием силы F1 (в обоих ва-
риантах) сила реакции со стороны уступа приложена к пра-
вому концу доски, а под действием силы F2 к центру доски.
Поэтому при написании правила рычага относительно этих
точек она вклада не даст. Из правила рычага имеем:
L L L 3L
F1 = FP + mg − F A (точка опоры — правый конец доски)
2 4 2 4
L L L
F2 = F P + F A (точка опоры — середина доски).
2 4 4

О т в е т. Отношение сил F1 /F2 ∼


= 1,038.
38. При вычислении потенциальной энергии воды как
целого нам не нужно следить за тем как части жидкости
перемешиваются между собой. Камень, лёгший на дно ак-
вариума, вытеснит жидкость объёмом VК , следовательно,
такой же объём жидкости добавится вверху на уровне во-
ды в аквариуме. Предположим, что остальные части жид-
кости не двигались, и тогда наш процесс эквивалентен пе-
ремещению жидкости объёмом VК со дна на поверхность.
Следовательно потенциальная энергия воды увеличится на
DE = rВ VК · gh, где h — начальный уровень воды в аквари-
уме, rВ — плотность воды, VК — объём камня.
188 Решения задач

К решению задачи 39

О т в е т. Потенциальная энергия воды увеличится за


счёт переноса вытесненной камнем воды на поверхность.
39. Рассмотрим все этапы движения крайнего левого ку-
бика. Если пройденное им расстояние меньше 5 см (т. е. тя-
нут только один кубик), то сила, которую нужно для это-
го прикладывать равна, по условию, 10 Н. Как только ле-
вый кубик касается следующего, сила скачком меняется до
20 Н, так как приходится тянуть уже два кубика. Продол-
жая рассуждения таким же образом, получим, что искомый
график таков:
Обратите внимание, что после 20 см можно тянуть всю
конструкцию с постоянной силой F = 40 Н сколь угодно дол-
го.
40. Сила, с которой второй динамометр действует на
первый, равна T2 . Силы, действующие на первый (верхний)
динамометр вверх и вниз, равны F = T1 = mg + T2 .
О т в е т. К верхнему динамометру приложена сила
F = 6 H.
41. Пусть m — масса динамометра, M — масса груза,
а T1 , T2 , T3 — показания соответствующих динамометров.
Каждый динамометр показывает вес груза, привешенного
к нему снизу, поэтому T1 = 90 Н = 2mg + Mg, T2 = mg + Mg,
T3 = 30 Н = Mg. Решая систему трёх уравнений, находим:
M = 3 кг, m = 3 кг, T2 = 60 Н.
О т в е т. Показания среднего динамометра равны 60 Н.
Масса динамометра равна 3 кг.
42. Запишем условия равенства давления, которое
верхний кубик оказывает на среднее тело, и давления,
Решения задач районных туров 189

которое два верхних тела оказывают на нижний кубик:


m3 m + m3
= 2 . Также запишем условия равенства давле-
S3 S2
ний между всей конструкцией и столом и давлением,
которое два верхних тела оказывают на нижний кубик:
m2 + m3 m + m2 + m3
= 1 . Из этого уравнения легко найти,
S2 S1
что m2 + m3 = m1 S2 /(S1 − S2 ). Подставим это выражение
в первое уравнение, m3 = m1 = 90 г. (При выводе послед-
него уравнения использована численная связь площадей,
S3 + S2 = S1 ). Теперь найдём m2 = m1 S2 /(S1 − S2 ) − m1 = 70 г.
О т в е т. Искомые массы суть m3 = 90 г и m2 = 70 г.
43. Выпишем условия равновесия плеч рычага, содер-
жащиеся в условии задачи: L1 (m1 g − 2rВ g · V /3) = m2 L2 Sg и
L1 (rV − 2 · rВ /3 − V ) = m2 L2 . Из этих двух уравнений легко
выразить искомый объём куба V = m2 L2 /(L1 (r − 2 · rВ /3)) = 75 см3 .
О т в е т. Объём алюминиевого куба равен 73,8 см3 .
44. Удлинение нижней пружины равно Dl2 = 2mg/k2 , а
её полная длина l2 = l02 + Dl2 . Удлинение верхней пружины
равно Dl1 = mg/k1 , а её полная длина l1 = l01 + Dl1 . Искомое
расстояние от потолка до нижнего шарика равно их сумме:
l = l1 + l2 .
О т в е т. Расстояние от потолка до нижнего шарика рав-
но l = 32 см.
45. Из-за связи поршней твёрдым стержнем возника-
ет сила F, создающая в жидкости давление p. Жидкость
несжимаема, поэтому она не будет перетекать в узкую часть
трубы меньшего объёма, и поршни останутся неподвижны-
ми. Условие их равновесия pS2 = F + pS1 .
О т в е т. Давление в жидкости равно p = F/(S2 − S1 ).
46. Для плавающего тела действующая на него сила тя-
жести равна весу вытесняемой им жидкости. Для всех тел с
одинаковой плотностью отношение объёма и массы, очевид-
но, фиксировано. Поэтому у плавающих тел с одинаковой
плотностью отношение объёма погруженной части ко все-
му объёму также совпадают. Следовательно, у плота будет
выступать 1/4 всего объёма, на половину диаметра бревна.
О т в е т. Плот выступает на высоту R.
47. Центр масс системы в состоянии равновесия бу-
дет находиться ровно над точкой опоры. Найдём эту точ-
190 Решения задач

ку. Выразим плотности кубиков: r1 = P1 /ga3 и r2 = P2 /gb3 .


Вес первого кубика в воде равен (r1 − r)a3 , а момент, созда-
ваемый им, есть (r1 − r)a3 (x + a/2), где x — искомая коор-
дината точки опоры. Аналогично можно написать выраже-
ние для момента, создаваемого другим кубиком. Координа-
та точки опоры выражается через условие равновесия мо-
ментов (правило рычага):
(r1 − r)a3 (x + a/2) = (r2 − r)b3 (L − x + b/2).
Разрешая уравнение относительно x, получим:
(r2 − r)b3 L 1 (r2 − r)b4 − (r1 − r)a4
x= − .
(r1 − r)a3 + (r2 − r)b3 2 (r1 − r)a3 + (r2 − r)b3

Результат имеет смысл, когда x < L. Допустим, для опре-


делённости, что вещество первого куба плотнее жидкости,
r1 > r. Тогда должно быть выполнено: r2 > r + (r1 − r)(a4 /b4 ).
В обратном случае следует обратить знаки.
О т в е т. Решение задачи существует, когда либо
P1 > rga и P2 /gb3 > r + (P1 /ga3 − r)(a4 /b4 ), либо P1 < rga3
3

и P2 /gb3 < r + (P1 /ga3 − r)(a4 /b4 ). В этих случаях система


находится в равновесии, если её опора расположена в точке
Q1 1 aQ1 − bQ2 P1 P2
x=L + , где Q1 = − a3 , Q2 = − b3 .
Q1 + Q2 2 Q1 + Q2 gr gr

48. Вода будет переливаться через верхний край труб-


ки, если её объём в нижнем сосуде больше, чем может вме-
стить узкая трубка, S0 H > Sh. (В данном случае это соотно-
шение выполняется). Объём воды, вытесненный из широко-
го сосуда равен (S0 − S)vt. Объём воды, текущий по трубке
есть Sut, где u — скорость воды в трубке, t — время дви-
жения поршня. Объём воды, вытесненной из нижнего со-
суда и протекшей в трубку, равен Sut = (S0 − S)vt, откуда
u = (S0 /S − 1)v. При некотором t = T верхний уровень воды
достигнет верхнего края трубки, тогда (S0 − S)vT = Sh.
О т в е т. Скорость течения воды в трубке u = v(S0 /S − 1).
Вода начнёт переливаться через верхний край трубки через
t > T = 0,11 сек.
49. Первая бусинка опустится на чашку весов спустя
Dt = L/v = 2 сек. Остальные бусинки будут опускаться через
Решения задач районных туров 191

К решению задачи 50

тот же интервал времени. В течение первых Dt = 2 сек дви-


жения нити весы показывают 0. Спустя 2Dt, весы покажут
массу m0 одной бусинки. Спустя 3Dt, весы покажут массу
2m0 , и т. д. Через 10 сек весы покажут массу пяти бусинок,
значит масса одной бусинки равна m0 = 3 г.
О т в е т. К концу 10-ой секунды весы будут показывать
15 г. Масса одной бусинки равна 3 г. Объём одной бусинки
равен V0 = m0 /r = M/5r = 1 см3 .
50. В начальный момент t = 0 сила натяжения прово-
локи равна весу двух слипшихся свинцовых цилиндров,
F0 = 2mg = 4 Н. По мере равномерного засыпания песка в
мешок в течение 60 сек, сила натяжения проволоки уве-
личивается прямо пропорционально массе песка до значе-
ния F = F0 + Mg = 104 Н. После отрыва нижнего цилиндра
от верхнего в конце 60-ой секунды, сила натяжения про-
волоки мгновенно падает до значения F1 = F0 /2, поскольку
подвешенным останется только один цилиндр.
О т в е т. График силы натяжения от времени приведён
на рисунке.
51. По определению давление равно силе отнесённой к
площади контакта тел: P = F/S = mg/S. Тогда давление сне-
гохода равно P1 = 5 кПа. При ходьбе человек большее время
опирается только на одну ногу. При этом давление на снег
больше, чем когда он стоит на двух ногах. Давление чело-
века массой 80 кг при указанной площади подошвы оказы-
вается равным P2 = 32 кПа. Снегоход утрамбовывает снег
192 Решения задач

до определённой плотности, если приложить большее дав-


ление, то снег продолжит сжиматься, и человек будет про-
валиваться.
О т в е т. Двигаясь по следу снегохода, человек будет
проваливаться в снег.
52. Выпишем условие равновесия системы в воде,
l l
(m1 g − F A1 ) = (m2 g − F A2 ) .
2 2
Поскольку архимедовы силы, действующие на шарики, рав-
ны F A1 = V1 (r1 − rВ ) и F A2 = V2 (r2 − rВ ), легко найти отноше-
ние объёмов этих шариков:
V2 r 1 − rВ
= .
V1 r 2 − rВ
С другой стороны, условие равновесия той же системы в
воздухе есть g · lm1 = g(l − 2x) · m2 , поэтому отношение объё-
мов шариков можно выразить в виде: V2 /V1 = r1 · l/r2 (l − 2x).
Сравнивая оба равенства, получаем: x = l · rВ (r1 − r2 )/2r2 (r1 − rВ ) = 10 см.
О т в е т. Шарик надо передвинуть на 10 см.
53. Обозначим за m — массу поршня, S1 — пло-
щадь сечения широкой части трубки A, S2 — площадь
сечения узкой части трубки A. Рассмотрим момент, ко-
гда поршень начинает приподниматься. На поршень дей-
ствует сила давления воздуха сверху p0 S1 , сила тяжести
mg, сила давления воды снизу, равная (rg(h − x) + p0 )S2 .
Тогда условие равновесие поршня в предельном случае:
(rg(h − x) + p0 )S2 = p0 S1 + mg,
(rg(h − x) + p0 )S2 = p0 S1 + mg ⇒ mg = rg(h − x)S2 + p0 (S1 − S2 ).
Пусть новый уровень воды в трубке B равен h , а поршень
поднялся на высоту y относительно исходного положения
(см. рисунок). Так как суммарный объём воды сохраняет-
S
ся, то, очевидно, (h − h )S2 = yS1 ⇒ y = (h − h ) 2 . Равенство
S1
mg
давлений внизу трубок A и B: p0 + + rg(y + x) = p0 + rgh .
S1
Подставляя полученные выражения для y и mg, можно вы-
разить h :
   
 1 p0 1
h= 2hS2 − − x (S1 − S2 ) = [10 · 3 − (10 − 1) · 1] м = 3 м.
S1 + S2 rg 7
Решения задач районных туров 193

К решению задачи 53

О т в е т. Вода в трубке установится на высоте 3 м.


54. Рассмотрим стержень как рычаг, находящийся в
равновесии, с точкой опоры, находящейся в его центре (в
точке приложения силы тяжести). Пусть расстояние меж-
ду точками крепления первой и второй пружиной равно
L = 10 см. Тогда плечо силы F1 = k1 Dx1 , действующей на
стержень со стороны первой пружины равно 2L = 20 см.
Плечо силы F2 равно L, F3 — 2L. Тогда по правилу рычага:
k1 Dx1 · 2L + k2 Dx2 · L = k3 Dx3 · 2L. Отсюда просто найти
(k3 Dx3 − k1 Dx1 ) · 2 2(1 · 10 − 1 · 5)
Dx2 = = см = 5 см.
k2 2

О т в е т. Растяжение второй пружины равно 5 см.


55. Пусть подвижный блок передвинули на расстояние l
вниз (см. рисунок), совершив таким образом работу Fl. Тело
на наклонной плоскости при этом сдвинулось на 2l и под-
нялось на высоту h, которую просто определить из подобия
H
треугольников: h = 2l . Так как трение в системе отсутству-
l
ет, то совершённая
 работа равна изменению энергии тела:
H
mg 2l = Fl. Отсюда просто определить F.
l
H
О т в е т. F = 2mg .
l
56. Сначала получим ответ на первый вопрос задачи.
Если положить груз в чашку A, то пружина № 2 при этом
не растянется. Таким образом, чашка A должна оказать-
ся на высоте 30 см от пола. Для этого удлинение пружи-
ны № 1 должно быть равно Dl = (2 м − 0,3 м) − 0,3 м = 1,4 м.
Определим, чему должна быть равна масса груза в чашке
194 Решения задач

К решению задачи 55

A, чтобы произошло такое удлинение пружины. Пружина


удлиняется под действием силы тяжести, действующей на
массивную чашку m A и груз M: (m A + M)g = k1 Dl. Отсюда
находим, что M = 2 кг. б) Теперь посмотрим, что будет, ес-
ли положить груз в нижнюю чашку. В этом случае растя-
нутся уже две пружины. Подсчитаем удлинение каждой из
(m + M)g
них в отдельности: Dl1 = A (всё аналогично первому
k1
Mg
случаю); Dl2 = (удлинение второй пружины происходит
k2
под действием силы тяжести, приложенной только к гру-
зу). Чтобы достать до пола, чашка B должна опуститься на
Dl = 1,4 м, как и в предыдущем случае. Получаем уравнение
mA + M M Dl
+ = . Решая его, находим M.
k1 k2 g
О т в е т. Чтобы чашка достала пола, на неё надо поло-
жить груз 1,3 кг.
57. Обозначим удельную теплоту сгорания топлива —
q. По определению КПД, h = A/mq. Таким образом, совер-
шённая работа A = mqh. После устранения неисправности
получится КПД h1 = 0,95h ≈ 19%.
О т в е т. После устранения протечки, КПД h1 ≈ 19%.
58. Общий расход топлива за 7 часов Q складывается
из трёх частей: расход топлива в первые 2 часа, в третий
час и в последние четыре часа пути: Q = Q1 + Q2 + Q3 . Под-
считаем каждую из этих величин. В первые два часа ав-
томобиль прошёл путь s1 = v1 t1 = 30 км/ч · 2 ч = 60 км. Так
как он двигался со скоростью 30 км/ч, то расход состав-
Решения задач районных туров 195

лял 7 л/100 км. Таким образом, за первые два часа пути он


60 км
потратил Q1 литров топлива: Q1 = 7 л · . Аналогично,
100 км
50 км/ч · 1ч 70 км/ч · 4ч
Q2 = 4 л· =2
л. Наконец, Q3 = 6 л · л. = 16,8
100 км 100 км
О т в е т. В бензобаке автомобиля осталось 50 л − (Q1 + Q2 + Q3 ) = 50 л − (4,2 л
литров топлива.
59. Всё время пока вода течёт по трубе мощность нагре-
вателя идёт на её нагрев. Поэтому в первом случае:
C pV (t1 − t p ) = Pt,

где V — объём бассейна, r = 1000 кг/м3 — плотность воды,


C — теплоёмкость воды. Отсюда
Pt 106 Дж/ сек · 21 · 60 сек
V= = = 20 м3 .
Cr(t1 − t p ) 4,2 кДж/ кг · град · 103 кг/м3 · 15 ◦ C

Пусть во втором случае изначально был заполнен объём V1 ,


тогда:
Cr(V − V1 )(t2 − t p ) + CrV1 (t2 − t0 ) = Pt, где t2 = 25 ◦ C, t0 = 15 ◦ C.
Отсюда
(t2 − t p ) Pt
CrV (t2 − t p ) + CrV1 (t0 − t p ) = Pt ⇒ V1 = V − = 10 м3 .
(t0 − t p ) (t0 − t p )

О т в е т. В бассейне содержалось 10 м3 воды.


60. Бидон можно например охладить в холодильнике
(керосин при этом не замерзнет), а затем вынуть и поста-
вить в комнате. Бидон запотеет как раз по уровню керосина
в нём, так как металлические стенки хорошо проводят теп-
ло и их температура равна температуре холодного керосина
там, где они соприкасаются, и совпадает с комнатной тем-
пературой над уровнем керосина. На холодных стенках бу-
дет конденсироваться влага из воздуха и уровень керосина
в бидоне будет чётко виден.
61. При попадании пули массы M в сугроб её кине-
тическая энергия падает от MV 2 /2 до нуля. Кинетическая
и тепловая энергии пули расходуются на плавление снега:
MV 2 /2 + McDT = lm. Отсюда находим массу снега m, кото-
рая может быть растоплена пулей.
196 Решения задач

О т в е т. Масса растопленного снега равна примерно


9,52 г.
62. Разница давлений пропорциональна разнице высот,
Dp = rgh. Откуда следует, что h = (p2 − p1 )/rg = 31,5 м.
О т в е т. Второе показание барометра снято на девятом
этаже.
63. После подвода тепла Q часть льдинки массой
m1 = Q/l = 0,094 кг = 94 г растаяла. Оставшаяся масса льда
m2 = m − m1 = 100 − 94 = 6 г. Обозначим массу дробинки за
mc тогда её объём равен Vc = mc /rc = mc /11,35 см3 (в этой
задаче плотность удобно считать в г/см3 , а объём в см3 ).
Объём самой льдинки равен V2 = m2 /rл = 6/0,8 = 7,5см3 . На
плаву вес тела равен весу вытесненной им воды (закон Ар-
химеда), и часть тела находится над поверхностью воды.
Поэтому, объём вытесненной воды равен суммарному объ-
ёму льдинки и дробинки. Тогда закон Архимеда запишется
в виде (m2 + mc )g = rв g(V2 + Vc ), откуда получаем
 
mc
6 + mc = 1 · 7,5 + .
11,35

Решая это уравнение, найдём mc .


О т в е т. Масса дробинки равна примерно 1,64 г.
64. Количество теплоты Q, отданное холодильником
в окружающее пространство, равно мощности теплоотдачи
P, умноженной на время T = 1 ч, Q = P · T. Электрическая
энергия Eсеть , полученная холодильником от сети равна по-
требляемой мощности Pсеть , умноженной на время T = 1 ч,
Eсеть = Pсеть · T. Количество теплоты Qохл , отданное охладив-
шейся водой равно:
Qохл = cm(t1 − t2 ) = 151,2 кДж.
Холодильник забирает энергию у воды и у сети, а отдаёт
энергию окружающей среде. Поэтому количество теплоты
Q, отданное холодильником в окружающее пространство,
равно электрической энергии, полученной от сети, плюс
количеству теплоты, полученному от охлаждённой воды.
Q = Eсеть + Qохл , тогда
Qохл 151200
Pсеть · T = PT − Qохл и Pсеть = P − = 300 − = 258 Вт.
T 3600
Решения задач районных туров 197

К решению задачи 65

О т в е т. Холодильник потреблял из сети мощность


258 Вт.
65. В течение первой минуты зависимость количества
отведённого тепла Q от времени линейная: Q(t) = P1 t. Все-
го, за первую минуту, было отведено Q1 = P1 t тепла. Потом
тепло отводилось ещё быстрее, и зависимость суммарного
отведённого количества теплоты от времени

Q(t) = Q1 + (P1 + P2 ) · (t − T).

Масса образовавшегося льда равна m(t) = Q(t)/l. При этом


за первую минуту образовалось

Q1 /l = 60000/340000 = 0,176 кг

льда, а за каждую следующую минуту прирастало по

(P1 + P2 )T/l = 120000/340000 = 0,352 кг.

О т в е т. График роста массы образовавшегося льда от


времени.
66. Средний модуль скорости — это отношение прой-
денного точкой пути к затраченному времени. Рассмотрим
движение вершины колеса на протяжении одного оборота.
На рисунке последовательные положения одной точки обо-
значены одинаковыми буквами. В течение одного оборота
198 Решения задач

К решению задачи 66

вершина A описывает две дуги, каждая из которых состав-


ляет 1/3 целой окружности и имеет радиус, равный длине
стороны колеса a. Таким образом, за один оборот путь точ-
ки A составляет 4pa/3. Поскольку за один оборот колесо
смещается на расстояние 3a, весь путь состоит из s/3a обо-
ротов. Значит, полный путь точки A равен 4ps/9, и средний
модуль скорости v = 4ps/9t.
О т в е т. Среднее значение модуля скорости точки, рас-
положенной в вершине колеса, равно v = 4ps/9t.
67. Обозначим искомую скорость v. Тогда пассажир на
поднимающемся эскалаторе имеет начальную скорость v,
направленную вверх по эскалатору. Через некоторое вре-
мя он вернётся в точку, в которой пассажиры поравнялись,
имея при этом скорость v, направленную вниз по эскала-
тору, т. е. окажется в такой же ситуации, как другой пасса-
жир. Поэтому время, которое он затратит на спуск, начиная
с этого момента, будет таким же, как у другого пассажира.
Таким образом, Dt — это время, через которое пассажир
вернётся в точку встречи. Поскольку проекция его скоро-
сти на эскалатор за это время изменится на 2v, aDt = 2v и
искомая скорость равна aDt/2.
Приведём также стандартное решение задачи. Обозна-
чим искомую скорость v, расстояние от точки встречи пас-
сажиров до основания эскалаторов L, времена движения
пассажиров t1 и t2 . Начальные скорости пассажиров рав-
ны v и направлены в противоположные стороны. Запишем
уравнения движения: L = vt1 + at21 /2 и L = −vt2 + at22 /2. При-
равняем правые части: vt1 + at21 /2 = −vt2 + at22 /2, v(t1 + t2 ) = a(t22 − t21 )/2,
откуда v = a(t22 − t21 )/2(t1 + t2 ) = a(t2 − t1 )/2 = aDt/2.
О т в е т. Скорость эскалаторов равна aDt/2.
Решения задач районных туров 199

68. Кузьма преодолел расстояние V · t = pR, двигаясь со


скоростью V по окружности радиуса R, t — время меж-
ду встречами Кузьмы и Матвея. Матвей пробежал за это
же время расстояние V · t + at2 /2 = 2R. Исключая время из
двух уравнений, получаем равенство pR + ap2 R2 /2V 2 = 2R,
из которого следует, что Матвей двигался с ускорением
a = 2(2 − p)V 2 /p2 R < 0, в то время как ускорение Кузьмы
равно V 2 /R.
О т в е т. Отношение ускорений Кузьмы и Матвея есть
p /2(p − 2) ∼ 4,32.
2
69. Выберем за начало отсчёта точку в пространстве,
где находился бомбардировщик в момент, когда он сбрасы-
вал 9-ую бомбу. Тогда расстояние между точками разры-
вов 9 и 11 бомб может быть определено следующим обра-
зом: DS = L + S2 − S1 , где L — расстояние между точками,
в которых находился самолёт в момент сбрасывания 9 и 11
бомб, S2 — горизонтальное смещение 11-ой бомбы за вре-
мя её полёта до земли, S1 — горизонтальное смещение 9-ой
бомбы за время её полёта до земли. В горизонтальном на-
правлении бомбы двигаются равномерно: S = V · T, со ско-
ростями, равными скорости самолёта в момент сбрасыва-
ния бомбы: S1 = V1 · T, S2 = V2 · T, где V1 — скорость бомбар-
дировщика в момент сбрасывания 9 бомбы, V2 — скорость
бомбардировщика в момент сбрасывания 11 бомбы, а T —
время падения бомб на землю. Так как самолёт движется
горизонтально, то время T не зависит  от скорости само-
2H
лёта и одинаково для всех бомб, T = . Следовательно,
g
a(2t)2
V1 = V0 + a · 8t, V2 = V0 + a · 10t, L = V1 (2t) + . Откуда по-
 2
лучаем: DS = 2V0 t + 18at2 + 2at 2H/g.
О т в е т. Расстояние между точками разрывов 9-ой и
11-ой бомб равно 129 м.
70. Обозначим скорости танков v1 , v2 , v3 , расстояние
между военной частью X и городом Y за S, а время дви-
200 Решения задач

жения первого танка за t. Тогда:

S = v1 t, (1)
S = v2 (t + 1 ч), (2)
S = v3 (t + 2 ч). (3)

Приравняв правые части (1) и (2), найдём t = 2 ч. Подставим


t в (1) и найдём S = 60 км. Подставим t и S в (3) и найдём
v3 = 15 км/ч.
О т в е т. Скорость третьего танка равна 15 км/ч.
71. Дистанция между кондукторами от времени показа-
на на рисунке. Кондуктор проходит вагон за 30 сек. Каж-
дые 30 сек скорость кондуктора первого вагона относитель-
но земли меняется скачком с 1,67 м/сек до 3,33 м/сек и
обратно. В те же моменты времени скорость кондуктора
второго вагона относительно земли меняется скачком от
1,94 м/сек до 3,61 м/сек. Через 90 сек кондукторы будут
находиться на задних площадках трамваев. За это время
первый трамвай пройдёт 2,5 м/сек · 90 сек = 225 м, а второй
2,78 м/сек · 90 сек = 250 м. Тогда расстояние между трамва-
ями будет 25 м, а между кондукторами 75 м. Кондукто-
ры начнут движение с хвостов к кабинам со скоростями
3,33 м/сек и 3,61 м/сек относительно земли, т.е̇. их ско-
рость сближения будет равна 6,94 м/сек. Тогда они порав-
няются ещё через 75 м/6,94 м/сек = 10,8 сек. На 90-ой се-
кунде кондуктор первого трамвая находился в 50 м от се-
редины пути. За 10,8 сек он продвинулся на 36 м, т. е. не
доехал 14 м до середины пути. Траектории кондукторов на
рисунке пересекаются через 100,8 сек после начала пути, на
расстоянии 14 м от середины пути к месту старта первого
трамвая.
О т в е т. Кондукторы трамваев встретятся с друг другом
через 101 сек после начала движения.
72. Выберем систему отсчёта, связанную с землей. То-
гда, первый автомобиль пройдёт до начала движения вто-
рого расстояние S = Vt. В системе отсчёта, связанной с пер-
вым автомобилем, второй автомобиль должен был пройти
до встречи то же самое расстояние, причём его скорость
движения была относительно земли была 3V , значит отно-
Решения задач районных туров 201

К решению задачи 70

сительно первого автомобиля будет 3V − V = 2V . Отсюда по-


лучается, что Vt = 2VT и, следовательно, t = 2T.
О т в е т. Вторая машина догонит первую через 1 час.
73. При равноускоренном движении средняя скорость
равна среднему арифметическому начальной и конечной
скорости. Тогда путь первого вагона равен S1 = (V0 + V1 )t1 /2,
где V1 = V0 + at1 — скорость электрички в момент, когда
первый вагон покинул туннель, а последний вагон въе-
хал в туннель, t1 — время, в течение которого первый ва-
гон был в туннеле. Путь второго вагона S2 = (V2 + V1 ) · t2 /2,
где V2 = V1 + at2 — скорость электрички в момент, когда
второй вагон покинул туннель, а t2 — время, в тече-
ние которого последний вагон был в туннеле. Поскольку
оба пути одинаковы и равны длине туннеля, выполняет-
ся равенство S1 = S2 . Вспомним, что t1 = 2t2 , выразим t1
через t2 , подставим в V1 и V2 и получим: V1 = V0 + 2at2 ,
V2 = V0 + 3at2 . Затем подставим все в S1 = S2 и получим:
(2V0 + 2at2 ) · 2t2 = (2V0 + 5at2 )t2 , откуда at2 = 2V0 , а это уже
подставим в выражение для V2 и получим ответ.
О т в е т. Скорость электрички, когда она покинула тун-
нель, была равна V2 = 7V0 .
74. Проекция скорости шайбы на ось, направленную
вдоль борта остаётся неизменной, так как в этом направ-
лении на неё не действуют никакие силы (в момент соуда-
рения шайбы о противоположный борт сила реакции борта
действует перпендикулярно борту). Чтобы шайба вернулась
202 Решения задач

К решению задачи 74

к хоккеисту, его скорость должна быть равна проекции ско-


рости шайбы, Ox: V = u cos a.
О т в е т. Угол между векторами V и u таков, что
cos a = V /u.
75. Мысленно проведём прямую l вдоль углов ступе-
нек. Будем искать пересечение траектории шайбы именно
с этой прямой, так как если шайбы пересекла прямую l, то
она обязательно упадёт именно на ступеньку под точкой пе-
ресечения. По оси X тело движется равномерно, x = ut, а
по Y равноускоренно, y = gt2 /2. Пусть x и y — координа-
ты пересечения траектории с прямой l, а t — время до пе-
gx2
ресечения, тогда t = x/u и подставим y = 2 . Преобразуем:
2u
h y gx 2u2 h
= tg a= = , откуда найдём x = = 7,5 м. В ступень-
a x2u2 g a
x 7,5
ках N= = = 18,75. Значит шайба перелетит полностью
a 0,40
через 18,75 ступенек, т. е. упадёт на 19-ой.
О т в е т. Шайба упадёт на 19-ой ступеньке.
76. Так как платформа двигалась в момент выстрела,
снаряд будет лететь по параболической траектории. Изме-
нение скорости платформы за время полёта снаряда равно
половине её первоначальной величины. Отсюда легко на-
ходится само время полёта снаряда и искомое расстояние.
Обозначим скорость платформы в момент выстрела через
v, скорость снаряда в проекции на направление движения
платформы также будет равна v. Искомое расстояние L да-
ётся формулой L = vt, где время полёта t вычисляется по
формуле t = 2v0 /g. С другой стороны, за это время проис-
ходит уменьшение скорости платформы вдвое, v/2 = v − at,
т. е. v = 2at.
Решения задач районных туров 203

К решению задачи 77

О т в е т. Расстояние полёта снаряда равно L = 8av20 /g2 .


77. Свет можно представить как поток частиц, в кото-
ром тарелка прочерчивает дорожку тени. Перейдём в систе-
му отсчёта, в которой «частицы света» неподвижны, где эта
дорожка, очевидно, будет прямой. Следовательно, она будет
прямой и в исходной системе отсчёта. Рассмотрим два поло-
жения тарелки: в момент старта и через некоторое время t.
Угол g между дорожкой и траекторией тарелки находим из
прямоугольного треугольника, стороны которого равны Vt
и Ct; tg g = C/V . Обозначив за u искомую скорость тени, по-
лучим треугольник со сторонами ut и Vt (см. рисунок) и уг-
лами a и g. Опустив высоту на верхнюю сторону треуголь-
ника, получаем ut(cos a + sin a/ tg g) = Vt. Сократив время
t, приходим к ответу.
О т в е т. Тень летающей тарелки движется по земле со
скоростью u = VC/(C cos a + V sin a).
78. В системе отсчёта земли векторы скоростей вагона
и игрушки складываются: V = V 1 + V 2 . Длина результиру-
ющего вектора находится с помощью теоремы Пифагора,
V = (V 21 + V 22 )1/2 ∼ 10,05 м/c.
О т в е т. Скорость игрушки относительно земли равна
примерно 10,05 м/c.
79. Если пренебречь массой верёвки, то на лодки с
её стороны действуют одинаковые силы: F1 = −F2 . Так как
массы лодок тоже одинаковы, то одинаковы и ускорения
a1 = −a2 . Двигаясь с этим ускорением, лодки проходят путь
L/2 (по симметрии задачи, этот путь, очевидно, будет оди-
наков для обеих лодок) за некоторое время t, L/2 = at2 /2.
Отсюда легко находим искомое время t = (L/a)1/2 .
204 Решения задач

К решению задачи 81

О т в е т. Лодки столкнутся через время t = (L/a)1/2 .


80. Скорость камня непосредственно после отрыва от ве-
рёвки будет направлена по касательной к окружности, по
которой он до этого двигался. Дальнейшая траектория кам-
ня представляет собой параболу, причём, камень в момент
отрыва находился в верхней точке параболы. Его горизон-
тальная скорость V постоянна, а горизонтальное перемеще-
ние равно S = Vt, где t = (2h/g)1/2 — время падения камня с
высоты h. Расстояние от мальчика до точки падения камня
можно найти по теореме Пифагора: d = (R2 + 2h · V 2 /g)1/2 .
О т в е т. Расстояние от точки на земле, где стоит маль-
чик, до точки падения камня равно (R2 + 2h · V 2 /g)1/2 .
81. Солнце находится далеко от поверхности Земли
(много дальше облаков), поэтому тени облаков на земле по-
вторяют их форму и движутся с теми же скоростями, что
сами облака. Скорости же облаков совпадают со скоростью
ветра на соответствующих высотах. Найдём вектор скоро-
сти движения точки пересечения теней облаков прямым по-
строением. Выберем единичный интервал времени, чтобы
векторы скоростей совпадали с соответствующими векто-
рами перемещений. Векторы, задающие новый смещённый
фронт тени, строим из точек, находящихся на границах те-
ней в предыдущий момент времени. Через концы векторов
проводим новые края параллельно начальным. Новые края
пересекаются в новой точке пересечения O . Тогда вектор
OO , от старого положения точки пересечения к новому по-
ложению и будет представлять вектор скорости движения
этой точки.
О т в е т. Вектор скорости движения точки пересечения
теней представлен на рисунке.
Решения задач районных туров 205

К решению задачи 81

82. Вагон, равномерно движущийся по окружности, яв-


ляется неинерциальной системой отсчёта, поэтому ускоре-
ния игрушки относительно земли aнач (aкон ) будут скла-
дываться из aнач (aкон ) в системе отсчёта вагона и соб-
ственного ускорения вагона. Ускорение вагона aц — цен-
тростремительное ускорение равно — aц = V 2 /R = 0,2 м/с2 ,
где V — скорость вагона, а R — радиус кругового участ-
ка железнодорожного пути. Абсолютные величины началь-
ного и конечного ускорений игрушки относительно зем-
ли вычисляются по теореме Пифагора: aнач = (a2ц + a2нач )1/2
и aкон = (a2ц.б. + a2кон )1/2 . Направления этих ускорений суть
cos a = aнач /aнач и cos b = aкон /aкон .
О т в е т. Абсолютные величины начального и ко-
нечного ускорений игрушки относительно земли суть
aнач = 0,728 м/с2 и aкон = 0,539 м/с2 . Направляющие ко-
синусы этих ускорений равны cos a = 0,962 и cos b = 0,929.
83. Рассмотрим положение равновесия. На нижний
блок действует вес груза Mg, направленный вниз, и две
силы натяжения нити T, направленные вверх. Посколь-
ку блок находится в равновесии, T = Mg/2. Следователь-
но, пружина 2 растянута на x2 = Mg/2k2 . Аналогично, пру-
жина 1 растянута с силой Mg/4, и её удлинение равно
x1 = Mg/4k1 . Поскольку пружина 1 удлинилась на x1 , верх-
ний блок опустился на x1 /2. Далее, поскольку верхний
блок опустился на x1 /2, а пружина 2 удлинилась на x2 ,
нижний блок опустился на (x1 /2 + x2 )/2 = x1 /4 + x2 /2. Под-
ставляя значения x1 и x2 , получаем искомую величину
h = Mg/16k1 + Mg/4k2 .
О т в е т. Нижний блок сместился на высоту h = 15 см.
206 Решения задач

К решению задачи 83

К решению задачи 84

84. На поршень действуют три силы сила давления воз-


духа F∂1 , сила давления воды F∂2 и сила F, с которой его
удерживают. Сила давления воздуха равна F∂1 = pa S, где
pa — атмосферное давление, а S = a2 — площадь порш-
ня. Давление воды на поршень неодинаково в разных его
частях, но общая сила давления может быть вычислена
как F∂2 = pср S, где pср — среднее давление воды (давление
в середине поршня). Поскольку на уровне h от дна дав-
ление равно атмосферному, pср = pa + rg(h − a/2). Посколь-
ку h > a/2, сила F∂2 больше F∂1 , и для удержания порш-
ня в равновесии необходимо прикладывать к нему силу,
направленную вправо. Сила F определяется из уравнения
F + F∂1 = F∂2 и равна F = rga2 (h − a/2).
О т в е т. Для удержания поршня в равновесии необходи-
мо прикладывать к нему силу 20 Н, направленную вправо.
85. Поскольку все грузы находятся в равновесии, сила
натяжения каждой нити равна силе тяжести соответствую-
Решения задач районных туров 207

К решению задачи 85

щего груза. Поскольку узелок, которым связаны нити, так-


же находится в равновесии, приложенные к нему силы на-
→ −
− → − →
тяжения скомпенсированы: T 1 + T 2 + T 3 = 0. Спроецируем
это равенство на оси x и y:

− T1 sin 45◦ + T2 sin 30◦ = 0,


T1 cos 45◦ + T2 cos 30◦ − T3 = 0.

Так как T1 = m1 g, T2 = m2 g и T3 = m3 g (m1 , m2 и m3 массы


левого, правого и среднего грузов соответственно), эти урав-
нения можно переписать в виде

m1 sin 45◦ + m2 sin 30◦ = 0,


m1 cos 45◦ + m2 cos 30◦ − m3 = 0.

и из них найти m2 = m1 sin 45◦ / sin 30◦ , m3 = m1 (cos 45◦ + sin 45◦ / tg 30◦ )
или m2 = 1,41 кг, m3 = 1,93 кг.
О т в е т. Масса среднего груза 1,93 кг, масса правого
груза 1,41 кг.
86. Добавочное давление, создаваемое столбом воды вы-
соты h, равно r0 gh, поэтому давление на глубине h равно
pa + r0 gh. Сила, с которой вода прижимает присоску к по-
верхности, равна F = (pa + r0 gh)S (S — площадь присоски).
Чтобы брусок держался, необходимо, чтобы эта сила была
не меньше, чем равнодействующая сил Архимеда и тяже-
сти, действующих на брусок. Объём бруска V = m/r, поэто-
му сила Архимеда равна Fa = r0 gV = (r0 /r)mg.
Итак: F > Fa − mg, (pa + r0 gh)S > (r0 /r)mg − mg, S > (r0 − r)mg/r(pa + r0 gh).
(r0 − r)mg
О т в е т. Минимальная площадь присоски равна S = .
r(pa + r0 gh)
208 Решения задач

К решению задачи 86

87. На сосуд (левый или правый) в вертикальном на-


правлении действуют сила тяжести сосуда, сила, с которой
давит атмосфера на дно сосуда, и сила, с которой вода давит
на дно сосуда. Пусть масса каждого сосуда равна m, пло-
щадь S, атмосферное давление равно P0 , а давление жидко-
сти на дне сосуда — Pж . Так как сосуд находится в равно-
весии, то:

mg + Pж S = P0 S ⇒ Pж = P0 − mg/S.

Поскольку сосуды одинаковы, то давление жидкости на дне


обоих сосудов будет одинаковым. Вода будет перетекать в
направлении того сосуда, в котором давление на уровне от-
верстия меньше. Давление на уровне дна в сосудах одина-
ково. Поскольку расстояние от отверстия до дна больше в
том сосуде, в который налито больше воды, в нём давление
на уровне отверстия будет меньше, и вода потечёт в его сто-
рону.
О т в е т. Вода будет перетекать в правый сосуд (см. рис. 34).
88. Масса пластинки равна r · Sh, где h — толщи-
на, а S — её площадь. Сила тяжести Fm = r · Shg, сила
давления FД = S · Dp и сила реакции оси N. В равновесии
проекции сил на ось перпендикулярную пластине равны:
Fm · sin a = FД . Откуда h = Dp/rg sin a.
О т в е т. Толщина пластинки, находящейся в состоянии
равновесия при разности давлений Dp равна h = Dp/rg sin a.
89. В точке, где находится альпинист, на верёвку, натя-
нутую над пропастью, действуют три силы: две равных друг
Решения задач районных туров 209

К решению задачи 89

другу и направленных вдоль верёвки силы натяжения и вес


альпиниста. Чтобы альпинист смог перебраться на другую
сторону, эти силы должны быть скомпенсированы, а си-
ла натяжения верёвки не должна превышать критическую
силу разрыва T. Выберем, для простоты рассмотрения,
ситуацию, когда альпинист находится на середине верёв-
ки: Mg + T1 + T2 = 0. Проекции этого уравнения на верти-
кальную и горизонтальную оси суть T1 cos a − T2 cos a = 0,
2T1 · sin a − Mg = 0, где cos a = H/L. Из уравнений найдём
силу натяжения T1 = Mg · L/2(L2 − H2 )1/2 = 1281H > T, сле-
довательно, канат порвётся.
О т в е т. Альпинист не сможет перебраться через про-
пасть по верёвке.
90. Перейдём в систему отсчёта «потолок», связан-
ную с точкой крепления блока. Она — неинерциальна, по-
этому, записывая уравнения движения, необходимо учесть
силы инерции. В инерциальной
 системе отсчёта II-ой за-
кон Ньютона таков: Fk = maИ , где aИ — ускорение те-
ла в инерциальной системе отсчёта. Пусть aС — уско-
рение неинерциальной системы относительно инерциаль-
ной, а aН — ускорение тела в этой неинерциальной си-
стеме,
 тогда aИ = aН + aС . Второй закон Ньютона даст нам:
Fk = m(aН + aС ). Перенеся в левую часть и введя обозначе-
ние FИ = −maС , получим: Fk + FИ = maН . То есть в неинер-
циальной системе отсчёта появляется сила инерции, ко-
торая равна минус масса тела, умноженной на ускорение
неинерциальной системы отсчёта.
В векторные уравнения будут такими: Mg + FИ1 + T = Ma,
mg + FИ2 + T = ma. Так как нить нерастяжима и остаёт-
ся всё время натянутой, то ускорения обоих тел равны
по модулю. Спроектируем эти уравнения на вертикаль-
210 Решения задач

ную ось, считая, что система движется вниз с ускорени-


ем aС : Mg − MaС − T = Ma, maС + T − mg = ma. Исключая T
из этих уравнений, найдём a = (M − m)(g − aС )/(M + m). Что-
бы оба тела имели ускорение, направленное в одну сто-
рону, тело, поднимающееся вверх в системе отсчёта «по-
толок», должно иметь отрицательное ускорение в системе
отсчёта «пол», a − aС < 0. Раскроем последнее неравенство:
(M − m)(g − aС )/(M + m) − aС < 0 или aС > (M − m)g/2M. Рас-
смотрим теперь второй случай, когда система движется с
ускорением aС1 вверх: Mg + MaС1 − T = Ma, −maС1 + T − mg = ma.
Откуда имеем a = (M − m)(g + aС1 )/(M + m). Оба тела име-
ли ускорение в одну сторону, поэтому тело, опускающее-
ся вниз в системе отсчёта «потолок», должно иметь поло-
жительное ускорение в системе отсчёта «пол»: aС1 − a > 0.
Раскроем это неравенство: aС1 − (M − m)(g + aС1 )/(M + m) > 0
или aС1 > (M − m)g/2m. Следовательно, aС < aС1 .
О т в е т. Ускорения грузов будут направлены в одну
сторону, если вертикальное ускорение (направленное вниз)
блока будет больше, чем (M − m)g/2M.
91. Лента транспортёра является инерциальной систе-
мой отсчёта. Шайба влетает на неё, имея составляющую
скорости V вдоль ленты и V поперёк ленты, т. е. началь-
ная скорость шайбы относительно транспортёра равна и на-
правлена под углом 45◦ к ленте. После шайба будет дви-
гаться относительно ленты равнозамедленно с ускорением,
равным по модулю F/m, и по прямой (под углом 45◦ к лен-
те). Чтобы шайба переехала √ транспортёр, её остановочный
путь должен быть больше 2H. Остановочный путь тела с
начальной √скоростью V1 под действием ускорения a равен
V12 ( 2V )2 √ mV 2
, тогда > 2H, т. е. √ > H.
2a 2F/m F 2
О т в е т. Чтобы шайба переехала через√ транспортёр, его
ширина не должна превосходить mV /(F 2).
2
92. Скорость растяжения пружин постоянна, значит си-
лы, действующие на них, скомпенсированы, F(t) = k1 x1 (t) = k2 x2 (t),
где x1 (t) и x2 (t) суть удлинения пружин от времени.
Последнее равенство задаёт кинематическую связь меж-
ду удлинениями и скоростями пружин: x2 = x1 · (k1 /k2 ) и
v2 = v1 · (k1 /k2 ). Свободный конец пружины движется с по-
Решения задач районных туров 211

К решению задачи 92

стоянной скоростью V = v1 + v2 , что позволяет выразить


v1 = V · k2 /(k1 + k2 ) — постоянную скорость конца первой
пружины, прикреплённого к шайбе (она же — скорость
шайбы). Соответственно, F(t) = k1 v1 t = k2 v2 t.
О т в е т. Скорость шайбы равна v1 ∼ 0,067 м/c. Зависи-
мость F(t) = 1,6t показана на рисунке.
93. Метеорологический зонд будет удерживаться, по-
ка сила атмосферного давления, действующая на пласти-
ны, превосходит Архимедову силу, поднимающую шар,
Sp0 > Vg(rВ − rНе ), где rВ — плотность воздуха.
О т в е т. Площадь пластин должна быть больше 640 см2 .
94. По условию задачи лыжник движется равномер-
но и прямолинейно, действие всех сил на него скомпенси-
ровано. Поскольку сила трения лыж о воду пренебрежи-
мо мала, таких сил всего три: сила тяжести, направлен-
ная вертикально вниз, сила реакции, действующая со сто-
роны воды на лыжи, перпендикулярная поверхности лыж
(эта сила будет иметь такой же угол с вертикалью, что и
угол между лыжами и горизонтом), а также сила натяже-
ния троса: T + N + mg = 0. Проекции этого векторного урав-
нения на горизонтальную и вертикальную оси координат
T − N sin a = 0, N cos a − mg = 0, позволяют найти тригоно-
метрическую функцию искомого угла.
О т в е т. Тангенс искомого угла равен tg a = T/mg.
95. Пусть сила натяжения нити равна T. Запишем вто-
рой закон Ньютона для обоих грузов в проекции на верти-
кальную ось и учтём, что ускорения a1 и a2 равны по моду-
лю и противонаправлены,

−m1 g + T = m1 a, −m2 g + T = −m2 a.


212 Решения задач

Вычтем из первого уравнения второе и получим a = g(m2 − m1 )/(m2 + m1 ).


Так как ускорения тел равны по модулю и начальные скоро-
сти равны нулю, то до встречи каждое тело должно пройти
h at2 
путь h/2. Тогда = , или t = h/a. Когда массы тел рав-
2 2
ны, система грузов находится в безразличном равновесии и
будет оставаться в покое. Если первый груз тяжелее второ-
го, то ответ не изменится, но a надо брать положительным.
О т в е т. Грузы окажутся на равном расстоянии от блока
g(m2 − m1 )
через t = (h/a)1/2 , где a = .
m1 + m2
96. Систему грузов можно рассматривать как один груз,
имеющий массу (m + M). Этот груз подвешен на двух нитях,
имеющих равные силы натяжения T. Проекция уравнения
сил на вертикальную ось даёт (m + M)g − 2T = 0.
О т в е т. Искомая сила натяжения нити равна T = g(m + M)/2.
97. Пробка выскальзывает из отверстия, когда T + FД = 0,
где T — сила натяжения нити, FД — сила давления стол-
ба жидкости. Сила давления даётся формулой FД = S · rgH
(S — площадь пробки) и линейно зависит от высоты стол-
ба воды и от площади пробки. Величина силы T опре-
деляется из уравнения для сил, действующих на брусок,
T + FА + mg = 0. Значение архимедовой силы FА определя-
ется глубиной hп погружения бруска в воду FА = s · rghп .
Таким образом, справедливо равенство T = s · (rghп − r1 ga),
где s = a2 — площадь сечения бруска, а условие выдёргива-
ния пробки есть S · rgH = s · (rghп − r1 ga), где H = L + hп . Нас
интересует ситуация, когда архимедова сила максимальна,
т. е. брусок полностью погружен в воду, а давление на проб-
ку минимально, т. е. вода над краями бруска не выступает.
В этом случае S · rg(L + a) = a3 (rg − r1 g).
О т в е т. Площадь пробки равна S = a3 (r − r1 )/r(L + a).
98. Будем пренебрегать размерами тела. Направим ось
OX по касательной к окружности в точке закрепления тела
и запишем уравнение сил, действующих на него, в момент
освобождения: N + mg + F = ma. В проекциях на оси это
уравнение запишется как OX: F cos(b − a) − mg sin a = ma,
OY: N + F sin(b − a) − mg cos a = 0. После  подстановки
 значе-
1 F mg
ний углов, из первого уравнения: a = √ − . Из вто-
m 2 2
Решения задач районных туров 213

рого уравнения следует, что тело оторвётся от поверхности 


шара (сила реакции N обратится в ноль) при F ≥ mg √ 3/2,
что исключается условиями задачи. Если F < mg/ 2, тело √
соскальзывает вниз по поверхности √ шара. При F = mg/ 2
тело покоится, а при mg > F > mg/ 2 двигается вверх по по-
верхности шара. 
О т в е т. Тело оторвётся от поверхности
√ шара при F ≥ mg 3/2,
иначе ускорение тела равно a = F/(m 2) − g/2.
99. Масса трубы велика, поэтому её собственным дви-
жением, вызванным взаимодействием с попавшим внутрь
неё малым телом, можно пренебречь. Трения нет, поэто-
му компонента скорости пылинки, направленная вдоль оси
трубы (vX ), и перпендикулярная ей (vY ), сохраняются.
vX = 8000 cos 30◦ H = 6928 м/с; vY = 8000 · sin 30◦ = 4000 м/с.
Продольная компонента при этом сохраняет и своё направ-
ление, а поперечная меняется как при вращении по окруж-
ности со скоростью, равной абсолютной величине попереч-
ной компоненты. Время прохождения трубы t = 10/vX ∼ 0,00144 сек
определяется только продольной скоростью пылинки, а её
поперечная скорость задаёт силу, с которой пылинка дей-
ствует на стенку трубы m · v2Y /R ∼ 4 · 104 H.
О т в е т. Тело покинет трубу через 0,00144 сек. Сила,
с которой пылинка действует на стенки трубы в процессе
движения равна 4 · 104 H.
100. Если доску массой m1 = 10 кг можно передвинуть,
приложив к ней силу в F = 50 H, то коэффициент тре-
ния для дерева, принятый в задаче, равен m = F/m1 g = 1/2.
Тогда сила натяжения нити, перекинутой через блок и
привязанной к ящику массы m2 , равна T = m · m2 g. При
равномерном движении доски сила трения, действующая
на неё со стороны пола, будет равна F1 = m(m1 + m2 )g, а
со стороны ящика — F2 = mm2 g. Доска будет равномер-
но скользить вдоль пола, если приложить к ней силу
F = T + F1 + F2 = T + m(m1 + m2 )g + mm2 g = F(1 + 3m2 /m1 ) = 650 Н.
О т в е т. Доска будет равномерно скользить вдоль пола,
если приложить к ней силу 650 H.
101. Со стороны нити на каждый шарик действует век-
торная сумма двух сил натяжения, ma = T1 + T2 . Поскольку
нить невесома, величины этих сил равны. Проекция урав-
214 Решения задач

К решению задачи 101

нения на горизонтальную ось: maX = T − T · sin a. Горизон-


тальное ускорение постоянно, значит расстояние L/2 (оди-
наковое для каждого шарика вследствие симметрии задачи)
они пройдут за время t, такое, что L/2 = aX t2 /2.
О т в е т. Шарики столкнутся через время t = (Lm/T(1 − sin a))1/2 .
102. Если нагреватель включён, то лёд начинает таять.
При этом его объём уменьшается, что и приводит к опус-
канию поршня. Пока под поршнем есть и лёд, и вода, их
температура равна 0 ◦ C. Рассмотрим промежуток времени t.
За это время нагреватель передаёт льду количество теплоты
Q = Pt (P — мощность нагревателя). Этого количества теп-
лоты достаточно, чтобы растаяла масса льда m = Pt/l. Это
приводит к уменьшению объёма на DV = m/r2 − m/r1 . Если
объём вещества под поршнем уменьшается на DV , поршень
опускается на расстояние x = DV /S = Pt(1/r2 − 1/r1 )/lS.
Следовательно, скорость поршня v и мощность нагревателя
P связаны следующими соотношениями:
 
P 1 1 lSv
v= − , P= 1 1 .
lS r2 r1 −
r2 r1

О т в е т. Скорость опускания поршня равна


−5
1,12 · 10 м/с = 0,67 мм/мин.
103. При опускании льда в доверху наполненный со-
суд часть воды выльется. Поскольку этот процесс происхо-
дит гораздо быстрее теплообмена и связанного с ним таяния
льда, будем считать, что вытекающая вода имеет темпера-
туру t1 . По закону Архимеда масса вытесненной воды равна
m. Таким образом, в сосуде находится лёд массой m (при на-
чальной температуре t0 ) и вода массой M − m (при началь-
ной температуре t1 ). Уравнение теплового баланса в сосуде
есть c(M − m)(t1 − t) = l · m + cm(t − t0 ), где t искомая темпера-
тура. Отсюда t = (c(M − m)t1 − l · m + cm · t0 )/cM.
Решения задач районных туров 215

О т в е т. В сосуде установится температура 0,7 ◦ C.


104. Тепло, выделившееся в проводниках с током, идёт
на нагревание воды в термосе. Обозначим температуру ки-
пения жидкости Tk . Работа электрического поля по пере-
мещению зарядов в проводниках за время Dt равна прира-
щению внутренней энергии жидкости, UI · Dt = cm · (Tk − T).
Выражая ток в последнем равенстве по закону Ома как
I = U/R и раскрывая R через удельное сопротивление, се-
чение и длину, получим Dt = r · Lcm · (Tk − T)/4SU 2 .
О т в е т. Вода в термосе закипит через время Dt = r · Lcm · (Tk − T)/4SU 2 .
105. Обозначим поток воды в трубах J1 и J2 . Пол-
ный поток воды, вытекающий из смесителя, равен их сум-
ме, JП = J1 + J2 . В смесителе горячая вода отдаёт часть
тепла холодной воде, в результате чего их температуры
выравниваются до некоторой t. Из уравнения теплового
баланса следует, что J1 (t1 − t) = J2 (t − t2 ). Откуда находим
t = (J1 t1 + J2 t2 )/(J1 + J2 ). Аналогичное уравнение получает-
ся из закона сохранения тепловой энергии, J1 t1 + J2 t2 = JП t.
Предыдущее равенство показывает, что при пропорцио-
нальном увеличении потоков воды в обеих трубах тем-
пература вытекающего потока не изменяется. Таким об-
разом, когда вытекает максимальный поток воды с дан-
ной температурой, один из кранов открыт полностью. Ес-
ли открыть оба крана полностью, то пойдёт поток воды
2J0 с температурой t0 = (t1 + t2 )/2. Чтобы получить мак-
симальный поток воды температуры ниже t0 , надо оста-
вить холодный кран полностью открытым, а поток воды
из горячего уменьшить; если требуется температура боль-
ше t0 , то надо, чтобы горячий кран был полностью от-
крыт. Если холодный кран открыт полностью (J1 = J0 ),
а поток воды из горячего крана равен J2 , то темпе-
ратура вытекающей воды равна t = (J0 t1 + J2 t2 )/(J0 + J2 ).
Отсюда J2 = J0 (t − t1 )/(t2 − t), и суммарный поток воды:
JП = J0 + J2 = J0 (t2 − t1 )/(t2 − t) при t1 < t < (t1 + t2 )/2. Если
горячий кран открыт полностью (J2 = J0 ), а поток во-
ды из холодного крана равен J1 , то температура воды
равна t = (J1 t1 + J0 t2 )/(J1 + J0 ). Отсюда J1 = J0 (t2 − t)/(t − t1 ),
а суммарный поток во JП = J0 + J1 = J0 (t2 − t1 )/(t − t1 ) при
t2 > t > (t1 + t2 )/2.
216 Решения задач

К решению задачи 105

О т в е т. Итоговый график потока воды (в литрах в се-


кунду) от температуры показан на рисунке. При t1 < t < (t1 + t2 )/2
Jпол = J0 (t2 − t1 )/(t2 − t); при (t1 + t2 )/2 < t < t2 Jпол = J0 (t2 − t1 )/(t − t1 ).
106. Тепловой энергии массы воды mв , выделившей-
ся при её охлаждении до 0 ◦ C, Q1 = cmв · Dt, недостаточно,
чтобы растаял весь лёд: Q2 = lmльда > Q1 . Положение спа-
сает высвободившаяся при падении потенциальная энергия
ghmв . Это позволяет оценить минимальную высоту падения
как h ∼ (Q2 − Q1 )/gmв .
О т в е т. Лёд падал с высоты h ∼ 77,14 м.
107. Как видно из графика, в калориметре происходит
два термодинамических процесса: остывание нагретого тела
и нагрев холодного. Пренебрегая потерями тепла в калори-
метре, будем считать теплоты этих процессов одинаковыми.
Тогда имеется возможность найти искомое отношение масс.
В самом деле, количество теплоты, выделяющееся за 8 мин
при остывании более нагретого тела, равно Q1 = cm1 · 9 ◦ C.
Количество теплоты, поглощаемое за 8 мин более холодным
телом, равно Q2 = cm2 · 3 ◦ C. В это время в калориметре на-
блюдается равенство теплот Q1 = Q2 , дающее искомое отно-
шение масс.
О т в е т. Отношение масс тел, помещённых в калори-
метр, равно m1 /m2 = 3 ◦ C/9 ◦ C = 1/3.
Решения задач районных туров 217

108. Обозначим напряжение батареи U, а сопротивле-


ние резисторов R. Мощность, выделяющаяся при подклю-
чении к батарее одного резистора, равна U 2 /R. Если тем-
пература резистора установилась, то вся эта мощность рас-
сеивается в окружающее пространство. По условию мощ-
ность теплоотдачи пропорциональна разности температур
резистора и окружающей среды обозначим коэффициент
пропорциональности a. Тогда U 2 /R = a(t1 − t0 ). При после-
довательном подключении двух резисторов сопротивление
цепи будет равно 2R, поэтому выделяющаяся мощность
U 2 /2R. Поскольку резисторы совмещены торцами, тепло-
отдача может происходить только с двух свободных тор-
цов. Таким образом, условия теплоотдачи не изменятся,
и её мощность будет равна a(t2 − t0 ) (t2 — температура в
этом случае): U 2 /2R = a(t2 − t0 ). Отсюда a(t1 − t0 ) = 2a(t2 − t0 )
и t2 = (t0 + t1 )/2 = 29 ◦ C.
О т в е т. Резисторы нагреются до 29 ◦ C.
109. Сила взаимодействия заряженных бусинок F на-
правлена горизонтально, а сила тяжести mg — вертикаль-
но. Их уравновешивает сила натяжения нити T, направлен-
ная противоположно их векторной сумме, вдоль нити. Обо-
значим искомое расстояние между бусинками h. Из подо-
бия треугольников, отношение расстояния бусинки от оси
симметрии d/2 к разности высот шарика и подвеса h рав-
но отношению F и mg: F/mg = d/2h. С другой стороны, d/2
связано с h по теореме Пифагора: (d/2)2 + h2 = L2 . Выражая
h через d, получаем уравнение, из которого находим рассто-
яние d:
F d
= p 2 2 .
mg 2 L − d /4
О т в е т. Бусинки находятся друг от друга на расстоянии
2L
d= p 2 2 2 .
m g /F + 1
110. До замыкания ключа сила тока через всё рези-
сторы одинакова, поскольку в параллельно соединённых
участках цепи выделяется одинаковая мощность при од-
ном и том же напряжении. Следовательно, сопротивле-
ния резисторов, на которых выделяется одинаковая мощ-
ность, равны. Мощности, выделяющиеся на резисторах
суть P1 = I2 · R1 и P2 = I2 · R2 . Напряжения на них равны
218 Решения задач

К решению задачи 110

U1 = IR1 и U2 = IR2 . При этом суммарное напряжение рав-


но U = U1 + U2 . Отсюда получается выражение для тока
I = (I2 · R1 + I2 · R2 )/(IR1 + IR2 ) = (P1 + P2 )/U = 0,75 А в каж-
дом из параллельно соединённых участков. Сопротивления
резисторов R1 = P1 /I2 = P1 · U 2 /(P1 + P2 )2 ∼ = 178 Ом, R2 = P2 · U 2 /(P1 + P2 )2 ∼
= 89 Ом
После замыкания ключа, напряжения на всех резисторах
одинаковы и равны U/2. Отсюда можно найти мощности,
выделяющиеся на резисторах: P1 = (U/2)2 /R1 = (P1 + P2 )2 /4P1 ,
P2 = (P1 + P2 )2 /4P2 .
О т в е т. На резисторах выделяются мощности P1 = 56,25 Вт
и P2 = 112,5 Вт.
111. Сила, действующая на шарик, равна F = q · E. Уско-
рение шарика есть a = F/m = q · E/m. Шарик не ударится о
потолок, если v2 /2a < H, т. е. v < (2q · EH/m)1/2 . В этом слу-
чае время полёта t = 2v/a = 2m · v/q · E. Если шарик ударится
в потолок, то t = 2[v − (v2 − 2a · H)1/2 ]/a.
О т в е т. График зависимости времени, за которое ша-
рик упадёт на пол, в зависимости от его начальной скорости
приведён на рисунке. Максимальное время падения шарика
равно 1,55 сек при скорости 7,75 м/с.
112. Обозначим полный ток через схему I, напряжение
на всей схеме U, напряжение на левом элементе U1 , напря-
Решения задач районных туров 219

жение на правом элементе U2 . Представим, что мы увели-


чиваем напряжение на схеме, начиная с нуля. При малых
напряжениях сопротивления всех элементов равны R. По-
этому общее сопротивление схемы есть 3R/2. Режим № 1
I(U) = 2/3 · U/R. В силу закона Ома и свойств последова-
тельного соединения падение напряжения на левом элемен-
те оказывается вдвое больше падения напряжения на двух
остальных элементах. Следовательно, U1 = 2U/3. Когда об-
щее падение напряжения равно 3UКР /2, напряжение на ле-
вом элементе равно UКР , и он переходит в режим с сопро-
тивлением 2R. При этом максимальное напряжение пер-
вого режима U1 max = 3UКР /2. Режим № 2. Важно отметить,
что при U = 3UКР /2 напряжение на параллельных элемен-
тах U2 = UКР /2 < UКР , и первым изменит своё состояние ле-
вый элемент. После того, как сопротивление левого резисто-
ра увеличится, U2 уменьшится до U2 = UКР /4, а сопротив-
ление параллельных резисторов останется равным R. Если
считать, что при переходе во второй режим внешнее напря-
жение U = 3UКР /2 не изменялось, то U1 должно возрасти до
6UКР /5, т. е. после перехода мы попадаем во внутреннюю
точку второго режима. И если мы будем продолжать пони-
жать U, до тех пор пока U1 > UКР , мы будем оставаться во
втором режиме. Минимальное напряжение второго режима
U2 min = 5UКР /4. Очевидно, что режимы № 1 и № 2 пере-
крываются, и мгновенный режим, в котором окажется схе-
ма, определяется не только текущим значением напряже-
ния (тока) в ней, но также и всей предысторией работы схе-
мы. Далее схема оказывается эквивалентна резистору 5R/2
и I(U) = (2/5) · U/R. При этом на параллельно соединённых
элементах падает 1/5 общего напряжения U2 = U/5. При об-
щем напряжении 5UКР падение напряжения на параллель-
но соединённых элементах равно UКР , и они переходят в
режим с сопротивлением 2R. Таким образом, U < 5UКР со-
ответствует U2 max = 5UКР . Режим Режим № 3. При переходе
из режима № 2 в режим № 3 напряжение на левом элемен-
те меняется от 4UКР до 10UКР /3 (превосходит критическое),
и в процессе перехода сопротивление левого резистора не
меняется. Напряжение же на правом элементе повышается
с UКР до 5UКР /3. И аналогично для минимального напря-
220 Решения задач

К решению задачи 112

К решению задачи 113

жения третьего режима получим: U3 min = 3UКР . В третьем


режиме схема эквивалентна резистору 3R, и I(U) = U/3R.
О т в е т. Вольт-амперная характеристика складывает-
ся из трёх отрезков прямых: 1) I = 2U/3R, U < 1,5UКР ; 2)
I = 2U/5R, 1,25UКР < U < 5UКР ; 3) I = U/3R, 3UКР < U.
113. Эквивалентная схема лампы и провода представ-
лена на рисунке. Когда в сеть включена лампочка, рас-
сеивающая мощность W1 , в схеме течёт ток I1 = W1 /U.
Провода, по которым течёт такой ток, рассеивают мощ-
ность W2 , значит их сопротивление равно r = W2 /I12 . Когда
в цепь включили другую лампочку, ток в схеме изменился:
I2 = W3 /U. Изменилась и мощность, рассеиваемая провода-
ми: W4 = I22 · r = W2 (W3 /W1 )2 .
О т в е т. Мощность, рассеиваемая проводами, в сети с
новой лампочкой равна W4 = 0,004 Вт.
114. Представим шар как последовательное соедине-
ние трёх резисторов: верхнего полушария (медного), по-
Решения задач районных туров 221

К решению задачи 114

К решению задачи 115

лоски (алюминиевой) и нижнего полушария (тоже мед-


ного). Сопротивление шара R = U/I = 1,95 · 10−5 Ом есть
сумма сопротивлений всех трёх частей: R = Rв + Rп + Rн .
Рассчитаем сопротивление полоски. Её сечение — коль-
цо радиусом r = 10 см и толщиной d = 1 мм. Его пло-
щадь равна S = 2prd = 628 мм2 . Поэтому сопротивление по-
лоски Rп = rAl a/S = 9,55 · 10−8 Ом. У железной полоски та-
ких же размеров сопротивление Rп = rFe a/S = 3,18 · 10−7 Ом.
При замене алюминиевой полоски на железную сопротив-
ление шара увеличится на величину Rп − Rп и станет рав-
но R = R + Rп − Rп = 1,98 · 10−5 Ом. Через шар пойдёт ток
I = U/R = 5,06 А.
О т в е т. Ток через шар станет равен 5,06 А.
115. Начертим схему, эквивалентную данной в условии.
Лампочка 3 горит ярче остальных, поскольку напряжение
на ней равно напряжению на клеммах источника. Лампоч-
ки 1 и 4 горят одинаково ярко, так как напряжение на них
одинаково. Лампочка 2 горит ярче лампочки 1, поскольку
через лампочку 2 течёт вдвое больший ток.
О т в е т. № 1 и № 4; № 2; № 3.
116. Обозначим скорость птицы u, угол, который её тра-
ектория составляет с горизонтом, b, а время, в течение ко-
торого летел камень (и птица) — t. Тогда, поскольку через
222 Решения задач

время t камень и птица оказались в одной точке,

vt cos a = ut cos b,
vt sin a − gt2 /2 = ut sin b.

Поскольку через время t камень оказался в верхней точ-


ке своей траектории, его вертикальная скорость в этот
момент равнялась нулю: v sin a − gt = 0. Разделим первые
два уравнения на t и подставим gt = v sin a во второе:
v cos a = u cos b, v sin a/2 = u sin b. Возведём их в квадрат и
сложим: u2 = v2 (cos2 a + sin 2a/4) = v (1 − 3 sin a/4).
2 2

3
О т в е т. Скорость птицы u = v 1 − sin2 a.
4
117. Заметим, что 4,8 · 10 −10 езссе = 1,6 · 10−19 Кл. Вос-
пользуемся следующей формулой: A = Pt = IUt = qU, то есть
в любой системе единиц: Единица работы = Единица
заряда · Единица напряжения. Следовательно: Едини-
ца работы СИ/Единица работы ССЕ = Единица за-
ряда СИ/Единица заряда ССЕ · Единица напряжения
СИ/Единица напряжения ССЕ = 4,8 · 10−10 /1,6 · 10−19 · 1/300 = 107 .
О т в е т. 1 Джоуль = 107 единиц работы ССЕ.
118. В начальный момент времени, t = 0, точка находи-
лась в точке x = 0. До тех пор, пока |x| < x0 = 2 м, ускорение
точки равно нулю, т. е. движение равномерное. Через вре-
мя t1 = x0 /v0x , координата точки будет равна x0 . Далее, при
x > x0 , некоторое время ускорение точки будет постоянным,
отрицательным и направленным против скорости. В резуль-
тате равнозамедленного движения точка остановится в мо-
мент времени t = t1 + t2 , t2 = v0x /|a0x |, где a0x = −1 м/сек2 .
Нетрудно заметить (см. график), что для возвращения в на-
чало координат точке также понадобится время t1 + t2 . При
t = 2(t1 + t2 ) координата примет начальное значение, однако
скорость поменяет знак. Далее движение повторится в обла-
сти отрицательных x, и скорость поменяет знак ещё раз. В
итоге через время 4(t1 + t2 ) = 12 сек, материальная точка бу-
дет иметь исходные значения проекций координаты и ско-
рости.
О т в е т. Материальная точка будет иметь исходные зна-
чения проекций координаты и скорости через 12 сек.
Решения задач районных туров 223

К решению задачи 118

119. Чай будет быстрее остывать в том стакане, в ко-


тором происходит помешивание. Необходимо отметить, что
вода имеет достаточно малую теплопроводность, и теплооб-
мен может происходить только за счёт конвекции. Переме-
шивание приводит к более активной передаче тепла меж-
ду частицами воды тем самым, увеличивая скорость теп-
лообмена внутри стакана. В результате помешивания над
чашкой возникают движения воздуха, которые увеличива-
ют скорость оттока тёплого воздуха и водяного пара. Другой
канал теплопередачи — металлическая ложка в стакане. У
ложки достаточно большая теплопроводность, она быстро
нагревается, забирая и передавая в воздух тепло.
120. Во всех системах измерения единицы измерения
величин связаны такими же формулами, как и сами ве-
личины. Ед. теплоёмкости = (Ед. энергии/Ед. массы)/
Ед. температуры = . . . = Ед. (скорости)2 /Ед. температу-
ры = . . . = (Ед. ускорения · Ед. длины)/Ед. температуры.
О т в е т. Ед. теплоёмкости (Х) = 9,8 · 0,3/(5/9) Ед. теп-
лоёмкости (СИ) = 5,3 Ед. СИ. Теплоёмкость воды c (Х)
= 4200/5,3 = 790 Ед. теплоёмкости (Х).
121. Модуль горизонтальной проекции скорости мячика
равен V · cos a и не меняется ни во время свободного полёта
(сопротивления воздуха не учитываем), ни после ударов о
стенку и потолок. За время полёта T мячик пролетел по го-
224 Решения задач

ризонтали путь 4 · r. Следовательно, VT · cos a = 4 · r. Откуда


получается искомый ответ.
О т в е т. Скорость мячика при броске равна 4 · r/T · cos a.
122. Обозначим высоту башни H, а V — скорость рас-
пространения пламени вниз. Тогда скорость распростране-
ния пламени вверх будет, по условию, равна sV . Точки
возгорания делят башню на 3 участка: нижний, высотой
H/10, который горит со скоростью V ; средний — высотой
L, который горит со скоростью (s + 1)V ; верхний, высотой
(9H/10) − L, который горит со скоростью sV . Время полного
сгорания башни — максимальное из времён сгорания каж-
дого куска. С увеличением среднего участка башни в два ра-
за: a) время горения нижнего участка не изменится; b) вре-
мя горения среднего участка увеличится в 2 раза; c) время
горения верхнего участка уменьшится. Предположим, что
при расстоянии L между точками возгорания, дольше всего
горел средний участок. Тогда при расстоянии между точ-
ками возгорания 2L дольше всех также горит средний уча-
сток, причём время горения должно увеличится в 2 раза.
По условию задачи видим, что это не так. С уменьшением
среднего участка башни в два раза: a) время горения нижне-
го участка не изменится; b) время горения среднего участ-
ка уменьшится в 2 раза; c) время горения верхнего участ-
ка увеличится. Предположим, что при расстоянии L меж-
ду точками возгорания, дольше всего горел верхний уча-
сток. Тогда при расстоянии между точками возгорания L/2
дольше всех также горит верхний участок, и время сгора-
ния башни увеличится. По условию задачи видим, что это
не так. Итак, при расстоянии L между точками возгорания,
дольше всего горел нижний участок, и для времени его сго-
H
рания справедливо: = t1 = 60 ч. При расстоянии между
10 · V
точками возгорания 2L, дольше всех горит средний участок
(если бы дольше горел по прежнему нижний участок время
сгорания бы не изменилось), поэтому для времени сгора-
ния справедливо: 2L/V (s + 1) = t2 = 61 ч. Исключая из двух
последних уравнений V , найдём высоту башни до пожара.
О т в е т. Высота башни до пожара была равна 541 м.
123. Задачу удобно решать графически. На рисунке при-
ведён пространственно-временной график работы светофо-
Решения задач районных туров 225

К решению задачи 122

ров. Штрихами обозначены промежутки времени, в течение


которых включён красный свет. Безостановочное движе-
ние автомобилей возможно со скоростями Vn = 2L/(2n + 1)T,
где L — расстояние между светофорами, T — период ра-
боты светофора, n = 0, 1, 2, . . . Помимо этого возможны
ещё и траектории типа A (см. рисунок), которые мож-
но назвать неустойчивыми. Двигаясь по ним, машина
приходит к одному светофору в момент его открытия, а
следующий проезжает перед самым закрытием. Возмож-
ные скорости вдоль таких траекторий выражаются форму-
лой Vn = L/nT (n = 1, 2, 3, . . .). Объединяя две эти форму-
лы, получим все возможные периоды работы светофоров
T = 2L/nV (n = 1, 2, 3, . . .), при которых возможно безоста-
новочное движение машин с постоянной скоростью V .
О т в е т. Безостановочное движение машин со скоростью
V возможно при следующих периодах работы светофоров
T = 2L/nV (n = 1, 2, 3, . . .).
124. В отличие от задачи 123, здесь нас интересует без-
остановочное движение с одинаковой скоростью V в обоих
направлениях. Это условие даёт возможность определить
сдвиг по времени при работе шлагбаумов. Поскольку все
шлагбаумы находятся на равных расстояниях и работают
с одинаковым периодом, то машина всё время должна по-
падать в один и тот же момент цикла работы каждого шлаг-
баума. Пусть это будет момент открытия шлагбаума. Един-
ственный возможный вариант безостановочного движения
с одной и той же скоростью в двух направлениях между
двумя шлагбаумами изображён на рисунке (сдвиг по време-
226 Решения задач

К решению задачи 124

ни в работе соседних шлагбаумов равен t). Обозначим T —


период работы шлагбаума, L — расстояние между ними.
Максимально возможная скорость безостановочного равно-
мерного движения в обе стороны должна быть такова, что-
бы расстояние между двумя шлагбаумами машина прохо-
дила за время t = T/2. На другом рисунке представлен про-
странственно-временной график работы шлагбаумов. Без-
остановочное движение в обе стороны возможно со ско-
ростями Vn = 2L/(2n + 1)T (n = 0, 1, 2, 3, . . .). Помимо этого
возможны ещё и траектории типа A, которые мы назовём
неустойчивыми. Двигаясь по ним, машина приходит к од-
ному шлагбауму в момент открытия его, а следующий про-
езжает перед самым закрытием. Возможные скорости вдоль
таких траекторий будут выражаться формулой Vn = 2L/2nT
(n = 1, 2, 3, . . .). В первом режиме работы шлагбаумов (при
T1 = 0,9t, T0 = 1,1t) возможны оба типа траекторий, и ответ
в этом случае будет L = nV · T/2. Во втором режиме работы
шлагбаумов (при T0 = 0,9t, а T1 = 1,1t) неустойчивые тра-
ектории невозможны. Они могут быть реализованы толь-
ко при T0 ≥ T1 . Следовательно, в этом случае ответ будет
L = (2n + 1)V · T/2.
О т в е т. Если каждый шлагбаум периодически откры-
вается на период T0 = 1,1t, а затем закрывается на время
T3 = 0,9t, то равномерное безостановочное движение машин
со скоростью V в обе стороны возможно, если расстояния
между шлагбаумами L = nVT/2. При втором режиме рабо-
ты шлагбаумов L = (2n + 1)VT/2.
125. Обозначим высоту отметки 1 литр как h и рас-
смотрим один из шариков, летающих в мензурке. Его энер-
гия равна 3mgh/4 (отсчитываем потенциальную энергию от
Решения задач районных туров 227

К решению задачи 124

дна мензурки). Пусть в момент начала движения мензур-


ки высота шарика над дном была равна h1 . Перейдём в си-
стему отсчёта мензурки. Если мензурка движется с ускоре-
нием g/2 вниз, эффективное ускорение свободного падения
в такой системе равняется g/2. Поэтому начало движения
мензурки эквивалентно уменьшению ускорения свободного
падения наполовину. При таком изменении кинетическая
энергия шарика не меняется, а его потенциальная энергия
уменьшается на mgh1 /2. Полная энергия шарика будет рав-
няться 3mgh/4 − mgh1 /2. Шарик будет долетать до высоты
h (отметки 1 литр), если его энергия превосходит mgh/2. В
результате получаем условие:

3 1 1
mgh − mgh1 ≥ mgh,
4 2 2

означающее, что h/2 ≥ h1 . Таким образом, если в момент


начала движения шарик находится ниже отметки 500 мл,
он будет долетать до отметки 1 литр, а если выше то не
будет. Рассмотрим движение шарика от отметки 750 мл
к дну мензурки, до начала её движения. Первую часть
пути (от отметки 750 мл до 500 мл) он проходит за
время t1 = (2h/4g)1/2 . Весь путь он проходит за время
t2 = (6h/4g)1/2 . Следовательно,
√ √ шарик проводит в нижней

части мензурки ( 3 − 1)/ 3 = 42% всего времени. В произ-
вольный момент времени примерно 42% шариков окажется
в нижней части мензурки и, следовательно, достигнет от-
метки 1 литр, если мензурку начать опускать.
228 Решения задач

К решению задачи 126

О т в е т. Примерно 42% шариков будет долетать до от-


метки 1 литр, если мензурку начать опускать с ускорением
g/2.
126. В состоянии равновесия, когда монета покоится от-
носительно тазика, она движется с ускорением a. На неё
действует две силы — сила реакции опоры N, направлен-
ная перпендикулярно поверхности тазика, в точке, где ле-
жит монета, и сила тяжести mg. Графическим построением
(см. рисунок), учитывая, что ma в 2 раза больше чем mg,
легко найти направление силы реакции опоры. Сила реак-
ции опоры направлена перпендикулярно касательной к по-
верхности. Следовательно, надо найти точку на кривой ка-
сательная, в которой будет перпендикулярна направлению
вектора N.
О т в е т. Монета будет находится на высоте z = 1,75 + 0,25 см.
127. Положение пакета не изменится, так как при на-
гревании останется прежней сила Архимеда, действующая
на него. Пренебрегая объёмом оболочки пакета, выразим
силу Архимеда: F = rвоздуха Vg = rлёгкого газа Vgrвоздуха /rлёгкого газа = mлёгкого газа grвозд
В самом деле, масса лёгкого газа остаётся постоянной, ведь
пакет герметичный. Отношение плотностей двух идеальных
газов, находящихся при одинаковых давлениях и темпера-
турах также постоянно.
Примечание. В процессе изменения температуры в ком-
нате возможны временные изменения положения пакета
вследствие движения воздуха при нагревании.
Решения задач районных туров 229

К решению задачи 128

128. Когда машина разгоняется, на неё действуют че-


тыре силы, изображённые на рисунке — две силы реак-
ции дороги N1 и N2 , сила тяжести mg, приложенная к цен-
тру тяжести, и сила трения скольжения F, приложенная к
точке контакта заднего (переднего) колёса с дорогой. Тре-
ние скольжения F = mN1 . Мы можем записать второй закон
Ньютона для горизонтальной и вертикальной составляю-
щих ускорения и закон рычага для всех сил. Закон рыча-
га (или закон равенства моментов) выражает тот факт, что
автомобиль не начинает вращаться вокруг центра масс и,
значит, сумма моментов всех сил относительно центра масс
равна нулю. Таким образом, мы получим три уравнения ди-
намики:

N1 + N2 = mg, F = ma, (N1 L − N2 L)/2 − Fh = 0.

Решая эту систему уравнений, мы получим ускорение ав-


томобиля, при условии, что ведущие колёса пробуксовы-
вают: a = (mg/2)/(1 − mh/L). Мощность, которая необходи-
ма для поддержания пробуксовки колёс, равна Fv(t) = ma2 t
и растёт со временем. Когда это значение станет боль-
ше P, ускорение уменьшится, и пробуксовка колёс пре-
кратится. Пройденный за это время путь очевидно равен
s = at2 /2 = P2 /(2m2 a3 ).
О т в е т. Путь до окончания пробуксовки равен s = 4P2 /(m2 m3 g3 ) · (1 − mh/L)3 .
Исследование ответа: Ответ подразумевает дальней-
шее исследование, так как множитель в скобках может
быть отрицательным (хотя для обычных автомобилей это
230 Решения задач

большая редкость). Нетрудно показать что ситуация ко-


гда (1 − mh/L) = 0, соответствует отрыву передних колёс от
земли, что исключено в условии задачи. При дальнейшем
уменьшении этого параметра устойчивый разгон с пробук-
совкой с опорой на четыре колёса становится невозможным,
и решение теряет смысл.
Примечание 1. В случае с задним приводом неочевидно,
что пробуксовка прекратится сразу же после уменьшения
ускорения. Тем не менее, легко показать, что при уменьше-
нии ускорения сила трения скольжения mN1 падает медлен-
нее, чем ma. Это и значит, что при уменьшении ускорения
сила трения будет меньше максимально допустимой силы
трения покоя (т. е. силы трения скольжения) и пробуксов-
ки не будет.
Примечание 2. В решении пренебрегается силами со-
противления. Обычно они заметно меньше, чем сила сухого
трения колёс о поверхность.
129. Поскольку трение отсутствует, полный импульс
системы из N + 1 бусинки сохраняется. Обозначим мас-
су легких бусинок за m, тогда масса тяжёлой бусинки по
условию равна 2m, а масса всех бусинок — (N + 2)m. Для
импульса получаем: p2 = 2 · 2m · E. После соударений им-
пульс сохраняется, и для кинетической энергии получа-
p2 4mE 2E
ем: E = = = . Выделившееся тепло рав-
2(N + 2)m 2(N + 2)m N+2
но разности начальной и конечной кинетических энергий:
N
Q = E − E = E.
N+2
N
О т в е т. Выделилось тепло E.
N+2
130. Коэффициент полезного действия определяется
как отношение полезной к затраченной в данном процес-
се работы. Полезная работа в данном случае будет рав-
на изменению потенциальной энергии воды в океане по-
сле погружения модуля и извлечения балласта. Обозна-
чим глубину погружения через h. Тогда: Aполезн = r0 Vgh.
Затраченная работа равна работе по подъёму балласта и
вычисляется согласно определению. Сила, которую необ-
ходимо приложить для очень медленного подъёма груза,
F = Mg − r0 Vбалласта g = Mg(1 − r0 /r). Этой силе соответствует
работа Aзатр = Mg(1 − r0 /r)h. Коэффициент полезного дей-
Решения задач районных туров 231

ствия, h = Aполезн /Aзатр = r0 V /M(1 − r0 /r). Заметим также,


что неравенство h < 1 обеспечивается условием погружения
модуля с балластом.
О т в е т. КПД описанного в условии способа погружения
h = r0 V /M(1 − r0 /r).
131. Угол, при котором начнётся скольжение резины
по наклонной доске, определяется из известного условия:
tg a = m1 , где m1 — коэффициент трения покоя. После нача-
ла движения сила трения будет равна Fтр = N · m2 , где m2 —
коэффициент трения скольжения. Запишем уравнения дви-
жения в проекциях на горизонтальную и вертикальную оси
координат:
OX : mg · sin a − Fтр = ma и OY : mg · cos a − N = 0.
Из этих уравнений можно выразить ускорение a = g(sin a − m2 cos a).
Зная длину доски L можно найти искомое время, поль-
зуясь известными формулами равноускоренного движения:
t = (2L/a)1/2 .
О т в е
т. Кусок резины достигнет другого края доски за
p
2L 1 + m21
время t = .
g(m1 − m2 )
132. Спроектируем силы, действующие на автомо-
биль, на ось параллельную дороге и направленную вниз:
fтр + mg sin a = ma или fтр = m(a − g sin a). Поскольку для
a = 1 м/с2 получается, что a < g sin a, то сила трения должна
быть направлена вверх (fтр < 0), а не вниз, как исходно пред-
полагалось при записи уравнения второго закона Ньютона
(см. рис.). Теперь спроектируем все силы на ось, перпен-
дикулярную дороге: N − mg cos a = 0. Кроме того, |fтр | = mN,
откуда m = |fтр |/N = |a − g sin a|/(g cos a).
О т в е т. Чтобы автомобиль мог двигаться вниз по скло-
ну с постоянным ускорением, коэффициент трения должен
быть равен m ∼ = 0,856.
133. Мощность, развиваемая электровозом, равна W = Fv,
где F — сила тяги поезда, v — его скорость. Движение
грузчиков можно представить следующим образом: груз-
чик разгоняется из состояния покоя, затем движется с по-
стоянной скоростью, проходя путь L, затем останавливает-
ся. Импульс системы «поезд + грузчик» равен P = P0 + mu,
232 Решения задач

К решению задачи 132

где m — масса грузчика с грузом, u — его скорость от-


носительно поезда. Видно, что пока скорость грузчика по-
стоянна, следовательно, добавка к импульсу, связанная с
движением грузчика, не зависит от времени, т. е. сила
F = DP/Dt = F0 . Следовательно, при движении грузчика с по-
стоянной скоростью W = W0 . В момент разгона грузчика
импульс системы увеличивается на mu, а в момент тор-
можения импульс системы уменьшается на mu. В момент
разгона (торможения) грузчика сила тяги поезда должна
была увеличиться (уменьшиться), так как ускорение по-
езда постоянно. Пусть F = F0 + f. Если разгон длится вре-
мя t, то дополнительная совершённая работа будет равна
DA = tDW = tfv = vp, где p — изменение импульса системы,
связанное с движением грузчика. Пусть в момент разгона
грузчика скорость поезда была равна v1 , а в момент тор-
можения v2 > v1 , тогда полное изменение работы будет рав-
но DAп = v1 mu − v2 mu = mu(v1 − v2 ) = −muaT = −maX, здесь
T — время движения грузчика через поезд, X — его сме-
щение вдоль поезда, a = (v2 − v1 )/T эффективное ускорение
грузчика.
О т в е т. Работа, совершённая электровозом, равна A1 = A − maL.
134. Поскольку закон изменения скорости не задан,
то решить эту задачу можно, только применив закон со-
хранения и превращения энергии. В верхней точке на-
клонной плоскости полная механическая энергия тела рав-
на сумме его потенциальной и кинетической энергий:
E1 = mgh + mV02 /2. За начало отсчёта потенциальной энер-
Решения задач районных туров 233

гии принят уровень, совпадающий с нижней точкой на-


клонной плоскости, в которой полная механическая энер-
гия равна кинетической энергии тела E2 = mV02 /2. Рабо-
та силы трения равна изменению механической энергии
A = E2 − E1 .
О т в е т. Работа силы трения равна A = −mgh.
135. Сначала определим, с какой силой F1 тормозная си-
стема действует на трос. Учитывая наличие двух тормозных
колодок, из правила рычага получаем, что F1 = 2mFr2 /r1 .
Тормозная система может сообщить человеку ускорение
a = (F1 /m) − g = [2mFr2 /r1 m] − g, направленное вверх. За вре-
мя падения T человек проходит путь h1 = gT 2 /2 и приобре-
тает скорость gT. Рассматривая отрезок в процессе движе-
ния космонавта после включения тормозов до полной оста-
новки у земли, получим h2 = g2 T2 /2a, где h2 — высота на-
чала торможения. Суммируя отрезки h1 и h2 , получим ми-
нимальную высоту начальной точки полёта.
О т в е т. 2 Минимальная высота начальной точки полёта
gT
H= .
2(2 − mgr1 /mFr2 )
136. Обозначим переменную массу мелка m(t). Второй
закон Ньютона для движения мелка по горизонтали,

m(t)a = −mm(t)g

показывает, что его ускорение постоянно и равно a = −mg.


Путь s, пройденный мелком с ускорением mg за время t,
равен s = Vt − mgt2 /2. Если начальная масса мелка m0 тако-
ва, что sh < m0 , то мелок остановится раньше, чем сотрёт-
ся. В этом случае, время его движения t = V /mg, а пройден-
ный путь равен s = V 2 /2mg. Иначе, если мелок истирает-
ся полностью, то пройденный им путь есть s = m0 /h, а вре-
мя движения даётся наименьшим положительным корнем
квадратного уравнения m0 /h = Vt − mgt2 /2.
О т в е т. Если начальная масса мелка больше hV 2 /2mg,
то его время движения до остановки равно t = V /mg. Иначе,
мелок истирается полностью за время t = t − (t 2 − 2m0 /mhg)1/2 .
137. Для решения этой задачи естественно обратиться
к закону сохранения энергии. По условию задачи, вначале
шарик лежит на поршне (шарик поднимают над поршнем
234 Решения задач

уже потом), что позволяет оценить величину силы трения


поршня о стенки трубы, |Fтр | > g(m + M). Видно, что вели-
чина силы трения достаточна, чтобы ограничить перемеще-
ние поршня z (поскольку удары шарика о поршень абсо-
лютно упругие, шарик, в отсутствие силы трения, мог бы
вытолкать поршень сколь угодно далеко). Через достаточно
большое время движение в системе прекратится, а вся по-
тенциальная энергия перейдёт в работу сил трения. Тогда
для системы «шарик + поршень» можно записать закон со-
хранения энергии для начального и конечного состояния в
виде: mg(h + z) + Mgz = Fтр z. Откуда немедленно следует от-
вет.
О т в е т. За достаточно большое время поршень опустит-
ся на глубину z = mgh/(Fтр − gm − gM).
138. В условии задачи оговаривается отношение моду-
лей сил натяжения в верхней и нижней точках траектории
тела, однако, направление векторов этих сил не уточняется.
В зависимости от выбора их направления (вверх или вниз)
задача допускает два различных ответа. В нижней точке
траектории выполняется уравнение Tн − mg = mV2 H/l, где
Tн — сила натяжения, l — длина стержня и Vн — ско-
рость шарика в нижней точке его траектории. Если сила
натяжения, действующая на шарик, направлена вверх, то
аналогичное равенство для верхней точки запишется в виде
−T1в + mg = mV1в 2 /l, где V
1в — первая возможная скорость
шарика в верхней точке его траектории. Учитывая равен-
ство Tн = 2T1в , получаем отсюда первый ответ T1в . Другой
ответ (T2в ) получится, если сила натяжения, действующая
на шарик, направлена вниз. Тогда, уравнение для верхней
точки будет иметь вид: T2в + mg = mV2в 2 /l, где V
2в — вторая
возможная скорость шарика в верхней точке его траекто-
рии.
О т в е т. Отношение скоростей шарика в нижней и верх-
ней точках его траектории равно: Vн /V1,2в = (2T − mg)1/2 (mg ± T)1/2 .
139. Шнур подчиняется закону Гука, и его относитель-
ное удлинение в момент отрыва нижнего магнита извест-
но. В момент когда нижний магнит отрывается от стола он
действует на верхний с силой равной своему весу. Учиты-
вая медленность процесса, записываем второй закон Нью-
тона для верхнего груза, F + m1 g = T, где F есть сила взаи-
Решения задач районных туров 235

модействия частиц, T — сила натяжения нити. Из третье-


го закона Ньютона следует, что в начальный момент (ко-
гда нижний груз находится в бесконечности) и в момент
отрыва справедливы равенства F = 0 и F = mg. Учитывая
закон Гука, для начального и конечного (момент отрыва)
состояний получим систему уравнений m1 g = Tн = kDxн и
(m + m1 )g = Tк = kDxк , где Dxк и Dxн суть конечное и началь-
ное удлинения шнура. Формула Dxк = 3Dxн следует непо-
средственно из условия задачи, откуда (m + m1 )/m1 = 3.
О т в е т. Масса нижнего магнита равна 2m1 .
140. Поскольку спутник неподвижен относительно пла-
неты, период его вращения совпадает с периодом вращения
самой планеты TС = TП . На спутник массы m действует сила
притяжения планеты массы M, F = gMm/R2 = mgr2 /R2 , где
R — искомое расстояние от центра планеты до её спутни-
ка. Эта сила является центростремительной для вращения
спутника с периодом T, следовательно, F = mw2 R = 4p2 Rm/T 2 .
Приравнивая правые части этих равенств, 4p2 Rm/T 2 = gmr2 /R,
легко выразить расстояние R3 = gr2 T2 /4p2 .
О т в е т. Расстояние от центра планеты до её спутника
R = g1/3 r2/3 T2/3 /41/3 p2/3 .
141. В задаче требуется найти работу силы, которая из-
меняется во времени. Силы, действующие на санки с вед-
ром, изображены на первом рисунке. Поскольку скорость
санок постоянна, векторная сумма действующих на них сил
равна нулю:
N + mg + F(t) + Fтр = 0.

Проекции на оси этого векторного уравнения суть:

Fтр = F(t) · cos a, m(t) · g = N + F(t) · sin a.

Выразим силу реакции опоры из второго уравнения систе-


мы с учётом того, что Fтр = kN, и получим выражение для
силы трения Fтр = k(m(t) · g − F(t) · sin a). Подставляя это вы-
ражение в первое уравнение системы, получим зависимость
силы, с которой мальчик тянет санки от массы воды в вед-
ре:
F(t) = k · m(t)g/(cos a + k · sin a).
236 Решения задач

К решению задачи 141

К решению задачи 141

Масса воды в ведре изменяется по закону m(t) = m0 − m0 · t/n.


В конце пути (через время tМ ) в ведре осталась половина на-
чального количества воды m0 /2 = m0 − m0 tМ /n. Из последне-
го уравнения найдём время движения, tМ = n/2, а затем —
пройденный путь S = V0 · n/2. На втором рисунке приведён
график зависимости силы от пути. Работа равна площади
под этим графиком. Подставляя значения начальной силы
F0 = k · m0 g/(cos a + k sin a) и максимального пути SМ , най-
дём искомую работу A = 3k · gm0 V0 n · cos a/8(cos a + k · sin a).
О т в е т. Работа, которую совершил мальчик, равна
A = 3k · m0 g · V0 n · cos a/8(cos a + k · sin a).
142. В начальный момент закреплённые грузы (см. ри-
сунок) находятся в равновесии и, следовательно, результи-
рующая сила, действующая на каждый груз, равна нулю.
Условия равновесия неподвижных грузов суть 2T = m1 g и
T = m2 g, где T — сила натяжения нити. Отсюда получаем
соотношение масс грузов m1 = 2m2 . Когда гайка начинает
Решения задач районных туров 237

К решению задачи 142

скользить по нити, на неё действуют две силы: сила тяжести


и сила трения m1 a = m1 g − Fтр . На нить, по которой скользит
гайка, действует сила, равная по величине Fтр и противо-
положная ей по направлению. Следовательно, сила натяже-
ния нити, перекинутой через блоки, равна половине силы
трения T1 = 0,5Fтр = 0,5m1 (g − a). Уравнение движения вто-
рого груза имеет вид m2 aX = m2 g − 0,5m1 (g − a). C учётом со-
отношения масс грузов, несложно заметить, что aX = a. Сле-
довательно, сила натяжения нити, перекинутой через бло-
ки, равна половине силы трения T1 = 0,5Fтр = 0,5m1 (g − a).
Уравнение движения второго груза имеет вид m2 aX = m2 g − 0,5m1 (g − a).
C учётом соотношения масс грузов, несложно заметить, что
такое уравнение удовлетворяется, если aX = a.
О т в е т. Ускорение второго груза совпадает с постоян-
ным ускорением гайки относительно земли и равно a.
143. Запишем закон сохранения импульса в век-
торной форме m1 V 1 + m2 V 2 = (m1 + m2 )V. Направле-
ние движения частиц после столкновения неизвестно,
поэтому надо рассматривать два варианта решения:
m1 V1 − m2 V2 = (m1 + m2 )V и m1 V1 − m2 V2 = −(m1 + m2 )V .
В первом случае соотношение масс выражается
формулой m1 /m2 = (V2 + V )/(V1 − V ), а во втором —
m1 /m2 = (V2 − V )/(V1 + V ).
О т в е т. Отношение масс частиц равно m1 /m2 = (V2 + V )/(V1 − V )
при V1 > V и m1 /m2 = (V2 − V )/(V1 + V ) при V2 > V .
238 Решения задач

К решению задачи 144

144. Максимальная высота подъёма маятника, которо-


му сообщили скорость V , равна h = V 2 /2g. Максимальный
угол, на который отклоняется шарик есть amax = arccos(1 − V 2 /2gL)  V /g1/2 L1/2
(последнее приближённое равенство получается, если коле-
бания малые). Если угол a достаточно большой (amax ≤ a),
то наличие стенки никак не скажется на периоде коле-
баний маятника T0 = 2pL1/2 /g1/2  2p(Lamax /V ). В против-
ном случае (amax > a) шарик будет мгновенно изменять свою
скорость на противоположную при каждом ударе о стену.
Период колебаний маятника при этом будет меньше T0
ровно на удвоенное время движения шарика от положе-
ния, в котором угол отклонения равен a, до максимально-
го отклонения (угол равен amax ): T1 = T0 − 2ta . При гармо-
нических колебаниях угол отклонения шарика изменяется
как a = amax cos(2pt/T), поэтому ta = T0 /2p arccos(a/amax ), и
T1 = 2[p − arccos(a/amax )]L1/2 /g1/2 . Зависимость периода ко-
лебаний маятника от величины его начальной скорости
представлена на рисунке.
О т в е т. Если начальная скорость маятника неболь-
шая (V ≤ aL1/2 /g1/2 ), то его период колебаний как и
у обычного маятника T0 = 2pL1/2 /g1/2 . Иначе, когда
V > ag1/2 L1/2 , период колебаний маятника становится мень-
ше, T1 = 2[p − arccos(ag1/2 L1/2 /V )]L1/2 /g1/2 .
Решения задач районных туров 239

К решению задачи 145

145. Выясним, может ли произойти разрыв стержня,


когда груз ещё не оторвался от конуса. Рассмотрим пре-
дельный случай, когда сила реакции опоры равна нулю. Ис-
пользуя второй закон Ньютона, запишем векторное уравне-
ние сил (см. рисунок), действующих на груз: ma = T + mg.
Проекции этого уравнения на выбранные оси координат
суть ma cos a = mg sin a и ma sin a = T − mg cos a. Из урав-
нений следует, что T = mg/ cos a. При a = 60 ◦ C, в частно-
сти, получаем величину силы натяжения нити, T = 2mg.
Очевидно, что она меньше силы, которую выдерживает
стержень. Отсюда следует, что для того, чтобы произошёл
разрыв стержня груз должен подняться над конусом. Из
выражения для силы натяжения T следует, что в момент
разрыва стержня угол между стержнем и вертикалью со-
ставляет b = arccos(1/3). Горизонтальная компонента силы
T сообщает телу центростремительное ускорение, поэтому:
3mg · sin b = mw2 r, где r = L · sin b есть радиус окружности,
по которой вращается груз. Отсюда несложно выразить ча-
стоту вращения стержня, при которой происходит его раз-
рыв.
О т в е т. Стержень разорвётся при частоте вращения
w = (3g/L)1/2 .
146. Рассмотрим изменение скорости частиц при уда-
ре о движущийся поршень. После удара о поршень, дви-
жущийся со скоростью U, скорость частицы будет равна
V1 = V − 2U. Запишем изменение кинетической энергии ча-
стицы при ударе о поршень DEк = 2m · U(V − U). За вре-
мя Dt до поршня долетят частицы, находящиеся в объёме
S(V − U) · Dt. Число этих частиц равно n = rS(V − U) · Dt/m.
240 Решения задач

Тогда полное изменение кинетической энергии поршня за


время Dt равно работе силы трения Aтр = 2rUS(V − U)2 · Dt,
поскольку скорость поршня остаётся постоянной. Разделив
последнее выражение на Dt, найдём мощность силы трения.
О т в е т. Мощность силы трения поршня в трубе равна
N = rSU(V − U)2 .
147. Поскольку процесс изохорный, работа равна ну-
лю DA = 0, а подведённое тепло полностью расходуется
на увеличение внутренней энергии газа DQ = DU. Урав-
нения состояния газа в начале и в конце процесса суть
P0 V0 = R(n1 + n2 )T0 и P1 V0 = (n1 + n2 )RT1 , где количество во-
дорода n1 , количество гелия n2 . Вычитая из второго урав-
нения первое, найдём изменение температуры в процес-
се: DT = DP · V0 /((n1 + n2 )R). Изменение внутренней энер-
гии смеси газов определяется подведённым к нему теплом:
DU = (3/2n1 + 5/2n2 )R · DT = DQ. Исключая из двух предше-
ствующих уравнений DT и учитывая, что n1 /n2 = 2m1 /m2 ,
получим DQ/(DP · V0 ) − 1,5 = m2 /(m1 + 2m2 ). Откуда следует
ответ.
О т в е т. Отношение массы водорода к массе гелия в сме-
си равно
 
 
m2 DQ DQ
= − 1,5 2− .
m1 V0 · DP V0 · DP

148. По определению теплоёмкости газа c = Q/DT. Пе-


редаваемое системе за время DT тепло равно Q = I2 r · Dt.
Расширение газа происходит при постоянном давлении p.
Объём, занимаемый газом, увеличивается за время Dt на
DV = Sv · Dt. Записывая уравнения состояния газа в момен-
ты времени разделённые промежутком Dt, pV0 = v · RT0 и
p(V0 + DV ) = v · RT1 , и вычитая одно уравнение из друго-
го, получим выражение p · DV = vR · DT, позволяющее найти
связь Dt и DT: Dt = vR · DT/pSv. Отсюда получим выражение
для теплоёмкости газа c = I2 rvR/pSv и, разделив найден-
ную теплоёмкость на число молей, получим окончательный
ответ.
О т в е т. Теплоёмкость одного моля газа в этом процессе
равна cv = I2 rR/pSv.
149. Перерисуем графики процессов в осях P − V (см. рис.).
Рассмотрим цикл 2 − 3 − 4 − 2. В процессе 4 − 2 работу со-
Решения задач районных туров 241

К решению задачи 149

вершают над газом, т. е. A4−2 отрицательна. В процессе


2 − 3 газ совершает работу при изотермическом расширении
A2−3 > 0, но она меньше по абсолютной величине, чем A4−2 .
Следовательно, работа газа в цикле 2 − 3 − 4 − 2 — A2−3−4−2
отрицательна. Теперь рассмотрим цикл 2 − 1 − 3 − 2. В про-
цессе изотермического сжатия 3 − 2 над газом совершают
работу, т. е. A3−2 отрицательна. В процессе изобарическо-
го расширения 1 − 3 газ совершает работу, но эта работа
меньше по абсолютной величине, чем A3−2 . Следовательно,
работа газа в цикле 2 − 1 − 3 − 2 — A2−1−3−2 также отрица-
тельна. Из графика очевидно, что по абсолютной величине
|A2−3−4−2 | > |A2−1−3−2 |. Но, так как обе эти работы отрица-
тельны, то A2−1−3−2 > A2−3−4−2 .
О т в е т. Газ совершил большую работу за цикл 2 − 1 − 3 − 2.
150. При низкой температуре весь гелий сжижается и
лежит на дне, его вес равен F0 , а масса — m = F0 /g. При
высокой температуре гелий полностью переходит в газооб-
разное состояние, и давление газа на все стенки становится
одинаковым. Давление найдём из уравнения состояния для
идеального газа, pV = RTF0 /gm. Длина грани равна V 1/3 , а
площадь грани — V 2/3 .
О т в е т. Сила, действующая на стенки куба при высокой
температуре, равна F = pV 2/3 = RTV 2/3 F0 /gm.
151. В принципе, вскипятить воду с помощью двух ки-
пятильников можно двумя способами, включая их либо па-
раллельно, либо последовательно. При включении в сеть с
напряжением 220 В мощность, потребляемая кипятильни-
ком, изменится. Поэтому, прежде всего, надо подумать о
242 Решения задач

том, чтобы не сжечь кипятильник. Вычислим, во сколько


раз увеличивается потребляемая кипятильником мощность
N = U 2 /R при включении его в сеть с напряжением 220 В.
Из формулы k = U12 /U22 = 4 > 0,2 следует, что потребляемая
мощность увеличится в 4 раза. Следовательно, включение
кипятильника, рассчитанного на 110 В, в розетку с напря-
жением 220 В недопустимо. Для того, чтобы уменьшить по-
требляемую мощность, надо увеличить сопротивление, и,
следовательно, включить кипятильники последовательно.
Общее сопротивление цепи из двух последовательно вклю-
чённых кипятильников будет 2R. Q = Nt — количество теп-
лоты, которое надо сообщить чайнику с водой, для того что-
бы вода закипела. Подставляя значения напряжений и со-
противлений в формулу T/t = U12 R0 /U22 R, легко найти, что
вода закипит за половину обычного времени T = t/2.
О т в е т. Воду можно вскипятить парой последовательно
включённых кипятильников за половинное время T = t/2.
152. Для того, чтобы стакан находился в равновесии,
сила тяжести должна быть уравновешена силой Архиме-
да, F A = mg. Действующая на стакан сила Архимеда по-
стоянна, так как вес стакана не меняется. Отсюда следу-
ет, что разность уровней воды внутри и снаружи стакана
одинакова в обоих случаях. Процесс изотермический, сле-
довательно, P1 (H1 + h)S = P2 (H2 + h)S, H1 = H/2. Давление
воздуха внутри стакана равно сумме атмосферного давле-
ния и давления столба жидкости высотой h, P1 = P0 + rgh, а
P2 = P0 (1 − 0,01k) + rgh. В итоге,
(P0 + r · gh)(H/2 + h)
H2 = −h + .
P0 (1 − 0,01 · k) + mg/S
О т в е т. Разность высот погружений стакана равна:
DH = 0,01kP0 (H/2 + m/rS)/(P0 (1 − 0,01k) + mg/S).
153. Абсолютную величину изменения температуры си-
стемы легко найти из уравнения баланса энергии: CS DT = mgh,
в котором полная теплоёмкость системы CS является сум-
мой теплоёмкостей одного моля одноатомного газа и извест-
ной теплоёмкости груза: CS = C + 3/2R. Из уравнения ба-
ланса легко выразить изменение температуры DT, а затем
найти конечную температуру системы T1 = T0 − DT.
Решения задач районных туров 243

К решению задачи 154

К решению задачи 154

О т в е т. Конечная температура системы равна T1 = T0 − mgh/(C + 3R/2).


154. Резисторы 1 и 2 включены в схеме параллель-
но, поэтому эту схему можно перерисовать как показано на
рис. 1 и рис. 2. Напряжение между точками A и B равно
2 В — это напряжение на полюсах батарейки 1 (или 2). На-
пряжение между точками B и C равно 6 В, поэтому напря-
жение между точками A и C, т. е. на каждом из резисторов
равно 8 В.
О т в е т. Через каждый резистор течёт ток 2 А. Через
батарейку 3 течёт суммарный ток, равный 4 А. Поскольку
батарейки 1 и 2 одинаковые, через каждую из них течёт
половина суммарного тока, т. е. 2 А.
155. Кинетическая энергия частицы до попадания в
конденсатор равна изменению её потенциальной энергии в
процессе разгона qf = mv2 /2. Скорость частицы вдоль пла-
стин не изменяется, поэтому время пролета через конденса-
тор t = L/v. Напряжённость электрического поля в конден-
саторе E = U/d обеспечивает частице ускорение в направле-
нии положительно заряженной пластины a = F/m = qE/m = qU/md
и путь частицы поперёк конденсатора S = at2 /2 = qUL2 /2v2 md = UL2 /4f.
Этот путь должен быть меньше, чем 3d/4, что и позволяет
оценить сверху разность потенциалов U < 3fd2 /L2 .
О т в е т. Разность потенциалов, при которой частица мо-
жет вылететь из конденсатора U < 3fd2 /L2 .
244 Решения задач

156. Куски проволоки различаются двумя параметрами


длиной L и площадью поперечного сечения S. Теплоёмко-
сти обоих кусков равны C1 = C2 . Поскольку теплоёмкость об-
разца пропорциональна его объёму C ∼ LS, то из равенства
теплоёмкостей следует, что L1 S1 = L2 S2 . Удлинение прово-
локи под собственной тяжестью Dl ∼ mg/k, где k — коэффи-
циент Гука для проволоки. Он масштабируется как k ∼ S/L,
следовательно, Dl ∼ L2 . Электрическое сопротивление R, в
свою очередь, масштабируется как R ∼ L/S или R ∼ Dl/C. По
условию, Dl1 = 2Dl2 , следовательно и R1 = 2R2 .
О т в е т. Сопротивление первой проволоки будет в 2 раза
больше, чем второй.
157. Из вольт-амперной характеристики (ВАХ) следу-
ет, что через элемент X ток начинает протекать, когда на-
пряжение на нём оказывается большим, чем U0 . Если ток
уже начал протекать через элемент X, то при дальнейшем
увеличении подаваемого напряжения (по абсолютной вели-
чине) падение напряжения на элементе X не будет менять-
ся, оставаясь равным U0 — в этом случае мы будем гово-
рить, что элемент X открыт. В противном случае, элемент
X закрыт. Пока состояние элементов X в схеме, изображён-
ной на рис. 38, не изменяется, ток в цепи линейно меняет-
ся с изменением напряжения. Таким образом, ВАХ состоит
из линейных отрезков, начальные и конечные точки кото-
рых определяются моментами, когда один из элементов X в
схеме открывается. Кроме этого, имеется возможность про-
текания неограниченного тока в положительном направле-
нии, т. е., когда напряжение достаточно велико — этому со-
ответствует вертикальная прямая на графике ВАХ. Далее,
в тексте решения элементы X называются диодами. Левая
ветвь графика соответствует случаю, когда верхний диод от-
крыт, и ток течёт через него и через верхнее сопротивление
R. Горизонтальный участок ВАХ соответствует состоянию,
когда все три диода закрыты. Затем, открывается нижний
правый диод, и ток течёт через него и через сопротивление
2R. Затем, открывается левый нижний диод, и ток неогра-
ниченно возрастает. На графике обозначено V = U0 .
О т в е т. График зависимости тока от напряжения для
схемы показан на рисунке.
Решения задач районных туров 245

К решению задачи 157

158. В пространстве между первыми двумя пласти-


нами частица находится в ускоряющем поле, а между
вторыми двумя пластинами — в замедляющем. Сила,
действующая на частицу в первом пространстве, равна
F = −ФQ/H12 − mg = 0 (Ф — потенциал средней пластины),
и траектория частицы в этом пространстве — прямая.
Смещение частицы вдоль этой прямой равно L1 = 346 см.
Во втором пространстве сила, действующая на частицы
есть F = QФ/H23 − mg = −5H, что придаёт частице ускоре-
ние g* = 5g. Время пролета частицы между второй и третьей
пластинами до высшей точки траектории равно t = vY /g* ,
а наибольшая высота подъёма — Hmax = v2Y /2g* . Подстав-
ляя численные значения, получим Hmax = 0,0625 м < H23 .
Смещение до наивысшей точки траектории во втором полу-
пространстве равно L2 = vX t = v2 cos 30◦ sin 30◦ /g* = 0,21 м.
Полное горизонтальное смещение частицы до столкновения
с нижней пластиной есть L = 2(L1 + L2 ).
О т в е т. Горизонтальное смещение частицы от началь-
ного момента до момента полной остановки равно 7,34 м.
159. Можно перерисовать схему так, как показано на
рисунке. Лампочки 1 и 2 не включены в новую схему, так
как они подключены параллельно с проводами, т. е. «зако-
рочены» и гореть не будут. Лампочка горит тем ярче, чем
больше на ней напряжение. Самая яркая лампочка, тогда
будет 6, так как напряжение на ней максимально и равно
напряжению источника. Из оставшихся лампочек 3, 4, 5
246 Решения задач

К решению задачи 159

К решению задачи 161

наиболее ярко будет гореть 4, так как ток через неё боль-
ше чем токи через лампочки 3, 5 и равен сумме последних.
Наконец лампочки 3 и 5 подключены параллельно и светят-
ся одинаково слабо.
О т в е т. (1, 2), (3, 5), 4, 6.
160. Единственный ответ, имеющий физическую раз-
мерность, соответствующую калориям — с) 1,16 · 10−3 Вт · час.
161. Показания тока на A1 должны уменьшиться, если
сопротивление реостата увеличивают. Показания на A2 —
увеличатся, падение напряжения на первом участке — тоже
увеличится. Таким образом, подходит ответ b).
162. График зависимости проекции скорости мячика на
вертикальную ось, направленную вверх, будет выглядеть
как показано на рисунке (в момент отскока шарик мгновен-
но меняет свою скорость на противоположную). Правиль-
ный ответ — e).
163. КПД тепловой машины можно оценить по
PV -диаграмме её циклического процесса, найдя отноше-
Решения задач районных туров 247

ние площади цикла (полезной работы), к полной площади,


заметаемой циклом на диаграмме (площадь самого цикла
плюс площадь под циклом, что соответствует полной со-
вершаемой работе). Проверив все предложенные варианты
ответов, убеждаемся, что наименьшим КПД будет обладать
вторая машина. Правильный ответ — b).
164. Если не учитывать сопротивление воздуха, то мо-
дуль ускорения мячика немедленно после отскока был бы
равен g — ускорению свободного падения. В действительно-
сти, сразу после отскока, сила сопротивления воздуха равна
по модулю mg, поэтому правильный ответ, конечно, d).
165. Правильный ответ — b.
166. Правильный ответ — c.
167. Правильный ответ — d.
168. Правильный ответ — f.
169. Правильный ответ — d.
170. Правильный ответ — b.
171. Правильный ответ — c.
172. Правильный ответ — b.
173. Правильный ответ — d.
174. Правильный ответ — c.
175. На брусок действуют три силы (см. рисунок): си-
ла тяжести mg, сила реакции земли R (её обычно пред-
ставляют в виде суммы двух проекций: нормальной силы
реакции N и силы трения Fтр ) и приливная сила, возни-
кающая из-за притяжения со стороны Луны, F такая, что
|F| = GMm/R2 = mgЛ . Сила тяжести направлена вниз, а при-
ливная сила — в любую сторону (в зависимости от фазы
Луны). Обозначим a — угол наклона силы R к вертикали.
Так как |Fтр | ≤ mN, то
Fтр mN
tg a = ≤ = m.
N N
Поэтому угол a не может превышать величину amax = arctg m,
которое достигается, когда |Fтр | = mN (проскальзывание).
Чтобы брусок не проскальзывал, векторная сумма сил, дей-
ствующих на него должна быть равна нулю (вектора сил
должны замыкать треугольник).
Когда траектория Луны пересекает вектор RМ , существу-
ет такое направление вектора F, что треугольник не замы-
248 Решения задач

К решению задачи 175

К решению задачи 175

кается, и тело не остаётся в покое. Если же окружность век-


тор не пересекает, то треугольник всегда можно замкнуть.
Случай касания окружности и вектора RМ является крити-
ческим. При этом угол между векторами RМ и F прямой,
и вытекающее отсюда равенство F = mg · sin amax определяет
минимальный коэффициент трения, при котором треуголь-
ник ещё замыкается.
О т в е т. Брусок не сдвинется с места, если коэффициент
его трения о землю
gЛ gЛ
m ≥ tg arcsin = p .
g g 1 − g2Л /g2

Примечание. Задача минимизации приливной силы ре-


шена графически на рисунках, сопровождающих задачу.
Разумеется, её решение также может быть найдено при по-
мощи производных. Обозначим угол, который приливная
сила образует с вертикалью, b (он может быть произволен)
Решения задач районных туров 249

К решению задачи 175

К решению задачи 175

К решению задачи 175


250 Решения задач

и рассмотрим ситуацию, когда сила трения достигла свое-


го максимального значения |Fтр | = mN. Условия равновесия
бруска в проекции на горизонтальную и вертикальную оси
суть: mg = N + F · sin b и mN = F · cos b. Из них можно найти
величину приливной силы:
m · mg
F = m · gЛ = .
cos b + m · sin b
Её минимум достигается для такого угла b0 , при кото-
ром производная силы обращается в ноль: Fb = 0 (функция
−F выпукла на интервале [0, 2p]). Можно убедиться, что
b0 = arctg m. Кривая критических значений коэффициента
трения m от величины отношения ускорений (gЛ /g)min изоб-
ражена на рисунке, она иллюстрирует тот же ответ, что уже
был получен при графическом способе решения.
176. Описанная в условии система представляет со-
бой «комбинированный маятник», поочерёдно совершаю-
щий по половине различающихся длительностями перио-
дов колебаний. При упругом столкновении двух тел оди-
наковой массы выполняются законы сохранения импульса:
mv1 = mu1 + mu2 , и энергии: mv21 = mu21 + mu22 .
В случае, когда одно из тел покоилось, система этих
уравнений допускает два решения. Первое соответствует
ситуации, когда столкновения не произошло (разумеется,
энергия и импульс при этом сохраняются), а второе реше-
ние описывает случай упругого удара: u1 = 0, u1 = v1 (нале-
тающее тело останавливается, а покоившееся приобретает
его скорость). Первый шарик, ударившись о второй, оста-
навливается. Маятник на более длинном подвесе начинает
движение и, совершив половину периода колебаний, ока-
зывается в нижней точке, обладая скоростью, равной на-
чальной по величине, но обратной ей по направлению. Эта
скорость полностью передаётся соседнему маятнику, кото-
рый так же совершает половину периода колебаний. Период
колебаний системы складывается из двух полупериодов ко-
лебаний маятников T = (T1 + T2 )/2. Подставляя выражения
для периодов, приходим к окончательному ответу.
1/2 1/2
О т в е т. Период колебаний системы равен p(l1 + l2 )/g1/2 .
177. Покоящийся относительно вагона шарик совершает
вместе с ним ускоренное движение под действием сил тяже-
Решения задач районных туров 251

сти, натяжения нити и архимедовой силы:

ma = mg + T + F A .

Это не позволяет использовать закон Архимеда в его стан-


дартной формулировке, применимой только в случае покоя-
щейся жидкости или газа. Для определения неизвестной ар-
химедовой силы мысленно заменим шарик таким же объё-
мом окружающего его воздуха, масса которого M, очевидно,
превосходит массу шарика m. Этот новый объём оставал-
ся бы неподвижным относительно вагона без воздействия
какой-либо удерживающей нити, таким образом уравнение
его движения имело бы вид:

ma = Mg + F A .

Почленное вычитание уравнений позволяют исключить


неизвестное выражение для силы Архимеда, создаваемой
ускоренно движущимся газом. В результате проектирова-
ния полученного векторного равенства

(m − M)a = (m − M)g + T = 0

на выбранные координатные оси (см. рис.) получаем систе-


му, первое из равенств которой,

(m − M)aX = −T · sin a,

означает, что при M > m ускорение сонаправлено с осью OX


(aX > 0, вагон поворачивает направо). Из второго равенства
системы,
(M − m)g = T · cos a,
можно определить величину центростремительного ускоре-
ния: aX /g = V 2 /gR, а следовательно и радиус закругления
железнодорожных путей R.
О т в е т. Железнодорожные пути заворачивают в пра-
вую сторону по ходу поезда, радиус закругления путей ра-
вен R = ctg a · V 2 /Rg.
178. Пусть ось X направлена вдоль стержня (возмож-
ного направления смещения шарика). Совместим 0 оси
252 Решения задач

К решению задачи 177

X с точкой равновесного положения шарика до поворота


стержня. После поворота координата равновесного положе-
ния шарика X0 (амплитуда колебаний) вычисляется, ис-
ходя из условия равенства проекции силы тяжести на на-
правление стержня и компенсирующей её силы упругости:
k · X0 = mg · sin a. При смещении шарика на расстояние X от-
носительно нового положения равновесия, его координата,
очевидно, станет равной X0 + X, а ускорение может быть
определено по второму закону Ньютона:

ma = mg + N + F · (X0 + X).

Проектируя это векторное равенство на направление стерж-


ня, получим: m · X  = −k(X0 + X) + gm · sin a. Подставляя в
последнее равенство координату равновесного положения
шарика, приходим обычному к уравнению гармонических
колебаний:
X  = −k · X/m,
решение которого хорошо известно. √ √
О т в е т. Период колебаний шарика равен T = 2p · m/ k,
амплитуда колебаний равна A = X = (mg · sin a)/k.
179. В условии задачи описан принцип действия маг-
нитного масс-спектрометра, прибора для определения изо-
топного состава вещества. Если в конденсаторе заряженная
частица прошла разность потенциалов U, то её скорость V
при попадании в поперечное магнитное поле определяется
исходя из закона сохранения энергии: mV 2 /2 = qU. В маг-
нитном поле поперечном скорости частицы она испытывает
Решения задач районных туров 253

К решению задачи 178

К решению задачи 180

действие силы Лоренца mV 2 /R = qVB√ и движется по дуге


полуокружности, радиус которой R ∼ m легко выразить из
предшествующих равенств. Как видно, для частиц с одина-
ковыми зарядами радиусы кривизны траекторий√относятся √
между собой как корни из масс частиц, R1 /R2 ∼ m1 / m2 .
О т в е т. Радиус орбиты альфа-частицы в четыре раза
больше радиуса орбиты протона.
180. Направим горизонтальную ось строго на юг (па-
раллельно плоской поверхности острова), а другую, «вер-
тикальную» ось — перпендикулярно ей. Проекции векто-
ра ускорения свободного падения на выбранные оси суть
gX = g sin a и gY = −g cos a. Здесь применимы обычные ки-
нематические соотношения. Координаты стрелы, выпущен-
ной туземцем вертикально вверх с начальной скоростью V0 ,
равны: X(t) = g sin a · t2 /2 и Y(t) = V0 t − g cos a · t2 /2. Время
полёта стрелы есть T = 2V0 /g cos a, дальность полёта стрелы
L = X(T) = 2V02 · sin a/g cos2 a. Для того, чтобы выпущенная
стрела вернулась обратно, её необходимо направить точно
против вектора ускорения свободного падения.
О т в е т. Стрела упадёт на расстоянии 2V02 · sin a/g · cos2 a.
Чтобы выпущенная стрела вернулась обратно, её необходи-
мо направить на юг, под углом a к вертикали.
181. Действующая на мальчика сила трения имеет две
составляющие: одна создаёт центростремительное ускоре-
254 Решения задач

ние, обеспечивающее движение вместе с колесом по окруж-


ности, другая — обеспечивает увеличение линейной ско-
рости при увеличении частоты вращения колеса. В случае
равномерного разгона угловая скорость колеса смеха воз-
растает со временем по линейному закону w(t) = w0 · t/T. До
тех пор, пока мальчик не скользит относительно колеса, мо-
дуль его линейной скорости так же возрастает пропорци-
онально времени V (t) = w(t) · R = a1 t, коэффициент пропор-
циональности в этой зависимости даёт величину тангенци-
ального ускорения a1 = w0 · R/T. Кроме этого, движение по
окружности обуславливает наличие центростремительного
ускорения. Полное ускорение мальчика определяется сум-
мой двух указанных взаимно перпендикулярных составля-
ющих a2 = a21 + [w2 (t) · R]2 . Единственной силой, способной
сообщить мальчику такое ускорение, является, очевидно,
сила трения покоя, величина которой не может превосхо-
дить mN. 
w02 R T2
m·a=m t4 + ≤ m · mg.
T2 w02

Заметим, что в момент начала проскальзывания последнее


неравенство превращается в равенство, что даёт возмож-
ность найти момент времени начала проскальзывания:
 
T m2 g 2 T 2
t* = 4
− 1.
w0 R2 w02

Угол поворота f(t* ), пройденный мальчиком до начала


скольжения при вращательном движении с постоянным уг-
ловым ускорением, находим аналогично пути, пройденного
при равноускоренном поступательном движении, f(t* ) = w0 · t2* /2T.
Число оборотов по найденному углу определяется равно
n = f(t* )/2p. Решение t* теряет смысл и становится чисто
мнимым в двух крайних случаях: 1) когда T/w0 < 0, тогда
t2* > T2 (мальчик удерживается на колесе до самого оконча-
ния разгона), 2) когда m < w0 · R/gT, и сила трения столь ма-
ла, что мальчик начинает скользить уже в момент начала
движения колеса смеха.
О т в е т. При подходящей силе трения о поверхность
колеса смеха мальчик начнёт соскальзывать ещё до окон-
Решения задач районных туров 255

чания разгона, через некоторое время после его нача-


ла. Число оборотов, которое он при этом сделает равно
n = ((mgT/w0 R)2 − 1)1/2 /4p. Если сила трения слишком мала,
мальчик может начать скользить сразу после начала движе-
ния. Если она слишком велика, мальчик удержит на колесе
до самого окончания разгона.
182. Обозначим искомую жёсткость пружины k. Ес-
ли в результате взаимодействия с тележкой пружина сжи-
мается на величину x, то упругая сила F = kx, с которой
пружина действует на тележку, обеспечивает ей и находя-
щемуся на ней предмету ускорение a, kx = (M + m)a. По-
скольку предмет бьётся при ускорениях, превосходящих
a0 , то a = kx/(M + m) < a0 . Пружина сжата максимально, ко-
гда вся кинетическая энергия тележки перешла в потен-
циальную энергию пружины, kx2max /2 = (M + m)V 2 /2. Под-
ставляя xmax в предыдущее неравенство и выражая оттуда
k, получим первое условие: k < aI0 (M + m)/VI. Таким обра-
зом, если пружина слишком жёсткая, предмет разобьётся.
С другой стороны, если пружина чересчур мягкая, она не
сможет предотвратить столкновение тележки со стенкой, и
предмет разобьётся при ударе. Чтобы этого не произошло,
xmax не должно превышать L. Из xmax ≤ L получаем оценку
k ≥ (M + m) · V 2 /L.
О т в е т. Чтобы хрупкий предмет на тележке не раз-
бился, жёсткость пружины должна удовлетворять двойному
неравенству: (M + m) · VI/L < k ≤ aI0 (M + m)/VI.
183. Обозначим температуру воды и воздуха над ней T,
массу стакана M, массу воздуха в стакане m, высоту столба
воздуха в стакане, установившуюся в конце наблюдения, lX .
Сразу после погружения динамометр показывает разницу
между действующей на стакан силой тяжести и выталкива-
ющей силой Архимеда, Fнач = (M + m)g − r0 gl0 S. Так как ста-
кан погружают в воду быстро, то он не успевает обменяться
теплом с окружающей водой. Адиабатическое сжатие воз-
духа при погружении до объёма l0 S сопровождается нагре-
ванием воздуха до некоторой температуры T1 . Затем, в те-
чение некоторого времени в стакане устанавливается тер-
модинамическое равновесие между воздухом и водой, в ре-
зультате которого воздух остывает до исходной температу-
ры T, а объём его уменьшается, что вызывает уменьшение
256 Решения задач

плавучести стакана. Когда стакан ещё находился над водой


(до погружения), он целиком (на объём LS) был наполнен
воздухом при атмосферном давлении p0 и температуре T.
Его состояние будем описывать уравнением p0 LS = mRT/m.
В конце наблюдения, на глубине H, газ находится под дав-
лением p0 + r0 g(H + lX ), и занимает объём lX S при той же
температуре T. Уравнение состояния газа принимает вид:
[p0 + r0 · g(H + lX )]SlX = m · R · T/m. Приравнивая левые части
уравнений состояния, находим отсюда lX :
   
1 p0 2 4p0 L 1 p0
lX± = ± H+ + − H+ .
2 gr0 gr0 2 gr0

Физический смысл имеет только положительный корень


lX+ > 0, который определяет новое значение выталкиваю-
щей архимедовой силы |F A | = r0 glX+ · S. После установления
в системе термодинамического равновесия динамометр по-
казывает новую разницу DF = Fнач − Fк .
О т в е т. Различие между показаниями динамометра в
начале и в конце наблюдения составляет DF = r0 · g · S · (lX+ − l0 ).
184. Найдём ускорение a1 клина относительно плоско-
сти. Так как, по условию задачи, брусок по сравнению с
клином лёгкий, силой его воздействия на клин можно пре-
небречь. Клин двигается по наклонной плоскости под дей-
ствием силы тяжести Mg (M — масса клина) и силы реак-
ции плоскости N. В проекции на ось OX второй закон Нью-
тона для клина записывается в виде Ma1 = Mg sin a, откуда
a1 = g sin a.
Перейдём в систему отсчёта, связанную с клином. В ней
на брусок, кроме силы тяжести mg (m — масса бруска)
и силы реакции клина N1 , действует сила инерции ma1 ,
направленная параллельно плоскости в сторону, противо-
положную движению клина по ней. В проекции на ось
OY второй закон Ньютона для бруска записывается в виде
mg sin(a − f) − ma1 cos f = ma2 , где a2 — искомое ускорение
бруска относительно клина.
О т в е т. Брусок поедет относительно клина с ускорени-
ем a2 = −g · sin f · cos a.
185. Из всех возможных траекторий движения пловца
из точки A в точку B через берег нужно найти такую, чтобы
Решения задач районных туров 257

К решению задачи 184

К решению задачи 184


258 Решения задач

пловец затрачивал наименьшее время на её преодоление,


и сравнить время движения по этой траектории с L/V1 —
временем движения из A в B напрямую. В поисках опти-
мальной траектории достаточно рассмотреть те из них, ко-
торые симметричны относительно отрезка OO , так как лю-
бая несимметричная траектория заведомо невыгодна. Дей-
ствительно, любая несимметричная траектория, более вы-
годная, чем прямолинейный отрезок AB, например траек-
тория ACDB, может быть ещё оптимизирована, например
до траектории ACC B (точки C и C симметричны относи-
тельно OO ).
1) Графический метод поиска экстремума. Симметрич-
ная траектория определяется одним параметром, например,
x или углом ∠a = ∠PAC. Обозначим возможное расстояние
от точек A и B до берега за s. Время движения по отрезку
AC (и AC ) составляет t1 = AC/V1 = s/V1 cos a, а время дви-
жения по отрезку CC — t2 = CC /V2 = (L − s · tg a)/V2 . На бе-
рег забегать выгодно, когда
L/V1 > 2 · s/V1 · cos a + (L − 2s · tg a)/V2 .
Неравенство должно выполняться хотя бы при одном зна-
чении a, когда траектория ACC B — самая оптимальная
(когда правая часть неравенства минимальна). При выбо-
ре таковой можно воспользоваться оптической аналогией,
вспомнив, что из всех возможных траекторий свет выбира-
ет наикратчайшую, наиболее выгодную по времени: наибо-
лее выгодным для пловца окажется двигаться, следуя ходу
луча, испытывающего преломление при переходе границы
разделения оптических сред с различными показателями
преломления, n1 = c/V1 и n2 = c/V2 . Случай, когда пловец бе-
жит вдоль берега, соответствует явлению полного внутрен-
него отражения: sin a = n2 /n1 = V1 /V2 . Последнее соотноше-
ние позволяет выразить тригонометрические функции угла
через скорости пловца в воде и на суше и подставить их в
верхнее неравенство:

L V2 − V1
s< √ .
2 V2 + V1
Если расстояние до берега превосходит найденную величи-
ну, то выбегать на берег, в любом случае, невыгодно.
Решения задач районных туров 259

К решению задачи 185

К решению задачи 185

2) Аналитический метод поиска экстремума. Мы па-


раметризовали возможные траектории пловца переменной
x. В этом случае исследуемое на экстремум неравенство,
представится в следующем виде:
 2     
2 V1 V1 V1 V1 2
2 −1 +L·x· · 1− +L 1− > s2 .
2
x
V2 V2 V2 V2

Можно искать минимум левой части этого неравенства,


дифференцируя его по x и затем приравнивая производную
нулю, но проще выяснить, когда дискриминант неравенства
положителен (только в этом случае неравенство имеет реше-
ние). Этот путь также приводит к ответу, полученному при
использовании оптической аналогии.
О т в е т. Выбегать на берег может быть выгодно, если до
него не слишком далеко.
186. Будем считать, что вода в стакане ещё не кипит.
Вращающийся стакан с водой есть неинерциальная систе-
ма, и на любую частицу жидкости в нём действуют си-
ла тяжести и сила инерции. Плоскости поверхности воды
в каждой точке перпендикулярна равнодействующей этих
сил. Тангенс угла наклона касательной к поверхности воды
260 Решения задач

в точке x определяется отношением величин сил, действу-


ющих на частицу жидкости в этой точке:
W2 · R W2 · x
tg a(x) = + .
g g
Уравнение поверхности жидкости может быть найдено пу-
тём непосредственного интегрирования этого равенства. Од-
нако, такой метод выходит за рамки школьной програм-
мы. Чтобы найти уравнение поверхности, мы воспользуем-
ся аналогией с известной кинематической задачей о полё-
те тела, брошенного под углом к горизонту. В пользу та-
кой аналогии говорит пропорциональность тангенса угла
наклона касательной переменной положения частицы жид-
кости x, очевидная из приведённого выражения, что явля-
ется признаком параболической траектории. В самом деле,
рассмотрим движение тела, выпущенного из точки O с на-
чальными скоростями VX0 и VY0 под действием ускорения
свободного падения g. Тангенс угла наклона касательной в
точке x к траектории равен:
g · t + VY0 VY0 g·x
tg b(x) = = + 2 .
VX0 VX0 VX0
Сравнивая два равенства, легко заметить, что угол накло-
на касательной к баллистической траектории такой же, как
угол при поверхности жидкости, если VX0 = g/R, VY0 = RW.
Формулы, описывающие кинематику движения под углом
к горизонту хорошо известны. В частности, используя соот-
ношение y = VY0 · t + g · t2 /2, получим искомое уравнение по-
верхности жидкости.
О т в е т. Поверхность жидкости примет параболическую
W2 · R W2
форму, её уравнение есть y = x + x2 + y0 , где y0 опре-
g 2g
деляется из полного объёма воды в стакане.
187. Обозначим y — расстояние от капли до закры-
того конца трубки после установления равновесия. Цен-
тростремительное ускорение обеспечивается разностью дав-
лений на каплю изнутри и снаружи: mw2 y = pR2 (p A − p).
Давление внутри мензурки легко найти с помощью закона
Бойля–Мариотта: p A x = py. Из этих уравнений найдём

1 ± 1 − 4ax
y1,2 = ,
2a
Решения задач районных туров 261

К решению задачи 186

К решению задачи 186


262 Решения задач

где a = mw2 /p A pR2 . Оба корня положительны, хотя толь-


ко один из них соответствует устойчивому решению. Рас-
смотрим функцию F(y) = m · w2 y − (p A − p(y)) · pR2 . Если она
больше нуля, каплю «тянет» наружу, иначе — внутрь. По-
скольку y > 0, функция F(y) имеет тот же знак, что и функ-
ция yF(y), которая является параболой. Для y = y1 (y1 —
с «плюсом») функция F(y) будет положительна, и каплю
«потянет» наружу. Следовательно, это положение неустой-
чиво; устойчивым является другой корень — y2 (с «ми-
нусом»). Можно также заметить, что в предельном случае
очень медленного вращения мензурки, 4a · x  1, второй ко-
рень y2 ∼= x, в то время как первый — y1 ∼ = −x, что, конеч-
но, не имеет смысла. Если капля исходно находилась на
оси вращения x = 0, то она не сдвинется с места: y = 0. При
0 < x < 1/4a, капля перемещается по направлению к откры-
тому концу мензурки, x < y < 1/2a, и покидает её («вытря-
хивается»), если вращение происходит достаточно быстро
a > 1/2L. При L > x > 1/4a, уравнение положения капли не
имеет действительных решений. В этом случае, капля не
может находится в равновесии в трубке.
О т в е т. Если исходное положение капли в
1 p · R2 p
мензурке x< · A
< L, то её новое положе-
4
 
mw 2

p A p · R2 4x · mw2
ние y = · 1− 1− . Если угловая ско-
2mw2 p A p · R2
рость вращения мензурки превосходит величину
w = (p A p · R) /(2 · mL) , капля вылетает из неё наружу.
1/2 1/2
188. Условие задачи об остановке поезда ровно в мо-
мент, когда последний вагон покидает туннель, позволяет
связать длину состава поезда со временем нахождения по-
следнего вагона в туннеле. Время, необходимое для тормо-
жения поезда на отрезке пути длиной S, равно t = [2S/a]1/2 .
Тогда, время нахождения в туннеле последнего вагона за-
пишется, очевидно, как TN = [2L/a]1/2 . Время нахождения в
туннеле первого вагона равно TN = [2(L + Nl)/a]1/2 − [2Nl/a]1/2 .
Решение системы этих уравнений относительно N приводит
к формальному ответу. Остаётся только заметить, что состав
поезда должен иметь целое число вагонов, таким образом,
ответ будет иметь смысл только, если N — целое число.
Решения задач районных туров 263

К решению задачи 189

О т в е т. Если числа T1 и TN таковы, что число


N = L(TN2 − T 2 )2 /4lT 2 T 2 — натуральное, то N равно чис-
1 1 N
лу вагонов в составе поезда.
189. Гравитационное поле входит в число потенциаль-
ных полей. Их основным свойством является то, что работа
сил поля не зависит от пути, а определяется только пере-
мещением, иными словами, для таких полей можно опре-
делить понятие потенциала. Поскольку начальная и конеч-
ная точки траектории тела находятся на одной высоте, пол-
ная работа силы тяжести равна нулю. Графиком зависи-
мости y(t) является парабола с повёрнутыми вниз рогами.
Vy(t) — прямая линия, пересекающая ось абсцисс в точке,
где V достигает максимума. Проекция ускорения тела на
вертикальную ось не изменяется во времени, отрицательна
и равна −9,8 м/с2 .
О т в е т. Работа силы тяжести по всей траектории равна
нулю, необходимые графики представлены на рисунке.
190. Путь мячика до удара по горизонтали равен L = vT.
Путь мячика по вертикали до удара равен H = gt2 /2 = Nh.
264 Решения задач

В результате подстановки имеем, LN = v[2Nh/g]1/2 . Усло-


вие непопадания мячика на N + 1 ступеньку и ступеньки с
ещё большими номерами представится в виде неравенства:
LN < bN. По аналогии выписывается условие непопадания
на N − 1 и всё предшествующие ступени. Получаем интер-
вал скоростей, при которых первый отскок произойдёт со
ступеньки с номером N.
О т в е т. Чтобы первый отскок шарика пришёлся на сту-
пеньку с номером N, его скорость должна принадлежать ин-
тервалу значений: [Ng/2h]1/2 > v/b > [g(N − 1)/2h]1/2 .
191. Скорость струи гелия, очевидно, будет некото-
рой функцией высоты V (h). Из закона сохранения най-
дём импульс DP(h), передаваемый телу, расположенному
на высоте h, струёй сечением S жидкости плотности r,
DP(h) = DM · V (h) = rSDt · V 2 (h). Зависимость скорости струи
от высоты может быть найдена, исходя из закона сохране-
ния энергии: V 2 (h) = V02 − 2gh. Начальная скорость струи ге-
лия, V02 = 2g(2h1 − h2 ) — из уравнений: rS[V02 − 2g · h1 ] = mg и
rS[V02 − 2gh2 ] = 2mg, выражающих условия равновесия ко-
нуса в струе фонтана. Откуда, сразу получаем максималь-
ную высоту подъёма струи hmax .
О т в е т. Максимальная высота подъёма струи равна
2h1 − h2 .
192. Время горения шнура n1 = 1 подожженного с двух
концов одновременно, равно T1 = T/2 = 30 сек. Таким спосо-
бом следует сжечь один шнур. Время горения оставшейся
части второго шнура, подожженного одновременно с пер-
вым и потушенного в момент полного сгорания первого
шнура есть DT2 = T − T1 = 30 c. Следует изготовить 5 та-
ких шнуров, n2 = 5. Время горения оставшейся части вто-
рого шнура, подожженного сразу с обоих концов равно
T2 = DT2 /2 = 15 сек. Таким способом, следует сжечь 1 из пя-
ти изготовленных 30-секундных шнуров.
Оставшееся время горения шнура, прогоревшего до это-
го время T, равно n3 = 4, DT3 = T − T2 = 45 сек. Следует из-
готовить 4 таких шнура. Соединяя каждый из оставшихся
четырёх 30-секундных шнуров с 15-секундными, получаем
4 шнура с заданным временем горения.
Решения задач районных туров 265

К решению задачи 193

О т в е т. Можно изготовить 4 шнура с временем горения


45 сек.
193. После первого столкновения с доской шарик отско-
чит от неё так, что угол падения будет равен углу отраже-
ния. Можно, обозначив длину доски через L и рассматривая
соударения одно за другим, находить координату каждой
последующей точки соударения и, таким образом, сосчи-
тать число ударов шайбы о доски. Такой расчёт сложен —
можно найти ответ проще. Вместо того, чтобы отражать тра-
екторию шарика от доски, оставим траекторию прямой, но
отразим от доски OA вторую доску OC. Будем повторять
эту операцию до тех пор, пока прямая AB (AB параллель-
на CD) будет пересекаться с отражениями досок. Для того,
чтобы восстановить траекторию шарика с помощью второ-
го рисунка, нужно сложить его по линиям OA, OC1 , OA1 ,
OC2 , а затем посмотреть на линию AB «на просвет». Вид-
но, что число соударений шайбы с досками равно четырём
(см. рисунок). Можно доказать, что так как ∠DOB = a (B —
точка, где шайба покидает угол, т. е. лежащая на окруж-
ности AC1 A1 C2 A2 ), для произвольного
  угла между досками
p
a, число соударений равно − 1, где квадратные скобки
a
означают целую часть дроби.
194. Поскольку стол равномерно разгоняли, а затем тор-
мозили в течение одного и того же времени T, — его тор-
мозили с тем же ускорением −a, и его движение также бы-
ло прямолинейным. Блюдце будет скользить по столу, если
сила трения mmg не может обеспечить ему то же ускорение,
266 Решения задач

К решению задачи 193

что и столу, то есть если mg < a. Перейдём в систему отсчё-


та, в которой стол неподвижен. В этой (неинерциальной)
системе отсчёта, в каждый момент времени, действует си-
ла инерции ma, направленная в сторону, противоположную
ускорению стола. Перемещение блюдца в этой системе от-
счёта не должно превосходить L. На первом интервале вре-
мени T блюдце разгоняется под действием силы ma − mmg.
За время T оно проходит расстояние x1 = (a − mg)T2 /2 и
приобретает скорость V = (a − mg)T. В последующие момен-
ты времени ускорение a и сила трения mmg действу-
ют в одну сторону и тормозят блюдце с силой ma + mmg
до полной его остановки. Блюдце остановится через вре-
мя t = V /(a + mg) = (a − mg)T/(a + mg), пройдя за это время
расстояние x2 = Vt − (a + mg)t2 /2 = (a − mg)2 T2 /(2a + 2mg). Пе-
ремещение блюдца x1 + x2 = aT 2 (a − mg)/(a + mg) не должно
превосходить L. Отсюда следует, что L должно удовлетво-
рять ограничению
aT 2 (a − mg)
L≥ .
a + mg
Примечание. Задачу можно решать и в исходной си-
стеме отсчёта. Пока стол разгоняется, блюдце также раз-
гоняется, но с меньшим ускорением (ускорение блюдца mg
обеспечивается силой трения о стол). Затем стол начинают
тормозить, а блюдце продолжает разгоняться. Когда ско-
рости стола и блюдца сравниваются, блюдце останавлива-
ется относительно стола и затем начинает скользить по
столу в противоположном направлении, к его центру. Нас
интересует интервал времени, в течение которого блюдце
Решения задач районных туров 267

двигалось от центра стола к краю, то есть пока скорости


стола и блюдца не сравнялись. Сила трения о стол, дей-
ствующая на блюдце, всегда направлена в одну сторону,
разгоняя его. Координата блюдца xb (t) = mgt2 /2, скорость
Vb = mgt. Координата стола x задаётся выражением x = at2 /2
при T ≥ t и x(t) = aT2 /2 + aT(t − T) − a(t − T)2 /2 при t ≥ T,
скорость стола равна V = aT − a(t − T). Блюдце остановит-
ся относительно стола в момент времени t0 = 2aT/(mg + a),
когда V = Vb . При этом блюдце пройдёт по столу путь
x(t0 ) − xb (t0 ) = aT 2 (a − mg)/(a + mg), который не должен пре-
восходить L. Отсюда следует, что L должно удовлетворять
ограничению, полученному выше.
О т в е т. Радиус стола должен удовлетворять условию
aT 2 (a − mg)
L≥ .
a + mg
195. КПД описанного в задаче двигателя, работающе-
го по циклу Карно, равен v = (100 ◦ C − T)/(100 ◦ C + 273 ◦ C),
где T — температура окружающей среды (выраженная в
градусах Цельсия). Обозначим через M массу запасённо-
го топлива. Работа по преодолению сил сопротивления на
всём маршруте производится за счёт запасённого топлива,
2FL = vH l(1 − h)M, здесь F — сила сопротивления карново-
за, l — теплота сгорания топлива, vH — КПД двигателя
ночью. Днём КПД станет меньше, и поезд пройдёт лишь
расстояние L , определяющееся из нового баланса энергии:
L F = vD l(1 − h)M/2, откуда L = LvD /vH = L(100 ◦ C − TD )/(100 ◦ C − TH ).
О т в е т. Поезд не доедет L − L = 4,2 км.
196. Шар находится в равновесии над полом, так как
действующие на него сила тяжести и сила Архимеда ском-
пенсированы. Поскольку при изменении атмосферного дав-
ления меняется объём шарика, меняется и выталкиваю-
щая сила, действующая на него. Следовательно, и дли-
на верёвки, которую шарик может поднять над полом, из-
меняется. Обозначим массу резиновой оболочки шара че-
рез M, объём шарика при давлении p0 как V0 , объём ша-
рика при давлении p1 через V1 , а расстояние от шари-
ка до пола при давлении p1 как H1 . Кроме того, вве-
дём r0 и r1 — плотности воздуха при давлениях и соот-
ветственно. Тогда баланс сил при давлении p0 имеет вид
268 Решения задач

(M + m)g + lHg = r0 gV0 . При давлении p1 баланс также со-


хранился, (M + m)g + lH1 g = r1 gV1 . Вычтем из второго урав-
нения первое (так что вес резиновой оболочки сократится) и
выразим величину H1 = H + (r1 V1 − r0 V0 )/l. Давление возду-
ха p0 можно связать с его плотностью r0 = mв /Vв = p0 Mв /RT
из уравнения для идеального газа, записанного для неко-
торой массы воздуха mв , имеющей при комнатной тем-
пературе T объём Vв . Аналогичное соотношение связыва-
ет давление p1 с плотностью r1 = p1 Mв /RT. Объём шари-
ка, V0 = mRT/(p0 + Dp)Mг , можно найти из уравнения иде-
ального газа для гелия массы m, имеющего при комнат-
ной температуре T давление p0 + Dp. Аналогично вычис-
ляется объём V1 = mRT/(p1 + Dp)Mг . Подставляя найденные
величины в формулу для новой высоты, получаем ответ
H1 = H + mDp(p1 − p0 )Mв /l(p0 + Dp)(p1 + Dp)Mг .
О т в е т. Шарик поднимется над полом выше.
197. Обозначим средние скорости молекул гелия и ксе-
нона в правой части сосуда как V1 и V2 , массы молекул ге-
лия и ксенона — m1 и m2 соответственно. Обозначим кон-
центрации гелия и ксенона в левой части сосуда в начале
процесса n1 и n2 , а количества вещества гелия и ксенона в
левой части сосуда в начале — n1 и n2 , температуру газа в
левой части сосуда T (так как отверстие маленькое и откры-
вается только на небольшое время, то будем считать, что эта
температура не изменяется). В приближении идеального га-
за молекулы гелия и ксенона вылетают независимо. Рас-
смотрим сначала истечение гелия. Предположим для про-
стоты, что молекулы в газе двигаются только вдоль взаим-
но перпендикулярных осей OX, OY, OZ со скоростью V1 (те
же предположения делаются в школьном курсе при выводе
основного молекулярного уравнения p = nkT). Тогда за вре-
мя Dt из левой части сосуда вылетят молекулы гелия, на-
ходящиеся в цилиндре длиной V1 Dt и имеющие скорость в
направлении оси OX. Их количество равно N1 = n1 V1 DtS/6
(множитель 1/6 появляется, так как из 1/3 всех молекул,
движущихся вдоль оси OX, только половина движется к от-
верстию). То же самое справедливо для вылетевших моле-
кул ксенона. Их количество равно N2 = n2 V2 DtS/6. Поэто-
му N1 /N2 = n1 V1 /n2 V2 . Этот результат остаётся справедлив,
даже если мы не делаем никаких дополнительных предпо-
Решения задач районных туров 269

К решению задачи 197

ложений относительно характера движения молекул в га-


зе, поскольку N1 должно быть пропорционально n1 хотя бы
из размерных соображений: N1 ∼ n1 V1 DtS, где коэффициент
пропорциональности хоть и отличается от 1/6, но является
не зависящим от свойств газа числом и сокращается при
делении N1 на N2 . Определим отношения объёмов и кон-
центрации для обоих газов. Так как мерой средней кинети-
ческой энергии молекул является температура, а она одина-
кова для гелия и ксенона (по условию первоначально в сосу-
де было тепловое равновесие),
√ √ то m1 V12 /2 = n2 V22 /2 ∼ T, сле-
довательно, V1 /V2 = m2 / m1 . Таким образом, для отноше-
ния концентрации газов получается следующее выражение:
n1 /n2 = m1 M2 /m2 M1 . Подставляя промежуточные результа-
ты в формулу для отношения средних концентраций гелия
и ксенона и учитывая, что m1 /m2 = M1 /M2 , вычисляем от-
вет.
О т в е т. После того, как отверстие закрыли, в правой
части сосуда установилось следующее отношение концен-
трации молекул гелия и ксенона:
   3
NHe m1 M2 m2 m1 M2
= · = ·
NXe m2 M1 m1 m2 M1

198. Для нахождения количества теплоты DQ, кото-


рую необходимо подвести к газу под поршнем для совер-
шении им заданной работы воспользуемся первым зако-
ном термодинамики: DQ = DA + DU. Уравнение состояния
270 Решения задач

идеального газа pV = vRT позволяет связать работу, совер-


шаемую расширяющимся газом при постоянном давлении,
с изменением температуры: DA = vRDT. Последнее соот-
ношение позволяет выразить изменение внутренней энер-
гии газа при нагревании через совершённую им работу
DU = 3vR · DT/2 = 3 · DA/2. КПД нагревателя связывает ко-
личество переданного газу тепла DQ с полной энергией
DW , подведённой к подъёмному устройству: DQ = h · DW .
Полный КПД подъёмника h0 , очевидно, равен отноше-
нию совершённой им механической работы к подведён-
ной энергии и вычисляется в соответствии с формулами
h0 = DA/DW = hDA/(DA + DU) = 2h/5 = 1/5.
О т в е т. КПД подъёмного устройства равен 20%.
199. Для выяснения динамики изменения температу-
ры в выбранной точке стержня рассмотрим его небольшой
элемент DL, концы которого имеют различную температу-
ру T1 > T2 . Очевидно, что тепловая энергия будет переда-
ваться от более нагретого конца к холодному (вдоль оси x).
Если количества тепловой энергии, втекающей и вытекаю-
щей из элемента длины стержня, не равны друг другу, его
внутренняя энергия и, следовательно, температура, будут
изменяться во времени DT = DQ/C = k(DT1 − DT2 ) · Dt. В зави-
симости от вида пространственного распределения темпера-
тур на рассматриваемом участке возможны как рост, так и
уменьшение его температуры во времени. В случае «умень-
шающейся по оси x крутизны» количество втекающего теп-
ла превосходит вытекающее и участок разогревается. Ана-
логично показывается, что участки с «возрастающей кру-
тизной» должны постепенно остывать. В случае же линей-
ной зависимости T(x) температура участка остаётся посто-
янной: втекающий в стержень и вытекающий из него теп-
ловые потоки точно равны друг другу. При стационарном
распределении температура изменяется вдоль стержня по
линейному закону: T(x, t) = 100 ◦ C − x · (100 ◦ C − 56,2 ◦ C)/L.
При этом оказывается, что в точке нахождения нафтали-
на температура не достигает величины, достаточной для его
плавления. Остаётся показать, что и до момента установле-
ния стационарного распределения нафталин не мог начать
плавиться. Действительно, в начальный момент темпера-
тура всех точек стержня, кроме x = 0 была меньшей, чем
Решения задач районных туров 271

К решению задачи 199

К решению задачи 199

температура стационарного распределения. В процессе теп-


лопередачи происходил постепенный нагрев средней части
стержня и приближение температуры его точек к стацио-
нарной. При этом температура любой точки стержня, кроме
его концов может достигнуть стационарного значения толь-
ко при условии достижения стационарного распределения
во всех остальных точках. В противном случае, в выбран-
ной точке температура начнёт понижаться. До тех пор, по-
ка правый конец стержня находится при температуре ки-
пения ацетона, нафталин не может начать плавиться. На-
личие небольшой массы нафталина существенно не меняет
распределения температур в стержне. Нафталин расплавит-
ся лишь после того, как весь ацетон выкипит.
О т в е т. К моменту, когда расплавится нафталин, весь
ацетон уже выкипит.
200. Обозначим напряжение, приложенное к нагрева-
тельной схеме — U, v1 и v2 — количество вещества водо-
рода и гелия соответственно, Dt — некоторый малый про-
межуток времени, T1 и T2 — температуры, которые имели
272 Решения задач

К решению задачи 199

К решению задачи 199

К решению задачи 199


Решения задач районных туров 273

в начале этого промежутка времени водород и гелий соот-


ветственно, DT1 и DT2 — изменение температур водорода и
гелия за промежуток времени Dt (ввиду малости Dt предпо-
лагается, что изменение температур небольшое, DT1  T1 ,
DT2  T2 ), R1 (T1 ) и R2 (T2 ) — сопротивления, которые име-
ли в начале этого промежутка времени элементы, нагре-
вающие водород и гелий соответственно (ввиду малости Dt
предполагается, что изменение этих сопротивлений в тече-
ние данного промежутка небольшое). Так как поршень теп-
лоизолирующий, температуры водорода и гелия будут раз-
ными в каждый момент времени. Давления же газов в лю-
бой момент времени будут одинаковы, так как в противном
случае поршень бы двигался. Через электрическую цепь
идёт ток I = U/(R1 + R2 ), поэтому в течение промежутка вре-
мени Dt первый элемент выделит тепловую энергию водо-
рода при постоянном его объёме (так как поршень не двига-
ется по условию задачи). Баланс энергий, E1 = Cv1 · v1 · DT1 ,
здесь Cv1 — молярная теплоёмкость водорода при постоян-
ном объёме. Аналогично, для гелия баланс энергий имеет
вид E2 = Cv2 · v2 · DT2 , где Cv2 — молярная теплоёмкость ге-
лия при постоянном объёме. Водород — двухатомный газ,
а гелий — одноатомный, их молярные теплоёмкости при
постоянном объёме равны Cv1 = 5R/2, Cv2 = 3R/2. Следова-
тельно, E1 /E2 = R1 (T1 )/R2 (T2 ) = 5v1 · DT1 /3v2 · DT2 . Отноше-
ние DT1 /DT2 выразим из отношения уравнений состояния,
записанных для каждого из газов в начале и в конце проме-
жутка времени Dt, v1 · DT1 /v2 · DT2 = V1 /V2 . Такое же выра-
жение можно получить для связи температур T1 и T2 , что
даёт зависимость:
 
3V2 v2 V1
R2 (T2 ) = · R1 T2 .
5V1 v1 V2

Остаётся выразить молярные объёмы через массы газов и


заметить, что полученная формула справедлива при любом
T2 .
О т в е т. Функция сопротивление второго элемента от
температуры равна
 
3V2 Tm2 V1
R2 (T) = ·R .
5V1 2m1 V2
274 Решения задач

201. Механическое равновесие системы поршней дости-


гается, если давления во всех секциях цилиндра одина-
ковы (иначе, на поршни будут действовать силы, приво-
дящие их в движение). Как найти величину этого давле-
ния? Поскольку сосуд теплоизолирован, а собственная теп-
лоёмкость поршней в расчёт не принимается, то суммар-
ная внутренняя энергия газа будет, очевидно, сохранять-
ся. В этих условиях, искомое установившееся давление бу-
дет простым «средним по объёму»: давление в каждой ча-
сти цилиндра pi даёт вклад в установившееся давление P
в соответствии со своим «весом», Vi /V , где V — полный
объём цилиндра. Связь внутренней энергии идеального од-
ноатомного газа с его давлением и объёмом даётся уравне-
нием состояния: ui = 3/2 · pi Vi (использовано уравнение со-
стояния идеального газа). В результате подстановки имеем:
3/2(p1 V1 + p2 V2 + . . . + pN VN ) = 3/2P (V1 + V2 + . . . + VN ).
О т в е т. Во всех секциях сосуда установятся одинаковое
давление P = (p1 V1 + p2 V2 + . . . + pN VN )/V .
202. В условии задачи предполагается, что устройства,
расположенные внутри цилиндра, принимают участие в
теплообмене. Внутренняя энергия газа в двух половинах
сосуда до перетекания одноатомного идеального газа рав-
на соответственно u1 = 3/2 · vRT1 и u2 = 3/2 · vRT2 . Полная
энергия системы до перетекания газа, с учётом теплоты, ак-
кумулированной устройствами, находящимися внутри ци-
линдра, равна W = 3/2 · vR(T1 + T2 ) + C · T1 . Соответственно,
полная энергия после прихода системы в равновесие запи-
шется как W  = 3vRT + CT . Полная энергия внутри тепло-
изолированного сосуда сохраняется.
О т в е т. Искомая температура равна T = [3/2vR(T1 + T2 ) + CT1 ]/(3vR + C).
203. Найдём возможные положения равновесия для
описанной в задаче системы. Пусть заряды остановились
на некотором расстоянии x друг от друга (см. рис. 1 к ре-
шению задачи), для определённости заряд −2q (чёрный ша-
рик) поместим в положении равновесия на верхней нити,
а заряд q (серый шарик) — на нижней. Натяжение левой
нити обозначим T1 , правой — T2 , будем считать, что ле-
вая нить — короткая (длиной L/2). Сила кулоновского вза-
имодействия между зарядами равна FKL = 2kq2 /x2 , причём
Решения задач районных туров 275

все силы, включая FKL , направлены вдоль оси OX, так как
по условию радиусы цилиндров очень малы, и оба заряда
движутся практически по одной прямой. Силы, с которой
однородное электрическое поле действует на заряды, равны
по модулю qE и 2qE. Все описанные силы изображены на
рис. 1. Запишем условие, что силы, действующие на каж-
дый заряд, скомпенсированы в положении равновесия. Для
заряда q оно имеет вид (в проекции на ось OX)

T2 + qE − FKL − T1 = 0.

Для заряда −2q запишем аналогичное равенство:

T2 − 2qE + FKL − T1 = 0.

Вычтем из первого уравнения второе, чтобы натяжения


нитей сократились, и подставим выражение для кулонов-
ской силы FKL — получим расстояние x между зарядами
в положении равновесия, x = (4kq/3E)1/2 . Несложно также
найти величину y, характеризующую положение верхнего
заряда относительно цилиндров. Длина левой нити равна
L/2. С другой стороны, её нетрудно выразить через x и y,
L/2 = 2y + x. Отсюда y = L/4 − (kq/3E)1/2 .
Замечание. Выше мы предполагали, что силы, действу-
ющие на шарики параллельны оси OY. Это верно, если най-
денная величина x много больше радиусов цилиндров. Если
на нижней нити находится заряд −2q, а на верхней заряд q
(см. рис. 2), то условия равновесия имеют тот же вид, что
и раньше. Следовательно, таким же останется и расстояние
между зарядами в положении равновесия x. Однако y те-
перь отмечает положение заряда q, а положение заряда −2q
даётся суммой x + y.
Полученное решение, однако, не единственное. Если за-
ряды окажутся на одной половинке нити — оба снизу
(см. рис. 2) или оба сверху, можно показать, что действу-
ющие на них силы не смогут быть скомпенсированы. Дей-
ствительно, действующая на оба заряда результирующая
сила будет направлена в ту сторону, в которую внешнее по-
ле тянет больший по модулю заряд. Заряд −2q доедет до
самого цилиндра (в случае рис. 3, влево), утягивая за собой
276 Решения задач

заряд q. Остальные, не изображённые на рис. 3 силы, явля-


ются внутренними по отношению к данной системе. Таким
образом, на рис. 3 также изображено возможное положение
равновесия системы. Существует ещё два положения равно-
весия — когда заряды «слипаются» (см. рис. 4, 5). Итак,
мы обнаружили пять различных положения равновесия в
системе (изображённые на рис. 1, 2, 3, 4, 5). Для того, что-
бы выяснить, какое же из них на самом деле реализуется в
системе, следует исследовать их на устойчивость. Положе-
ние на рис. 1, очевидно, неустойчиво. Действительно, если
верхний заряд −2q сдвинется немного влево (а другой за-
ряд, соответственно, вправо), кулоновское взаимодействие
FKL между ними уменьшится. Заряды от этого ещё боль-
ше раздвинутся, так что система «свалится» в положение,
изображённое на рис. 3. Если же верхний заряд чуть сдви-
нется вправо, то кулоновское взаимодействие FKL между
ними увеличится, и заряды ещё больше притянутся, так что
система «свалится» в положение, изображённое на рис. 4.
Аналогичное рассуждение показывает, что состояние, изоб-
ражённое на рис. 2, также оказывается неустойчивым. Су-
ществует ещё третье положение неустойчивого равновесия
(см. рис. 6) в некотором смысле симметричное состоянию,
изображённому на рис. 3. Действительно, если чуть сдви-
нуть заряд −2q в сторону верхней нити (против часовой
стрелки), он поедет влево по верхней нити, и система «упа-
дёт» в состояние, изображённое на рис. 4. Если же чуть
сдвинуть заряд −2q в сторону нижней нити (по часовой
стрелке), он поедет влево по нижней нити, потянет за собой
второй заряд, и система «упадёт» в состояние, показанное
на рис. 5. Итак, положения равновесия, представленные на
рис. 1, 2, 6, являются неустойчивыми, а положения, пока-
занные на рис. 3, 4, 5 устойчивыми, причём положения 4, 5
абсолютно устойчивы в них система имеет практически бес-
конечную отрицательную энергию (точечные заряды про-
тивоположного знака расположены бесконечно близко друг
к другу). Поэтому, даже обладая некоторой скоростью, си-
стема не сможет «проскочить» эти положения. Состояние,
изображённое на рис. 3, хотя и устойчиво, но характери-
зуется конечной энергией. Обладая кинетической энергией,
превосходящей по модулю энергию кулоновского взаимо-
Решения задач районных туров 277

действия, система сможет «проскочить» его. На рис. 7 кре-


стиками изображены неустойчивые положения заряда −2q,
а квадратиками устойчивые положения. Соответствующие
им положение второго заряда восстанавливаются однознач-
но, поэтому мы не указываем его, чтобы не загромождать
рисунок. Все положения равновесия на рис. 7 пронумеро-
ваны. То или иное устойчивое равновесное состояние реа-
лизуется в зависимости от начальных условий задачи. При
этом каждое из устойчивых решений характеризуется сво-
ей областью притяжения (бассейном аттракции) начальных
условий. Например, если в начальный момент заряд −2q
находился в положениях между состоянием 1 и состоянием
3, или между состояниями 3 и 2, система стремится в по-
ложение 3. Предположим, что кинетическая энергия систе-
мы невелика (в соответствии с условием задачи нить и за-
ряды лёгкие). Тогда описанное в предыдущем пункте «про-
скакивание» положения равновесия 3 не происходит. Ана-
логично, если больший по модулю заряд находится в обла-
сти между состояниями 1 и 4 или 4 и 6 система стремится
в положение 4. Если больший по модулю заряд находится
в оставшейся области, система стремится в положение 5.
Кроме того, когда x = L/2, положения равновесия 1 и 3 (а
также 2 и 6) сливаются. При x > L/2 положения равновесия
1, 2 исчезают. Проанализируйте для этих ситуаций картину
устойчивости самостоятельно.
Если начальные положения заряда −2q выбирают слу-
чайно (считают случайной величиной, распределённой рав-
номерно вдоль всей длины нити), то для каждого из устой-
чивых состояний можно указать вероятность его реализа-
ции. Например, состояние 3 будет наблюдаться с вероят-
ностью, равной отношению длины заштрихованной зоны к
полной длине нити, т. е. (L − x)/2L. Состояние 4 появляется
с вероятностью (L − y)/2L), а состояние 5 — с вероятностью
(x + y)/2L.
О т в е т. В зависимости от начальных условий, шарики
расположатся в одной из трёх устойчивых конфигураций.
204. На провод длиной l, по которому течёт ток I, распо-
ложенный перпендикулярно однородному магнитному по-
лю B, действует сила Ампера F = IBl, причём направление
силы перпендикулярно как направлению провода, так и на-
278 Решения задач

К решению задачи 202

К решению задачи 202

К решению задачи 202

К решению задачи 202

К решению задачи 202


Решения задач районных туров 279

К решению задачи 202

правлению поля. Предложенную в задаче систему можно


разбить на куски, состоящие из прямолинейных проводов в
однородном поле, найти силы, действующие на каждый ку-
сок и затем построить векторную сумму всех сил. Построе-
ние облегчается симметрией задачи — результирующая си-
ла будет перпендикулярна плоскости AA , так что достаточ-
но для каждого куска провода искать проекцию силы Ампе-
ра на ось OX. Такой способ решения, хотя и является гро-
моздким, приводит к правильному ответу. Можно, однако,
заметить, что выражение для проекции силы Ампера на ось
OX, F = IBL cos a, где a — угол между направлением силы
Ампера и осью OX (и, одновременно, угол между проводом
и направлением AA — см. рис. 2) не меняется, если вме-
сто исходного провода рассмотреть перпендикулярный OX
провод длиной l cos a, по которому течёт ток I. Например,
проекция силы Ампера не изменится, если провод ab на
рис. 2 заменить проводом cd с таким же током. Заменим все
провода системы по этому принципу. Провода 1 − 3 − 5 − 2 и
1 − 4 − 2 (см. рис. 1) поменяются на куски 1 − 2, расположен-
ные справа от AA ; 2 − 7 − 1 и 2 − 6 − 1 поменяются на кус-
ки 2 − 1, расположенные слева от AA (см. рис. 2, 3). Для
четырёх образовавшихся проводов несложно найти проек-
ции сил Ампера на OX. Для каждого провода проекция рав-
на −4IBd, поэтому полная сила равна 16IBd и направлена
против оси OX.
О т в е т. Полная сила, действующая со стороны поля на
провод равна 16IBd.
205. Предположим, что бусинки не находятся на краю
перекладины. На каждую бусинку действуют FK = kq2 /r2 —
сила кулоновского отталкивания со стороны второй бусин-
280 Решения задач

К решению задачи 204

К решению задачи 204

К решению задачи 204


Решения задач районных туров 281

ки (r — расстояние между бусинками), FЛ = qBwR — сила


Лоренца, притягивающая бусинку к центру вращения (R —
расстояние от бусинки до точки O), N сила реакции стерж-
ня, перпендикулярная стержню. Сумма всех этих сил долж-
на обеспечивать центростремительное ускорение бусинки,
т. е. должна быть равна FЦ = mw2 R и направлена к центру
вращения O. В проекции на направление стержня силе Ку-
лона, отталкивающей бусинки друг от друга, противосто-
ит проекция силы Лоренца, стремящаяся их сблизить. По
условию задачи магнитное поле велико, значит, сила Куло-
на не сможет растолкать бусинки так, чтобы хоть одна из
них сместилась до края перекладины. Таким образом, по-
ложение равновесия бусинок — симметричное относитель-
но ножки каркаса, причём бусинки не достигают его края.
Обозначим через a — угол между ножкой каркаса и линией,
соединяющей бусинку с точкой O. Расстояние между бусин-
ками r = 2l tg a, бусинки вращаются по окружности радиу-
са R = l/ cos a. Условие движения бусинки по окружности в
проекции на направление стержня имеет вид

−kq2 /r2 + qwBR sin a = mw2 R sin a

и может быть переписано в виде tg3 a = kq2 /4l3 w(qB − mw).


При сильном магнитном поле правая часть последнего ра-
венства мала, значит, искомый тангенс также мал. Заме-
няя tg a на сам угол a, получаем a ∼= (kq2 /4l3 w(qB − mw))1/3 .
Полученное решение устойчиво: если рассмотреть отклоне-
ние бусинки из положения равновесия (например, к краю
перекладины), можно заметить, что сила Кулона умень-
шится, а сила Лоренца увеличится, так что возникнет воз-
вращающая к положению равновесия результирующая си-
ла. Случай смещения бусинки в противоположном направ-
лении рассматривается аналогично. Характер зависимости
равновесного угла a от величины круговой частоты вра-
щения показан на рисунке. Приведённый график постро-
ен в обезразмеренных величинах, нормировочный множи-
тель A = (kq2 m/4)1/3 /l, циклотронная частота W = qB/m. Ес-
ли каркас не вращается, бусинки движутся только под дей-
ствием расталкивающей силы Кулона, смещающей их на
противоположные края проволоки. Если каркас вращается,
282 Решения задач

К решению задачи 205

на движущиеся в магнитном поле бусинки действует сила


Лоренца, сводящая бусинки друг к другу. Наименьшее рас-
стояние между ними достигается, когда круговая частота
вращения каркаса равна половине циклотронной частоты.
При дальнейшем увеличении угловой скорости вращения
каркаса нарастающая сила реакции проволоки, действую-
щая на бусинки, начинает существенно противодействовать
силе Лоренца, и в момент, когда угловая скорость достига-
ет величины циклотронной частоты, полностью компенси-
рует её. В этот момент движение бусинок снова определяет-
ся только действием силы Лоренца, расталкивающей их к
противоположным упорам проволоки.
Если заменить знак заряда одной из бусинок, действу-
ющая на неё сила Лоренца будет сдвигать бусинку к краю.
Так как магнитное поле по условию сильное, то сила Ку-
лона (теперь притягивающая отрицательно заряженную бу-
синку к ножке) не сможет ей противодействовать. Значит,
отрицательный заряд сместиться на самый край проволоки
и остановится там лишь под действием силы реакции упо-
ра на конце перекладины. Положительный заряд при этом
останется практически у ножки. Возможна также ситуация
когда заряды слипнутся у ножки каркаса. Какая из этих
двух ситуаций реализуется, зависит от начального располо-
жения бусинок.
206. Бесконечная металлическая пластина создаёт по-
ле E = s/2e0 . Однако, заряженный шарик влияет на рас-
пределение электронов на пластине, создавая в ней своё
Решения задач районных туров 283

«изображение». Если шарик с зарядом −q находится на


расстоянии l от пластины, то поле пластины со стороны
шарика, согласно методу зеркальных изображений, будет
суперпозицией поля E и поля точечного заряда +q, нахо-
дящегося за пластиной симметрично шарику. Когда ша-
рик находится в точке A, его расстояние до пластины рав-
но l = (L − 2pR)/4. Таким образом, энергия электростатиче-
ского взаимодействия пластины и шарика в точке A рав-
на W A = −qE · l + W0 , где W0 = −k · q2 /(4 · l). Когда шарик на-
ходится в точке B, его энергия взаимодействия с пласти-
ной WB изменится: первое слагаемое целиком перейдёт в
кинетическую энергию. Второе слагаемое тоже изменит-
ся, но вычислить это изменение мы не можем, посколь-
ку формально в точке B выражение для W0 (l = 0) обраща-
ется в бесконечность. Обозначим изменение энергии, свя-
занное с этим слагаемым, через DW . Тогда часть энергии
WB − W A = qE · l + DW перешла в кинетическую энергию лен-
ты. Рассматривая аналогично перемещение шарика из точ-
ки B в точку C получим, что на этом этапе в кинетиче-
скую энергию перешло WC − WB = qE · l − DW . За половину
оборота транспортёра, когда шарик переместился из точки
A в точку C, лента приобрела кинетическую энергию 2qE · l.
За 10 оборотов лента транспортёра приобретёт кинетиче-
скую энергию 40qE · l = MV 2 /2, что соответствует скорости
V = (80qE · l/M)1/2 .
О т в е т. Через 10 оборотов скорость транспортёра станет
равна 
10q · s(L − 2p · R)
V= .
Mε0

207. Переменное магнитное поле порождает вихревое


электрическое, которое, воздействуя на заряженные части-
цы в течение времени Dt, сообщает им некоторую ско-
рость. После того, как магнитное поле полностью вклю-
чилось, разогнанные электрическим полем частицы начи-
нают двигаться в нём вдоль некоторых окружностей. Рас-
смотрим в качестве контура окружность радиуса x в ка-
тушке. Обозначим величину напряжённости вихревого по-
ля на этой окружности |boldsymbolE(x). Это поле созда-
ёт ЭДС индукции в нашем контуре, равную по модулю
284 Решения задач

К решению задачи 206

|U| = 2pxE. С другой стороны, ЭДС индукции нашего кон-


тура связана с изменением потока магнитного поля через
контур по закону U = −DФ/Dt = −px2 B0 /Dt. Отсюда можно
найти E = B0 /(2x · Dt). На заряженную частицу, находящу-
юся на расстоянии x от центра, таким образом, действует
электрическая сила FKL = qE, которая за время Dt прида-
ёт заряду импульс p = FKL · Dt = qx · B0 /2. Если задана масса
частицы m, то её скорость V = qxB0 /2m. Далее частица бу-
дет двигаться под действием силы Лоренца qVB0 по окруж-
ности радиуса x/2 с одинаковой угловой скоростью. Пред-
положим, что через некоторое время все частицы, двига-
ясь каждая по своей окружности с центрами в точках A1
и A2 , преодолели угол a и оказались в точках B1 и B2 .
Тогда все они будут лежать на одной прямой, образующей
угол a/2 с прямой, соединявшей их до начала движения
(∠B1 OA1 = ∠B2 OA2 = a/2, как углы опирающиеся на дугу с
центральным углом). Итак, все частицы будут находиться
на одной прямой.
О т в е т. Траектории частиц суть окружности различных
радиусов, соприкасающиеся в точке O. Все частицы в каж-
дый момент времени расположены вдоль одной прямой.
208. Пусть Ф и Ф — магнитные потоки, пронизыва-
ющие один виток питающей обмотки трансформатора при
подведении напряжения в первом и втором случаях соот-
ветственно. Если пренебречь падением напряжения на ак-
тивном сопротивлении обмотки, оказывается, что всё под-
ведённое к питающей обмотке входное напряжение компен-
Решения задач районных туров 285

К решению задачи 207

К решению задачи 207

К решению задачи 207


286 Решения задач

сируется возникающей в ней ЭДС индукции U1 = n1 · DФ/Dt.


Выходное напряжение равно ЭДС индукции в другой обмот-
ке. При его вычислении следует учесть, что из-за особен-
ности устройства сердечника каждый виток вторичной об-
мотки пронизывается магнитным потоком, вдвое меньшим,
чем в первичной U = n2 · D(Ф/2)/Dt. Коэффициент транс-
формации оказывается вдвое меньшим, чем в «стандарт-
ном» трансформаторе с таким же отношением обмоток,
U1 /U = 2n1 /n2 . Проводя аналогичные рассуждения в случае
обратного включения трансформатора, приходим к похоже-
му выражению для отношения напряжений в этом случае:
U1 /U = n1 /2n2 . Неизвестное отношение числа витков n1 /n2
может быть исключено из выражений для коэффициентов
трансформации, что даёт для выходного напряжения при
обратном включении U1 = U1 /4.
О т в е т. При обратном включении трансформатора вы-
ходное напряжение составит 10 В.
209. Сила электрического тока, протекающего через со-
противление R2 , легко вычисляется на основании закона
Ома для пассивного участка цепи i2 = (f3 − f4 )/R2 . Анало-
гично рассчитывается ток через сопротивление R1 : i1 = (f2 − f4 )/R1 .
При вычислении алгебраической суммы токов, втекающих
в устройство C, необходимо учесть токи, ответвляющиеся
на сопротивления R1 и R2 : IS = I1 + (I3 − i2 ) − (I2 + i1 ). Вели-
чина заряда, накапливаемая на устройстве за заданное вре-
мя, равна произведению разности втекающих и вытекаю-
щих в него токов на длительность временного промежутка
Q = IS · t.
О т в е т. Величина заряда, накапливаемая на устройстве
за время t, равна
 
f3 − f4 f2 − f4
Q = t · I1 + I3 − I2 − − .
R2 R1

210. Попытка решения задачи на основе законов Нью-


тона привела бы к существенным математическим труд-
ностям. Воспользуемся законом сохранения механической
энергии K + U = const, учтя кинетическую энергию движе-
ния частиц K и потенциальную энергию электростатиче-
ского взаимодействия между ними U (электростатические
силы являются потенциальными; эффектами, связанными
Решения задач районных туров 287

и излучением ускоренно движущимися зарядами прене-


брегаем), и законом сохранения проекции суммарного им-
пульса системы на направление возможного движения ча-
стиц — ось x (вдоль этого направления действуют толь-
ко внутренние силы). Из сохранения x-проекции импуль-
са (PS )X = const и условия равенства масс частиц следует,
что сумма скоростей частиц должна сохраняться во време-
ни. Поскольку в случае нестационарного решения, соответ-
ствующего «неупругому столкновению» скорости обеих ча-
стиц должны стать одинаковыми, их величина должна со-
ставлять половину начальной скорости налетающей части-
цы. В этом случае закон сохранения энергии приобретает
вид:
mV02 m(V0 /2)2 q1 · q2
=2· +k ,
2 2 L
откуда немедленно следует условие для величин зарядов ча-
стиц q1 · q2 = L · mV02 /4k. Частицы должны иметь одноимён-
ные заряды. Если это условие не выполняется, система при-
дёт в стационарное состояние лишь когда частицы находят-
ся на бесконечно большом расстоянии друг от друга, что со-
ответствует обращению в нуль потенциальной энергии их
взаимодействия. В этом случае законы сохранения энергии
и импульса формально принимают тот же вид, что и в слу-
чае абсолютно упругого лобового столкновения двух шаров
одинаковой массы. Возможное решение такой задачи хоро-
шо известно из элементарного курса физики: одно из тел
должно остановиться, а другое, в итоге, приобретёт началь-
ную скорость первого.
Начнём с рассмотрения случая одноимённо заряженных
частиц. Поскольку между ними действуют силы отталки-
вания, остановка исходно двигавшейся частицы возмож-
на только, если она не обгонит покоившуюся (в против-
ном случае, после прохождения точки максимального сбли-
жения, сила отталкивания начнёт ускорять первую части-
цу). При условии q1 · q2 > L · mV02 /4k, соответствующему бо-
лее эффективному отталкиванию частиц, имеем: {V1 (∞) = 0,
V2 (∞) = V0 }. При обратном неравенстве, q1 · q2 < L · mV02 /4k,
реализуется решение {V2 (∞) = 0, V1 (∞) = V0 }. Налетающая
из бесконечности частица обгонит покоившуюся и после
некоторого торможения, сопровождающего сближение ча-
288 Решения задач

стиц, начнёт вновь разгоняться вплоть до приобретения сво-


ей начальной скорости.
В случае частиц, имеющих разноименные заряды, ки-
нетическая энергия налетающей из бесконечности частицы
будет возрастать на этапе сближения и уменьшаться при
разлёте, её изменение равно работе электростатических сил.
Из симметрии следует, что работы этих сил при разгоне и
торможении будут равны друг другу по абсолютной вели-
чине. Следовательно, налетающая частица, в конечном ито-
ге, приобретёт прежнюю скорость.
Приведённые рассуждения допускают наглядную гео-
метрическую интерпретацию. Дело в том, что закон сохра-
нения энергии задаёт симметричную поверхность в про-
странстве с координатами {q1 , q2 , V0 }, которая называется
гиперболоидом. Часть этой поверхности (c V0 > 0) представ-
лена на рисунке. Каждое горизонтальное сечение такой фи-
гуры, соответствующее некоторому уровню кинетической
энергии налетающей частицы, представляет собой две ги-
перболы в плоскости {q1 , q2 }. Каждая система двух частиц
с зарядами q1 и q2 задаётся точкой в плоскости {q1 , q2 }. Ес-
ли соответствующая точка попадает в пространство между
гиперболами (крыльями гиперболоида, изображёнными на
рисунке), то частицы асимптотически не взаимодействуют,
возвращаясь на бесконечности в свои исходные состояния.
Если точка попадает внутрь одного из крыльев гиперболо-
ида — частицы обмениваются состояниями. Наконец, если
точка оказывается в точности на поверхности гиперболои-
да, взаимодействующие частицы делят начальную кинети-
ческую энергию поровну.
О т в е т. Если заряды частиц таковы, что q1 · q2 < L · mV02 /4k —
налетающая из бесконечности частица восстановит свою на-
чальную скорость, а первоначально покоившаяся частица
остановится. Если q1 · q2 = L · mV02 /4k, то обе частицы при-
обретут одинаковые скорости, равные половине начальной.
Наконец, в случае q1 · q2 > L · mV02 /4k, налетающая частица
остановится, а её скорость приобретёт покоившаяся.
211. Поскольку шарик движется по дуге окружности,
рассматриваемая механическая система подобна маятнику,
укреплённому на невесомой нерастяжимой нити длиной R.
Роль натяжения нити играет сила реакции опоры, направ-
Решения задач районных туров 289

К решению задачи 210

ленная перпендикулярно проволоке, т. е. к центру кольца.


При отсутствии электрического поля положение устойчиво-
го равновесия шарика находится в нижней точке кольца,
вблизи него и возможны колебания. Их уравнение имеет
стандартный вид:
ma = mg + N.
В случае малой амплитуды происходят гармонические ко-
√ √
лебания с круговой частотой, w0 = g/ R. При включении
электрического поля уравнение движения шарика приобре-
тает более сложный вид:
ma = mG + N, G = g + qE/m.
Частота малых
√ √ колебаний в этом случае определяется фор-
мулой w1 = G/ R, где ускорение свободного падения заме-
нено на модуль вектора G (сумма гравитационной и элек-
трической силы может быть направлена как вниз, так и
вверх). Условие уменьшения частоты колебаний вдвое при-
водит к очевидному уравнению на величину напряжённости
электрического поля E:
|G| = |g + qE/m| = g/4.
Оно имеет два решения, соответствующие противополож-
ным направлениям вектора G. Первое решение, E = −3 · mg/4q,
290 Решения задач

соответствует колебаниям шарика в нижней части коль-


ца: g + qE/m = g/4. Второе решение, E = −5 · mg/4q, — ко-
лебаниям шарика в верхней точке кольца: g + qE/m = −g/4.
Заметим, что в отличие от обычного математического ма-
ятника, для которого верхнее положение является всегда
неустойчивым, при включении электрического поля, верх-
нее положение может стать устойчивым. Знак «−» в полу-
ченных выражениях напоминает о том, что электрическое
поле должно быть направлено так, чтобы обусловленная им
сила была противоположна силе тяжести.
О т в е т. Возможно два значения величины вектора на-
пряжённости электрического поля: E = 3 · mg/4q или E = 5 · mg/4q.
При q > 0 вектор E должен быть направлен вверх, в против-
ном случае — вниз.
212. Неизбежное затухание электрического сигнала
(разности потенциалов между сердцевиной нервного волок-
на и окружающей средой) вследствие конечного сопротив-
ления клеточной мембраны приводит к необходимости су-
ществования механизма, усиливающего нервный импульс
(создание хорошей изоляции нервных волокон привело бы
к существенному утяжелению нервной системы). Как те-
перь хорошо известно, роль такого усилителя в нервных
клетках играют несбалансированные друг с другом встреч-
ные диффузные потоки ионов Na и K, регулируемые кле-
точной мембраной, пропускание ионов которой носит се-
лективный характер. Современные модели нервных аксонов
значительно сложнее описанной простейшей схемы (напри-
мер для правильного описания распространения коротко-
го импульса в волокне необходим учёт наличия электри-
ческой ёмкости между его внутренней проводящей частью
и внешней средой). Даже в рамках сделанных в условии
приближений строгий расчёт входного сопротивления си-
стемы представляет весьма сложную задачу, по-видимому,
решаемую только численными методами. Однако, в слу-
чае малой проводимости стенки, вполне правдоподобные
оценки сопротивления можно получить, мысленно заменяя
длинное непрерывное волокно бесконечной системой корот-
ких ячеек (колец) со стремящейся к нулю толщиной DL,
каждая из которых содержит по два дискретных элемента:
«продольное» сопротивление сердцевины волокна и «попе-
Решения задач районных туров 291

К решению задачи 212

К решению задачи 212

речное» сопротивление утечки через мембрану. «Продоль-


ное» сопротивление сердцевины такого кольца даётся вы-
ражением DR1 = r · DL/pr2 , а «поперечное» сопротивление
утечки — выражением DR2 = r0 · h/(2pr · DL). Эквивалент-
ная электрическая схема всего волокна имеет вид бесконеч-
ной цепочки, содержащей одинаковые звенья. Замена такой
цепочки на ещё более простую эквивалентную схему поз-
воляет получить уравнение для входного сопротивления:
R = DR1 + R · DR2 /(R + DR2 ).
Очевидно, что из двух корней последнего уравнения фи-
зический смысл имеет только положительный корень:

DR1 DR21
R= + + DR1 · DR2 .
2 4

Устремляя к нулю длину звена цепочки, находим входное


сопротивление волокна R и втекающий в него ток I = U0 /R.
О т в е т. Входное сопротивление нервного волокна в рас-
сматриваемой модели равно R = (rr0 h/2p2 r3 )1/2 . Ток равен
I = U0 · (2p2 r3 /rr0 h)1/2 .

К решению задачи 212


292 Решения задач

213. Обозначим Vz — проекцию начальной скорости


частицы на ось Oz. Опишем характер движения частицы
до соударения с плоскостью. Вертикальная проекция ско-
рости Vz = V sin a сохраняется, так как в проекции на эту
ось не действуют внешние силы. В проекции на горизон-
тальную плоскость частица движется по окружности с по-
стоянной скоростью Vxy = V cos a. Таким образом, траекто-
рия движения частицы до столкновения — спираль. Ско-
рость частицы по модулю постоянна, так как магнитная си-
ла не совершает работы. Найдём параметры траектории ча-
стицы. Движение по окружности радиуса R = Vm · cos a/qB
с периодом обращения T = 2pm/qB в плоскости Oxy обес-
печивается силой Лоренца, qB · Vxy = mVxy 2 /R. После столк-
новения с плоскостью траектория частицы осталась спира-
лью. Вертикальная компонента скорости в процессе удара
изменила своё направление на противоположное. Так как
заряд частицы уменьшился вдвое, радиус спирали и пери-
од обращения увеличился в 2 раза: R = 2R, T = 2T. Итак,
до столкновения траектория частицы — спираль, намотан-
ная на вертикальный цилиндр радиуса R, после столкнове-
ния траектория частицы — спираль, намотанная на верти-
кальный цилиндр радиуса 2R. По условию задачи цилин-
дры имеют общую точку с координатами {x = 0, y = 0, z = 0}
(точка старта, там, где траектории заведомо пересекаются).
Следовательно, цилиндры касаются друг друга, имея об-
щую вертикальную прямую, проходящую через начало ко-
ординат. Поэтому понятно, что траектории до и после со-
ударения с плоскостью могут пересекаться только над точ-
кой старта. Частица до соударения оказывалась над точ-
кой старта каждый раз, совершив полный оборот, т. е. че-
рез моменты времени T, 2T, 3T, 4T, . . . , nT после нача-
ла движения (n — целое число оборотов, которое частица
совершит до соударения). Зная вертикальную компоненту
скорости частицы, легко понять, что частица до соударе-
ния окажется в эти моменты над точкой старта на высоте
TV · sin a, 2TV · sin a, 3TV · sin a, 4TV · sin a, . . . , nTV · sin a
соответственно, причём верхняя точка nTV · sin a совпада-
ет по высоте с расположением плоскости nTV · sin a = H.
После соударения период обращения частицы увеличил-
Решения задач районных туров 293

К решению задачи 213

ся в 2 раза, следовательно, она будет совершать полный


оборот в 2 раза медленнее и пролетает над точкой стар-
та в 2 раза меньшее количество раз (n/2 раз), а имен-
но в точках H − 2TV · sin a = (n − 2)TV · sin a, (n − 4)TV · sin a,
(n − 6)TV · sin a, . . . , 2TV · sin a, 0. Во всех этих точках тра-
ектория частицы пересечёт саму себя.Так как число этих то-
чек по условию равно N, следовательно, n = 2N.
О т в е т. Плоскость расположена на высоте H = 2NTV · sin a = 4pm · NV · sin a/
214. Обозначим, x — расстояние между частицей в по-
ложении равновесия и стержнями, h — глубина, на кото-
рой частица будет находиться под стержнями, когда её бу-
дут поднимать, Q — искомый заряд частицы. Если стерж-
ни медленно поднимать, возможно, что частица будет нахо-
диться в состоянии равновесия под действием силы тяже-
сти mg и кулоновских сил притяжения к каждому стерж-
ню FKL . Итак, на частицу действуют силы, изображённые
на следующем рисунке, причём FKL = s · |Q|/2pε0 x. Чтобы
частица находилась в равновесии, сумма сил, действую-
щих на неё, должна обращаться в ноль. В проекции на
горизонтальную ось это условие тривиально выполняется
из симметрии. В проекции на вертикальную ось оно да-
ёт нам mg = 2FKL · cos a = s|Q| · cos a/pε0 x. Подставляя сюда
найденные из геометрических соображений x = (R2 + h2 )1/2
и cos a = h/x, получаем квадратное уравнение для h,

mg = s · |Q|h/2pε0 (R2 + h2 )1/2 .


294 Решения задач

К решению задачи 213

Уравнение имеет два корня — положительный и отрица-


тельный. Отрицательный корень соответствует случаю, ко-
гда частица расположена выше стержней — он неустойчи-
вый. Положительный корень,
mg
h+ = R r .
s 2 Q2
− mg2
p2 ε2

устойчив и действителен (а следовательно, физичен), когда


m < s 2 Q2 /p2 ε20 g2 .
Уточним вопрос устойчивости корней. Исследуем, как
ведёт себя правая часть уравнения, при случайном смеще-
нии частицы из положения равновесия. На графике пока-
зано поведение функции

f(h/R) = (h/R)/(1 + (h/R)2 )1/2 ,

выражающей функциональную зависимость от h правой ча-


сти полученного уравнения. Равновесие устойчиво, если
при малом увеличении h (при опускании частицы) функ-
ция f увеличивается (кулоновская сила начинает сильнее
тянуть вверх частицу), и наоборот, при небольшом умень-
шении h (при поднимании частицы) функция f уменьшает-
ся (частицу начинает тянуть вверх слабее). Очевидно, что
устойчивым является только положительный корень h+ .
О т в е т. Частицу можно поднять, медленно поднимая
стержни вверх, если Q > m1/2 pε0 g/s.
215. После соединения сфер проводником заряды на
первой и третьей сферах перераспределятся так, чтобы по-
тенциалы соединённых сфер сравнялись. В соответствии
Решения задач районных туров 295

К решению задачи 213

К решению задачи 213


296 Решения задач

с этим перераспределением зарядов изменится и полная


энергия системы. Задачу удобно решать, воспользовавшись
энергетическими соображениями. Вычитая из начальной
энергии (когда сферы не соединены) системы конечную
энергию, которая получится после соединения, найдём вы-
делившееся количество теплоты. В соответствии с теоремой
Гаусса равномерно заряженная сфера радиуса R с зарядом
Q создаёт на расстояниях r = R такой же потенциал, как ес-
ли бы весь заряд располагался в центре. На поверхности
сферы (r = R) этот потенциал обращается в f(R) = kQ/R и
при дальнейшем приближении к центру остаётся постоян-
ным и равным kQ/R (так же в соответствии с теоремой Гаус-
са). Пусть на сферах радиусов R1 , R2 , R3 (считая от центра)
находятся заряды q1 , q2 , q3 соответственно. Найдём, какой
потенциал создаёт эта система зарядов. Для нашей систе-
мы потенциал в каждой точке складывается из потенциа-
лов, которые создаёт в этой точке каждая из сфер. В области
r = R1 первая сфера создаёт потенциал kq1 /R1 , вторая — по-
тенциал kq2 /R2 , третья — потенциал kq3 /R3 . Таким обра-
зом, в области r = R1 потенциал системы равен
kq1 kq2 kq3
Y1 (q1 , q2 , q3 ) = + + .
R1 R2 R3

В области r = R2 потенциалы второй и третьей сфер останут-


ся теми же, а потенциал первой сферы станет равен kq1 /R2
(так как эти точки лежат уже снаружи от первой сферы).
Поэтому
kq kq kq
Y2 (q1 , q2 , q3 ) = 1 + 2 + 3 .
R2 R2 R3
Аналогично, в области r = R3 потенциал системы равен
kq1 kq2 kq3
Y3 (q1 , q2 , q3 ) = + + .
R3 R3 R3

Найдём энергию этой системы зарядов. Заряд q1 первой


сферы находятся в потенциале Y1 , заряд q2 второй сфе-
ры — в потенциале Y2 , заряд q3 третьей сферы — в по-
тенциале Y3 .
q Y +q Y +q Y kq21 kq22 kq23 kq1 q2 kq1 q3 kq2 q3
E= 1 1 2 2 3 3 = + + + + + .
2 2R1 2R2 2R3 R2 R3 R3
Решения задач районных туров 297

Здесь мы учли, что просто суммируя энергию всех заря-


дов, мы учтём энергию каждой пары зарядов дважды, по-
этому в предыдущей формуле есть множитель 1/2. Подстав-
ляя в неё значения q1 = q, q2 = −q, q3 = q и R1 = R, R2 = 2R,
R3 = 3R, получаем начальную энергию нашей системы —
EH = 5/12 · kq2 /R. Чтобы узнать, как перераспределились
заряды, нужно приравнять потенциалы Y1 и Y3 , положив
q2 = −q (заряд второй сферы не изменился), и
kq1 kq kq3 kq1 kq kq3
− + = − +
R1 R2 R3 R3 R3 R3

и q3 + q1 = 2q. Отсюда можно найти, что q1 = q · R1 (R3 − R2 )/R2 (R3 − R1 ) = q/4


и q3 = q + q · R3 (R2 − R1 )/R2 (R3 − R1 ) = 7/4 · q. Выражение для
полной энергии системы в конце при этом даёт EK = 11/48 · kq2 /R.
Разность EK − EH даёт выделившуюся теплоту.
О т в е т. При соединении внешней и внутренней сфер
выделяется теплота Q = 3/16 · kq2 /R.
216. Рассмотрим силы, действующие на бусинки. На
первую, ближайшую к оси вращения бусинку действуют
две силы, направленные к оси: сила Лоренца, qBw · x1 ,
и кулоновская сила взаимодействия между бусинками,
kq2 /(x22 − x21 ), (здесь x1 и x2 — расстояния от оси до стацио-
нарного положения первой и второй бусинок соответствен-
но). На вторую бусинку также действуют две силы — сила
Лоренца, qBw · x2 , (направленная к оси) и кулоновская си-
ла, kq2 /(x2 − x1 )2 , (направленная от оси). В стационарном
состоянии бусинки движутся по окружностям. В проекции
на спицу, равенства сил выглядят так:
kq2 kq2
m1 w2 x1 = qBwx1 + , m2 w2 x2 = qBwx2 − .
(x2 − x1 )2 (x2 − x1 )2

Перепишем полученные уравнения в следующем виде:


1 1
b1 x1 = , b2 x2 = − ,
(x2 − x1 )2 (x2 − x1 )2

в котором мы ввели параметры b1 = m1 · w2 /kq2 − Bw/kq и


b2 = m2 · w2 /kq2 − Bw/kq. Параметр b1 > 0, если частота вра-
щения w > W1 = q · B/m1 — циклотронной частоты первой бу-
синки, а при w < W1 параметр b1 < 0. Итак, в зависимости от
298 Решения задач

того, как соотносится частота вращения бусинок w с харак-


терными частотами задачи W1 и W2 (W1 < W2 ), меняются зна-
ки параметров b1 и b2 , а значит и решения системы уравне-
ний. При w > W2 (b2 > 0) действительных решений у системы
не существует, так как левые части уравнений движения по-
ложительны, а правые — отрицательны. Поэтому при боль-
ших w обе частицы улетают на бесконечность. При малых
частотах вращения, w < W1 , (b2 < 0, b1 < 0) первое уравнение
системы неприменимо. Первая частица установится на оси
вращения (x1 = 0), причём кулоновскую силу, действующую
на неё, будет компенсировать сила реакции оси, неучтён-
ная в самом первом уравнении. Разрешая второе уравнение
движения, которое остаётся справедливым, получим ответ:

kq2
x1 = 0, x2 =
3
.
qBw − m2 w2
Чтобы убедиться в устойчивости полученного решения,
необходимо рассмотреть малый сдвиг частицы в окрестно-
сти найденной точки равновесия x1,2 = x1,2 + dx1,2 , разло-
жить члены уравнений движения в ряд по dx1,2 и убедить-
ся, что ведущая поправка по dx1,2 соответствует возвращаю-
щей силе. Такой анализ лежит за пределами школьной про-
граммы, поэтому ниже мы ограничимся лишь некоторыми
оценочными соображениями об устойчивости рассматрива-
емой системы. Первая частица удерживается силой реак-
ции оси и прижимающей к оси кулоновской силой. Положе-
ние частицы устойчиво. Сдвинем немного вторую частицу
от центра. Тогда, отталкивающая её от центра сила Кулона
ослабнет, а прижимающая к центру сила Лоренца увеличит-
ся, в результате, частица сместится обратно к центру. При
случайном сдвиге к центру — ситуация противоположна.
Положение второй частицы также устойчиво. Осталось рас-
смотреть ситуацию, когда W1 < w < W2 (b2 < 0, b1 > 0). Скла-
дывая уравнения
1
b1 x1 + b2 x2 = 0, b2 x2 = − ,
(x2 − x1 )2
откуда находим:
 
b1 b21 b22
x2 = − x2 = x1 =
3 3
x , 2, .
b2 1 −b2 (b1 + b2 ) b1 (b1 + b2 )2
Решения задач районных туров 299

Полученное решение является неустойчивым. Действитель-


но, если рассматривать обе частицы как единую систему,
то, по отношению к этой системе, кулоновская сила оказы-
вается внутренней, и уравнение движения для такой систе-
мы получается сложением исходных уравнений. Если сдви-
нуть обе частицы на одинаковое расстояние D от центра, то
суммарная сила Лоренца уже не сможет обеспечить центро-
стремительное ускорение:

b1 (x1 + D) + b2 (x2 + D) > 0

так как по результатам предыдущего пункта b1 > |b2 |. В этом


случае, обе частицы уйдут на бесконечность.
О т в е т. Единственным устойчивым положением бу-
синок будет следующее: x1 = 0, x2 = [kq2 /(qBw − m2 w2 )]1/3 .
Оно реализуется при малых частотах вращения спицы,
w < qB/m1 . Если частота вращения спицы превосходит цик-
лотронную частоту первой бусины — бусины уходят на бес-
конечность.
217. Электромагнитное равновесие на Ио наступит, ко-
гда сумма сил, действующих на любой свободный электрон,
присутствующий в проводящем веществе атмосферы Ио, бу-
дет равна нулю, т. е. если электрическая (кулоновская) и
магнитная (лоренцевская) силы будут равны по модулю,
q · E = q · BV , где q — заряд электрона, E — величина элек-
трического поля, V — скорость свободного электрона атмо-
сферы Ио в магнитном поле Юпитера B. Диаметр Ио равен
d = 2R = 3640 км = 3,64 · 106 м; скорость Ио относительно
магнитного поля Юпитера суть V = VR − VK = 5,77 · 104 м/с.
О т в е т. Искомая разность потенциалов равна U = E · d = B · (VR − VK )d ∼
= 420 к
218. Траектория частицы внутри цилиндра состоит
из кусков окружности. Если радиус этой окружности ра-
вен r, а скорость частицы V , то mV 2 /r = qVB. Посколь-
ку шарик возвращается в исходную точку, точки после-
довательного соударения частицы c цилиндром A и B
должны быть видны из центра цилиндра O под углом
2b = 2pk/s, где k и s — целые произвольные числа. Очевид-
но a = 90 ◦ C − b (частица влетает перпендикулярно к поверх-
ности), R · sin b = r · sin b ⇒ r = R · tg(pk/s). Откуда получают-
ся искомые значения скоростей, V = q · BR · tg(pk/s)/m, где
300 Решения задач

К решению задачи 217

k, s — любые целые числа. Перебирая целые k и s, можно


добиться, чтобы со сколь угодной точностью tg(pk/s) был
равен любому наперёд заданному действительному числу.
О т в е т. Возврат шарика в точку A может произойти
при любой скорости.
219. Хотя капля ртути и маленький шарик — тела
с несхожими механическими свойствами, тем не менее, в
сформулированных условиях, траектории обоих тел ока-
зываются одинаковыми. Хотя, при каждом ударе о плос-
кость капля ртути полностью теряет свою скорость, между
столкновениями её движение аналогично движению шари-
ка до первого удара о плоскость (сравните рисунки к ре-
шению). После каждого удара о пластину заряды шарика
и капли изменяют свой знак, но остаются постоянными по
модулю (i — номер удара шарика или капли о пластину
qi = ±2Q. Ускорение шарика и капли при движении меж-
ду пластинами после удара равно a = g + qi · E/m. Измене-
ние проекций скорости (и ускорения ) шарика при ударе о
пластину (ось OX направлена горизонтально, OY — верти-
кально) суть VX = −VX , VY = VY . Поскольку начальная ско-
рость равнялась нулю, траектория шарика до первого уда-
ра является прямой, направленной вдоль равнодействую-
щей сил тяжести и электрического поля, т. е. составляю-
щая угол f = arctg(qE/mg) с горизонтом. При каждом ударе
о пластину векторы скорости и ускорения шарика одина-
ково изменяют своё направление относительно вертикали,
не изменяя своей величины. Они остаются сонаправленны-
ми, а траектория шарика представляет собой ломаную, со-
стоящую из отрезков прямых, наклонённых к горизонтали
Решения задач районных туров 301

К решению задачи 219

К решению задачи 219

под углами ±f. При каждом ударе о пластину капля ртути


полностью теряет свою скорость. Между столкновениями её
движение аналогично движению шарика до первого удара
о пластину. Таким образом, траектория капли — ломаная,
состоящая из отрезков, наклонённых к горизонту под угла-
ми ±f. Поскольку траектории (но не скорости движения по
ним!) шарика и капли совпадают, оба тела испытают одина-
ковое число столкновений с вертикальными плоскостями:
NК = NШ .
О т в е т. Капля ударится о вертикальные плоскости N
раз, столько же раз сколько и шарик.
220. Индукция магнитного поля — псевдовекторная
величина, её направление определяется избранной систе-
мой отсчёта. Псевдовекторной величиной является и си-
ла Лоренца, действующая на заряженные тела, движущи-
еся в магнитном поле. В зависимости от знака заряда на-
правление силы Лоренца меняется на p. Это, в частно-
сти, приводит к тому, что при решении данной задачи тре-
буется учесть два случая: положительного и отрицатель-
302 Решения задач

ного заряда. Заряженная частица, влетающая в однород-


ное магнитное поле перпендикулярно его линиям, движет-
ся по дуге окружности, R = qV /mB. В рассматриваемой си-
туации траектория частицы состоит из полудуг окружно-
стей. Радиусы дуг в верхней полуплоскости больше, чем в
нижней, поскольку B2 > B1 . В случае положительного заря-
да частицы (q > 0) центры образующих её траекторию дуг
будут постепенно перемещаться. Пометим как X коорди-
наты точек пересечения траекторией границы раздела по-
лей, X2N = 2(R2 − R1 ) · N, X2N+1 = 2(R2 − R1 ) · N + 2R2 . Выпи-
шем условие попадания в точку M положительно заря-
женной частицы: X2N = L — решений нет; X2N+1 = L —
решение есть при N = 0, . . . , [B1 /(B2 − B1 )] (прямоуголь-
ные скобки обозначают целую часть числа), при этом
q = 2mV · ((N + 1)/B2 − N/B1 )/L. Чтобы число возможных ре-
шений N было конечным, необходимо, чтобы q > 0. В слу-
чае отрицательно заряженной частицы (q < 0), центры об-
разующих её траекторию дуг будут смещаться вправо. Те-
перь, будем обозначать X — координаты точек нечёт-
ных и чётных пересечений границы: X2N = 2(R1 − R2 ) · N,
X2N+1 = 2(R1 − R2 ) · N − 2R2 . Выпишем также аналогичное
условие попадания в точку M для отрицательно заря-
женной частицы: X2N = L — решение есть при любом N,
q = −2mV · (N/B1 − N/B2 )/L; для N > [B1 /(B2 − B1 )], X2N+1 = L),
при этом q = −2mV · (N/B1 − (N + 1)/B2 )/L. Так же как и в
предыдущем случае, требование q < 0 ограничивает набор
допустимых целых значений N, для которых данное реше-
ние имеет смысл. В заключение, объединяя вместе оба ре-
шения, получаем две бесконечные серии решений, справед-
ливые при целых неотрицательных N.
О т в е т. Допустимыми значениями зарядов частицы да-
ются двумя бесконечными сериями решений, справедливых
для неотрицательных N.
221. При протекании электрического тока, на сопро-
тивлении выделяется количество теплоты, пропорциональ-
ное квадрату величины тока I. В задаче предлагается рас-
смотреть процесс выделения тепловой энергии в процес-
се, происходящем в электрическом колебательном конту-
ре. Как обычно, наиболее простое решение задачи со-
стоит в выводе закона сохранения при превращении со-
Решения задач районных туров 303

К решению задачи 220

К решению задачи 220


304 Решения задач

К решению задачи 222

вокупной энергии, запасённой в конденсаторе и катуш-


ке, в тепловую энергию. Начнём с выражений для энер-
гий, запасённых в конденсаторе, WC = q2 /2C, и в катуш-
ке, WL = L · I2 /2. Тепловые энергии, QS = WC + WL , выде-
ляющиеся на резисторе, DQR = R · I2 Dt, и на проводах,
DQr = r · I2 · Dt, за время Dt, подчиняются соотношениям:
QR /Qr = R/r, QR + Qr = WC + WL .
О т в е т. На резисторе выделится тепло QR = (WC + WL ) · R/(R + r).
222. Построим все изображения источника в зеркалах.
Их будет 3: 1) изображение источника в первом (большом)
зеркале, 2) изображение источника во втором (маленьком)
зеркале, 3) изображение в первом зеркале, отражённое вто-
рым зеркалом, совпадающее с изображением во втором зер-
кале, отражённым первым зеркалом (см. рис. 1). Первое
изображение будет видно в области, заштрихованной на
рис. 2. Второе изображение будет видно в области, заштри-
хованной на рис. 3. Третье изображение будет видно в обла-
сти, заштрихованной на рис. 4. Область, где видно 2 изоб-
ражения, заштрихована на рис. 5.
223. Обозначим, D — расстояние от девочки до вит-
рины, d — расстояние между стёклами. Соответствующие
изображения приведены на рисунке. Каждое из стёкол вит-
рины отражает девочку как зеркало. Изображение ak будет
Решения задач районных туров 305

К решению задачи 222

К решению задачи 222


306 Решения задач

К решению задачи 222

К решению задачи 222


Решения задач районных туров 307

К решению задачи 223

казаться девочке меньше, чем изображение bn, так как на-


ходится дальше и видно под меньшим углом. Угол, под ко-
торым девочка видит изображение ak, равен углу, под ко-
торым она видит кусок bm второго изображения, поэтому
bm/bn = h/H = 5/6. Из подобия треугольников Dack и Dbcm
следует ak/bm = ac/bc, т. е. H/h = (2D + 2d)/(2D).
О т в е т. Расстояние между стёклами витрины равно
d = D/5 = 20 см.
224. Из рисунка видно, что тень от линзы пред-
ставляет собой кольцо; внешняя окружность кольца со-
ответствует тени от ободка линзы, а светлое пятно в
середине определяется ходом лучей сквозь собирающую
линзу. Внешняя окружность кольца тени — круг радиу-
са x1 = R(f + a)/a, площадью s1 = (f + a)2 pR2 /a2 . Линза со-
здаёт действительное изображение источника за экраном
на расстоянии a1 от линзы, определяющимся формулой
1/a + 1/a = 1/f, так что a1 = af/(a − f). Поэтому светлое пят-
но на экране также есть круг. Его радиус x2 = R(a1 + f)/a1 ,
площадью s2 = (a1 − f)2 pR2 /a21 . Итого, площадь тени есть
s1 − s2 = (2 + a)pR2 /a = 2pR2 (здесь a = a/f = 2).
О т в е т. Площадь тени равна 2pR2 .
308 Решения задач

К решению задачи 224


Решения задач городских туров

225. Размерность силы (Ньютон) H = кг · м/с2 . В неиз-


вестной стране приняты другие величины, которые мы обо-
значим как H1 , кг1 , м1 , с1 . Выражение для единиц измере-
ния силы в новой системе есть H1 = кг1 · м1 /с21 . Найдём от-
ношение новых и старых единиц силы: H1 /H = (кг1 /кг) · (м1 /м) · (с2 /с21 ).
Из условия задачи кг1 /кг = a, м1 /м = b, с1 /с = g. Подставляя
известные отношения в предшествующую формулу, полу-
чим ответ: H1 /H = ag/b2 .
О т в е т. [Новая ед. силы] = ag/b2 · [Старая ед. силы].
226. Давление лунохода массы mЛ на занимаемую
им лунную поверхность S равно p = mЛ gЛ /S. Обозначим
некоторый «эффективный» размер лунохода lЛ такой, что
mЛ = rЛ VЛ ∼ l3Л . Площадь под луноходом S ∼ l2Л . Введём ана-
логичный эффективный размер для модели lМ и, напи-
сав такие же соотношения для её массы mМ и площа-
ди SМ , получим соотношения подобия: mЛ /mМ ∼ l3Л /l3М и
SЛ /SМ ∼ l2Л /l2М . Теперь остаётся записать аналогичное соот-
ношение подобия для давлений, оказываемых луноходом и
его моделью на грунт на поверхности Луны и в лаборатор-
ных условиях на Земле: pЛ /pМ = (gЛ /gЗ ) · (lЛ /lМ ). Величину
pМ считаем известной, откуда pЛ = pМ · (gЛ /gЗ ) · (lЛ /lМ ).
О т в е т. Размер оригинала лунохода может отличаться
от его земной модели в следующее число раз: lЛ /lМ = gЗ /gЛ ∼ = 6,1.
227. Обозначим скорость Лисы u, скорость Младшего
Брата v, время, за которое Младший Брат оседлал коня, T.
Пока Младший Брат седлал коня, Лиса убежала на рассто-
яние uT. Найдём время t1 , которое Младший Брат провёл
на коне: v · t1 − u · t1 = uT, t1 = u · T/(v − u). Аналогичное вре-
мя для Старшего Брата найдём, заменив v на 2v и T на
2T: t2 = 2u · T/(2v − u). Видно, что t1 > t2 , т. е. Младший Брат
провёл больше времени на коне.
О т в е т. Младший брат дольше гнался за лисой, чем
Старший Брат.
228. Так как движение на каждом из участков равномер-
ное и прямолинейное (по условию), то зависимость коорди-
наты от времени — линейная. Построим эту зависимость
(см. рисунок). Очевидно, что спортсмены поравняются то-
310 Решения задач

К решению задачи 228

гда, когда эти ломаные пересекутся (разумеется, после точ-


ки (0, 0)). По графику нетрудно увидеть, что в этой точке
время равно 200 с.
О т в е т. Спортсмены поравняются через 200 сек.
229. Сначала на столе лежит кубик из пластилина.
Пусть его масса — m, площадь — S, длина ребра — a. В
этом случае давление на стол равно p = mg/S = rП ag. Когда
на пластилиновый кубик положили стальной, масса пласти-
линового кубика не изменилась, добавилась масса стально-
го кубика, равная mС = 27 · rС a3 g. Давление, оказываемое на
стол после того, как пластилиновый куб расплющился, рав-
но p1 = g((3 · a)3 rС + a3 · rП )//2S ∼
= 7571 Па.
О т в е т. Давление на стол стало равно примерно 7571 Па.
230. В момент, когда нить рвётся, цилиндр ещё на-
ходится в состоянии равновесия. Запишем условие равен-
ства сил, действующих на цилиндр. Пусть T — сила на-
тяжения, m — масса цилиндра, H — высота цилиндра,
h — высота его погруженной части, S — площадь его се-
чения, rВ — плотность воды, rТ — плотность тела. То-
гда: T = m · g − rg · V = rТ S · Hg − rВ S · hg = g · S(rТ H − rВ h). Это
уравнение верно для всех трёх планет (разумеется, вместо g
и h надо подставить соответствующие значения этих вели-
чин на планетах). Так как по условию нить рвётся при од-
ной и той же силе натяжения, то можно приравнять силы
Решения задач городских туров 311

на Земле и Луне. Путём несложных алгебраических преоб-


разований получим: H · (gЛ − gЗ ) · (rТ /rВ ) = gЛ · hЛ − gЗ · hЗ . От-
куда получаем H ∼ = 0,1 · (rВ /rТ ). Теперь напишем выраже-
ние для силы натяжения нити в условиях Марса и при-
равняем эту силу к силе натяжения нити на Земле (или
Луне): gЗ · (HrТ /rВ − hЗ ) = gМ · (HrТ /rВ − hМ ). Из полученного
уравнения найдём глубину погружения, при которой рвётся
нить на Марсе.
О т в е т. На Марсе нить порвётся на глубине примерно
0,35 м.
231. Пусть x1 — координата первой белки, x2 — ко-
ордината второй белки, l — количество воды, попадающее
в чашку за одну секунду, k — жёсткость пружины, vч —
скорость чашки. Так как сумма сил, действующих на чаш-
ку, равна нулю (чашка движется равномерно), получаем:
k(x1 − vч · t) = k · (x2 + vч · t) + ltg. Поскольку x1 = v1 t и x2 = v2 t,
значит x1 + x2 = (v1 + v2 ) · t. Из этих двух уравнений получа-
ем искомую скорость чашки: vч = (v1 − v2 )/2 − l · g/2k.
О т в е т. Скорость чашки равна 0,95 см/c.
232. Пусть Вася прошёл по дорожке полный круг. Так
как скорость собаки относительно хозяина равна скорости
Васи, то за это же время Закусай относительно Васи дол-
жен пройти такое же расстояние. Пройденное ими рассто-
яние — 2pR. Один оборот Закусая относительно Васи 2pr.
Следовательно, Закусай сделает вокруг Васи R/r оборотов.
Пусть Вася двигается по часовой стрелке, а Закусай бега-
ет в том же (противоположном) направлении. Закусай бу-
дет гавкать каждый раз, когда оказывается на радиусе, со-
единяющем Васю и ёлку с внутренней (внешней) стороны
(см. рисунок). Так как Закусай гавкнул в первый момент, в
начале движения Васи он как раз находился в такой точке.
Из рисунков видно, что Закусай будет гавкать, перебегая
(не добегая) до полного оборота угол, равный пройденному
Васей от точки начала движения. Таким образом, когда Ва-
ся пройдёт полный оборот, Закусай сделает на один «гавк»
меньше (больше), чем оборотов. Он гавкнет R/r − 1 = 5 или
R/r + 1 = 3 раз.
О т в е т. Пока Вася пройдёт полный круг, собака гавк-
нет либо три, либо пять раз.
312 Решения задач

К решению задачи 232

К решению задачи 232


Решения задач городских туров 313

К решению задачи 233

233. Пусть длина размотанной части нити равна L.


Разобьём процедуру погружения на три части. Сначала ку-
бик погружают так, что глубина не превосходит a (a —
ребро кубика). Так как зависимость плотности от глубины
линейна, то можно ввести «среднюю плотность», т. е. за-
менить слой жидкости с переменной плотностью, в кото-
рый погружен кубик, на слой жидкости с некоторой сред-
ней плотностью rC = (r(0) + r(L))/2. Заметим, что сила Ар-
химеда в жидкости с такой плотностью равна силе Архиме-
да, действующей на кубик со стороны жидкости с первона-
чальным распределением плотности F A = rC · g · a2 · L. Сила
натяжения нити равна T = mg − F A . Затем кубик погружают
так, что объём его погруженной части не меняется. Сред-
няя плотность в этом случае равна rC = (r(L − a) + r(L))/2, а
сила натяжения нити есть T = mg − rC · ga2 L. Когда нижняя
грань кубика окажется на глубине 40 см, его плотность ста-
нет равна средней плотности жидкости, и он перестанет то-
нуть. При этом сила натяжения нити станет равна 0 и при
дальнейшем сматывании нити изменяться не будет — нить
просто будет провисать.
О т в е т. График силы натяжения нити приведён на ри-
сунке к решению задачи.
234. Так как все тела неподвижны, то их веса равны
силам натяжения нитей, на которых они висят. Трение в
314 Решения задач

системе отсутствует, поэтому сила натяжения нити посто-


янна вдоль её длины. Следовательно, правый крайний груз
также имеет массу m. Теперь определим массы остальных
грузов. Каждый из них висит на нити, привязанной к оси
блока. Каждый такой блок находится в равновесии под дей-
ствием трёх сил натяжения. Две из них направлены вверх и
равны mg, значит, третья равна 2mg. Следовательно, каж-
дый из этих грузов имеет массу 2m.
О т в е т. Масса правого груза равна 10 кг, массы осталь-
ных грузов — 20 кг.
235. Поток воды из каждого крана зависит только от
уровня воды в данный момент. Поэтому, когда открыты оба
крана, скорость вытекания воды из сосуда при некотором
уровне воды просто будет складываться из скоростей выте-
кания воды через каждый кран в отдельности при том же
уровне. Скорость вытекания воды пропорциональна скоро-
сти уменьшения уровня воды. Как видно из графика, эта
скорость на интервале от 20 до 15 см примерно постоян-
на для обоих кранов. При открытом первом кране уровень
уменьшается от 20 до 15 см примерно за 8 сек, при от-
крытом втором — примерно за 4 сек. Скорости уменьше-
ния уровня равны соответственно 5 см/8 сек и 5 см/4 сек.
Тогда скорость вытекания при двух открытых кранах рав-
на 5 см/8 сек + 5 см/4 сек, а уровень уменьшается с 20 до
15 см за время 5 см/(5 см/8 сек + 5 см/4 сек) ∼ = 2,7 сек.
О т в е т. Если открыть оба крана, то уровень воды опу-
стится за 2,7 сек.
236. Обозначим: V2 — объём гири, l — расстояние от
A до B, m = 10 кг — измеряемая масса груш, r0 — плот-
ность воды. Пусть при измерении в воздухе крючок нужно
установить на расстоянии x от точки A, тогда по правилу
рычага: r2 V2 g · l = mg · x. Пусть теперь груши взвешиваются
под водой. Суммарная внешняя сила, действующая на гру-
ши, равна r0 mg/r1 − mg и направлена вверх. Внешняя сила,
действующая на гирю, равна (r2 − r0 )V2 g и остаётся направ-
ленной вниз. Поэтому крючок надо будет устанавливать с
той же стороны от A, что и гиря (точка B). (При этом гру-
ши будут сверху стержня.) Пусть крючок надо установить
на расстоянии x слева от точки A. Применяя правило ры-
Решения задач городских туров 315

К решению задачи 236

чага,  
 r0
x · mg − 1 = gl(r2 V2 − r0 V2 ),
r1
и выражая x из измерений на воздухе (см. предыдущее ра-
венство) получаем:
1 − r0 /r2
x = = 2x.
r0 /r1 − 1

Остаётся правильно нанести отметку на шкалу.


О т в е т. Положение крючка, при котором весы находят-
ся в равновесии, отмечено на шкале рисунка к решению.
237. Длина отрезка SC , который преодолевает собака за
один пробег «туда-обратно», уменьшается до нуля по мере
подхода мальчика к вершине, поэтому полный путь собаки
складывается из большого количества всё уменьшающихся
отрезков (см. рисунок). Однако отрезок пути S, который за
каждый такой период проходит мальчик, также уменьшает-
ся. Можно заметить, что отношение путей мальчика и соба-
ки k = SC /S за один период «туда-обратно» не меняется от
пробега к пробегу. Так как полный путь мальчика 100 м,
полный путь собаки в k раз больше и равен 100 м · SC /S.
Найдём теперь k, рассмотрев произвольный пробег собаки
до вершины и обратно. Обозначим скорость мальчика V ,
скорость собаки вверх по горе — V1 , вниз — V2 . Пусть в
момент очередной встречи собаки с мальчиком их отделя-
ет от вершины расстояние L, время до следующей встречи
обозначим t. Время, за которое собака добегает до верши-
ны равно L/V1 . Следовательно, собака вернётся от верши-
ны к мальчику за время (t − L/V1 ). Собака бежит до верши-
ны расстояние L, затем встречает мальчика на расстоянии
l = V2 · (t − L/V1 ) от вершины. Мальчик при этом проходит
расстояние Vt = L − l = L − V2 (t − L/V1 ). Из этого уравнения
316 Решения задач

К решению задачи 237

выражаем t = L(1 + V2 /V1 )/(V2 + V ). Теперь, зная t, можно


легко найти S = Vt = 4 · L/9, SC = L + l = L + V2 (t − L/V1 ) = 14 · L/9
и вычислить отношение k = SC /S = 7/2. Таким образом, путь
собаки равен 100 · 7/2 = 350 метров.
Примечание. То, что отношение путей собаки и мальчи-
ка SC /S за один период «туда-обратно» не меняется от про-
бега к пробегу можно обнаружить не только с помощью при-
ведённых вычислений, но и графическим способом постро-
ив график зависимости от времени координаты мальчика и
координаты собаки (ломаная OBCDEF . . .) и заметив, что
он состоит из пропорционально уменьшающихся треуголь-
ников.
О т в е т. К тому времени, как мальчик достигнет верши-
ны, собака успеет пробежать 350 м.
238. Обозначим V — начальный объём пластилина,

V — требуемый объём пластилина в воде. Натяжение ни-
ти постоянно вдоль её длины, грузы неподвижны, зна-
чит, вес правого груза уравновешен той же силой натяже-
ния T, что и вес левого (mg), значит, масса правого гру-
за равна m. Средний груз висит на двух нитях, его тянет
вверх сила 2T. Следовательно, его масса равна 2m, а объ-
ём V = 2m/r2 = 1,48 дм3 . Так как плотности крайних гру-
зов равны, то равны и их объёмы, VK = m/r1 . Пусть те-
перь равновесие установлено в воде. Разность силы тяже-
сти mg и силы Архимеда r0 gVK , действующая на каждый
из крайних грузов, уравновешивается новой силой натяже-
ния нити T  , поэтому T = mg − r0 gVK = mg(1 − r0 /r1 ). На ку-
сок пластилина (имеющего теперь объём V  и соответству-
Решения задач городских туров 317

ющую массу M = r1 V  ) действует сила тяжести M g = r2 gV 


и сила Архимеда r0 gV  , результирующая которых равна
(r2 − r0 )V  g и уравновешивается удвоенной силой натяже-
ния нити 2T  . Поэтому (r2 − r0 )V  g = 2T = 2mg(1 − r0 /r1 ). От-
сюда получаем, что V  = 2m(1 − r0 /r1 )/(r2 − r0 ) = 10, 37 дм3 ,
а DV = V  − V ∼ = 8,88 дм3 .
О т в е т. Для равновесия в воде нужно прилепить к гру-
зу дополнительно примерно 8,88 дм3 пластилина.
239. Направление следа на окне совпадает с направ-
лением скорости капель дождя относительно автомобиля.
Эта скорость является векторной разностью скорости ка-
пель относительно земли u и скорости транспортного сред-
ства V относительно земли. Вектор u не обязательно на-
правлен вертикально, а V в любом случае горизонтален.
Конец вектора скорости автомобиля лежит посередине меж-
ду концами векторов скоростей автобуса V1 и грузовика V2
(см. рис. 1). Отсюда следует построение (см. рис. 2), позво-
ляющее получить ответ:
1) Провести две горизонтальные прямые, пересекающие
данные по условию следы AD и BF.
2) Опустить из точек A и B перпендикуляры AE и BG.
3) Из произвольной точки C, лежащей на верхней гори-
зонтальной прямой, опустить перпендикуляр CI.
4) Отложить отрезок IH = (ED + GF)/2.
5) Прямая CH.
О т в е т. Прямая CH.
Примечание. Перед этим построением можно сделать
параллельный перенос прямых, данных в условии. В част-
ности, возможно построение, при котором точки A, B и C
(а также E, G и I) совпадают, а H — середина отрезка DF.
При этом важно, что отрезки DE, FG и HI не представляют
скорости автобуса, грузовика и автомобиля соответственно
(т. е. не пропорциональны им), так как дождь, вообще гово-
ря, не вертикален.
240. При падении тела в воздухе сила тяжести разго-
няет тело, а сила сопротивления воздуха его тормозит. Си-
ла сопротивления растёт с увеличением скорости тела и че-
рез некоторое время сравнивается с силой тяжести — те-
ло перестаёт разгонятся, достигнув максимально возмож-
318 Решения задач

К решению задачи 239

К решению задачи 239

ной скорости. Предположим, сила трения о воздух зави-


сит от скорости по закону Fтр (V ). Тогда максимальная ско-
рость определяется условием равновесия Fтр (Vmax ) = Mg, ко-
торое реализуется в точке пересечения пунктирных прямых
на рис. 1. Если парашютист берёт гирю, действующая на
него сила тяжести увеличивается, и тело перестанет раз-
гоняться, лишь когда сила трения достигнет нового (боль-
шего) значения. Открытый парашют изменяет вид зависи-
мости Fтр (V ) так, что Vmax , соответствующая весу челове-
ка, оказывается безопасной для приземления. С момента
открытия парашюта до момента, когда парашютист дости-
гает новой скорости Vmax , сила сопротивления превосходит
силу тяжести, следовательно, скорость движения парашю-
тиста уменьшается. Если раскрыть парашют на слишком
большой скорости, человек может получить травмы из-за
резкого изменения силы трения (стропы парашюта сильно
дёрнут вверх). Открывать парашют, пока гиря ещё не вы-
пущена из рук, опасно и неэффективно, так как при этом
увеличивается тормозной путь парашюта и возрастает риск
приземлиться со слишком большой скоростью. Такому па-
рашютисту придётся начинать торможение на большей вы-
Решения задач городских туров 319

К решению задачи 240

соте, что эквивалентно уменьшению достигаемой им мак-


симальной скорости. Когда человек отпускает гирю, он на-
чинает замедляться. Это связано с уменьшением силы тя-
жести, действующей на человека. Отпустив гирю, следует
подождать некоторое время и только затем раскрывать
парашют. Требуемые графики приведены на рис. 2: линия
1 представляет собой время движения парашютиста как
функцию конечной скорости гири, линия 2 время движе-
ния гири как функцию её конечной скорости. Точка A, где
графики пересекаются соответствует ситуации, когда пара-
шютист летит вместе с гирей до земли. При этом време-
на полёта гири и парашютиста совпадают. Для гири точка
A соответствует, очевидно, максимальному времени полё-
та, а для парашютиста — минимальному. С другой сторо-
ны, гиря не может удариться о землю со скоростью боль-
шей некоего значения Vmax , определяющегося весом гири
согласно соотношению. Поэтому монотонные графики 1 и 2
при приближении к этому значению скорости становятся
вертикальными.
241. Причина, по которой устанавливается постоян-
ная скорость движения системы, подробно обсуждалась в
предыдущей задаче. Так как шары одинаковые и двигают-
ся с равными скоростями на большом расстоянии друг от
друга, действующие на них силы сопротивления воздуха
FC равны по абсолютной величине. Силы, действующие на
каждое тело, компенсируются, так как по условию шары
двигаются с постоянной скоростью. Пусть для определён-
ности r2 > r1 , тогда второй шар будет лететь ниже первого
(такая ситуация более устойчива, так как центр масс систе-
320 Решения задач

К решению задачи 240

мы находится ниже, чем в противоположном случае). Урав-


нения движения для первого (верхнего) шара, r1 Vg + T = FC ,
где T — искомая сила натяжения нити; для второго (ниж-
него) шара: r2 Vg − T = FC . Вычитая из первого выражения
второе, получаем ответ.
О т в е т. Сила натяжения нити равна T = Vg(r2 − r1 )/2.
Примечание. Если считать, что первый шар тяжелее, от-
вет будет другого знака. Также в этом случае поменяются
знаки для силы натяжения в уравнениях.
242. Мячик движется равноускоренно в поле силы тяже-
сти. В начальный момент проекция скорости на вертикаль-
ную ось равна V0 . Через время V0 /g скорость обращается в
ноль, а ещё через такое же время её проекция равна −V0 . В
моменты удара о землю скорость мгновенно меняет направ-
ление на противоположное.
О т в е т. График зависимости скорости мяча от времени
показан на рисунке.
243. Если x < H − l0 , то пружина не касается бруска. В
этом случае давление определяется формулой: p = rgx. Если
x > H − l0 , пружина будет сжиматься и давить на брусок. Ве-
личина сжатия равна (l0 + x − H), действующая сила по за-
кону Гука F = k(l0 + x − H). Полное давление во втором слу-
чае равно p = r · gx + F/S = x(rg + k/S) + k · (l0 − H)/S.
О т в е т. График зависимости давления от высоты брус-
ка x показан на рисунке.
244. Обозначим искомую силу трения скольжения Fтр .
Сила трения всегда направлена в сторону, противополож-
ную движению. Кроме силы трения, на брусок действу-
Решения задач городских туров 321

К решению задачи 242

К решению задачи 243


322 Решения задач

ют силы тяжести и реакции опоры, которые вместе созда-


ют скатывающую силу F, направленную вниз вдоль плос-
кости. Поскольку брусок может покоиться на наклонной
плоскости, F < Fтр . Когда брусок тащат вверх, сила тре-
ния и проекция силы тяжести направлены в одну сторону,
когда вниз — в противоположные. Поэтому F1 = Fтр + F2 ,
F2 = Fтр − F. Из последних двух уравнений выражаем силу
трения.
О т в е т. Искомая сила равна (F1 + F2 )/2.
245. Обозначим объём воды V0 = 1 м3 . Когда контей-
нер оказался залитым водой, соль начала растворяться,
вследствие чего масса контейнера стала уменьшаться. Вы-
числим, сколько соли из контейнера растворится. Так как
в одном литре может раствориться k = 0,35 кг соли, то в
1 м3 = 1000 л растворится 350 кг. Таким образом, после
окончания процесса растворения масса контейнера соста-
вит m = M − 1000k. С другой стороны, вода начинает на-
сыщаться солью, её плотность возрастает, что приводит к
увеличению архимедовой силы, действующей на контей-
нер и поплавок. Найдём плотность раствора после окон-
чания процесса. Так как по условию уровень жидкости в
резервуаре не меняется, при смешении 1 л (1 кг) воды
и k кг соли получается раствор массой (1 + k), занимаю-
щий объём, равный сумме объёмов исходных компонент,
1/r0 + k/r. Таким образом, плотность насыщенного рас-
твора составит: rН = r · r0 (1 + k)/(r + kr0 ). Контейнер всплы-
вёт, если полная сила тяжести, действующая на контейнер
и пробку, mg + rП Vg, окажется меньше, чем выталкиваю-
щая сила, действующая на поплавок и контейнер с солью,
rН g(V + m/r). Подставляя приведённые в условии значения,
находим, что 1640H = m · g + rП V · g < rhoН g · (V + m/r) = 1773H.
О т в е т. Контейнер с поплавком всплывёт.
246. Пронумеруем блоки так, как показано на рис. 1.
Пусть спасатели протащили свой конец верёвки на рассто-
яние a, а плита сдвинулась на расстояние b. При этом бло-
ки 2, 4 и 5 неподвижны, блок 3 переместится на расстояние
b влево, а блок 1 также сместится влево на некоторое рас-
стояние, которое мы обозначим c. При перемещении плит и
блоков верёвки изменят форму. Новые положения верёвок
Решения задач городских туров 323

К решению задачи 246

К решению задачи 246

показаны жирными линиями на рис. 2 (верёвка спасателей)


и 3 (вторая верёвка). Поскольку верёвки нерастяжимы, дли-
на каждой из них не изменится: a − 2c − b = 0, c − 3b = 0. Из
этих уравнений a = 7b. Обозначим силу, действующую на
плиту со стороны верёвок, F . По «золотому правилу» ме-
ханики Fa = F b. Поэтому F = 7F.
О т в е т. Результирующая сила, действующая на брусок,
равна F = 7F.
Примечание. Возможно другое решение, когда вычисля-
ется натяжение каждой нити. Нить, за которую тянут спа-
сатели, имеет, очевидно, натяжение F. Как видно из равно-
весия блока 1 (влево его тянут две нити с натяжением F, а
вправо — только одна), привязанная к нему нить имеет на-
тяжение 2F. Аналогично, нить привязанная к блоку 3 имеет
натяжение 4F. Сумма сил натяжения всех нитей привязан-
ных к плите равно 4F + 2F + F = 7F.
247. Выпишем условия равновесия для тел системы.
Для тела массы m имеем T1 + T2 = mg. Аналогично для гру-
за M запишем уравнение T1 = Mg. Блок невесом и неподви-
324 Решения задач

К решению задачи 246

жен, поэтому условия равновесия для него запишутся в ви-


де: T = 2T1 . Получаем систему трёх уравнений с четырьмя
неизвестными. Полностью разрешить её невозможно, одна-
ко можно найти искомое отношение масс тела и груза. Из
неё следует, что T = 2Mg, T2 = (m − M)g. Рычаг AC находит-
ся в равновесии, поэтому момент силы T равен моменту си-
лы T2 . Запишем правило рычага, 2MgL = 2L(m − M)g, где
M 1
L = AB. Из последнего уравнения следует, что = .
m 2
О т в е т. Тело в два раза тяжелее груза.
248. Плотность r2 больше плотности r1 , поэтому, по за-
кону Архимеда, второй шар глубже погружен в воду. Ко-
гда воду спускают, именно второй шар, более плотный, кос-
нётся дна аквариума первым. Значит, аквариум наклонится
вправо (в сторону более плотного шара).
О т в е т. Аквариум наклонится вправо.
249. Верёвка, перекинутая через блок B, натянута пер-
вым грузом m1 . Выпишем для него условие равновесия,
T1 = m1 g. Сам блок невесом, поэтому T = 2T1 . С другой сто-
роны, блок B уравновешен грузом M, T = Mg. Это возможно
только при условии, что M = 2m1 .
О т в е т. Масса груза M равна 100 г.
250. После того как на левый поршень положили груз,
он сместился вниз на расстояние x. Площадь правого порш-
ня в пять раз больше, чему у левого, поэтому он сместит-
ся вниз на расстояние x/5. (Объём жидкости не изменил-
ся.) Изменение давления над правым поршнем, связанное с
появлением груза, равно mg/S1 . Помимо этого, опускание
Решения задач городских туров 325

К решению задачи 249

К решению задачи 250

левого поршня приводит к уменьшению высоты давящего


столба жидкости и, следовательно, падению давления над
правым поршнем на величину rgx, а из-за опускания пра-
вого поршня давление над ним возрастает на rgx/5. Общее
изменение давления над правым поршнем,
mg
Dp = + rgx − rgx/5,
S1

уравновешивается изменением величины силы Гука k(x/5)/S2 = Dp.


Вода не выливается из колена, если x/5 = 20 см, значит,
x = 1 м. Подставляя x = 1 м в уравнение и полагая, что
g = 10 Н/м, находим m = 12 кг.
О т в е т. Масса груза m = 12 кг.
326 Решения задач

К решению задачи 252

251. Время, за которое водомерка, покинувшая озеро A,


достигает B равно t1 = s/(u + v) = 40 мин. Аналогично, вре-
мя, которое тратит водомерка из B, чтобы добраться до A
равно t2 = s/(v − u) = 120 мин. За время своего движения во-
домерка встретит на своём пути тех насекомых, которые
уже находились в дороге (N1 ) и тех, которые выплывут из
B за то время, пока она сама преодолевает этот путь (N2 ),
т. е. искомое число встреченных насекомых N = N1 + N2 . Во-
домерки отправляются в путь каждые минуту (t = 1 мин), то
N1 = t2 /t = 120 и N2 = t1 /t = 40.
О т в е т. N = 40 + 120 = 160 водомерок.
252. Нарисуем все силы, действующие на палку (см. ри-
сунок). Запишем условия равновесия блоков: T2 r = T1 R (ра-
венство плеч рычагов относительно точки O). Запишем так-
же условие равновесия палки (равенство плеч рычагов от-
носительно центра масс палки): T2 y = T1 x. Сравним оба ра-
венства. Очевидно, что y = r, а x = R, т. е. центр масс палки
находится ровно под точкой O. Искомое расстояние, таким
образом, равно: l = x + L/2 = R + L/2 = 2r + L/2 = 70 см.
О т в е т. Расстояние от левого крюка до правого конца
палки равно 70 см.
253. Сначала опустим поплавок известной плотности в
жидкость. Измерим линейкой, на сколько поднялся уровень
жидкости в стакане. Обозначим это число за h1 . Потом ак-
куратно надавим на поплавок так, чтобы он весь погрузился
под водой; уровень жидкости в стакане при этом обозначим
Решения задач городских туров 327

за h1 . Отсюда мы можем найти плотность неизвестной жид-


кости: r = r1 h1 /h1 , где r1 — известная плотность поплав-
ка. Повторяя процедуру со вторым поплавком, получим:
r2 = rh2 /h2 . В этом уравнении h2 и h2 — величины анало-
гичные h1 и h1 . Подставляя значение r из опыта с первым
поплавком, получим: r2 = r1 h1 h2 /h1 h2 .
254. Сила, с которой конструкция давит на пол, опреде-
ляется результирующей сил тяжести конструкции, вытал-
кивающей силы и упругой силы, с которой пружина (рас-
тянутая) тянет конструкцию ко дну, N = Fтяж + Fпруж − Fводы .
Пользоваться законом Архимеда нельзя, так как вода не
подтекает под конструкцию. Поэтому силу, с которой вода
выталкивает конструкцию, будем искать как равнодейству-
ющую сил давления воды. При h < a очевидно, что Fводы = 0.
Посмотрим, что будет при a ≤ h ≤ 2a. В этом случае столб во-
ды будет оказывать выталкивающее действие на конструк-
цию, Fводы = 2rgha2 (h − a). При h ≤ 2a столб воды прижима-
ет конструкцию к полу, Fводы = 2rgha2 (h − a) − 4rgha2 (h − 2a).
Учтём, что Fпруж = kDx = 0,5ka = const, так что действие пру-
жины просто даёт постоянный вклад в давление.
О т в е т. График силы давления конструкции на пол
приведён на рисунке. Он представляет собой три участка,
на каждом из которых поведение силы давления является
линейным.
255. При решении настоящей задачи закон сообщаю-
щихся сосудов применить впрямую не получится, так как
жидкости, которыми заполнена трубка, различны. Мыслен-
но отметим уровень от дна, на котором начинается столбик
воды в левом колене. Заметим, что на этом же уровне в
правом колене давление верхнего столбика масла высотой
h2 и ртути высотой x равно давлению столбика воды высо-
той h1 в левом колене. Давление столбика жидкости высо-
той a равно rga, где r — плотность жидкости. Таким обра-
зом, для нашей системы получаем: r2 h1 g = r3 h2 g + r1 xg, от-
куда x = (r2 h1 − r3 h2 )/r1 . Далее, из очевидных соображений,
h1 + h3 = h2 + x, откуда h3 = x + h2 − h1 = 0,103 м.
О т в е т. Разность уровней жидкости в коленах трубки
составляет 10 см 3 мм.
328 Решения задач

К решению задачи 254

К решению задачи 255


Решения задач городских туров 329

256. Найдём, на сколько изменится радиус ворота с


верёвкой при подъёме груза на высоту H. Так как вит-
ки верёвки всё время ложатся один на другой, то пло-
щадь круга, который образует ворот с верёвкой, возрастёт
на Hd — площадь, которую занимает кусок верёвки толщи-
ны d и длины H. Тогда, если R1 — конечный радиус, то
Hd = pR21 − pR20 . Следовательно, R1 = (Hd/p + R20 )1/2 ∼
= 20 см.
Пусть радиус ворота с верёвкой равен R. По правилу ры-
чага, рабочему приходится прикладывать силу F = mgR/L.
Если в начале подъёма она равна F0 = mgR0 /L, то в кон-
це — F1 = mgR1 /L. За один оборот эта сила изменяется на
DF = mgDR/L = mgd/L, т. е. за каждый оборот сила всегда
изменяется на одну и ту же величину, малую по сравнению
с самой силой F. Пусть Ti = T(Fi ) — время, потраченное на
i-ый оборот ворота (Fi — средняя сила, которую приходится
прикладывать рабочему за время этого оборота, связь меж-
ду Ti и Fi даётся графиком на рисунке). Полное затраченное
на подъём груза время тогда равно
„ «
P
Ti · DF
t= Ti = i
d .
i
mg
L

Член суммы Ti DF имеет смысл площади маленького пря-


моугольника на графике зависимости F(T) с основанием DF
и высотой Ti . Сумма таких площадей площадь фигуры под
кривой F(T), между вертикальными прямыми F = F1 , F = F2
и осью F. В фигуру укладывается примерно 36 клеток пло-
щадью DS = 2H · 0,25 сек. Вся площадь S ∼
= 18 Н · сек. Затра-
ченное время T = SL/mgd = 36 сек.
О т в е т. Рабочий поднимет груз на высоту 19 м за
36 сек.
257. Обозначим через b сторону ледяного куба. Най-
дёмb из соотношения между массой и объёмом льда,
M
b= 3 . Глубина, на которую тело погружено в воду, це-

ликом определяется соотношением между средней плотно-
стью тела и плотностью воды. Так, ледяной куб со сто-
роной b первоначально плавает, погрузившись в воду на
глубину h0 = brЛ /r0 . Предположим сначала, что кубик ме-
330 Решения задач

К решению задачи 256

ди проплавил во льду отверстие глубиной x < a (так что


вся растаявшая вода выливается). При этом x связано с
начальной температурой меди уравнением теплового ба-
ланса cМ · mt = lrЛ · xa2 , где m = rМ · a3 — масса меди, t —
температура меди, отсчитываемая от 0 ◦ C. Левая часть
последнего выражения представляет собой теплоту, от-
данную медью при остывании, а правая часть — тепло-
ту, затраченную на плавление льда объёмом xa2 . Отсю-
c r ta
да находим x = М М = kta, для краткости мы ввели па-
lrЛ
раметр k = rМ cМ /lrЛ . Так как теперь внутри льда содер-
жится медь, средняя плотность плавающего тела увели-
чилась по сравнению с первоначальной. Найдём эту сред-
нюю плотность, разделив массу льда с медью на объём
льда, rСР = (M + m − x · rЛ · a2 )/b3 . В последнем равенстве в
числителе мы учли, что часть льда объёмом xa2 растаяла
и вылилась. Таким образом, глубина погружения составит
h(t) = brСР /r0 = h0 (1 + rМ · a3 /M) − h0 · ktrЛ · a3 /M. Это линей-
ная убывающая зависимость. Для того, чтобы x было мень-
ше a (чтобы вся вода, образовавшаяся изо льда, вытека-
ла), параметр kt должен быть меньше единицы. Это име-
ет место при температурах t < t0 = l · rЛ /(rМ cМ ). Начиная с
температуры t = t0 , при увеличении t, вода перестаёт выли-
ваться из углубления (так как cЛ < c0 и вода, образовавша-
яся в результате таяния льда, имеет меньший объём, чем
исходный лёд). Таким образом, cСР перестаёт изменяться,
Решения задач городских туров 331

К решению задачи 257

и h(t) на участке t > t0 будет равной h(t0 ). Возможен так-


же случай, когда медь насквозь проплавляет лёд и тонет.
Это происходит
 если cМ mt > lrЛ ba2 , т. е. при температурах
M lrЛ
t > t1 = 3 . Очевидно, в этом случае глубина погруже-
rЛ acМ rМ
ния сравняется с первоначальной глубиной h0 .
О т в е т. Требуемый график изображён на рисунке.
258. Теплота, уходящая в единицу времени непосред-
ственно на нагрев воды, определяется разностью мощностей
плитки и потерь тепла. Поэтому теплота, идущая на на-
грев за время T секунд, будет равна площади трапеции, за-
штрихованной на рисунке. Из графика видно, что в момент
времени T мощность потерь численно равна (100 + T/2) Вт.
Если T — искомое время, то высота рассматриваемой тра-
пеции равна T, её стороны 400 и 400 − T/2, а её пло-
щадь равна T(400 − T/4). Эта площадь должна быть рав-
на cm · Dt = 4200 · 2 · 10 = 84 кДж. Данное условие сводится к
квадратному уравнению T2 − 1600T + 336000 = 0, решая ко-
торое, находим его наименьший корень T ∼ = 249 сек. Наи-
больший корень не имеет физического смысла, так как со-
ответствует ситуации, когда мощность потерь превосходит
500 Вт.
О т в е т. Вода нагреется на 30 ◦ C приблизительно за
249 сек.
259. Обозначим искомую температуру через t3 . Тепло-
отдача складывается из потока тепла через боковые стен-
ки, дно и поверхность. Мощность теплоотдачи через дно и
] свободную поверхность воды равна для трёх сосудов, со-
ответственно, A(t1 − t), A(t2 − t) и A(t3 − t), здесь A — ко-
332 Решения задач

К решению задачи 258

эффициент пропорциональности, одинаковый для всех трёх


сосудов. Мощность теплоотдачи через боковые стенки (кро-
ме аналогичной зависимости от разности температур) про-
порциональна площади боковых стенок сосудов, а следова-
тельно, их высоте, и, значит, и объёму сосуда. Коэффи-
циент пропорциональности обозначим за B. Тогда тепло-
отдача через боковые стенки равна BV1 (t1 − t), BV2 (t2 − t) и
V3 (t3 − t). Суммарная мощность теплоотдачи i-го сосуда рав-
на Pi = (ti − t) · (A + BVi ). В установившемся режиме она рав-
на мощности кипятильника, и, следовательно, одинакова во
всех случаях. Поэтому: 60(A + B) = 40(A + 2B) = X(A + 4B),
где X = t3 − t. Решение системы: A = B, X = 24. Откуда по-
лучаем t3 = 44 ◦ C.
О т в е т. Воду в сосуде можно нагреть до температуры

44 C.
260. Пусть масса жидкости X равна mX , а масса её ис-
следованного образца — mX0 = 1 г. Вначале жидкость X,
налитая в воду, будет нагреваться, а вода — охлаждаться.
Когда температура жидкости X достигнет TK , она начнёт
выкипать, но вода продолжит охлаждаться. В предельном
случае, когда ровно вся жидкость X выкипит, температура
воды достигнет TK . Теплоёмкость налитой жидкости равна
m m
CX X , а полная теплота парообразования — QП X . Тогда
mX0 mX0
уравнение теплового баланса даёт:

mX m
CX (TK − TX ) + QП X = cВ rВ VВ (TВ − TK ).
mX0 mX0
Решения задач городских туров 333

К решению задачи 260

V
Но mX = VX rX (TX ) = mX0 X . Где V0 (T) — объём исследо-
V0 (TX )
ванного образца при температуре T. Подставляя численные
160 ◦ C − TX мл
данные, получаем уравнение на TX : = V0 (TX ).
70 град
Изобразив правую часть уравнения на графике, можно най-
ти, что искомое TX ∼ = 66 ◦ C.
О т в е т. Минимальная температура, при которой вся
добавленная в воду жидкость выкипит приблизительно рав-
на 66 ◦ C.
261. Пусть за один вдох/выдох через грелку прохо-
дит некоторое количество воздуха с теплоёмкостью C. Уста-
новившуюся температуру грелки обозначим за T. Тогда
при выдохе, за счёт охлаждения воздуха до температуры
грелки, она получает количество теплоты Q1 = C(TЛ − T).
При вдохе проходящий через грелку воздух нагревается
до её температуры, и грелка отдаёт количество теплоты
Q2 = C(T − T0 ). Также грелка отдаёт тепло в окружающую
среду через боковые стенки. Мощность теплоотдачи пропор-
циональна разности температур грелки и окружающей сре-
ды: P = a(T − T0 ). Пусть лыжник делает один выдох и вдох
за время t. Считая, что температура грелки установилась,
можно написать уравнение теплового баланса:

a TЛ − T
Q1 = Q2 + Pt ⇒ 1 + t = .
C T − T0
334 Решения задач

Пусть в первом случае период дыхания лыжника равен t1 .


Тогда
a T −T
t1 = Л 1 − 1 = 2.
C T1 − T0

Во втором случае период дыхания уменьшился вдвое. Под-


ставляя в уравнение теплового баланса T = T2 и t = t2 = t1 /2,
выразим
TЛ + 2T0 4 ◦
T2 = =− C ≈ −1,3 ◦ C.
3 3

О т в е т. Грелка нагрелась до температуры T2 ≈ −1,3 ◦ C.


262. В стационарном состоянии сила натяжения верёв-
ки, перекинутой через подвижный блок, равна mg. Так как
подвижный блок невесом, сила натяжения верёвки соеди-
нённой с куском льда равна 2mg и остаётся постоянной.
Следовательно, остаётся постоянной разность сил тяжести
и Архимеда 2mg = mл g − rVпогр g, где mл — текущая масса
льда, а Vпогр — объём погруженной части льда. По мере та-
яния льда mл уменьшается, следовательно уменьшается и
Vпогр . Это будет происходить пока mл не станет равна 2m, а
Vпогр не станет равным нулю. Можно также мысленно раз-
бить лёд на две части, одну, нетающую, массой 2m и дру-
гую, тающую, вес которой скомпенсирован силой Архиме-
да, равной весу вытесненной жидкости. Растаявший лёд как
раз заполнит объём, вытесняемый им ранее. Поэтому уро-
вень воды будет оставаться постоянным, и при этом вода
выливаться не будет. Пусть m0 — начальная масса льда,
1 m0
V0 = — начальный объём погруженной части. Из усло-
2 rл
вия равновесия имеем:

1 r 2m
2mg = m0 g − m0 g ⇒ m0 = 1 r .
2 rл 1−
2 rл

Система выйдет из равновесия, когда растает масса льда

2m
Dm = m0 − 2m = 2rл = 25 · г.
−1
r
Решения задач городских туров 335

Пусть искомая температура воды t. Тогда:

cV r(tк − t) = lDm + cDm(t − 0 ◦ C) ⇒


V r · tк + (0 ◦ C − l/c) · Dm
⇒t= ≈ 9,0 ◦ C.
V r + Dm

О т в е т. Когда лёд поднимется из воды, её температура


будет примерно равна 9 ◦ C.
263. Заметим, что пылинки не обгоняют друг друга,
так как те из них, что имеют большую скорость, нахо-
дятся дальше от центра. Рассмотрим пылинки, находящи-
еся в начальный момент на расстоянии r1 и r2 от цен-
тра облака. Через время t они окажутся на расстояни-
ях r1 = r1 + Hr1 t = r1 (1 + Ht) и r2 = r2 (1 + Ht) от центра. Те-
перь рассмотрим вещество, находившееся в начальный мо-
мент между сферами радиусами r1 и r2 , центр которых
совпадает с центром облака. Масса этого вещества рав-
на m = r0 (4pr13 /3 − 4pr23 /3). Поскольку пылинки не обгоня-
ют друг друга, через время t это вещество будет находить-
ся между сферами радиусами r1 и r2 , центр которых сов-
падает с центром облака. Поэтому средняя плотность этого
вещества равна r = m/(4pr1 3 /3 − 4pr2 3 /3) = r0 /(1 + Ht)3 . Мы
видим, что эта плотность не зависит от r1 и r2 , таким обра-
зом, облако останется однородным. Радиус облака опреде-
ляется местоположением самых дальних пылинок и равен
R = R(1 + Ht).
О т в е т. Плотность пыли на расстоянии x от центра об-
лака через время t после взрыва равна r = r0 /(1 + Ht)3 , если
x < R(1 + Ht) и r = 0, в противном случае.
264. Рассмотрим силы, действующие на кубики. На оба
кубика действует сила тяжести (mg и Mg). На каждый ку-
бик действует сила реакции со стороны другого кубика (N1
и N2 ). Кроме того, на левый кубик действует сила натяже-
ния нити T, а на правый — сила реакции поверхности N3 .
Ускорение левого кубика a1 направлено перпендикулярно
нити, а правого a2 — горизонтально. Запишем второй за-
кон Ньютона:
− → − → − →
m g + N + T = m−
1
→a ,
1
→ →
− − →

M g + N 2 + N 3 = M−

a 2.
336 Решения задач

К решению задачи 263

К решению задачи 264

Спроецируем первое уравнение на ось x1 , перпендику-


лярную нити, а второе — на горизонтальную ось x2 :
mg sin a − N1 cos a = ma1 , N2 = Ma2 . Поскольку кубики не
отрываются друг от друга, горизонтальные проекции их
ускорений равны: a1 cos a = a2 . Кроме того, по третьему за-
кону Ньютона N1 = N2 . Решая получившуюся систему урав-
нений, находим ускорения кубиков.
О т в е т. Ускорения кубиков в начальный момент равны
mg sin a mg sin a cos a
a1 = , a2 = .
m + M cos2 a m + M cos2 a

265. Пронумеруем балконы сверху вниз, а массу бал-


кона обозначим m. С помощью закона сохранения энергии,
Решения задач городских туров 337

К решению задачи 265

mgh = mv21 /2, вычислим скорость√ v1 , с которой первый бал-


кон налетит на второй: v1 = 2gh. Поскольку столкновения
между балконами абсолютно неупругие, то, после столкно-
вения первого балкона со вторым, оба будут иметь одну
скорость u2 , которую можно найти из закона сохранения
импульса, mv 1 = 2mu2 , u2 = v1 /2. Аналогично можно найти
скорость vn = u2n + 2gh, которую будут иметь n балконов
перед столкновением с (n + 1)-ым — из закона сохранения
энергии, nmgh + nmu2n /2 = nmv2n /2, а также скорость un , ко-
торую будут иметь n балконов сразу после того, как группа
из n − 1 балкона столкнётся с n-ым: (n − 1)mvn−1 = nmun . От-
n−1
куда un = vn−1 . Таким образом, можно последовательно
n
15
вычислить скорости v2 , u3 , v3 , u4 и, наконец, v4 = gh.
4
(n + 1)(2n + 1)
Кроме того, можно показать, что vn = gh. Гра-
3n
фик этой зависимости представлен на рисунке.
О т в е т. Скорость 4-го балкона в момент удара о землю
15
равна v4 = gh.
4
266. Если Вася движется со скоростью v в течение вре-
мени t, то он поднимется на расстояние vt. Значит, что-
бы определить максимальную длину лестницы, надо найти
на графике точку, соответствующую максимальному значе-
нию vt, или, что то же самое, минимальному отношению
(1/t)/v. Поскольку (1/t)/v есть угловой коэффициент пря-
мой, проходящей через начало координат и данную точку
338 Решения задач

К решению задачи 266

графика, то задача сводится к тому, чтобы найти прямую с


минимальным угловым коэффициентом. Таковой является
касательная к графику, проведённая из начала координат
(прямая 1 на рис. 1). Таким образом, необходимо прове-
сти касательную и после этого вычислить длину лестницы,
например, по координатам точки касания: v0 = 1,33 м/сек,
1/t0 = 0,029 сек−1 . Отсюда l0 = 46,2 м. Можно использовать
и любую другую точку касательной. Теперь рассмотрим слу-
чай, когда Вася на забирается по эскалатору, движущему-
ся со скоростью u. Если скорость Васи равна v, то рассто-
яние, которое он успеет пройти, равно (v − u)t. Это произ-
ведение максимально, когда отношение (1/t)/(v − u) мини-
мально, т. е. когда минимален угловой коэффициент пря-
мой, проходящей через точку (u, 0) и данную точку графи-
ка. Таким образом, необходимо провести касательную к гра-
фику из точки (u, 0) (прямая 2 на рис. 1) и по её угловому
коэффициенту определить длину эскалатора аналогично то-
му, как это делалось для неподвижной лестницы. Проведя
эту процедуру для нескольких значений u, можно получить
итоговый график (см. рис. 2).
267. Нить всегда натянута перпендикулярно стержню,
поскольку кольцо невесомо и не испытывает трения о стер-
жень. Следовательно, взаимное положение клина, бруска,
нити и кольца не будет изменяться, и они будут сколь-
зить вниз по рельсам как целое. На брусок действуют си-
Решения задач городских туров 339

К решению задачи 266

лы тяжести, натяжения нити и реакции со стороны клина


(см. рис. 1). Они придают бруску ускорение a, направленное
параллельно рельсам:
− −
→ → − →
g + N 1 + T = m−

a.
Проекция уравнения на вертикальную ось есть mg − T = ma sin a.
Теперь рассмотрим вместе клин, брусок, нить и кольцо. На
эту систему действуют сила тяжести (m + M)g, сила реак-
ции рельс N, сила трения Fтр и сила реакции F со стороны
стержня (эта сила действует на кольцо и уравновешивает
силу натяжения нити, поэтому F = T). Вся система приоб-
ретает ускорение a,
→ −
− → − → −→
(m + M) g + F + N + F тр = (m + M)−

a
или, в проекциях на оси x и y, (m + M)g sin a − Fтр = (m + M)a,
N − T − (m + M)g cos a = 0. Возможны два варианта: либо си-
стема покоится (a = 0), либо движется. В последнем случае
Fтр = mN. Решая получившуюся систему уравнений, получа-
ем выражение для ускорения, границы применимости этого
решения определяются условием a > 0.
340 Решения задач

К решению задачи 267

К решению задачи 267

О т в е т. Ускорение бруска и клина равно


(m + M)(sin a − m cos a) − mm
a=g
m + M − mm sin a

(m + M) sin a
при m < , иначе a = 0.
(m + M) cos a + m
268. Кроме силы тяжести, на уголок действуют силы
со стороны потока воды. Когда поток меняет направление,
он передаёт уголку импульс. Следовательно, на уголок дей-
ствуют силы, приложенные в точке крепления уголка A и
в точке изгиба B. Вычислим силу F, действующую в точ-
ке B. Во-первых, она направлена под углом 135 ◦ C к сто-
ронам уголка, поскольку она противоположна по направле-
нию изменению импульса воды при повороте. Чтобы вы-
числить её величину, обозначим площадь сечения уголка
S, скорость воды v, её плотность r. Тогда масса воды, про-
текающей по уголку за время t, равна rSvt, √ её импульс
pнач = p
√кон = rSv
2 t, изменение импульса Dp = 2rSv2 t, и си-
ла F = 2rSv2 . Если скорость воды удвоить, то эта сила уве-
личится в 4 раза. Найдём условие равновесия уголка. Мо-
Решения задач городских туров 341

К решению задачи 268

менты сил, действующих на него, скомпенсированы. Будем


вычислять моменты относительно точки A. Тогда сила, дей-
ствующая со стороны оси, и сила, возникающая из-за изме-
нения направления течения воды в точке A, не имеют мо-
ментов. Если обозначить
√ длину стороны уголка l, то плечо
силы F равно l/ 2. Сила тяжести приложена в центре масс
уголка. Обозначим расстояние от точки A до центра масс l1 .
Тогда плечо силы тяжести равно l1 sin(a − a0 ), где a0 — угол
между направлениями из точки A на √ точку B и на центр
масс (см. рис. 1). Таким образом, lF/ 2 = Fm l1 sin(a − a0 ).
При удвоении скорости воды сила √ F увеличивается в 4 раза.
Если b — искомый угол, то 4F/ 2 = Fm l1 sin(b − a0 ). Следо-
вательно, sin(b − a0 ) = 4 sin(a − a0 ). Осталось найти угол a0 .
Центр масс уголка находится посередине между середина-
ми его сторон (см. рис. 2). Поэтому tg a0 = 1/3. В итоге име-
ем: b = a0 + arcsin(4 sin(a − a0 )), a0 = arctg(1/3). Подставляя
a = 30 ◦ C, получаем b = 71,7 ◦ C.
О т в е т. Вода будет бить из уголка под углом 71,7 ◦ C к
горизонту.
269. Поскольку в условии сказано, что в третьем слу-
чае стержень составляет малый угол с вертикалью, в даль-
нейшем будем считать, что силы взаимодействия стержня
с бусинками направлены практически вертикально. Пусть
бусинка движется по спирали под действием некоторой вер-
тикальной силы F (в первом случае это сила тяжести, в тре-
тьем — равнодействующая сил тяжести и упругости стерж-
342 Решения задач

К решению задачи 268

ня). Кроме неё, на бусинку действует сила реакции со сто-


роны спирали. Очевидно, что сила реакции пропорциональ-
на силе F, и поэтому вертикальное ускорение бусинки так-
же пропорционально силе F, aверт = b · F. Здесь b — коэф-
фициент пропорциональности, зависящий от геометриче-
ских характеристик спирали и массы бусинки m. Анало-
гичное утверждение верно и для бусинки, движущейся по
вертикальному стержню. Её вертикальное ускорение так-
же пропорционально действующей силе F, aверт = b · F. (В
этом случае коэффициент b равен обратной массе бусин-
ки.) Обозначим вертикальные ускорения бусинок во всех
трёх случаях a1 , a2 , a3 . В первых двух случаях бусинки
двигаются под действием силы тяжести, поэтому a1 = amg,
a2 = b · mg. В третьем случае на верхнюю бусинку действует
сила mg + T, а на нижнюю mg − T, здесь T — сила упруго-
сти стержня. При этом вертикальные ускорения обеих бу-
синок равны a3 = a · (mg + T) = b · (mg − T). Рассматривая вы-
ражения для ускорений a1 , a2 , a3 как систему совместных
уравнений, выразим a3 через a1 и a2 : a3 = 2a1 a2 /(a1 + a2 ).
Во всех трёх случаях бусинки проходят одно и то же рас-
стояние L = 1/2a1 T12 = 1/2a2 T22 = 1/2a3 T32 . Из последнего ра-
венства нетрудно выразить T3 .
О т в е т. Не имея начальной скорости, бусинки  преодо-
T12 + T22
леют вертикальное расстояние за время T3 = .
2
270. Сложность задачи заключается в том, что она состо-
ит из нескольких самостоятельных задач, каждая из кото-
рых должна быть решена для получения корректного, обос-
Решения задач городских туров 343

нованного ответа. В начале необходимо доказать, что тре-


угольник, форму которого приняло резиновое кольцо —
равносторонний. Затем следует решить задачу про груз,
подвешенный на двух симметричных нитях, и найти си-
лу натяжения резинового кольца. Потом надо решить за-
дачу о том, как изменяется коэффициент упругости в зави-
симости от длины резинового кольца, и найти длину рас-
тянувшегося кольца. И, наконец, надо решить геометриче-
скую задачу по определению расстояния BD (см. рис. 1),
зная все линейные размеры и углы ромба ABCD. Каждая
из этих задач достаточно проста, но их совмещение создаёт
определённые трудности. Сначала докажем, что треуголь-
ник ACD — правильный. Для этого рассмотрим элемент
резинки, непосредственно прилегающий к спице, напри-
мер кусочек рядом с точкой A (см. рис. 1). На него дей-
ствуют две силы натяжения F, которые по модулю равны
между собой, и сила реакции со стороны спицы T. Так
как резинка покоится, то векторная сумма этих сил рав-
на нулю. В проекции на направление спицы сила T не
действует, значит, проекции на эту ось обеих сил F долж-
ны уравновешивать друг друга. Так как силы F одинако-
вые по модулю, это возможно лишь при условии равен-
ства обозначенных на рисунке углов. Верхний из этих уг-
лов равен 60 ◦ C, следовательно, угол DAC равен 60 ◦ C. Тре-
угольник ACD симметричен, следовательно, он является
правильным и равен треугольнику ABC. Теперь рассмот-
рим точку подвеса груза и найдём силу натяжения резин-
ки. На эту точку действуют сила тяжести со стороны гру-
за и две симметричных силы натяжения резинки. Так как
груз покоится, то векторная сумма этих сил равна нулю. В
проекции на вертикальную ось это даёт mg = 2F cos 30 ◦ C,
откуда легко выразить F = mg/2 · cos 30 ◦ C. Теперь опреде-
лим, чему равен коэффициент упругости резинки длиной
2L. Для этого проведём мысленный эксперимент: прило-
жим одинаковые силы F к двум резинкам, изготовленных
из одной и той же резиновой ленты. Пусть одна из ре-
зинок имеет длину L и жёсткость k, а другая — длину
2L и неизвестную жёсткость k1 . Предположим, резинка
длиной L растянулась на DL. Понятно, что каждая поло-
винка длинной резинки в точности соответствует резинке
344 Решения задач

К решению задачи 269

длиной L и растянется на столько же. Значит закон Гу-


ка для короткой и длинной резинки имеет, соответствен-
но, вид F = kDL, F = 2k1 DL. Решая эту систему уравнений,
находим коэффициент упругости для резинки длины 2L:
k1 = k/2. Зная k1 , найдём удлинение резинового кольца дли-
ной 2L под действием силы F: 2DL = F/k1 = mg/k · cos 30 ◦ C.
Из соображений симметрии следует, что каждая сторо-
на треугольника ACD, образованного резинкой, удлини-
лась одинаково, тогда можно найти длину одной сторо-
ны AD = (2L + mg/k · cos 30 ◦ C)/3. И наконец, теперь можно
найти и расстояние BD = 2AD cos 30 ◦ C.
О т в е т. Искомое расстояние равно BD = 4L · cos 30 ◦ C)/3 + 2mg/3k.
271. Казалось бы, нахождение длины тени ничем не от-
личается от нахождения длины тени стены такой же высо-
ты: рассмотрим стену высотой H и солнце, освещающее её
под углом a (см. рис. 1). Длина тени, очевидно, оказывается
равной x = H ctg a. При малых a длина тени стены H ctg a
стремится к бесконечности. Однако этот, справедливый для
тени стены ответ, не всегда годится для столба. В этом лег-
ко убедиться, рассмотрев систему сверху (см. рис. 2). Необ-
ходимо учесть размер Солнца, ибо углом, под которым мы
его видим, нельзя пренебрегать. Тень, отбрасываемая стол-
бом от удалённого точечного источника, по мере удаления
от столба становится всё уже и уже и, в конце концов, исче-
Решения задач городских туров 345

К решению задачи 269

зает. Она имеет сверху форму треугольника или трапеции,


в зависимости от высоты Солнца над горизонтом. Очевид-
но, что при большой ширине столба, когда отличие фор-
мы тени столба от прямоугольника мало, длина тени рав-
на x = H · ctg a (так было бы, если вместо столба находилась
стена). Неполнота этого выражения становится очевидной,
если представить себе что a → 0. В этом случае H ctg a стре-
мится к бесконечности, однако, длина тени столба не может
быть больше s. Вычисление s опирается на соображения
подобия: r/s = R/(L + s), откуда s = r · (L + s)/R ∼
= rL/R = 15 м.
Итак, длина тени (в направлении луча!) не больше, чем s,
а значит её проекция на землю не больше, чем s1 = s · cos a.
Таким образом, длина тени на земле равна минимальной
из величин x, s1 , l = min(x, s1 ). Видно, что при малых уг-
лах s > s1 , а при a > aкр наоборот, s < s1 , sin aкр = H/s = 1/2,
aкр = p/6 < 60 ◦ C. Следовательно, график будет состоять из
куска синусоиды и куска графика котангенса, переходящих
друг в друга в точке aкр .
О т в е т. График зависимости длины l тени столба от вы-
соты Солнца над горизонтом (в градусах) показан на рисун-
ке.
272. Обозначим массу лежащего на наклонной плоско-
сти груза M, а висящего груза — m. Запишем второй закон
Ньютона в проекции на направление нити для каждого из
грузов в первом опыте: mg − T = m · a1 (уравнение для вися-
346 Решения задач

К решению задачи 271

К решению задачи 271

К решению задачи 271


Решения задач городских туров 347

щего груза, T — натяжение нити), Mg sin a + T = Ma1 (для


лежащего груза). Сложив эти два уравнения, исключим
силу натяжения нити: Mg sin a + mg = a1 (M + m). Из этого
уравнения можно выразить ускорение системы в первом
опыте a1 . Во втором опыте система грузов может ехать ли-
бо вправо, либо влево. Если система едет влево, то, сложив
уравнения движения для грузов в проекции на направление
движения, получим Mg sin a − mg = (M + m) · a2+ . Если же
система едет влево, будем иметь −Mg sin a + mg = (M + m) · a2− .
Таким образом, в зависимости от направления движения
слагаемые меняют знак:

±Mg sin a ∓ mg = (M + m) · a2±

(плюс соответствует движению системы налево, минус —


направо). Отсюда можно найти ускорения a2± . Согласно
условию задачи, путь системы, пройденный за некоторое
время t в первом эксперименте, в два раза длиннее, чем
во втором: S1 = 2S2± . Так как при равноускоренном движе-
нии S = a2 t/2, ускорения в первом и втором эксперименте
также отличаются в два раза: a1 = 2a2± . Если подставить
сюда найденные выражения для ускорений, окажется, что
либо M/m = 1/(3 sin a) (когда система во втором опыте едет
вправо), либо M/m = 3/ sin a (когда система во втором опы-
те едет влево).
О т в е т. В зависимости от направления движения си-
стемы во втором опыте, отношение масс грузов может быть
либо M/m = 1/(3 sin a) либо M/m = 3/ sin a.
273. В этой задаче удобно перейти в систему отсчёта
центра масс. Решение, в принципе, возможно получить и
в исходной системе отсчёта (в лабораторной), но при этом
возникают значительные технические трудности (которые
не смог преодолеть ни один из участников олимпиады,
действовавший таким способом). Для определённости, бу-
дем рассматривать тот шарик, который сначала был сле-
ва (см. рис. 1). Его скорость в системе центра масс даёт-
ся формулой u = V /2. Длина нити подобрана так, что ко-
гда она натягивается, угол между ней и вектором скорости
равен 45 ◦ C. В этот момент шарики упруго взаимодейству-
ют. При этом проекция скорости шариков на направление
348 Решения задач

К решению задачи 273

нити u меняет знак, а перпендикулярная нити составляю-


щая u⊥ остаётся неизменной. Эти проекции равны. Значит,
вектор скорости первого шарика просто «развернулся» на
90 ◦ C (см. uнов ); то же самое произошло и со вторым ша-
риком, так как в этой системе отсчёта шарики ведут себя
симметрично. Итак, в рассмотренной системе отсчёта шари-
ки будут двигаться вдоль сторон квадрата L со скоростями
V /2, при этом центр квадрата будет перемещаться со ско-
ростью V /2 в сторону, куда системе первоначально переда-
ли импульс (на имеющейся картинке — вверх). Вернёмся
в исходную (связанную с землей) систему отсчёта. Чтобы
сделать это, к вектору скорости шариков, двигающихся по
периметру квадрата, надо добавить вектор u, направленный
вверх. Соответствия между траекториями в системе отсчёта
центра масс (для, например, левого шарика) и в системе от-
счёта земли изображено на рис. 2 (см. ABCDEA . . ., точки
D и C на правом рисунке совпадают).
О т в е т. Траектории состоят из отрезков прямых; лома-
ная траектория ABCEA для левого шарика изображена на
правом рисунке. Траектория уходит дальше вверх — её сле-
дует продлить периодически вверх, надстроив над верхней
точкой A новые точки B, C, E и т. д.
Решения задач городских туров 349

К решению задачи 273

274. Для решения этой задачи необходимо рассмотреть


действие сил на тело, имеющего конечные размеры, т. е.
учесть моменты сил. Кроме того, важно правильно опреде-
лить направление сил, приложенных к доске, тем более, что
это направление разное при рассмотрении первого и вто-
рого вопросов задачи. Так, при решении первой части за-
дачи, когда необходимо найти соотношение между коэффи-
циентами трениями при условии, что доска покоится, сила
трения, действующая со стороны куба на доску, направлена
вверх. Напротив, для ответа на второй вопрос задачи необ-
ходимо исследовать, не заклинивает ли куб доской; в этом
случае, очевидно, сила трения, действующая со стороны ку-
ба на доску, направлена вниз. Дополнительные сложности
носят чисто математический характер и связаны с тем, что
необходимо решить систему нестрогих неравенств. На дос-
ку действуют сила тяжести mg, две силы реакции опоры (со
стороны куба N2 и со стороны пола N1 ) и две силы трения
(сила трения о куб F2 и сила трения о пол F1 ). Запишем
второй закон Ньютона в целом для доски (см. рис. 1),


→ → −
− → − → −→
N 1 + N 2 + F 1 + F 2 + mg = 0.
350 Решения задач

Спроектируем это векторное уравнение на горизонтальную


и вертикальную оси:
0x : N2 − F1 = 0,
0y : mg − F2 − N1 = 0.
Теперь запишем правило моментов сил относительно ниж-
ней точки доски,
(mg · L cos a)/2 − F2 L · cos a − N2 L · sin a = 0.
Исключим из уравнений силу тяжести и выразим F1 и F2
через N1 и N2 : F1 = N2 , F2 = N1 − 2N2 tg a.
По условию задачи, доска должна покоиться, поэтому на
неё должны действовать силы трения, которые не превос-
ходят по величине максимума сил трения покоя, F1 ≤ m1 N1 ,
F2 ≤ m2 N2 . В результате, мы имеем два неравенства, кото-
рые совместны только, когда m1 ≥ 1/(2 tg a + m2 ). Если за-
фиксировать коэффициенты трения, то последнее неравен-
ство означает, что угол a не должен быть слишком мал. Для
ответа на второй вопрос задачи исследуем, при каких со-
отношениях коэффициентов трения доску заклинивает. За-
клинивание происходит в тот момент, когда силы трения
направлены так, как показано рис. 2, и их векторная сум-
ма равна нулю. Проекции этого уравнения отличаются от
предыдущего случая только знаком при F2 ,
0x : N2 − F1 = 0, 0y : mg + F2 − N1 = 0,
и, также как и в первой части задачи, напишем уравнение
моментов сил:
(mgL · cos a)/2 + F2 L · cos a − N2 L · sin a = 0.
Силы трения не должны превосходить максимальных зна-
чений сил трения покоя. Решив полученную систему нера-
венств, так же как и в первом случае, получим соотношение
коэффициентов трения, при котором доску будет заклини-
вать: m1 ≥ 1/(2 tg a − m2 ). Теперь подставим, вместо m2 и 1/m1
величину tg a. Полученное неравенство при этом не нару-
шается: 2 · tg a − tg a − tg a = 0 (мы получили крайнюю точ-
ку, в которой оно превращается в строгое равенство). Зна-
чит, при данных значениях коэффициентов трения доску
Решения задач городских туров 351

К решению задачи 274

К решению задачи 274

будет заклинивать, и куб будет невозможно сдвинуть впра-


во.
О т в е т. Области значений коэффициентов трения, при
которых доска останется неподвижна, и где произойдёт её
заклинивание, для случая a = 30 ◦ C показаны на рис. 3.
275. Перейдём в систему отсчёта, которая в момент вы-
стрела начинает двигаться без начальной скорости вниз с
ускорением g. В этой системе отсчёта снаряд летит по пря-
мой с постоянной скоростью v0 , а шар поднимается вверх
без начальной скорости с ускорением g и к моменту взрыва
проходит путь gT2 /2. Точка взрыва снаряда в этой систе-
ме отсчёта находится на окружности с центром в точке C,
где был произведён выстрел, и радиусом v0 T. Ближайшая
к конечному положению шара B точка этой окружности ле-
жит на отрезке BC. Таким образом, стреляющие должны
целиться в точку, находящуюся на высоте gT 2 /2 над ша-
ром. Тангенс угла a, под которым следует стрелять, легко
352 Решения задач

К решению задачи 274

К решению задачи 275

определить из рисунка,
AB gT 2
tg a = = tg f+ .
AC 2l cos f

О т в е т. Следует стрелять под углом


 
sin f + gT 2 /2l
a = arctg .
cos f

276. Обозначим массу клина и бруска m. Возможны три


варианта движения системы: 1) клин скользит по столу,
брусок скользит по клину; 2) клин стоит на месте, а брусок
скользит по клину; 3) клин и брусок неподвижны. Ситуа-
ция, при которой брусок неподвижен относительно клина,
Решения задач городских туров 353

а клин скользит по столу, противоречит закону сохранения


энергии. Первый вариант реализуется при достаточно ма-
лых коэффициентах трения, m < m1 . При увеличении коэф-
фициента трения реализуется второй вариант: m1 < m < m2 .
При ещё больших коэффициентах трения становится воз-
можным третий вариант: m > m2 . Определим критические
значения коэффициента трения m1 и m2 . Значение m2 со-
ответствует ситуации, при которой и клин, и брусок ещё
неподвижны, но брусок вот-вот начнёт проскальзывать, т. е.
сила трения покоя, действующая на него, уже достигла сво-
его максимального значения. Действующие на брусок силы
изображены на рис. 1. Два уравнения получим из второго
закона Ньютона в проекциях на оси x и y:
OX : mg sin 45 ◦ C − Fтр = 0, OY : N − mg cos 45 ◦ C = 0.
Кроме того, воспользуемся условием Fтр = m2 N. Решая си-
стему трёх уравнений относительно m2 , получаем m2 = tg 45 ◦ C = 1.
Теперь определим значение m1 . Оно соответствует ситуации,
при которой брусок едет по клину. Силы, действующие на
брусок и клин, изображены на рис. 2. Согласно третьему за-
кону Ньютона, на клин со стороны бруска действуют силы
Fтр и N. Сам клин при этом ещё неподвижен, но сила тре-
ния между клином и столом уже достигла максимального
значения. Запишем второй закон Ньютона для бруска, дви-
жущегося с ускорением a, в проекциях на оси x и y, а для
клина в проекциях на оси x1 и y1 (см. рис. 2). Для бруска
имеем уравнения:
OX : mg sin 45 ◦ C − Fтр = ma, OY : N − mg cos 45 ◦ C = 0,
а для клина
OX : N sin 45 ◦ C − Fтр cos 45 ◦ C = Fтр1 ,
OY : N1 = N cos 45 ◦ C + Fтр sin 45 ◦ C + mg.
Учтём, что Fтр = m1 · N и Fтр1 = m1 · N1 . Такая система урав-
нений непротиворечива, если коэффициент трения m1 удо-
влетворяет квадратному уравнению m21 + 4m1 − 1 = 0, физи-
ческий
√ смысл имеет при этом его положительный корень
m1 = 5 − 2.
354 Решения задач

К решению задачи 276

К решению задачи 276


Решения задач городских туров 355

К решению задачи 277

О т в е т. Брусок будет скользить по √ клину при m < 1, а


клин будет скользить по столу при m < 5 − 2.
277. Обратим внимание на то, что горизонтальная ком-
понента силы натяжения нити будет одинакова во всех точ-
ках каната. Действительно, рассмотрим произвольный ку-
сочек каната массой m с силой натяжения T A и TB на кон-
цах. Сумма действующих на него сил должна быть равна
нулю,

− →
− →

T A + T B + m g = 0.
В проекции на горизонтальную ось это даёт постоянство
горизонтальной компоненты силы натяжения. В нижней
точке каната сила его натяжения горизонтальна, следо-
вательно, горизонтальная компонента всюду выражается
как TX = T3 . Но тогда, по теореме Пифагора, легко счи-
тается вертикальная компонента силы натяжения на ле-
вом и на правом концах каната T1Y = T12 − TX 2 = T12 − T32 ,

T2Y = T2 − TX = T22 − T32 . Из равновесия каната ясно, что
2 2

их сумма равна mg. p p


T2 − T2 T2 − T2
О т в е т. Масса каната равна m = 1 3
+ 2 3
.
g g
278. Можно построить мнимое изображение ракеты,
зеркально симметричное относительно поверхности озера.
Рассмотрим прямой луч EB от изображения к глазу и точ-
ку пересечения C этого луча с поверхностью воды AD. На
самом деле, свет, конечно, идёт от ракеты, отражается от
поверхности воды в точке C и затем попадает в глаз как
будто бы он вышел из точки E (см. рис.) Если в точке C
окажется катер, то он закроет изображение ракеты, поэто-
356 Решения задач

К решению задачи 278

му на самом деле в задаче требуется найти скорость точки


C в момент, когда ракета находится на высоте H. Обозна-
чим расстояние от наблюдателя до точки C через x. Подо-
бие треугольников ABC и EDC даёт x/h = (L − x)/H, откуда
находим координату точки C: x = hL/(H + h). Найдём теперь
скорость u этой точки (см. рис.), предположив, что ракета
сдвинулась на маленькое расстояние по вертикали. Пользу-
ясь стандартными формулами, описывающими равноуско-
ренное движение, легко найти, что в интересующий
√ нас мо-
мент времени ракета имела скорость V = 2aH. Введём век-
тор v, параллельный V, ограниченный линиями, соединя-
ющими наблюдателя и ракету до и после рассматриваемого
промежутка времени. Треугольник, образованный точкой B
и вектором V подобен треугольнику, образованному точкой
B и вектором v. Поэтому v = Vx/L. Треугольник, образован-
ный векторами v и u прямоугольный, а угол, прилегающий
к катету u, равен a. Отсюда u = v ctg a = vx/h = Vx2 /(Lh).
Подставляя сюда найденное значение скорости V и коорди-
наты x, получаем ответ.
О т в е т. Скорость катера должна быть равна u = hL(2aH)1/2 /(H + h)2 .
279. Очевидно, что при отсутствии трения шайба по дну
скользит прямолинейно с постоянной скоростью. Что про-
исходит в момент наезда на стенку? Так как радиус закруг-
ления очень мал, центростремительное ускорение, связан-
ное с изменением направления скорости шайбы под дей-
Решения задач городских туров 357

ствием стенки, много больше g и гравитацией при этом


рассмотрении можно пренебречь. Также можно пренебречь
кривизной стенки по сравнению с кривизной закругления.
Из-за отсутствия трения сила со стороны поверхности сосу-
да всегда перпендикулярна этой поверхности. Поэтому по-
ка шайба преодолевает закругление, компонента скорости
вдоль границы дна и стенки не меняется (в этом направле-
нии проекция силы реакции стенки равна нулю). Из-за со-
хранения энергии не меняется также модуль скорости. По-
этому шайба, заехавшая на стенку, движется вначале под
тем же углом к горизонтали, под которым подъезжала к
стенке (см. рис. 1). Рассмотрим теперь траекторию движе-
ния шайбы по вогнутой вертикальной стенке в поле тяже-
сти (см. рис. 2). По сходным соображениям можно увидеть,
что шайба, скользящая без трения, оставляет на стенке та-
кой след, что если стенку развернуть, след превратиться в
параболу. На втором рисунке изображён след шайбы на раз-
вертке цилиндра. Действительно, стенка действует на шай-
бу только по нормали, следовательно, в проекции на саму
стенку эта сила не влияет на движение. Из сказанного вид-
но, что шайба съезжает со стенки под тем же углом a, что и
заезжает на неё (на первом рисунке все обозначенные углы
равны a). Следовательно, по дну она всегда прочерчивает
хорду одной и той же длины. Траектория в общем случае со-
стоит из чередующихся таких хорд и участков, проходимых
по стенке, также одинаковых между собой. Выясним, каким
путём шайба может вернуться в начальную точку. Оказыва-
ется, что это возможно только перпендикулярно диаметру,
проходящему через точку старта, т. е. по той же прямой,
по которой шайба стартовала. Это следует из того, что на
любой хорде обсуждаемой длины есть единственная точка,
удалённая от центра окружности на расстояние L, это се-
редина хорды. Теперь выясним угловую меру пути, прохо-
димого по хорде, и пути, проходимого (при данной скоро-
сти) по стенке. Угол g с вершиной в центре дна, построен-
ный на интересующей нас хорде, равен 2p/3. Угол a легко
определяется из геометрических соображений и равен p/3.
Тогда вычислим горизонтальную V0x = V0 cos a и вертикаль-
ную компоненты начальной скорости для шайбы на стенке.
Время прохождения шайбы по стенке находим, исходя из
358 Решения задач

К решению задачи 279

того, что вертикальная компонента скорости в конце долж-


на стать противоположной начальной t = 2V0y /g. По криво-
линейной оси x, направленной горизонтально вдоль стенки,
движение равномерно и пройденный вдоль этой оси путь
легко находится (длина пути по √ развертке боковой части
цилиндра): s = V0x t = V0 sin 2a/g = 3V 2 /2g. Следовательно,
2
центральный угол между точками, где шайба заезжает на
стенку и съезжает с неё, равен b = s/R. Участков по дну и
по стенке одинаковое количество — M, если конечно учиты-
вать начальную и конечную половинки хорды как одну хор-
ду. Угол, пройденный вокруг центра, при начальной ско-
рости u естественно приравнивается числу полных оборо-
тов: M(g + b) = 2p · N. Разрешая это уравнение относительно
u, получаем значение скорости V . При этом важно, чтобы
шайба не доехала до верхнего ребра и не вылетела из сосу-
да, т. е., чтобы кинетическая энергия не превышала потен-
циальную энергию подъёма до верхней кромки сосуда mgh,
для чего требуется V < (8gh)1/2 . Подставляя в это неравен-
ство V , найдём окончательно условие, при котором описы-
ваемое движение шайбы возможно.
О т в е т. Траектории нумеруются такими целыми чис-
 √ 
1 2h 3
лами M и N, что N < M + , а скорость шайбы равна
  
3 pR
4pgR N 1
V= √ − .
3 M 3
Решения задач городских туров 359

К решению задачи 279

К решению задачи 279


360 Решения задач

На рис. 3 показана гистограмма возможных значений


безразмерной скорости V /(gR)1/2 для шайбы, совершающей
не более пяти выездов на стенки в цилиндре, в котором вы-
сота в три раза больше радиуса.
280. Если d = 0, то силы, действующие на малую шайбу
со стороны больших, компенсируются. В этом случае ско-
рость лёгкой  шайбы после взаимодействия u = 0. Другой
случай, d > (R + r)2 − R2 , возникает, когда тяжёлые шай-
бы вообще не соприкасаются с лёгкой (а лишь друг с дру-
гом), тогда  скорость лёгкой шайбы также остаётся u = 0.
При 0 < d < (R + r)2 − R2 шайбы будут взаимодействовать.
Так как, по условию задачи, массы тяжёлой и лёгкой шайб
сильно различаются, изменением импульса тяжёлых шайб
в процессе их взаимодействия с лёгкой шайбой можно пре-
небречь. Проведём ось Ox через центры тяжёлой и лёг-
кой шайбы, в тот момент, когда они соприкоснулись друг
с другом. Так как шайбы не деформируемы, то лёгкая шай-
ба моментально приобретает такую скорость, что проек-
ции скоростей тяжелей и лёгкой шайбы на ось Ox рав-
ны друг другу: ux = u sin a = Vx = V cos a. Из соображений
симметрии очевидно, что вектор скорости лёгкой шайбы
перпендикулярен скоростям тяжёлых шайб. Угол a легко
найти, рассматривая прямоугольный треугольник OAB:
d
sin a = . Выражая из полученных равенств скорость u,
R+r
получаем ответ.
Отвp е т. Скорость маленькой шайбы после разлёта рав-
(R + r)2 − d2
на u = V .
d
281. Пусть a — малый угол между канатом, оття-
нутым в сторону, и канатом в нерастянутом состоянии
(см. рис., вид сверху). Растяжение каната тогда выра-
жается как dL = L/ cos a − L = 2L sin2 (a/2)/ cos a. При ма-
лых углах a справедливо cos a ∼ = 1, sin(a/2) ∼
= a/2, так что

dL = La /2. Сила натяжения каната получается тогда рав-
2

ной T = k · dL ∼
= k · La2 /2. Её можно связать с силой F с помо-
щью условия равновесия каната в точке приложения силы
F = 2T · sin a = 2T · a. Получившаяся система двух уравнений

F 3 kL
легко разрешается относительно T и a. Получим T = .
2 F
Решения задач городских туров 361

К решению задачи 281

Чтобы плита поехала, значение силы натяжения должно


сравняться с силой трения плиты, T = mmg. Отсюда сразу
получаем ответ для максимальной массы плиты. Добавоч-
ная сила натяжения увеличивает возможную массу. Первое
уравнение принимает вид T = T0 + kLa2 /2, так что в уравне-
kLa2 2F
нии для a будем иметь T0 + = . Видно, что a, полу-
2 a
чаемое из второго уравнения, меньше, чем из первого. Зна-
чит, достигаемая сила натяжения T = F/2a станет больше,
и появится возможность тащить плиту большей массы.
О т в е т. Максимальная масса плиты, которую может
сдвинуть человек, прикладывая к ней силу F есть m = (F2 · kL)1/3 /2mg.
Начальное натяжение каната с силой даёт небольшой выиг-
рыш.
282. Так как кольца двигаются по стержням без тре-
ния, нить всегда перпендикулярна стержням. Расстояние
от бусины до верхнего стержня x и до правого стержня y
в сумме не меняются и дают L: x + y = L. Следовательно,
траектория бусины y = L − x является прямой, наклонённой
под углом 45 ◦ C к вертикали. Заметим, суммарная сила ре-
акции нитей перпендикулярна траектории. Значит, един-
ственная сила, имеющая проекцию на направление движе-
ния сила тяжести. Проекция силы тяжести на траекторию,
mg · cos 45 ◦ C = ma, обеспечивает
√ постоянное по модулю и
направлению ускорение a = g/ 2. Путь бусинки равен диа-
гонали квадрата со стороной L/2, поэтому легко найти вре-
мя, за которое бусинка его пройдёт. √
О т в е т. Ускорение бусины a = g/ 2, она достигнет вер-
тикального стержня через время t = 2 L/g.
283. Задачу удобно решать, направив ось Oz перпен-
дикулярно плоскости, ось Oy — вниз по плоскости, ось
362 Решения задач

Ox — вдоль плоскости перпендикулярно оси Oy. Сила нор-


мальной реакции плоскости, действующая на обе шайбы,
рассматриваемые как целое, равна проекции силы тяже-
сти на ось z: N = 2mg · cos a. Так как шайбы скользят, мо-
дуль силы трения достигает величины Fтр = m0 N, а её на-
правление противоположно направлению скорости. Уско-
рение шайб определяется по второму закону Ньютона,
2ma = 2mg + Fтр + N. В верхней точке траектории вектор си-
лы трения (как и вектор скорости) горизонтален, поэтому
ускорение имеет компоненты aBx = m0 g cos a, aBy = g sin a,
aBz = 0. Для верхней шайбы второй закон Ньютона можно
записать в виде F12 = ma − mg, где F12 сила, действующая
на верхнюю шайбу со стороны нижней. Сила тяжести имеет
следующие проекции на выбранные оси:

mgx = 0, mgy = mg sin a, mgz = mg cos a.

Рассматривая векторное уравнение для силы F12 в проек-


ции на оси и подставляя в него ma и mg, можно заме-
тить, что в любой (и в верхней тоже) точке траектории си-
ла, действующая между шайбами, определяется только си-
лой трения, поскольку силы вдоль оси Oy скомпенсированы
(«невесомость» для тела, «летящего по параболе» в плоско-
сти под действием «ускорения свободного падения» g sin a).
Как следствие, сила |F12 | постоянна по величине,

F12 = mg 1 + m20 · cos a,

в любой точке траектории. Постоянна по величине и её


составляющая в плоскости движения. Значит, если верх-
няя шайба не проскользнула по нижней сразу, она не про-
скользнёт по ней до конца.
О т в е т. Верхняя шайба действует
на нижнюю с посто-
янной по величине силой F12 = mg 1 + m20 · cos a. Проскаль-
зывание не может начаться, если его нет с самого начала.
284. Поскольку температура T в процессе меняется,
а значит, меняются электрическое сопротивление R и вы-
деляемая нагревателем мощность P, надо рассмотреть ма-
ленький промежуток времени Dt, за который температура
Решения задач городских туров 363

не успевает заметно измениться. Приравняв выделившую-


ся энергию P · Dt = U 2 · Dt/(R0 + a · T) к энергии, необходимой
для нагревания, c · m · DT, мы можем связать малое прира-
щение времени Dt с малым приращением температуры DT:
cm · R0 ca
Dt = · DT + · T · DT.
U2 U2

Время закипания воды t (нагрева от T0 до TK ) можно найти


из этого (вообще говоря, дифференциального) уравнения,
связывающего приращения функции и её аргумента разны-
ми способами.
1 способ. Заметим, что отношение V = Dt/DT даёт вы-
ражение для тангенса угла наклона в касательной в точке
кривой, отображающей зависимость времени нагрева t от
температуры T (см. рисунок). Рассмотрим движение неко-
его гипотетического тела вдоль одной прямой, такое, что
величина t играет роль координаты тела, а величина T —
времени. Величина V здесь будет играть роль скорости дви-
жения. Линейный характер зависимости V от T указывает
на то, что рассмотренное гипотетическое движение являет-
ся равноускоренным и кривая t(T) является параболой. По-
этому можно воспользоваться аналогией с кинематической
задачей о равноускоренном движении.
2 способ. В настоящем решении мы проведём явное ин-
тегрирование последнего уравнения путём суммирования
всех приращений времени нагрева dt. Для этого разобьём
интересующий нас интервал температур TK − T0 = N · dT на
очень большое число N одинаковых маленьких промежут-
ков температуры dT. Вода нагреется от начальной темпера-
туры T0 до температуры T0 + dT за время:
cm · R0 ca
dt1 = · dT + · T0 · dT.
U2 U2

В течение следующего промежутка времени вода разогре-


ется от T0 + dT до T0 + 2 · dT и т. д. Очевидно, что время t
нагрева воды до температуры N · dT равно сумме всех про-
межутков времени dtn :
cm · R0 ca
dt2 = · dT + · (T0 + dT) · dT.
U2 U2
364 Решения задач

Арифметическая прогрессия во втором слагаемом последне-


го равенства даёт:


N  
cm · R0 ca ca
t= dtn = + 2 · T0 · (N · dT) + · (1 + 2 + . . . + N) · dT2 .
U2 U U2
n=1

Очень большое число N компенсирует малость интервала


dT в произведении (N · dT) и в его квадрате (N · dT)2 . По
сравнению с ними число N · dT 2 — очень мало, им можно
пренебречь, что и сделано в последнем выражении. В ито-
ге, получим
N · (N + 1) N2 N2
· dT 2 ∼
N
(1 + 2 + . . . + N) · dT2 = · dT2 = · dT 2 + = · dT2 .
2 2 2 2

Итак, время нагрева воды на N · dT = TK − T0 получается из


последнего равенства:
 
cm · R0 ca ca
t= 2 + 2 · T0 · (N · dT) + 2 · (N · dT)2 .
U U 2U

О т в е т. Время нагрева воды от начальной температуры


до температуры кипения равно
 
cm · R0 ca ca
t= + 2 · T0 · (TK − T0 ) + · (TK − T0 )2 =
U2 U 2U 2
cm · R0 ca
= · (TK − T0 ) + · (TK
2
− T02 ).
U2 2U 2

285. По условию, мощность теплоотдачи из бассейна


пропорциональна разности температур воды и окружающей
среды: P = a · Dt. Теплоотдача обуславливается тем, что по-
ступающая в бассейн вода теплее, чем вытекающая. Обо-
значим массу воды, поступающей в единицу времени в бас-
сейн из одного крана, m. Тогда приток тепла в единицу вре-
мени равен P = cm(tin − tout ) (c — теплоёмкость воды, tin и
tout — температура втекающей и вытекающей воды). Та-
ким образом, можно записать уравнения баланса для ситу-
аций, когда открыт кран только с горячей или только с тёп-
лой водой: a(T1 − T0 ) = cm(T1 − T1 ) и a(T2 − T0 ) = cm(T2 − T2 ).
Здесь T0 — температура окружающей среды. Поделим
Решения задач городских туров 365

К решению задачи 284

T1 − T0 T1 − T1
уравнения друг на друга:  = . Из этого урав-
T2 − T0 T2 − T2
T1 T2 − T1 T2
нения можно найти T0 = = 10 ◦ C. Обозна-
(T1 − T1 ) − (T2 − T2 )
чим температуру, которая установится в бассейне, если
открыть оба крана, T. Она удовлетворяет соотношению
a · (T − T0 ) = cm · (T1 − T) + cm · (T2 − T). Разделив это на первое
T−T T + T − 2T
уравнение, получим:  0 = 1 2  . Откуда находим и
T1 − T0 T1 − T1
температуру, установившуюся в бассейне:
T1 (T1 + T2 − T0 ) − T0 T2 T1 T1 − T2 T2
T= = .
T1 + T1 − 2T0 (T1 + T1 ) − (T2 + T2 )

О т в е т. Если открыть оба крана, в бассейне установит-


ся температура 46 ◦ C.
286. Когда утюг включён в сеть, его нагреватель пе-
риодически включается и выключается. Пусть мощность
нагревателя P, а мощность, отдаваемая утюгом в окру-
жающее пространство, равна P1 и P2 в первом и вто-
ром случае соответственно. Обозначим время, в течение
которого нагреватель включён и выключен как tвкл1 и
tвыкл1 в первом случае и tвкл2 и tвыкл2 во втором случае.
Тогда (P − P1 )tвкл1 = P1 tвыкл1 = (P − P2 )tвкл2 = P2 tвыкл2 = Q,
где Q = C(t2 − t1 ) (C — теплоёмкость утюга) — количе-
ство теплоты, необходимое для нагрева утюга от тем-
пературы t1 до температуры t2 . Из условия известно,
что в первом случае утюг работает k = 1/4 всего вре-
мени, т. е. k = tвкл1 /(tвкл1 + tвыкл1 ). Также известно, что
366 Решения задач

во втором случае время между последовательными мо-


ментами включения нагревателя в n = 4/3 раза мень-
ше, tвкл1 + tвыкл1 = n(tвкл2 + tвыкл2 ). Этой системы урав-
нений достаточно для решения задачи. Обозначим ис-
комую долю времени k2 , k2 = tвкл2 /(tвкл2 + tвыкл2 ). Тогда
из (P − P1 )tвкл1 = P1 tвыкл1 следует (P − P1 )k = P1 (1 − k) и
P1 = k1 P, и аналогично P2 = k2 P. Из вышеприведённых
уравнений следует, что tвкл1 /k = ntвкл2 /k2 . Подставляя это
выражение в соотношение (P − P1 )tвкл1 = (P − P2 )tвкл2 , по-
лучаем (P − P1 )k = (P − P2 )k2 /n, откуда nk(1 − k) = k2 (1 − k2 ).
Решение этого квадратного уравнения k2 = 1/2.
О т в е т. Нагреватель утюга будет работать в течение по-
ловины времени.
287. Обозначим поток тепла к первому шару q1 , а поток
тепла от третьего шара q3 . Тогда поток тепла от второго ша-
ра равен q1 − q3 . Также обозначим температуры мест соеди-
нения стержней t A и tB . Поскольку поток тепла по каждому
стержню пропорционален разности температур на его кон-
цах, имеют место следующие соотношения: q1 = a(t A − t1 ),
q3 = a(tB − t A ) = a(t3 − tB ), q1 − q3 = a(t2 − t A ). Здесь a — неко-
торый коэффициент пропорциональности. Исключим его из
уравнений:
q1 q3 q q −q
= = 3 = 1 3.
t A − t1 tB − t A t3 − t B t2 − t A
Из этой системы можно найти:
2t1 + 2t2 + t3
tA = = 40 ◦ C,
5
t + t + 3t3
tA = 1 2 = 70 ◦ C,
5
q3 t + t − 2t3 3
= 1 2 = .
q1 3t1 − 2t2 − t3 4

Таким образом, 3/4 тепла, поступающего к первому шару,


идёт от третьего, а 1/4 — от второго. Следовательно, третий
шар остыл на 0,3 ◦ C, а второй — на 0,1 ◦ C.
О т в е т. Температура второго шара 49,9 ◦ C, третьего
99,7 ◦ C.
288. Прежде всего, проградуируем ось скоростей. В на-
чальный момент скорость тела равна нулю — равна нулю
и сила трения о воздух, поэтому тело двигается лишь под
Решения задач городских туров 367

К решению задачи 287

действием силы тяжести и его ускорение равно g. Прове-


дём касательную к графику в точке t = 0. Её наклон ра-
вен, как следует из измерения по графику, 2 дел/сек, но,
с другой стороны, он же должен быть равен g = 10 м/сек2 .
Значит, каждое деление на оси скоростей соответствует
5 м/сек. Проще определить тепловую мощность, выделя-
ющуюся в момент времени t2 = 18 сек. К этому момен-
ту скорость тела практически перестаёт изменяться — си-
ла трения компенсирует силу тяжести mg. Скорость тела
определяется из графика и равна V2 = 50 м/сек. Выделя-
ющаяся тепловая мощность равна мощности силы трения
Pтр = Fтр · V2 = mg · V2 = 35 кВт. Чтобы определить мощность,
выделяющуюся в момент времени t1 = 5 сек, запишем вто-
рой закон Ньютона для тела: ma = mg − Fтр  , где a и F  , со-
тр
ответственно, ускорение и сила трения в этот момент вре-
мени. Скорость тела равна V1 = 38 м/сек. Ускорение мож-
но определить по наклону касательной в данной точке, оно
равно a = 4,2 м/сек2 . Выделяющаяся тепловая мощность
равна P = Fтр  · V = m(g − a) · V = 15,5 кВт. Формулу для вы-
1 1
деляющейся тепловой мощности можно получить и другим
способом с помощью закона сохранения энергии. Рассмот-
рим короткий промежуток времени Dt, за который скорость
тела изменилась с v до (v + Dv), а высота — с h до h − Dh. Ес-
ли бы трения не было, полная механическая энергия систе-
мы (mv2 /2 + mg · h) сохранялась бы. Из-за трения механиче-
ская энергия превращается в тепло, значит выделившееся
368 Решения задач

К решению задачи 288

за Dt тепло равно уменьшению механической энергии систе-


мы: DQ = EH − EK = m[v2 − (v + Dv)2 ]/2 + mg · Dh = −m · (2v · Dv + (Dv)2 )/2 + mg · Dh.
Изменение скорости Dv за рассматриваемый малый проме-
жуток времени Dt достаточно мало, чтобы можно было пре-
небречь слагаемым (Dv)2 по сравнению с 2v · Dv, поэтому
DQ ∼
= −m · v · Dv + mg · Dh. Тепловая мощность — это, по опре-
делению, P = DQ/Dt = −mv · Dv/Dt + mg · Dh/Dt = −m · v · a + mg · v = m(g − a) · v.
При выводе использованы формулы для мгновенной скоро-
сти v = Dh/Dt и ускорения a = Dv/Dt.
О т в е т. Тепловая мощность, выделявшаяся из-за тре-
ния о воздух в моменты времени t1 = 5 сек и t2 = 18 сек рав-
ны 15,5 кВт и 35 кВт соответственно.
289. Обозначим плотности воды, пробки и соли r0 ,
rП и r соответственно. Пусть при некоторой температу-
ре растворимость соли равна k. Определим плотность на-
сыщенного раствора соли при этой температуре. Рассмот-
рим некоторый объём воды V1 . Масса соли в граммах,
которая может раствориться в этой воде, равна kV1 . То-
гда масса раствора будет равна r0 V1 + k · V1 . Поскольку уро-
вень жидкости в резервуаре при растворении соли не ме-
Решения задач городских туров 369

няется, объём раствора равен сумме объёмов воды и соли


V1 + k · V1 /r. Таким образом, плотность насыщенного рас-
твора равна rР = (r0 + k)/(1 + k/r). Теперь запишем условие
всплытия мешка. Рассмотрим силы, действующие на си-
стему «мешок с солью + поплавок». Часть соли из меш-
ка (массой k · V1 ) растворяется, увеличивая плотность жид-
кости и уменьшая массу мешка с солью. Сила тяжести,
действующая на систему, состоит из веса пробки и ве-
са нерастворившейся соли и равна FТ = r1 Vg + (M − k · V1 )g.
Архимедова сила действует на пробку и мешок и рав-
на FА = rР g · V + rР g(M − k · V1 )/r. Поэтому условие всплытия
мешка суть FТ = FА , которое после, подстановки туда всех
известных величин, приобретает вид
 
r(r0 + k) M − k · V1
rП gV + (M − k · V1 )g = ·g· V + .
(r + k) r

Выражая из этого уравнения k и подставляя численные зна-


чения величин, получаем, что k = 330 г/л. По графику на-
ходим температуру, при которой соль имеет растворимость
330 г/л. Эта температура равна 18 ◦ C, поэтому систему на-
до нагреть на DT = 3 ◦ C. Вводя теплоёмкости воды, пробки
и соли c0 , cП и c соответственно, легко получаем ответ.
О т в е т. Теплота, необходимая для всплытия мешка,
равна Q = DT · (cП rП V + c0 r0 V1 + cM) = 13,9 МДж.
290. Условию задачи удовлетворяет, например, схема,
приведённая на рисунке. При напряжении меньше 1 В ток
через цепь не течёт. Когда напряжение превышает 1 В, ле-
вый диод открывается, и ток начинает идти через нижний
резистор (при этом зависимость тока от напряжения линей-
на). Когда напряжение достигает 2 В, открывается и сред-
ний диод, и ток начинает идти и через средний резистор.
В момент открытия среднего диода напряжение на нижнем
резисторе равно 1 В, и ток через него равен 1 мА. Когда
напряжение достигнет 3 В, откроется и правый диод, и ток
пойдёт через все резисторы. В момент открытия правого ди-
ода напряжение на нижнем резисторе равно 2 В (ток 2 мА),
на среднем 1 В (ток 1 мА), так что полный ток через цепь
составляет 3 мА. Сопротивление верхнего резистора подо-
брано так, чтобы при напряжении 4 В ток через схему со-
ставлял 7 мА при этом напряжения на резисторах равны
370 Решения задач

К решению задачи 290

К решению задачи 291

3 В, 2 В и 1 В (считая снизу вверх), а токи через них 3 мА,


2 мА и 2 мА соответственно.
291. Судя по тому, что напряжения U1 и U2 отли-
чаются не в 3 раза, вольтметр у знатока физики был не
идеальный. Обозначим сопротивление вольтметра R, а со-
противление каждого из резисторов r. Первая схема, ко-
торая возникла при измерениях, показана на рис. 1. В
этой схеме вольтметр показывает как раз то напряжение,
которое выдаёт источник. Значит, напряжение источника
равно U1 . Вторая схема дана на рис. 2. Она эквивалент-
на последовательному соединению резисторов сопротивле-
ниями Rr/(R + r) и 2r, поэтому вольтметр показывает на-
пряжение U2 = U1 Rr/(Rr + 2r) = U1 R/(3R + 2r). Отсюда мож-
но найти отношение сопротивлений резистора и вольтмет-
ра: r/R = U1 − 3U2 /2U2 = 1/6. В третьей схеме (см. рис. 3)
и эквивалентной ей (см. рис. 4) вольтметр показывает
U2 = U1 · 2R/(2R + r) = 4U1 U2 /(U1 + U2 ).
О т в е т. Вольтметр показал 2,77 В.
292. Сопротивление очень короткого участка цилин-
дра длины DL с удельным сопротивлением r выражается
формулой DR = r · DL/S, где S — площадь сечения цилин-
дра. Разность потенциалов на выбранном участке цилин-
дра, очевидно, Df = I · DR = IkL · DL/S. Просуммировать раз-
Решения задач городских туров 371

К решению задачи 291

К решению задачи 291

К решению задачи 291


372 Решения задач

ности потенциалов Df, чтобы найти разность потенциалов


на концах цилиндров f(L), можно, заметив, что это пло-
щадь под графиком функции f(L) = IkL/S (действительно,
для выбранного участка цилиндра разность потенциалов
численно равна заштрихованному кусочку под графиком на
рисунке. При этом, вследствие малости величины DL, от-
личием формы заштрихованного кусочка от прямоугольни-
ка можно пренебречь. Площадь прямоугольного треуголь-
ника с катетами L и IkL/S и соответствует искомой раз-
ности потенциалов). Возможно также воспользоваться пол-
ной формальной аналогией с равноускоренным движени-
ем тела (L аналогично времени t, постоянный множитель
Ik/S — ускорению a, выражение f(L) = IkL/S — мгновен-
ной скорости V (t) = at, искомая величина f — координа-
те тела x. Очевидно, x = at2 /2 ⇒ f = IkL2 /(2S)). Итак, если
потенциал начала стержня принять за ноль, потенциал на
расстоянии L от начала будет f = IkL2 /(2S). Собственно, в
этой задаче нам нужен только факт пропорциональности по-
тенциала квадрату расстояния от начала стержня f ∼ L2 .
Так как разность потенциалов между концами цилиндра
f(L) ∼ L2 = U0 , очевидно, разность потенциалов между нача-
лом и серединой цилиндра будет равна f(L/2) = U0 /4. Вто-
рой стержень включён в противоположном направлении,
так что разность потенциалов между концом второго стерж-
ня и его серединой по-прежнему равна U0 /4, значит, се-
редина второго стержня будет, очевидно иметь потенци-
ал [U0 − (U0 /4)]. Итак, разность потенциалов, между сере-
динами противоположно подключённых стержней составит
[U0 − (U0 /4)] − U0 /4 = U0 /2.
О т в е т. Показания идеального вольтметра будут равны
U0 /2.
293. В первую очередь заметим, что если схема состоит
из двух последовательно соединённых резисторов, то при
увеличении сопротивления одного из них сопротивление
всей схемы увеличится. Точно так же, если схема состоит
из двух параллельно соединённых резисторов, то при уве-
личении сопротивления одного из них сопротивление всей
схемы увеличится (поскольку проводимость R−1 этого ре-
зистора уменьшится, и проводимость всей схемы, равная
сумме проводимостей резисторов, также уменьшится). Уча-
Решения задач городских туров 373

К решению задачи 292

сток схемы правее прямой n на рис. 1 состоит из последо-


вательно соединённых реостата и резистора. Поскольку со-
противление реостата увеличивается, сопротивление участ-
ка схемы правее прямой n также увеличивается. Участок
схемы правее прямой n − 1 на рис. 1 состоит из параллель-
но соединённых резистора и ранее рассмотренного участка
схемы, сопротивление которого увеличивается. Поэтому со-
противление участка схемы правее прямой n − 1 также уве-
личивается. Продолжая рассуждения в том же духе, полу-
чим, что сопротивление участка схемы, лежащего справа от
любой отмеченной на рисунке пунктирной прямой, увели-
чивается при перемещении ползунка реостата. То же мож-
но сказать и об общем сопротивлении схемы. Поскольку об-
щее сопротивление схемы увеличивается, полный ток че-
рез неё (т. е. ток I1 ) уменьшается (см. рис. 2). Разность по-
тенциалов (т. е. напряжение) между точками A и B равна
I1 · R1 , разность потенциалов между точками B и C обозна-
чим U1 . Значит, разность потенциалов между точками A и
C равна I1 · R1 + U1 , но, с другой стороны, она равна U —
напряжению источника: U = I1 · R1 + U1 . Поскольку при пе-
ремещении ползунка I1 уменьшается, а U и R1 не изменя-
ются, U1 увеличивается. Значит, ток через резистор, на ко-
тором падает напряжение U1 , также увеличивается. Ток I2
равен разности тока I1 и тока через резистор, на котором
падает напряжение U1 . Первый уменьшается, а второй уве-
личивается, поэтому ток I2 уменьшается. Далее, поскольку
U1 = I2 · R2 + U2 , напряжение U2 увеличивается. Продолжая
такие рассуждения, получим, что все токи I1 , . . . , Ik , Ik+1
уменьшаются, а все напряжения U1 , . . . , Uk увеличивают-
ся. Но амперметр показывает ток Ik+1 , а вольтметр — на-
374 Решения задач

К решению задачи 293

К решению задачи 293

пряжение Uk . Значит, показания амперметра уменьшатся,


а вольтметра — увеличатся.
О т в е т. Показания амперметра уменьшатся, а вольт-
метра — увеличатся.
294. Обозначим сопротивления схемы R1 , R2 и R3 . Вос-
пользуемся формулами для сопротивления параллельных и
последовательных соединений, чтобы выразить полное со-
противление схемы во всех трёх случаях: первый случай
R1 · R2 /(R1 + R2 ) + R3 = 10 Ом, второй случай R2 · R3 /(R2 + R3 ) + R1 = 1 кОм,
третий случай R1 · R3 /(R1 + R3 ) + R2 = 10,02 Ом. Приведём к
общему знаменателю левые части каждого из этих уравне-
ний: L/(R1 + R2 ) = 10 Ом, L/(R2 + R3 ) = 1 кОм, L/(R1 + R3 ) = 10,02 Ом,
где L = R1 · R2 + R2 · R3 + R1 · R3 . Разделим второе и третье
уравнения на первое, R1 + R2 /(R2 + R3 ) = 100, R1 + R2 /(R1 + R3 ) = 1,002.
Рассмотрим эти два уравнения как систему с неизвестными
R1 и R2 :
R1 + R2 = 100(R2 + R3 ), R1 + R2 = 1,002(R1 + R3 ).
Выразим их через R3 и подставим эти выражения в самое
первое уравнение.
Решения задач городских туров 375

О т в е т. Сопротивления резисторов равны R1 = 997,6 Ом,


R2 = 6,020 Ом и R3 = 4,016 Ом.
295. Выпишем уравнения, характеризующие термоди-
намику процессов в приборе. Его сопротивление линейно
зависит от температуры и при минимальном токе (т. е. ко-
гда джоулево тепло, выделяющееся в приборе настолько ма-
ло, что температура прибора совпадает с комнатной) равно
R0 . Следовательно, при температуре T сопротивление при-
бора имеет вид R = R0 (1 + a · DT), а закон Ома записывается
как U = I · R0 (1 + a · DT), где DT = T − T0 — превышение тем-
пературы прибора над комнатной (T0 ), a a — температур-
ный коэффициент. По условию, мощность теплоотдачи с по-
верхности прибора пропорциональна перепаду температур
U · I = b · DT, где b — неизвестный коэффициент пропорци-
ональности. Из этих двух уравнений можно найти, как пе-
R I2
регрев прибора DT растёт с ростом тока I: DT = 0
.
(b − aR0 I2 )
Из полученной формулы видно, что DT неограниченно воз-
растает, когда величина тока приближается к значению
(b/aR0 )1/2 которое, таким образом, естественно отождеств-
ляется с известным по условию значением I0 . Это даёт воз-
можность выразить коэффициент b в уравнениях через a,
b = I02 a · R0 . Исключая DT из уравнений, получаем вольт--
амперную характеристику прибора (см. рисунок), выража-
ющую зависимость напряжения на нём, от протекающего
тока, U(I) = R0 I02 · I/(I02 − I2 ). В задаче спрашивается значе-
ние U(I0 /2), которое легко определяется из последнего ра-
венства.
О т в е т. Когда через прибор пропускают ток I0 /2, на-
пряжение на нём оказывается 2R0 I0 /3.
296. Обозначим радиус орбиты через R. Тогда расстоя-
ние между звёздами 2R. Скорость вращения звёзд u можно
найти из условия, что сила гравитационного притяжения
звёзд друг к другу обеспечивает центростремительное уско-
рение при движении по круговой орбите:

GM2 Mu2 GM
= ⇒u= .
4R4 R 4R
Заметим, что механическая энергия EM рассматриваемой
системы отрицательна, ведь, чтобы развести притягиваю-
376 Решения задач

К решению задачи 295

щиеся тела бесконечно далеко друг от друга, где они пе-


рестают взаимодействовать (где их потенциальная энергия
равна нулю), нужно совершить положительную работу. По-
GM2
тенциальная энергия равна EП = − (вспомним анало-
R
гичное выражение для энергии притягивающихся зарядов
kq2
q и −q в электростатике, EП = − ).
Для суммы кинетиче-
R
ских энергий обеих звёзд и потенциальной энергии их вза-
имодействия имеем:

Mu2 GM2 M GM GM2 GM2


EM = 2 − =2 − =− .
2 2R 2 4R 2R 4R

При исчезновении оболочки кинетическая энергия системы


уменьшится на DEK = −DM · u2 /2 = −GM · DM/8R. Потенци-
альная энергия системы возрастёт (она отрицательна, а её
абсолютная величина уменьшится), её изменение будет рав-
но DEП = GM · DM/2R. Система распадётся, если её энергия
станет равна нулю (действительно, при этом для того, чтобы
расстояние между звёздами стало очень большим, нужно со-
вершить нулевую работу). В результате получаем неравен-
ство: DEП + DEK ≥ |EM |, из которого следует, что звёздная
система распадётся, если DM ≥ 2/3 · M.
О т в е т. Система двух звёзд одинаковой массы распадёт-
ся, если масса оболочки, сброшенной одной из них, превос-
ходит 2/3 массы звезды.
Решения задач городских туров 377

297. Задачу удобнее решать, разбив систему на сле-


дующие части: жидкость, целый куб (без полости) плот-
ностью r1 , полость — тело отрицательной массы, плотно-
стью −r1 . Заметим, когда на куб без выреза «наклады-
вают» полость отрицательной массы, в области полости
плотность куба r1 и плотность полости −r1 компенсиру-
ют друг друга, так что получается область с нулевой плот-
ностью. Найдём, где расположены центры тяжести упомя-
нутых частей системы. Если куб наклонять медленно, то
поверхность жидкости будет всё время параллельна зем-
ле, и, следовательно, центр тяжести жидкости будет ле-
жать на перпендикуляре OF, проходящем через центр по-
лости (см. рис. 1). Центр тяжести полости находится в точ-
ке O, |OD| = k · r, AC — перпендикуляр к плоскости, про-
ведённый через точку опоры A. Центр тяжести целого ку-
ба (без выреза) расположен в точке B, |DB| = n · r/2. Силы
тяжести, действующие на все компоненты системы, изоб-
ражены на рис. 2. Рассмотрим вращение относительно точ-
ки A. Момент силы реакции пола относительно этой точки
равен нулю. Куб опрокинется, если момент силы тяжести
полости с водой оказывается больше или равен моменту си-
лы тяжести целого куба: (mЖ − |mП |) · |NP| ≥ mК · |BM|. Отре-
зок BM — плечо силы тяжести целого куба, NP — плечо
силы тяжести полости с жидкостью; mК = r1 (n · r)3 — мас-
са целого куба, mЖ = 2pr2 · r3 /3 — масса жидкости в поло-
сти, |mП | = 4pr1 · r3 /3 — масса полости вырезанной части ку-
ба. Осталось найти отношение |BM|/|MP|. Из рассмотрения
треугольников на рисунке легко получить
соотношение для
BM BN |DN| − |DB|
плеч соответствующих сил:
NP
= =
NO
. Из тре-
|OD| − |DN|
угольника DNA (см. рис. 1) легко найти |DN| = (nr · ctg a)/2.
После подстановки этого соотношения в предыдущее равен-
ство можно найти тригонометрическую функцию угла на-
клона куба, при котором наступает его опрокидывание

2(mЖ − mП )k + mК n
ctg a ≤ .
n(mК − mП + mЖ )

Остаётся только подставить сюда все массы, чтобы получить


окончательный ответ.
378 Решения задач

К решению задачи 297

К решению задачи 297

О т в е т. Куб опрокинется, если


4pk/3n(r2 − 2r1 ) + r1 n3
ctg a ≤ .
2p/3(r2 − 2r1 ) + r1 n3

298. Пусть пробка сдвинется на Dy вверх. Тогда вода


сдвинется на столько же вниз. Изменение потенциальной
энергии системы при этом равно DEП = (m2 − m1 )gDy = (r2 − r1 ) · S1 h · g · Dy,
здесь m2 — масса пробки, m1 — масса жидкости, которую
вытесняет пробка. При сдвиге пробки у системы появит-
ся и кинетическая энергия DEK = m2 V 2 /2 + MV12 /2 + mV22 /2.
Здесь V — скорость движения пробки, V1 — скорость дви-
жения воды в широкой части трубки, M — её масса, V2 —
скорость движения воды в узкой части трубки, m — её мас-
Решения задач городских туров 379

са. Выражая m2 , m и M через соответствующие плотности и


объёмы, имеем DEK = (r2 · S1 h · V 2 + r1 · S1 · (L1 − h)V12 + r1 · S2 L2 · V22 )/2.
Поскольку жидкость практически несжимаема, соотноше-
ние скоростей жидкости в широкой и узкой частях труб-
ки можно найти из условия S2 · V2 = S1 · V1 . Очевидно, что
V1 = V . Вдобавок, полная энергия в системе не меняется,
так как она замкнута, поэтому DEП + DEK = 0, следователь-
но, для скорости движения пробки получим выражение
2(r1 − r2 )ghDh
V2= .
r2 h + r1 (L1 − h) + r1 L2 S1 /S2

В первый момент сила, действующая на пробку, будет по-


стоянна, значит, движение можно считать равноускорен-
ным. Это позволяет, зная скорость, найти начальное ускоре-
ние пробки a = V 2 /(2 · Dy). Заметим, что (r1 − r2 ) > 0, поэтому
ускорение всегда направлено вверх.
О т в е т. Начальное ускорение пробки равно
(r1 − r2 )gh
a= .
r2 h + r1 (L1 − h) + r1 L2 S1 /S2

299. Поскольку удар кратковременный, можно считать,


что за время удара расстояние между вагонами не успевает
измениться. Пружина в начальный момент не деформиро-
вана, и её присутствие никак не сказывается на скорости ва-
гонов сразу после удара. Мы можем рассматривать отдельно
столкновение пустого вагона с груженым вагоном. Запишем
законы сохранения импульса и энергии (M и m — массы
груженого и пустого вагонов соответственно, V1 и u — их
скорости): MV = MV1 + mu, MV 2 /2 = MV12 /2 + mu2 /2. Учи-
тывая, что m = M/2, из предыдущих уравнений получим
V1 = V /3 и u = 4V /3. Отметим, что сразу после удара ско-
рость первого (испытавшего удар) пустого вагона равна u,
а скорость второго пустого вагона равна нулю. Благодаря
пружине, пустые вагоны будут обмениваться энергией, и
начнутся колебания. Однако общий импульс пары вагонов
после удара будет постоянен (на пустые вагоны не действу-
ют нескомпенсированные внешние силы) значит и скорость
VC движения центра масс пустых вагонов после удара по-
стоянна. Её можно найти из уравнения m · u + 0 = 2m · VC . От-
сюда VC = 2V /3. Относительно центра масс первый вагон в
380 Решения задач

начальный момент движется со скоростью 2V /3, а второй —


со скоростью (−2V /3) (см. рис.). Пружина начнёт сжимать-
ся. Найдём период и амплитуду гармонических колебаний
пустых вагонов на пружине (рассматриваем движение ваго-
нов в системе центра масс). При смещении каждого ваго-
на на Dx общая деформация пружины будет равно 2 · Dx, и,
следовательно, на каждый вагон со стороны пружины дей-
ствует сила F = 2k
· Dx = ma. Следовательно, период колеба-
m
ний равен T = 2p . Заметим, что ответ получился такой
2k
же, как для периода колебаний тела массой m на пружине
жёсткостью 2k. Дело в том, что середина пружины непо-
движна, т. е. каждое тело, фактически, колеблется на пру-
жине половинной длины. С другой стороны, известно, что
уменьшение длины пружины в n раз влечёт увеличение её
жёсткости во столько же раз. Таким образом, мы получили
вполне естественный ответ для T. Найдём теперь амплиту-
ду A колебаний вагонов в системе центра масс. Очевидно,
максимальное сжатие пружина будет иметь, когда каждый
вагон сместится к центру масс на A, а сами вагоны оста-
новятся, чтобы начать затем движение в противоположную
сторону. При этом (см. рис.) начальная кинетическая энер-
гия системы 2m(2V /3)2 /2 целиком перейдёт в потенциаль-
ную энергию пружины k(2A)2 /2. Приравнивая эти энергии,
найдём амплитуду колебаний

V 2m
A= .
3 k

Положение первого вагона относительно центра масс в мо-


мент времени t будет задаваться расстоянием
   
l 2pt l V 2m 2k
L3 = − A sin = − sin t.
2 T 2 3 k m

За время t гружёный вагон пройдёт расстояние L1 = V · t/3,


а центр масс пары пустых вагонов пройдёт расстояние
L2 = 2V · t/3. Теперь легко найти расстояние между груже-
ным и ближайшим к нему пустым вагоном через время t
после столкновения S = L2 − L1 − L3 .
Решения задач городских туров 381

К решению задачи 299

О т в е т. Расстояние между груженым и ближайшим к


нему пустым  вагоном
через
время t после столкновения
V 2m 2k l
равно S = t+ sin t − .
3 k m 2
300. Обозначим через S = 2s — площадь дна бочки.
Пока ведро полностью находится под водой, на него, про-
тиводействуя силе динамометра F1 , действуют сила тяже-
сти и архимедова сила: F1 = mg − F A = mg − r0 · gsh. Эта фор-
мула справедлива, пока ведро не начало подниматься над
бочкой, т. е., пока расстояние от дна ведра до дна бочки
y изменяется от L до H − h. Пусть теперь ведро подни-
мается над водой на Z. Обозначим уровень воды в боч-
ке в этот момент H1 . Очевидно, воды в бочке стало мень-
ше на (H − H1 )S. Полное количество воды в ведре и боч-
ке остаётся постоянным, поэтому (H − H1 )S = sZ. С другой
стороны, H1 = y + (h − Z) (см. рис. 1). Из последних уравне-
ний легко найти, с учётом S/s = 2, что Z = 2(y + h − H). Те-
перь определим зависимость показаний динамометра от y:
F2 = mg − F A = mg − r0 · g · s(h − Z) = mg − r0 · g · s(2H − 2y − h). Она
будет справедлива до тех пор, пока дно ведра не сравняет-
ся с уровнем воды в бочке. Это произойдёт при Z = h, когда
координата дна ведра станет равна y = H − h/2. Далее, когда
ведро полностью поднимется над водой, динамометр будет
показывать просто mg.
Учёт силы поверхностного натяжения в момент отры-
ва дна ведра от поверхности воды привёл бы к появлению
небольшого максимума вблизи точки y = H − h/2.
382 Решения задач

К решению задачи 300

К решению задачи 300

О т в е т. График зависимости показаний динамометра


от расстояния между дном ведра и дном бочки приведён на
рис. 2.
301. В начальный момент ускорения всех точек верёвки
направлены по касательной к цилиндру и равны по моду-
лю. Обозначим ускорение каждой точки верёвки a. Пусть
с начального момента прошло малое время t, на протяже-
нии которого движение можно считать равноускоренным.
За это время верёвка сместится на l = at2 /2 и наберёт ско-
рость v = at. Определим уменьшение потенциальной энер-
гии верёвки. С точки зрения положения центра масс, сме-
Решения задач городских туров 383

К решению задачи 301

щение верёвки эквивалентно переносу отрезка длиной l из


верхней части верёвки в нижнюю (см. рис.: кусочек из
положения 1 перенесли в положение 2). Масса этого от-
резка верёвки Dm = ml/(pR/2), а уменьшение высоты R.
Таким образом, изменение потенциальной энергии равно
DmgR = m · l/(pR/2) · gR. Эта энергия перешла в кинетиче-
скую энергию верёвки, m · l/(pR/2) · gR = mv2 /2. Подставляя
сюда выражения для l и v, получаем a = 2 · g/p. Теперь рас-
смотрим часть верёвки, заключённую между её нижним
концом и точкой A. Его масса равна DM = ma/(p/2). Ки-
нетическая энергия рассматриваемой части верёвки равна
сумме работ силы тяжести и силы натяжения верёвки T.
Работа силы натяжения отрицательна и равна −T · l, а ра-
бота силы тяжести равна уменьшению потенциальной энер-
гии, которое вычисляется аналогично уже рассмотренному
изменению потенциальной энергии всей верёвки: в данном
случае отрезок верёвки длины l опускается на R · sin a. Та-
ким образом, получаем: Dmg · R · sin a − T · l = DMv2 /2. Под-
ставляя сюда Dm и DM, получим окончательно m · l/(p · R/2) · gR · sin a − T · l = ma
откуда и определяем силу натяжения.
О т в е т. Сила натяжения равна T = (sin a − 2a/p) · 2mg/p
и обращается в ноль на концах верёвки.
302. Пружина имеет нулевую начальную длину, значит,
всё растяжение L является её удлинением. На груз дей-
ствуют сила упругости пружины FУ = k · L и сила тяжести
FТ = Mg. Разложим силу упругости на компоненты вдоль
осей (см. рис. 1). Для любого произвольного положения
384 Решения задач

пружины компоненты силы упругости вдоль соответствую-


щей оси пропорциональны координатам тела в данный мо-
мент времени. Запишем уравнения движения в проекциях
на оси координат и получим два уравнения

MaX = −kx, MaY = Mg − ky,

со следующим начальным условием на координаты и скоро-


сти: y(0) = L · sin a, x(0) = L cos a, VY (0) = VX (0) = 0. Решение
уравнений движения суть гармонические колебания вблизи
положения равновесия (0, Mg/k). Выражение для угловой
частоты колебаний тела на пружинке соответствует стан-
дартной формуле, а амплитуды колебаний определяются из
начальных условий, так что решение уравнений движения
имеет вид
       
k Mg Mg k
x(t) = L cos a · cos t , y(t) = + L sin a− cos t .
m k k M

Эти равенства задают траекторию движения груза в пара-


метрическом виде. Исключая время из решений, получим
явное выражение для траектории y = Mg/k + x(L sin a − Mg/k)/L · cos a.
Полученное уравнение задаёт прямую линию. Рассмотрим
случай, когда L · sin a < Mg/k, в этом случае координата y
всегда будет больше нуля. Чтобы определить длину траек-
тории рассмотрим крайние положения груза (точки A и B
на рис. 2). Длину этой траектории легко найти, рассмат-
ривая треугольник ABC. Убедитесь, что в случае, когда
L · sin a ≥ Mg/k (если, конечно, ничего не препятствует дви-
жению груза) ответ останется таким же.
О т в е т. Траектория является отрезком прямой длиной

Mg M 2 g2
2 L2 − 2L sin a + 2 .
k k

303. Представим движение капсулы как сумму поступа-


тельного движения по вертикали и кругового вращательно-
го движения. Основной идеей этой задачи является то, что
из-за линейной зависимости силы трения от скорости эти
Решения задач городских туров 385

К решению задачи 302

К решению задачи 302


386 Решения задач

составляющие движения будут независимы. Обозначим че-


рез k коэффициент пропорциональности между силой тре-
ния и относительной скоростью соприкасающихся поверх-
ностей. Начнём со случая, когда капсула движется вверх.
Направим ось Y вверх, по направлению движения, и разо-
бьём всё время движения капсулы на маленькие промежут-
ки Dt. Учтём, что импульс капсулы в проекции на ось Y ме-
няется благодаря действующим на капсулу импульсам сил
(тяжести mg и проекции силы трения [FТР ]Y = kVY ), т. е.

(mg + kVY ) · Dt = m · DVY .

Просуммируем такие уравнения для всех промежутков вре-


мени подъёма капсулы: mgt1 − k · H = mV1 , где t1 — время,
которое капсула двигалась вверх, V1 — модуль начальной
линейной скорости капсулы, H — максимальная высота
подъёма. Теперь просуммируем уравнения (1) от момента
максимального подъёма капсулы вплоть до момента, когда
она опустилась на первоначальную высоту: mgt2 + kH = V2 ,
здесь t2 — время, которое капсула опускалась, а V2 —
модуль линейной скорости капсулы при прохождении ис-
ходной высоты. Вычитая полученные равенства, получа-
ем mgT − 2k · H = mV . Отсюда легко выражается H, прав-
да, величина k остаётся пока неизвестной. Чтобы найти k,
выпишем закон изменения суммарного вращательного им-
пульса боковой поверхности (он меняется только благода-
ря горизонтальной составляющей силы трения [FТР ]X = kVX )
и разделим его на R, kVR Dt/R = m · DVR /R. Учитывая, что
VR /R = Da (угол поворота шайбы за рассматриваемый про-
межуток времени), и то что DVR /R — изменение угловой
скорости, несложно просуммировать такие уравнения за всё
время спуска капсулы от момента, когда она имела угло-
вую скорость W . Получим ka = mW , откуда легко выразить
неизвестный коэффициент k.
О т в е т. Максимальная высота, до которой поднялась
капсула относительно начальной высоты, равна H = a(gT − V )/2W .
304. Поскольку все столкновения, обсуждаемые в за-
даче, упругие, то при столкновениях кольца просто обме-
ниваются скоростями. Проще всего представить себе ситу-
ацию на системе из двух колец (см. рис. 1). Если пред-
Решения задач городских туров 387

К решению задачи 304

ставить себе, что кольца не взаимодействуют проходят од-


но сквозь другое графики зависимости их координаты от
времени изобразится отрезками парабол. Графику перво-
го запущенного кольца соответствует линия OABHKDEJ
и т. д., графику второго кольца (запущенного с опозданием)
линия GBCKIEF и т. д. На самом деле, конечно, кольца вза-
имодействуют сталкиваются. Это происходит сначала в точ-
ке B (см. рис. 1). Однако в результате столкновения коль-
ца просто обменяются траекториями: первое кольцо станет
двигаться по линии графика BCKI, а второе кольцо — по
BHKD. Затем кольца снова столкнутся в точке K и снова
обменяются скоростями и траекториями: первое кольцо ста-
нет двигаться по KDEF, а второе — по KIEJ. Таким обра-
зом, рисовать графики координатной зависимости колец от
времени можно без учёта их взаимодействий друг с другом,
важно лишь помнить, что первое кольцо всегда обменивает-
ся скоростями и траекториями с другими так, чтобы остать-
ся сверху, а нижнее — чтобы остаться снизу. Графики для
девяти колец рисуются аналогично.
О т в е т. Графики зависимости координат от времени
для верхнего и нижнего колец показаны жирными линиями
на рис. 2.
305. Важно, что в начальный момент стержни пер-
пендикулярны друг другу; действующая на каждый стер-
жень сила со стороны другого стержня параллельна перво-
388 Решения задач

К решению задачи 304

му стержню, и их рассматривать можно независимо. Сила


натяжения нижнего стержня не растягивает верхний, так
как перпендикулярна ему. Значит, верхний стержень рас-
тянут лишь силой тяжести, действующей на шарнир A, точ-
нее, её проекцией mg · cos a на направление верхнего стерж-
ня. Рассмотрим соединённые стержнем грузы A и B как
единую систему. В проекции на ось AB на эту систему дей-
ствует только проекция силы тяжести 2mg sin a (действи-
тельно, сила натяжения стержня AB является для такой
системы внутренней, а сила натяжения стержня OA пер-
пендикулярна рассматриваемой оси AB). Значит, по второ-
му закону Ньютона, проекции ускорения точек A и B на
ось AB равны друг другу и составляют a = g · sin a. Ускоре-
ние точки B складывается из проекции на ось AB (g · sin a)
и перпендикулярного ему ускорения, связанного с враще-
нием вокруг неподвижного шарнира A. Движение груза B,
связанное с этим вращением, перпендикулярно силе натя-
жения стержня AB, и, значит, обеспечивается только силой
тяжести, а соответствующая составляющая ускорения рав-
на g · cos a.
О т в е т. Сила натяжения верхнего стержня T = mg · cos a,
ускорение груза B в первый момент времени равно a = g.
306. Клин может оторваться от стола, если шайба доста-
точно тяжела. При этом клин станет поворачиваться вокруг
точки A. Чтобы этого не произошло, суммарный момент
сил, действующих на клин, относительно этой точки дол-
Решения задач городских туров 389

К решению задачи 306

жен быть равен нулю. Рассмотрим сначала случай m < tg a,


когда шайба скользит по клину. Пусть клин неподвижен. В
этом случае сила, с которой шайба действует на клин, скла-
дывается из силы трения F1 = mmg cos a и силы нормально-
го давления F2 = mg cos a. Если обозначить длину основа-
ния клина l, то плечи этих сил равны l sin a и l tg a · sin a
соответственно. Плечо силы тяжести клина равно l/3. Кро-
ме того, есть ещё момент силы реакции опоры (обозначим
его K), которая приложена в какой-то точке основания кли-
на:

mmg cos a · l sin a + Mg · l/3 = l · mg cos a · tg a sin a + K.

Это равенство возможно только в том случае, когда

mmg cos a · l sin a + Mg · l/3 > mg cos a · l · tg a · sin a + K.

или M/3 > m sin a(sin a − m cos a). Соответственно, клин ото-
рвётся от стола, когда

M
m> .
3 sin a(sin a − m cos a)

Теперь рассмотрим случай m > tg a, когда шайба не про-


скальзывает. В этом случае шайба давит на клин вертикаль-
но, и эта сила не имеет момента относительно точки A. По-
этому в такой ситуации клин от стола не оторвётся.
M
О т в е т. Отрыв возможен, если m < tg a и m > .
3 sin a(sin a − m cos a)
307. Рассмотрим систему в тот момент, когда первое
кольцо находится в верхней точке. Кольца, которые к этому
390 Решения задач

моменту находятся на стержне, имеют скорости, направлен-


ные вверх, и находятся друг от друга на различных расстоя-
ниях. Можно считать, что те кольца, которые ещё будут вы-
пущены, также находятся на продолжении стержня и сво-
бодно движутся по нему, имея такие скорости и находясь на
таком расстоянии от точки запуска, чтоб вылететь в нуж-
ный момент, имея скорость v0 . Рассмотрим случай, когда
на стержне находятся N + 1 кольцо, а число колец, которые
будут дополнительно выпущены вдоль стержня (и в данный
момент находятся под точкой бросания) — M. t — интервал
между последовательными бросками колец, по условию за-
дачи N + 1 = [v0 /(gt)]  1. Заметим, что центр масс системы
N + M + 1 кольца имеет постоянное ускорение g. Скорость и
высота центра масс относительно точки бросания суть
0 + gt + 2gt + . . . + (N + M)gt (N + M)gt
vc = ≈ ,
N+M+1 2

gt 2 N 2 gt 2
hc ≈ − (0 + 1 + 4 + . . . + (N + M)2 ) ≈
2 2(N + M + 1)
gt 2 N 2 gt 2 (N + M)2
≈ −
2 2 3

с учётом сделанных приближений (N, M  1). Движение


центра масс N + M + 1 кольца, очевидно, есть просто свобод-
ное падение тела, брошенного вертикально вверх с началь-
ной скоростью vc с высоты hc . Центр масс должен оказаться
в точке бросания в тот момент, когда в той же точке окажет-
ся и N + M + 1-е кольцо, т. е. через время Mt (здесь снова
используется N, M  1). Запишем:
gt 2 N 2 gt 2 (N + M)2 (N + M)gt g(Mt)2
− + (Mt) − = 0.
2 2 3 2 2

Решая уравнение относительно M, получаем 2 значения


M1 = −N и M2 = 2N. Очевидно, нас устраивает второе реше-
ние. Следовательно, время «схлопывания», отсчитываемое
от момента, в котором положение первого кольца наивыс-
шее, равно tСХЛ = Mt = 2Nt = 2v0 /g. Искомое время склады-
вается из времени схлопывания и времени подъёма первого
кольца до максимальной высоты: tИСК = tСХЛ + v0 /g = 3v0 /g.
Решения задач городских туров 391

Примечательно, что в сделанных приближениях (t  v0 /g —


«непрерывный поток») от t ответ не зависит.
О т в е т. Стопка колец упадёт на землю через время
3v0 /g.
308. В нижней части траектории у бусинки будет
только центростремительное ускорение равное a = u2 /R, где
u2 = 2gh — квадрат скорости в нижней точке. Для нахожде-
ния радиуса кривизны R в нижней точке лески нужно рас-
смотреть небольшой кусочек лески длиной Dl вблизи ниж-
ней точки (см. рис. 1). Он представляет собой дугу окружно-
сти радиусом R, угол этой дуги равен Df = Dl/R. На этот ку-
сок действует три силы, две силы натяжения T0 со стороны
других частей лески и сила тяжести gDm = mgDl/L, где m —
масса лески. Из условия равенства сил в вертикальном на-
правлении следует mgDl/L = 2T0 sin(1/2Dl/R). Для малых
углов sin(x) ∼= x, если x выражен в радианах. Учитывая,
что мы выбираем малый D  R, получим 1/R = mg/(T0 L).
Чтобы найти радиус кривизны, надо, таким образом, вы-
числить силу натяжения T0 . Для её нахождения мож-
но рассмотреть правую половину лески (см. рис. 2). На
неё действует сила натяжения T0 со стороны левой по-
ловины, которая направлена горизонтально, сила тяжести
mg/2 и сила со стороны подвеса T1 , направленная под уг-
лом a к горизонту. Условия равновесия запишутся в виде:
mg/2 = T1 sin a и T0 = T1 cos a. Откуда несложно получить,
что T0 = mg/2 · ctg a. Подставляя этот результат в выраже-
ние для центростремительного ускорения, получим ответ.
О т в е т. Ускорение бусинки в нижней точке верёвки
равно a = 4g tg a · h/L.
309. Предположим, что мотоцикл не проскальзывает.
На мотоциклиста действуют три силы: сила реакции опо-
ры N, направленная к оси цилиндра, сила тяжести mg и
сила трения F. Учитывая, что N направлена горизонталь-
но, получим F = mg и при этом F < mN. Сила N придаёт мо-
тоциклисту центростремительное ускорение a = v2 cos2 a/R,
где a — угол наклона траектории мотоциклиста к горизон-
ту. Следовательно, N = mv2 cos2 a/R и мы получаем условие
m > gR/(v2 cos2 a). Учитывая cos2 a = 4p2 R2 /(4p2 R2 + h2 ), по-
лучим ответ m > g/v2 · (R + h2 /(4p2 R)). Следует отметить, что
392 Решения задач

К решению задачи 308

К решению задачи 308


Решения задач городских туров 393

К решению задачи 309

данное условие является достаточным для случая, когда мо-


тоцикл едет без проскальзывания. В других режимах дви-
жения, может потребоваться большее значение m.
О т в е т. Коэффициент трения покоя между покрышка-
ми мотоцикла и поверхностью цилиндра должен превосхо-
дить величину g/v2 · (R + h2 /(4p2 R)).
310. Сделаем из нихрома спираль и поместим её в чай-
ник. Подключим спираль к источнику напряжения U. Про-
текающий в цепи ток I будем измерять амперметром. Счи-
тая спираль чисто активной нагрузкой, мы можем найти
мощность кипятильника, P = UI, и, измеряя время нагрева-
ния воды Dt1 , определить количество теплоты, сообщённое
системе Q = PDt1 . Эта теплота расходуется на нагревание во-
ды с температурой T  на DT градусов и на тепловые потери
QP1 (теплотой на нагревание чайника можно пренебречь,
так как чайник по условию задачи лёгкий). При постоян-
ных температурах нагретого тела T  и окружающей среды
T1 потери тепла Q1P будут пропорциональны времени, в те-
чение которого происходит теплообмен, и разности темпе-
ратур тела и окружающей среды, неизвестный коэффици-
ент пропорциональности, характеризующий интенсивность
теплообмена мы обозначим k: Q1P = k · (T  − T0 ) · Dt1 . Итак,
баланс тепла в системе имеет вид Q = cm · DT + QP1 , здесь
c — теплоёмкость воды, m — её масса. В реальном экспе-
рименте температуру окружающей среды мы можем с хоро-
шей точностью считать постоянной, но у нас нет возмож-
394 Решения задач

ности поддерживать постоянной температуру воды, темпе-


ратура воды будет расти, так что выражение для тепловых
потерь становится неверным (в точке кипения воды, конеч-
но, температура стабилизируется, но тут прибавятся допол-
нительные потери тепла на парообразование и это только
усложнит задачу). Этого можно было бы избежать, рассмат-
ривая нагревание достаточно горячей воды на очень малую
температуру, чтобы изменением температуры воды по от-
ношению к комнатной температуре можно бы было прене-
бречь. Но, вдобавок, нам неизвестна сама величина k. По-
этому, для определения тепловых потерь предлагается сле-
дующая схема эксперимента. Заметим время Dt1 , необхо-
димое на нагревание воды на некоторую температуру DT.
Затем выключим напряжение и заметим время Dt1 , необхо-
димое на охлаждение воды до первоначальной температу-
ры. Если мы будем замечать время, необходимое на нагре-
вание и охлаждение на несколько градусов при достаточно
высокой температуре T , то при оценке тепловых потерь,
можно считать, что температура нагреваемого тела прак-
тически не меняется по сравнению с температурой комна-
ты: T − T0 ∼= T  + DT − T0 . Иными словами, T можно счи-
тать в процессе теплообмена практически постоянной. Теп-
ловой баланс при охлаждении воды на DT запишется как
cm · DT = QP2 = k · (T  − T0 ) · Dt2 . Теперь можно найти потери
тепла при нагревании воды. Выразим k из предыдущей
формулы и подставим его в выражение для тепловых потерь
QP1 , в результате получим формулу QP1 = cm · DT · Dt1 /Dt2 .
Здесь потери тепла выражены через массу воды и вели-
чины, измеряемые экспериментально. Подставляя получен-
ное выражение для потерь в формулу, получим формулу
для определения массы воды.
О т в е т. Масса воды в чайнике может быть оценена сле-
дующим образом: надо измерить ток, текущий через нихро-
мовую спираль, время Dt1 , необходимое для нагревания во-
ды на некоторое измеренное число градусов DT при доста-
точно высокой температуре воды T  , и время Dt2 остывания
воды на DT при выключенном напряжении на спирали. Эти
измерения дают возможность рассчитать массу воды с до-
статочно высокой точностью m = IUDt2 /c(Dt2 + Dt1 ).
Решения задач городских туров 395

311. Так как давление насыщенных паров зависит толь-


ко от температуры, а по условию процесс изобарический
(давление не меняется и равно атмосферному P), то и
температура насыщенного пара меняться не может. Зна-
чит, сначала происходит испарение жидкости при посто-
янной температуре T0 . В ходе этого процесса объём па-
ра в данный момент V = v RT0 /P пропорционален количе-
ству вещества пара v в этот момент. Когда вся жидкость
испарится, количество вещества пара увеличится вдвое,
значит, и объём пара увеличится по сравнению с пер-
воначальным объёмом V0 вдвое. Изменение объёма па-
ра DV1 в процессе испарения жидкости легко вычислить:
DV1 = V0 = vRT0 /P. Работу газа в процессе испарения жид-
кости при постоянном давлении вычислим по формуле
DA1 = P0 DV = v · RT0 . Далее происходит нагрев пара на DT
градусов с дальнейшим увеличением объёма (при этом в
системе уже не осталось жидкости, и пар при таком на-
гревании уже не будет насыщенным). Запишем два урав-
нения состояния пара в начале и в конце процесса нагре-
вания: P · 2V0 = 2v · RT0 и P(2V0 + DV2 ) = 2v · R(T0 + DT). Рабо-
та в этом процессе равна DA2 = PDV2 , её легко вычислить,
вычитая из второго уравнения первое, DA2 = 2v · R · DT. Те-
перь легко найти изменение внутренней энергии содержи-
мого сосуда DU = Q − DA1 − DA2 .
О т в е т. Изменение внутренней энергии содержимого
сосуда равно DU = Q − v · R(T0 + 2DT).
312. Обозначим концентрации изотопов в левом от-
секе nЛ и nТ , а искомую относительную концентрацию
C0 = nЛ /(nЛ + nТ ). Так как давление в левом отсеке связано
с суммарной концентрацией основным уравнением молеку-
лярно-кинетической теории P1 = (nЛ + nТ )kT, концентрации
nЛ и nТ могут быть выражены через искомую величину
C0 : nЛ = C0 P1 /kT, nТ = (1 − C0 )P1 /kT. Аналогично, концен-
трации в правом отсеке выражаются через известную C1 :
nЛ = C1 P2 /kT, nТ = (1 − C1 )P2 /kT. Определим, сколько моле-
кул каждого изотопа пролетают через отверстия в перего-
родке за единицу времени. Для определённости, рассмот-
рим сначала лёгкий изотоп, пролетающий из левого отсе-
ка в правый. Так как температура — мера средней кине-
тической энергии молекул (kT ∼ mЛ V 2 /2), то среднеквадра-
396 Решения задач

тичная скорость молекулы будет обратно пропорциональ-


на корню её массы V ∼ (kT/mМ )1/2 . Вообще, в газе можно
вводить разнообразные средние скорости среднюю от моду-
ля скорости, среднюю проекцию вдоль какого-нибудь на-
правления. Понятно по размерности, что все эти скорости
должны быть пропорциональны друг другу, так что средняя
в любом смысле скорость будет обратно пропорциональна
корню из массы молекулы. Рассмотрим молекулы, которые
успеют через маленький промежуток времени Dt пролететь
через некоторое отверстие в перегородке (на рис. белые мо-
лекулы — те, что попадают за Dt в отверстие, чёрные —
те, что не попадают). Понятно, что если скорости всех мо-
лекул уменьшить в n раз (средняя скорость молекул при
этом также уменьшится в n раз), то через время nDt те же
молекулы (белые) пролетят сквозь отверстие. Значит, чис-
ло молекул, пролетающих через отверстие в единицу време-
ни уменьшится в n раз. Итак, число пролетающих молекул
пропорционально средней скорости молекул, и, значит, об-
ратно пропорциональна корню из их массы. С другой сто-
роны, если увеличить в k раз концентрацию молекул перед
отверстием, понятно, что и число пролетающих сквозь от-
верстие за время Dt молекул увеличится во столько же раз.
Итак, число молекул лёгкого изотопа, пролетающего из ле-
1/2
вого отсека в правый в единицу времени N1 ∼ nЛ /mЛ . Су-
ществует и обратный поток молекул — из правого отсека
в левый, причём, так как газ предлагается считать идеаль-
ным, потоки молекул не влияют друг на друга (это верно,
если длина свободного пробега молекул больше характери-
стических размеров отверстий). Соответствующее число ча-
стиц в единицу времени N1 ∼ nЛ /mЛ . Разница между N1
1/2

и N2 даёт поток частиц лёгкого изотопа из левого отсека в


правый PЛ = A(nЛ − nЛ )/mЛ = A(C0 P1 − C1 P2 )/(kTmЛ ), че-
1/2 1/2

рез A мы обозначили коэффициент пропорциональности,


не зависящий от массы молекул и их концентрации (и,
на самом деле, от температуры, хотя это и не использует-
ся в решении). Аналогично, для потока тяжёлых изотопов
PТ = A(nТ − nТ )/mТ = A((1 − C0 )P1 − (1 − C1 )P2 )/(kTmТ ) с тем
1/2 1/2

же коэффициентом пропорциональности. В стационарном


Решения задач городских туров 397

К решению задачи 312

случае (когда все концентрации перестали меняться) отно-


шение концентраций во втором отсеке пропорционально от-
ношению потоков, втекающих в этот отсек: PЛ /PТ = C1 /(1 − C1 ).
Подставляя значения потоков, получим уравнение, связы-
вающее относительные концентрации лёгкого изотопа до
перегородки и после неё.
О т в е т. Концентрация лёгкого изотопа в первом объёме
будет равна
1 − k + kC1 + kr − C1 kr)C1
C0 = ,
C1 + r − C1 r

где k = P2 /P1 , r = (mЛ /mТ )1/2 .


313. Пусть газ, совершая цикл ABCGA, получает от на-
гревателя тепло Q1 , отдаёт холодильнику тепло Q2 и со-
вершает работу A. Тогда по закону сохранения энергии
A = Q1 − Q2 , а КПД цикла ABCGA есть h = A/Q1 . Рассмот-
рим цикл CDEFC. Заметим, что он получается из цик-
ла ABCGA увеличением объёма в каждой точке в 2 ра-
за. Внутренняя энергия (U = const · vRT = const · pV ), совер-
шаемая газом работа (DA = pDV ), а, значит, и получаемое
или отдаваемое газом тепло (DQ = DU + DA) пропорциональ-
ны объёму или изменению объёма. Следовательно, в цикле
CDEFC газ получит от нагревателя тепло 2Q1 , отдаст холо-
дильнику тепло 2Q2 и совершит работу 2A. При этом важ-
но, что переход от получения тепла к отдаче и наоборот про-
исходит в соответственных точках циклов (т. е. точках с рав-
ными давлениями и отличающимися вдвое объёмами). Про-
хождение газом цикла AGCDEFCBA эквивалентно сначала
398 Решения задач

прохождению цикла CDEFC, а затем цикла ABCGA в об-


ратном направлении. Проходя цикл ABCGA в обратном на-
правлении, газ получает тепло Q2 , отдаёт тепло Q1 и совер-
шает работу A. Таким образом, в цикле AGCDEFCBA газ
получает от нагревателя тепло 2Q1 + Q2 и совершает рабо-
ту 2 · A − A. КПД цикла AGCDEFCBA равен A/(2 · Q1 + Q2 ).
Подставляя A = h · Q1 и Q2 = Q1 − A = (1 − h) · Q1 можно выра-
зить КПД в единицах h.
О т в е т. КПД цикла AGCDEFCBA равен h/(3 − h).
314. Система теплоизолирована, так что сумма потенци-
альной энергии оболочки и внутренней энергии газа посто-
янна. Решение предлагаемой задачи, вообще говоря, требу-
ет интегрирования функции давления p(V ) по объёму для
вычисления потенциальной энергии, запасённой в упру-
гой оболочке. Как обычно, в этом случае предполагает-
ся, что участники олимпиады проведут «численное инте-
грирование», т. е. определят площадь под графиком. Пер-
вый шаг решения состоит в нахождении начального объё-
ма шара по графику, исходя из условия p(V0 ) = p0 , полу-
чаем V0 = 1,1 л. Выпишем выражение для начальной энер-
гии системы: U = E(V0 ) + 3/2 · p0 V0 . Второе слагаемое в пра-
вой части этого выражения внутренняя энергия газа. Пер-
вое слагаемое запасённая в оболочке энергия, численно рав-
ная площади под графиком на интервале от 0 до V0 по
оси абсцисс. Действительно, энергия оболочки объёма V0 —
это, численно, работа по надуванию оболочки такого объё-
ма. Для раздувания оболочки на малую величину DV , ес-
ли оболочка уже надута до давления P, требуется совер-
шить работу PDV , т. е. площади узкого прямоугольника на
графике P(V ). Рассматривая процесс надувания оболочки
как большое количество раздуваний на маленькую величи-
ну DV , каждое из которых протекает при практически по-
стоянном давлении, получаем выражение для совершённой
работы как сумму площадей узких прямоугольников разной
высоты (с шириной DV ), которое при малых DV стремится к
площади под графиком. В конечном состоянии полная энер-
гия системы имеет вид U  = 2E(V /2) + 3/2 · V · p(V /2). Пред-
положим, что объём каждого шарика стал бы V0 /2 (полный
объём газа не изменился). При этом давление p(V0 /2) (опре-
деляем по графику) уменьшилось бы по сравнению с перво-
Решения задач городских туров 399

начальным, и вместе с ним уменьшилось бы второе слага-


емое в U  по сравнению с аналогичным слагаемым в U. С
другой стороны, видно, что удвоенная площадь под графи-
ком P(V ) на интервале от 0 до V /2 меньше площади под
тем же графиком на интервале от 0 до V (здесь существен-
ной оказывается только то, что P(V ) — неубывающий гра-
фик). Так что первое слагаемое в U  также меньше анало-
гичного слагаемого в U. Итак, видно, что если бы объём
шариков стал бы V0 /2, то энергия системы стала бы рав-
на U0 и была бы меньше исходной энергии U. На самом
деле энергия системы не меняется. Заметим, что энергия
системы в конечном состоянии U  (V ) — монотонная функ-
ция объёма, значит, в реальности U  > U0 , и, следователь-
но, V /2 > V0 /2. Итак, объём всего газа увеличится, а дав-
ление — уменьшится. Для получения ответа на второй во-
прос задачи следует найти точку (P, V ) на графике, для
которой 2E(V /2) + 3/2V · p(V /2) = E(V0 ) + 3/2V0 · p(V ). Здесь
E(V /2) представляет собой площадь под графиком на ин-
тервале от 0 до V /2.
О т в е т. После установки равновесия давление умень-
шится, а объём будет равен примерно 1,39 л.
315. Шар будет подниматься, если выполняется нера-
венство V (rв − rг ) > m, где V — объём оболочки, rв и
rг — плотность соответственно воздуха и газа, находяще-
гося в оболочке на той высоте, где находится шар. Если
V < m/(rв − rг ), то шар не полетит совсем. Воспользовавшись
уравнением состояния идеального газа P = (r/m)RT, пере-
пишем левую часть неравенства: V (rв − rг ) = V (Pв mв /RTв − Pг mг /RTг ).
Если давление и температура газа внутри оболочки равны
внешнему давлению и температуре (последнее будет выпол-
няться в условиях задачи, поскольку исследуется статиче-
ская задача ищется высота максимального подъёма), то ра-
венство можно продолжить, V (rв − rг ) = VPг (Pв mв − mг )/RTг .
Таким образом, подъёмная сила, действующая на поднима-
ющийся шар, постоянна до тех пор, пока оболочка не до-
стигнет своего предельного объёма. Высота, на которой это
происходит, называется высотой выполнения. Дальнейший
подъём происходит до тех пор, пока сила Архимеда не срав-
няется с силой тяжести: V0 rв (h) = m + V1 r1 (V1 r1 — масса га-
400 Решения задач

за, заключённого в оболочку). Это даёт для высоты подъёма


1 V0 r0
hm = ln .
a m + V 1 r1

Ясно, что чем меньшее значение стоит в знаменателе дроби,


тем больше будет высота hm . Этого можно добиться умень-
шением V1 . При этом для того, чтоб шар оторвался от зем-
ли, необходимо V > m/(rв − rг ). Предельное значение и даёт
искомый минимальный объём. Высота подъёма при этом
1 V0 r0
hm = ln .
a m + V 1 r1

О т в е т. Наибольшая высота подъёма воздушного шара


hm достигается, когда его наполнение минимально необхо-
димое, чтобы оторваться от земли. Шар при этом поднима-
ется с очень маленькой скоростью. Если баллон более на-
полнен, шар поднимается быстрее, но до меньшей высоты.
316. Из условия следует, что во всех точках ни-
же графика VK (T) пар находится в равновесии с жид-
костью, т. е. он насыщенный. Давление в этих точках
равно давлению насыщенного пара при соответствующей
температуре. В точках A и C, находящихся на графике
зависимости VK (T) пар становится насыщенным, но его
количество всё ещё равно v. Это позволяет найти дав-
ление насыщенного пара p A = vRT A /V A = 1,62 · 104 Па и
pC = vRTC /VC = 1,28 · 104 Па. Так как температуры в точках
C и D равны, а давление насыщенного пара зависит только
от температуры pD = pC . И наконец давление в точке B рав-
но pB = p A · (V A /VB ) = 1,33 · 104 Па. Опишем цикл ABCD: 1)
AB: изотермический процесс расширения; 2) BC: изохори-
ческий процесс охлаждения; 3) CD: изобарическое сжатие,
масса пара уменьшается; 4) DA: изохорический процесс на-
грева, масса пара увеличивается.
О т в е т. График процесса в координат P − V приведён
на рисунке. Давления насыщенного пара в угловых точ-
ках равны: p A = 1,62 · 104 Па, pC = 1,28 · 104 Па, pD = pC ,
pB = p A · (V A /VB ) = 1,33 · 104 Па.
317. Составим из восьми таких кубов куб вдвое больших
размеров. Центр «большого» куба является вершиной каж-
Решения задач городских туров 401

К решению задачи 316

дого из «маленьких», поэтому потенциал в этой точке ра-


вен 8f (f — искомый потенциал). С другой стороны, восемь
составленных вместе кубов можно рассматривать как один
куб вдвое больших размеров (при этом заряд каждой грани
равен 4q) и три квадратные пластины, сторона каждой из
которых вдвое больше, чем грани исходного куба (при этом
заряд каждой пластины равен 8q). Если взять какой-нибудь
заряженный объект и увеличить все его линейные размеры
в a раз, а заряд — в b раз, то потенциал, создаваемый в
соответственных точках, изменится
 в b/a раз, так как по-
тенциал даётся формулой k · qi /ri (сумма слагаемых вида
qi /ri для всех маленьких кусочков с зарядами qi располо-
женных на расстояниях ri ). Действительно, если увеличить
все линейные размеры в a раз, а заряд — в b раз, то каж-
дое слагаемое суммы увеличится в b/a раз. Поэтому потен-
циал, создаваемый «большим» кубом в его центре, равен
4f2 /2 = 2f2 , а «большой» пластиной в её центре 8f1 /2 = 4f1 .
По принципу суперпозиции 8f = 2f2 + 3 · 4f1 .
О т в е т. Потенциал в вершине куба равен f = 3f1 /2 + f2 /4.
318. Потенциал в центре большой пластины равен f.
Разобьём пластину на четыре одинаковых квадрата. По-
тенциал в центре пластины есть сумма четырёх потенциа-
лов, создаваемых зарядами на маленькой пластине, в уг-
лах каждой из маленьких пластин. Значит, потенциал, со-
здаваемый в углу маленькой пластины зарядами, распо-
ложенными на ней, равен f/4. Заряд на маленькой пла-
стине в четыре раза меньше, чем на большой qM = q/4.
402 Решения задач

К решению задачи 318

Разобьём большой и маленький квадраты на n квадрати-


ков (см. рис.). Сделаем число квадратиков n настолько боль-
шим, чтобы их можно было бы рассматривать как точечные
заряды. Пусть заряд каждого квадратика на маленькой пла-
стине qM . Тогда заряд каждого квадратика большой пласти-
ны есть 4 · qM , так как его площадь в четыре раза больше,
а заряд по пластине распределён равномерно. Потенциал
в углу пластин есть сумма потенциалов, создаваемых то-
чечными зарядами маленьких квадратиков. Расстояние от
центра любого квадратика до угла на большой пластине в
два раза больше, чем аналогичное расстояние на малень-
кой пластине RM = R/2. Отсюда можем записать, что потен-
циал в углу маленькой пластины состоит тз суммы боль-
шого количества слагаемых qM /Li , с разными расстояни-
ями Li : f/4 = qM /Li . Для большой пластины сумма вы-
глядит
 аналогично, но каждое слагаемое в 2 раза больше:
fБ = 4qM /(2Li ). Из двух последних формул легко найти
потенциал в углу большой пластины fБ .
О т в е т. Потенциал в углу данной пластины равен f/2.
319. Рассмотрим точку B на линии AA. Поместим нача-
ло второй такой же заряженной нити в точку C, так чтобы
отрезки OB и BC были равны: |OB| = |BC| = x (обозначим
длину этих отрезков через x). Из соображений симметрии
ясно, что эти нити будут создавать в точке B одинаковую
по величине напряжённость поля E1 = E2 , но из этого ещё
не следует, что напряжённость поля в точке B будет равна
нулю, поскольку мы не знаем, как направлены эти векто-
ры. Выясним, чему будет равна суммарная напряжённость
поля в точке B. Выберем маленький кусочек DL на первой
Решения задач городских туров 403

нити (обозначим его KM) и кусочек DL1 на второй нити,


так, чтобы кусочки были видны под одним и тем же ма-
лым углом da из точки B (см. рис. 1). Эти кусочки должны
быть малы настолько, чтобы их можно было бы рассмат-
ривать как точечные заряды. Напряжённость поля, созда-
ваемого точечным зарядом Dq = r · DL равна E1 = kr · DL/R2 .
Расстояние R от точки B до кусочка KM можно выразить
через длину отрезка OB: R ∼ = BM = x/ cos a. Длина кусоч-
ка DL может быть определена из прямоугольного треуголь-
ника KMF: DL = b/ cos(a + da) ∼ = b/ cos a, где определяется
из треугольника FMB: b = R · sin da ∼ = R · da. Отсюда для на-
пряжённости поля, создаваемого кусочком нити DL, имеем
E1 = r · da/x. Ответ не зависит ни от чего, кроме расстояния
от точки B до заряженной нити и угла da, следовательно, и
для напряжённости поля, создаваемого кусочком DL1 спра-
ведлив тот же ответ: E2 = r · da/x. Таким образом, рассмат-
риваемые два кусочка создают равную по величине и про-
тивоположно направленную напряжённость поля. Очевид-
но, что для каждого малого кусочка на одной нити суще-
ствует симметричный кусочек из другой нити. (То, что при
движении вдоль нити кусочки DL становятся на одной ни-
ти короче, а на другой длиннее не должно нас смущать, так
как одновременно изменяется расстояние до точки в кото-
рой мы вычисляем напряжённость. Кроме того, мы всегда
можем выбрать угол da достаточно малым.) Следовательно,
суммарная напряжённость поля, создаваемого двумя нитя-
ми в точке B, будет равна нулю. Учитывая то, что величины
напряжённости поля, создаваемого каждой нитью, равны,
а их суммарная напряжённость поля равна нулю, мы мо-
жем сделать вывод о том, что векторы напряжённости поля
направлены в противоположные стороны. Из соображений
симметрии векторы напряжённости поля должны быть на-
правлены под одинаковым углом к соответствующим лини-
ям (AA и BC). Единственным возможным вариантом будет
направление суммарного вектора напряжённости под углом
45 ◦ C (напряжённость от нити, исходящей из точки O —
биссектриса угла ABC, а от нити, исходящей из точки C —
биссектриса угла OBH).
О т в е т. Вектор напряжённости поля в точке B направ-
лен под углом 45 ◦ C к линии AA (см. рис. 2).
404 Решения задач

К решению задачи 319

К решению задачи 319


Решения задач городских туров 405

К решению задачи 320

320. В исходном состоянии полная энергия двух ша-


риков определяется формулой W0 = 2mg · h + kq2 /S, где k —
электрическая постоянная. В момент, когда шарики будут
слетать со спиц, полная энергия их будет равна WK = 2mg · H + kq2 /b + m · v2 .
Как следует из рисунка, h = (S · ctg a)/2, b = 2L · sin a и
H = L · cos a. Используя закон сохранения энергии WK = W0 ,
получим искомое значение скорости.
О т в е т. Скорости шариков в момент их слета со спицы
будут равны:

(2L sin a − S)(kq2 − 2mgSL cos a)
|V | = .
2mSL sin a

Бусинки вообще слетают со спиц, если kq2 > 2mgSL cos a


(только в этом случае выражение под квадратным корнем
в ответе больше нуля).
321. В этой задаче сопротивление жидкости, а, значит,
и выделяемая тепловая мощность изменяются с течением
времени. Чтобы найти выделившееся в системе тепло необ-
ходимо разбить процесс на много маленьких промежутков
Dt, в течение каждого из которых мощность практически
не меняется, так что выделившаяся теплота равна произ-
ведению этой мощности и Dt. Затем требуется сложить все
такие вклады. Видно, что каждое слагаемое при этом чис-
ленно равно площади узкого (шириной Dt) прямоугольника
под графиком мощности. Так что выделившаяся за всё вре-
мя теплота численно равна площади под графиком. Постро-
им этот график. Объём первой жидкости линейно убывает
с течением времени, V (t) = V0 − kvt. Сопротивление жидко-
406 Решения задач

К решению задачи 321

сти объёма V с удельным сопротивлением r, налитой в ци-


линдр площадью S равно rL/S = rV /S2 , здесь L — высота
уровня жидкости. Зависимость полного сопротивления це-
пи, подключённой к источнику тока, от времени есть, таким
образом, R(t) = (r1 V (t) + r2 V0 )/S2 , а выделяемая мощность
N(t) = I2 R(t). Построим график зависимости мощности от
времени при t < t1 = V0 /kv (t1 — время, за которое вытечет
первая жидкость). Далее сопротивление цепи не меняется,
значит, тепловая мощность постоянна (см. рис.). Количе-
ство выделившейся теплоты равно площади под графиком,
которую легко вычислить с помощью простых геометриче-
ских соображений.
О т в е т. В процессе выделилась теплота Q = I2 V02 (2r2 + r1 /2)/kvS2 .
322. Вырежем из параллелепипеда коробку (состоящую
из 5 граней) с толщиной стенок Dx и имеющую такие же
рёбра, как и исходный параллелепипед. Рассмотрим так-
же вторую коробку с такой же толщиной стенок и заряжен-
ную с той же плотностью заряда, но все рёбра которой в
k раз меньше рёбер первой. Напряжённость поля в центре
верхней (пустой) грани для каждой коробки будет одина-
ковым. Действительно, разобьём грани первой коробки на
много маленьких площадок, так что каждую из них мож-
но рассматривать как элементарный заряд. Сделаем анало-
гичное разбиение на второй коробке, так, чтобы у каждого
элемента первой коробки был её масштабированный аналог
на второй. Заряд каждого элемента первой коробки в k2 раз
Решения задач городских туров 407

больше, чем у второй коробки, зато расстояние от этого эле-


мента до центра пустой грани в k раз больше. Так как поле
каждого из элемента пропорционально его заряду и обратно
пропорционально квадрату расстояния, видно, что соответ-
ствующие элементы коробок создают одинаковые вклады на
которые можно разбить интересующее нас поле. Таким об-
разом, напряжённость поля, создаваемого коробкой зависит
только r и Dx. Из этих величин можно единственным спо-
собом составить величину, имеющую размерность напря-
жённости: rDx. Значит поле коробки EK = KrDx, где K —
безразмерный коэффициент пропорциональности. Разобьём
наш исходный параллелепипед на множество коробок со
стенками одинаковой толщины и пропорционально убыва-
ющей длиной рёбер. Напряжённость поля в интересующей
нас точке F для параллелепипеда с распределённым по объ-
ёму зарядом Q1 будет равно сумме полей всех коробок, т. е.
EП = Kra = KQ1 · a/V = K · Q1 /4a. Поверхностный заряд мож-
но рассматривать как заряд, размещённый в очень тонком
слое d на поверхности. Напряжённость поля, создаваемая
этим зарядом, суть E = Kr2 · d = KQ2 · d/V2 , где V2 — объ-
ём, в котором размещён поверхностный заряд. Подставляя
V2 = 12 · a3 d в выражение для E, получим E = K · Q2 /12a2 .
Поскольку поле в точке F равно нулю, то справедливо ра-
венство EП = −E, откуда непосредственно и следует ответ.
О т в е т. Отношение зарядов равно Q2 /Q1 = −3.
323. Рассмотрим тепло, выделяющееся в проволоке за
малый промежуток времени, и учтём, что всё оно идёт
на нагревание проволоки, поскольку, по условию зада-
чи, отвода тепла нет. Количество тепла, которое выде-
ляется в проволоке за время Dt запишем, используя за-
кон Джоуля–Ленца, DQ = U 2 (t) · Dt/(R0 + aR0 T). Посколь-
ку температура должна изменяться линейно, то T = b · t с
некоторым произвольным коэффициентом b. Следователь-
но, DQ = U 2 (t) · Dt/(R0 + ab · R0 t). Это тепло идёт на нагре-
вание проволоки на DT = DQ/C. Подставляя полученное
DQ = Cb · Dt, можно выразить напряжение.
О т в е т. Подводимое к проволоке √ напряжение должно
изменяться со временем как U(t) = bCR0 (1 + abt), b — по-
стоянная скорость роста температуры. На рисунке мы со-
поставили линейный рост температуры во временем (обез-
408 Решения задач

К решению задачи 323

размеренным) и корневой рост напряжения (обезразмерен-


ного), необходимый для поддержания линейного роста тем-
пературы.
324. Введём координату t для всех точек на резинке,
равную отношению длины резинки от этой точки до одно-
го из концов ко всей длине резинки, когда она находится в
нерастянутом состоянии. Таким образом, t равномерно «ну-
мерует» все точки на резинке и изменяется от 0 до 1 от одно-
го конца к другому. Рассмотрим теперь деформированную
резинку, помещённую в некоторое электрическое поле E.
Пусть точка резинки с координатой t имеет радиус-вектор
r, а координате (t + Dt) соответствует радиус-вектор (r + Dr),
причём Dt и Dr малы — рассматриваем малый участок ре-
зинки. Жёсткость этого участка равна K/Dt (жёсткость об-
ратно пропорциональна длине в нерастянутом состоянии).
Поэтому возникающая сила натяжения равна
T = K · Dr/Dt.
Здесь мы пренебрегли начальной длиной по сравнению с
длиной в нерастянутом состоянии, посчитав, что растяже-
ние равно Dr. Вектор T направлен по касательной к ре-
зинке, в сторону увеличения t. Теперь перейдём к рас-
смотрению условия равновесия каждого маленького участ-
ка резинки в поле. Пусть в точке с координатой t сила
натяжения равна T. Её величину в точке (t + Dt) обозна-
чим T + DT. Заряд такого участка, очевидно, равен Q · Dt.
Решения задач городских туров 409

Пусть электрическое поле в окрестности этого участка ре-


зинки равно E, тогда уравнение равновесия будет иметь вид
DT + Q · DtE = 0. Откуда следует, что
DT/Dt = −QE.
Уравнения движения для T и её производной DT/Dt ана-
логичны уравнениям движения частицы с зарядом (−Q)
в поле E. Известно, что при движении частицы в элек-
трическом поле сохраняется энергия, p2 /2m − Qf(r) = const.
Аналогичный закон сохранения для T несложно выводит-
ся из уравнений движения, T2 /2K − Qf(r) = const. Для лю-
бых двух точек на резинке он позволяет выписать равенство
F12 /2K − Qf1 = F22 /2K − Qf2 . В частности, положим f1 = 0 —
потенциал на левом конце резинки, а f2 = kq/L − kq/3L —
на правом. Тогда из последнего равенства сразу следует
связь между F1 и F2 .
О т в е т. Для удержания второго конца к нему должна
4 q
быть приложена сила F2 = F12 + kK .
3 L
Примечание. Задачу можно было также решать методом
виртуальной работы. Предположим, что мы зафиксирова-
ли форму, которую приняла резинка (например, помести-
ли её внутрь гладкого желоба) и после этого медленно по-
тянули за точку 2. Сдвинем резинку вдоль самой себя на
часть Dt. Такой сдвиг можно рассматривать как перенос
кусочка Dt из точки 1 в точку 2. Изменение энергии ре-
зинки DE равно разности энергий кусочка в точках 1 и 2.
Энергия кусочка длиной Dx складывается из потенциальной
энергии в поле внешних зарядов и упругой энергии растя-
жения DE = Dt · Qf + (K/Dt)Dx2 /2 = Dt(Qf + F2 /(2K)). Произ-
ведённая работа равна F1 · Dx1 − F2 · Dx2 , где Dx1,2 — путь,
пройденный соответствующим концом резинки. Снова име-
ем F1 · Dx1 − F2 · Dx2 = DE1 − DE2 . Откуда, с учётом равенства
K · Dx1,2 /Dt = F1,2 получается закон сохранения и ответ.
325. Так как размер металлического шара много мень-
ше диаметра диска можно считать, что он находится в од-
нородном электрическом поле E = s/(2ε0 ), где s = Q/pR2 —
поверхностная плотность заряда диска. Электростатическая
сила, действующая на шар равна F = E|q|, где q — суммар-
ный заряд, сосредоточенный на шаре. Так как электриче-
410 Решения задач

ского поля внутри сферы нет, потенциал в центре сферы


равен потенциалу на её поверхности, т. е. нулю (сфера за-
землена). С другой стороны, этот потенциал равен сумме
потенциалов создаваемых диском и распределением заря-
дов на шаре.
Рассчитаем потенциал в середине диска создаваемый
зарядами диска (так как r  R он равен потенциалу в
центре сферы). Для этого можно разбить диск на коль-
ца шириной d A и сложить потенциалы создаваемые все-
ми кольцами. Рассмотрим кольцо радиуса A, его заряд
равен 2ps A · d A. В своём центре оно создаёт потенци-
0 ) = s · d A/(2ε0 ). Суммируя по всем
ал dФ = 2ps A · d A/A/(4pε
кольцам и учитывая d A = R, получим, что потенци-
ал в центре сферы создаваемый зарядами диска равен
Ф1 = sR/(2ε0 ). Потенциал в центре сферы создаваемый за-
рядами на сфере в свою очередь равен Ф2 = q/(4pε0 r). Учи-
тывая, что Ф1 + Ф2 = 0 мы получим q = −2sprR. Сила дей-
ствующая на шар равна F = ps 2 rR/ε0 .
О т в е т. Электростатическая сила, с которой диск дей-
ствует на сферу, равна F = rQ2 /pε0 R3 .
326. Электростатическая система может быть проанали-
зирована в рамках механической статики. На рисунке мы
обозначили точкой G электростатический «центр тяжести»
трубки. Определить такую точку можно аналогично настоя-
щему центру тяжести: на заряженное тело в электрическом
поле действует сила, подобная гравитационной силе, дей-
ствующей на тело в поле тяжести Земли. Фигуру можно
разбить на маленькие кусочки и просуммировать все куло-
новские силы, действующие на каждый кусочек телефонной
трубки в отдельности. Из условия равновесия следует ра-
венство ∠CAG = a, а из симметрии задачи — AC = BC. Тогда
из двух первых равенств можно выразить AC = (L · cos a)/2.
Возможны два случая направления поворота: по и против
часовой стрелки. Затраченная при этом работа, очевидно,
будет разной.
О т в е т. Чтобы медленно повернуть фигуру в положе-
ние, когда отрезок ABB направлен вдоль поля нужно произ-
вести работу U1 = QEL(1 − cos a)/2 cos a или U2 = QEL(1 + cos a)/2 cos a.
327. Решение задачи можно последовательно уточнять
практически до бесконечности, поскольку, строгое вычис-
Решения задач городских туров 411

К решению задачи 326

ление ёмкости системы двух шаров требует, вообще го-


воря, учёта всех членов мультипольного разложения. По-
следнее выходит за рамки школьной программы. В самом
грубом приближении, кинетическую энергию шаров мож-
но вычислять, предполагая, что заряды шаров не изме-
няются. Подстановка конкретных числовых значений, со-
общаемых в условии задачи, показывает, что относитель-
ная ошибка в таком приближении составляет всего 9%. На
следующем шаге можно оценить, как влияет заряд пер-
вого шара на относительный потенциал второго. Третьим
шагом могла бы стать возможная оценка влияния диполь-
ного момента первого шара на потенциал второго шара.
Последовательность вычисления поправок можно продол-
жать. Следует, однако, заметить, что вклад каждой сле-
дующей поправки в результат примерно на порядок мень-
ше вклада предыдущей, поэтому дальнейшие уточнения ре-
зультата вскоре теряют смысл. Ниже, в решении, мы вос-
произведём первые приближения к решению задачи, со-
проводив их оценкой того, что дальнейшие поправки дей-
ствительно малы. Суммарный заряд системы сохраняется
Q = q1 + q2 , и равенство q1 /R1 + q2 /L = q1 /L + q2 /R2 выража-
ет закон сохранения энергии и позволяет найти заряды
q1 = Q cot(R1 cot L − R1 cot R2 )(R1 cot L + R2 cot L − 2R1 cot R2 )−1 ,
q2 = Q − q1 . Разность потенциалов в начальном приближе-
нии имеет вид:
 R1 R2 
Q L − R2 R2 −
U= + L
.
4pε0 R2 L + R1 L − 2R1 R2 R2 L + R1 L − 2R1 R2
412 Решения задач

Здесь, первое слагаемое в скобках выражает потенциал пер-


вого шара относительно второго, а второе слагаемое — по-
правку к потенциалу, которая обусловлена влиянием «мо-
нополя» первого шара. В этом же приближении вычислим
кинетическую энергию
 R1 R2 
QU Q2 L−
E (0)
= = L
.
2 8pε0 R2 L + R1 L − 2R1 R2

Теперь приступим к качественным оценкам величины сле-


дующего приближения. Введём «приведённый» радиус элек-
тростатической системы:
 R1 R2 −1
L−
L
.
R2 L + R1 L − 2R1 R2

Обозначим его R. Электрический дипольный момент систе-


мы удовлетворяет оценке P ∼ R−2 . Изменение потенциала,
связанное с влиянием дипольного момента, будет иметь вид
Df ∼ P · R−2 ∼ R−4 . Тогда, относительный вклад «дипольной»
поправки убывает обратно пропорционально кубу приве-
дённого радиуса Df/f ∼ R−3 . Процедура последовательного
уточнения решения может быть продолжена.
О т в е т. В грубом приближении кинетическая энергия
шаров есть E(0) = Q2 /8pε0 R, где R — приведённый радиус
электростатической системы. Учёт дипольного взаимодей-
ствия даёт относительную поправку Df/f ∼ R−3 .
328. Вначале выясним, при какой ситуации будет ка-
заться, что толщина стенок равна нулю. Очевидно, что для
этого крайние лучи, выходящие из бутылки, должны про-
ходить, касаясь внешней поверхности стекла. Таким обра-
зом, угол падения b такого луча на внутреннюю стенку бу-
1
тылки должен быть предельным, т. е. sin b = . По теоре-
n1
OA OB r R
ме синусов для OAB имеем: или = = .
sin b sin(p − a)
sin b sin a
Подставляя значение sin b в последнее равенство, получим
r 1
= . Проанализируем теперь, чему может быть рав-
R n1 sin a
но a. Так как n2 > n1 , то максимальное значение a равно
p/2.
Решения задач городских туров 413

К решению задачи 328

О т в е т. При взгляде сбоку будет казаться, что толщина


r 1
стенок бутылки равна нулю, если > .
R n1
329. Движение линзы можно представить как сложение
двух движений: параллельного сдвига линзы вдоль оптиче-
ской оси линзы Ox до начала колебаний и поворота лин-
зы вокруг горизонтальной оси Oy, перпендикулярной пер-
вой. Поворот линзы (при неизменном положении центра)
не приводит к смещению изображения источника располо-
женного на оптической оси. Действительно, изображение
остаётся на прямой соединяющей источник и центр линзы,
т. е. не может сместиться вверх или вниз. Смещение вдоль
исходной оптической оси невозможно из соображений сим-
метрии. При повороте линзы в другую сторону на тот же
угол смещение должно было бы сменить знак (если оно про-
порционально углу). Такой поворот, однако, получается от-
ражением системы относительно плоскости (xy) и, следова-
тельно, не может приводить к изменению x координат то-
чек. Смещение вдоль Ox поэтому равно нулю. Параллель-
ный сдвиг линзы вдоль Ox на величину D приводит к сдви-
гу изображения на величину D в том же направлении, так
как изображение источника, расположенного на бесконеч-
ности всегда находится на расстоянии F от линзы. Конеч-
ность расстояния до источника приводит в данном случае
к пренебрежимо малым поправкам, в чём вы можете убе-
диться сами. В итоге, траектория изображения представля-
ет собой горизонтальный отрезок, его длина равна размаху
колебаний линзы S = 2aF.
414 Решения задач

О т в е т. Ускорение изображения совпадает с горизон-


тальным ускорением линзы, которое и максимально, при
максимальном отклонении линзы, и равно amax = g · sin a · cos a ≈ ag.
330. Рассмотрим изображение круга, по которому летает
муха. Его центр будет находиться на расстоянии S от лин-
зы, так что 1/f = 1/S + 1/L и S = 15 см. Известно, что изобра-
жение создаваемое линзой будет масштабировано (растяну-
то) в S/L = 1/2 раз поперёк оптической оси и в (S/L)2 = 1/4
раз вдоль оптической оси. Аналогично будут преобразовы-
ваться компоненты скорости мухи. Максимальная и мини-
мальная скорости изображения мухи равны u/2 = 5 см/с и
u/4 = 2,5 см/с соответственно. Максимальная скорость до-
стигается, когда муха пересекает оптическую ось линзы, а
минимальная — в точке наибольшего удаления от оси. Дан-
ное решение использует приближение постоянного коэффи-
циента увеличения, так как r  L − f. Задачу можно решить
и точно, что приводит, однако, к длинным выкладкам. Мак-
симальная скорость будет достигаться теперь один раз на
круге в точке пересечения траектории изображения мухи
и дальней от линзы оптической оси. Точки же достижения
изображением минимальной скорости (их по-прежнему бу-
дет две) очевидно сместятся в сторону линзы.
О т в е т. Минимальная скорость изображения мухи рав-
на 2,5 см/с, она достигается в точке наибольшего удаления
насекомого от оптической оси линзы. Максимальная ско-
рость изображения равна 5 см/с и достигается в момент пе-
ресечения мухой оптической оси.
331. Если шайба при t = 0 имела только вращатель-
ную компоненту скорости, то каждая её точка двигалась
со скоростью V = wR. Под действием постоянной силы тре-
ния эта скорость убывала со временем по линейному зако-
ну: V (t) = wR − at, где a — ускорение шайбы под действи-
ем силы трения. Поскольку по V (t0 ) = 0, то это ускорение
a = wR/t0 . Рассмотрим теперь случай, когда шайба в на-
чальный момент времени одновременно участвует во вра-
щательном и в поступательном движении, а её скорость
U0 = [V02 + (wR)2 ]1/2 . Под действием постоянной силы трения
эта скорость линейно убывает со временем U(t) = U0 − at.
Сила трения не изменяет направление скорости шайбы
Решения задач городских туров 415

К решению задачи 331

(угол a на рисунке), а лишь тормозит её, приводя к останов-


ке через t1 = U0 · t0 /wR. Проекция силы трения в направле-
нии движения f = FТР · cos a, где cos a = V0 /U0 , обеспечива-
ет отрицательное ускорение шайбы, так что её перемеще-
ние вдоль стержня равно x(t) = V0 · t − a · cos at2 /2. Путь, ко-
торый пройдёт по стержню шайба до остановки, есть x(t1 ).
О т в е т. До остановки шайба пройдёт по стержню путь
s = V0 · U0 · t0 /2wR.
332. Вдоль горизонтальной оси на систему обруча и
шайбы не действуют никакие силы, поэтому импульс вдоль
этой оси сохраняется:

M · U = m · VX .

Здесь M и U суть масса и скорость обруча, а VX —


горизонтальная проекция скорости шайбы. Заметим, что
VX и U меняют знак синхронно: когда шайба движет-
ся по верхней части обруча, например, вправо, обруч ка-
тится влево. Для нижней части обруча — наоборот. Ско-
рости шайбы относительно центра обруча VОТН и отно-
сительно земли V связаны как V =ОТН +U. Отсюда мож-
но выписать связь компонент вектора скорости шайбы со
скоростью обруча (см. рис. 1): tg f = VY /(VX + U). Нако-
нец, для системы справедлив закон сохранения энергии
mg · R(1 + cos f) = 1/2 · mV 2 + 1/2 · MU 2 . Законы сохранения
и кинематическая связь суть три уравнения, определяю-
щие неизвестные горизонтальную и вертикальную проек-
ции скорости шайбы VX и VY , вместе со скоростью обруча
U. Решения уравнений удобно записать, воспользовавшись
безразмерными скоростями u = U/(2gR)1/2 и v = V /(2gR)1/2 ,
416 Решения задач

а также отношением масс обруча и шайбы m = M/m:



1 + cos f
u = cos f , vx = mu, vy = (1 + m)u tg f.
(1 + m)(m + sin2 f)

Интересно проследить, как «обмениваются» скоростя-


ми обруч и шайба при разных отношениях их масс m. На
рис. 2–4 изображены зависимости вертикальной компонен-
ты безразмерных скорости шайбы vy (сплошная линия) и
скорости центра обруча u (прерывистая линия) от поло-
жения шайбы в обруче f (измеряемого в радианах). Шай-
ба стартует из точки p (верхнего положения) без началь-
ной скорости vy (p) = 0 и скользит вниз vy > 0, к точке 0.
В нижней точке вертикальная скорость шайбы равна нулю
vy (0) = 0. Пройдя нижнюю точку, шайба начинает движение
в обратном направлении, вверх, vy < 0, возвращаясь в верх-
нее положение с vy (−p) = 0. Направление движения обруча
меняется на противоположное, когда шайба проходит через
точки f = ±p/2. Скорость обруча максимальна, когда шайба
находится в f = 0. Если массивная шайба скользит внутри
очень лёгкого обруча, m  1 (рис. 2 построен для m = 0,01),
то движение обруча приобретает характер толчков, происхо-
дящих, когда шайба находится вблизи нижней точки. Сама
шайба в этот момент скачком меняет свою скорость на про-
тивоположную так, как это происходит например при упру-
гом ударе о неподвижную стенку. Допуская увеличение m
(см. рис. 3), мы наблюдаем постепенное сглаживание харак-
тера движения системы. Наконец, движение лёгкой шайбы
внутри тяжёлого обруча, m  1 (на рис. 4 выбрано m = 100),
подобно гармоническим колебаниям.
О т в е т. Скорость центра обруча, когда шайба находится
под углом f к вертикали равна

U 1 + cos f
√ = cos f· .
2gR (1 + M/m)(M/m + sin2 f)

333. Мысленно разобьём пробирку на N  1 малых ча-


стей и выпишем равенства сил для двух малых элементов
массы, находящихся на расстоянии x от середины пробир-
Решения задач городских туров 417

К решению задачи 332

К решению задачи 332


418 Решения задач

К решению задачи 332

К решению задачи 332


Решения задач городских туров 419

ки, ⎧  
l
⎨ dm · w2 · − x = d pi · p · r2 − dm · g · cos f,
2
 
⎩ dm · w2 · l + x = d p · p · r2 − dm · g · cos f,
2 k

где d p — разность давлений слева и справа от каждого из


элементов. Число таких пар равно N/2. Просуммировав все
аналогичные пары уравнений, получим:
1 2 2 1
· r w = (pB − p A ) − · rg · l · cos f.
2 2
Поскольку предельная форма мениска — полусфера, то раз-
ность давлений на концах трубки есть p A − pB = r · g · l · cos f + 4s/r.
Подставляя это выражение в предыдущее уравнение, полу-
чим выражение для угла выливания жидкости из капил-
ляра, cos f = w2 l/g − 8s/rrgl. Поскольку | cos f| < 1, условие
определяет интервал допустимых значений параметров за-
дачи, при нарушении которых жидкость либо выливается
из капилляра сразу, либо не выливается
оттуда вовсе.
2
О т в е т. Если w l/g − 8s/rrgl < 1, жидкость начнёт вы-
 
ливаться, если f = arccos w l/g − 8s/rrgl .
2

334. Напряжённость электростатического поля внут-


ри равномерно заряженного шара E(r < R) = K · Q · r/R3 , где
K = 1/4pε0 может быть найдена с помощью теоремы Гаус-
са или исходя из двух известных фактов: напряжённость
электрического поля внутри равномерно заряженного сфе-
рического слоя равна нулю; вне равномерно заряженного
слоя электростатическое поле эквивалентно полю точечно-
го заряда, помещённого в центр этого слоя. На мезон, на-
ходящийся внутри ядра, действует сила, направленная к
центру ядра, величина которой пропорциональна смеще-
нию мезона от центра. Уравнение движения частицы под
действием такой силы mr = −qE полностью аналогично
уравнению колебаний под действием силы, подчиняющей-
ся закону Гука. Воспользовавшись указанной аналогией,
нетрудно получить выражение для частоты колебаний мезо-
на w02 = k/m = KqQ · r/R3 . Энергия, необходимая для выбива-
ния покоящегося мезона из центра ядра включает, во-пер-
вых, энергию, чтобы вывести мезон на поверхность ядра,
420 Решения задач

во-вторых, энергию удаления мезона с поверхности ядра на


бесконечность. Первая их энергий может быть найдена как
энергия гармонических колебаний с амплитудой, равной
радиусу ядра W1 = mw02 · R2 /2. Выражение для второй части
энергии аналогично общеизвестной формуле для энергии,
которую нужно сообщить телу на поверхности планеты для
того, чтобы оно навсегда её покинуло (определённый инте-
грал от электростатической силы в пределах от радиуса яд-
ра до бесконечности) W2 = KqQ/R. Полная энергия выбива-
ния покоящегося мезона из ядра тогда равна W0 = W1 + W2 .
Последнее равенство позволяет выразить радиус атомного
ядра через известную энергию W0 .
О т в е т. В рамках сделанных в условии задачи допуще-
ний, радиус ядра оценивается величиной R = w0−1 · (2W0 /3m)1/2 .
335. Из законов сохранения энергии и импульса следу-
ет, что в результате единичного соударения лёгкой и тяжё-
лой бусинок, двигавшихся в одном направлении со скоро-
стями v1 и u1 соответственно, величины их скоростей изме-
няются следующим образом:
Mv1 − 2Mu1 + mv1 Mu1 − mu1 + 2mv1
v2 = , u2 = .
(M + m) (M + m)

Полагая m  M, отсюда получаем, что v2 ≈ v1 − 2u1 , а u2 ≈ u1 − m · u1 + 2m · v1 ,


где m = m/M  1. Перепишем последние приближённые ра-
венства в виде отношения конечных разностей
Du Dv
= −m · u + 2m · v, = −2u,
Dn Dn

описывающих приращения скоростей бусин в течение их


элементарного взаимодействия друг с другом. В пределе
большого числа соударений n  1, такие уравнения могут
быть поняты как дифференциальные уравнения, в кото-
рых переменная n играет роль времени. Несложно убе-
диться, что такая система дифференциальных уравнений
эквивалентна уравнению затухающих гармонических коле-
баний для функции v(n): v + m · v + 4m · v = 0, решение ко-
торого (удовлетворяющее начальным условиям v(0) = v0 и

v (0) = 0) есть v(n) ≈ v0 · e−mn/2 · cos(n · 2 m) (здесь мы прене-
брегли вкладом порядка m по сравнению с m1/2 в аргумен-
Решения задач городских туров 421

К решению задачи 335

те косинуса, поскольку m  1). Бусинки перестанут сталки-


ваться, когда v ≤ u = −v /2 (маленькая бусинка перестанет
догонять большие). Последнее имеет место для чисел соуда-
рений, превосходящих
  
1 m−4
n ≥ − √ · arctg √ .
2 m 4 m

Прямоугольные скобки обозначают целую часть числа. Гра-


фик максимального числа соударений бусинок как функция
отношения их масс m показан на рис. 1. Из приведённо-
го графика видно, что сравнительное большое число соуда-
рений между бусинками возможно только, если отношение
масс m очень мало. В пределе nmax → ∞, когда m → 0.
О т в е т. Если отношение массы лёгкой бусинки к мас-
сам тяжёлых бусинок очень мало, m  1, число их столкно-
вений друг с другом не превосходит целой части
  
1 m−4
− √ · arctg √ .
2 m 4 m

Установившаяся скорость бусинки может быть √ оценена в


этом случае как v0 exp(−mnmax /2) · cos(nmax · 2 m). Её график
приведён на рис. 2.
336. Для вычисления установившейся скорости падения
градин запишем условие движения без ускорения при нали-
422 Решения задач

К решению задачи 335

чии компенсирующих друг друга сил тяжести и сопротивле-


ния воздуха:
4
0 = ma = mg − F(v) = pr3 rice g − ar2 v.
3
Решая полученное уравнение, находим скорость падения
градин v. Воздушный шар будет висеть неподвижно, если
сила Архимеда скомпенсирована силой тяжести и эффек-
тивной силой, действующей на него со стороны падающих
градин
DP
0 = MaH = F A − Mg − Fice = (r − rH )Vg − ,
Dt
последнюю будем вычислять через скорость изменения
импульса, передаваемого градинами шару. Для расчёта
передаваемого импульса мысленно разделим поверхность
шара на тонкие кольца, положение которых будем ха-
рактеризовать углом j с вертикалью (см. рисунок). Чис-
ло градин, ударяющих о выделенное кольцо площадью
DS = 2p · R2 · sin j · Dj за заданный интервал времени опреде-
ляется выражением
DN = n · vDt · DS · cos j = pR2 nv · sin(2j) · Dt · Dj.
Все градины, ударяющиеся о выделенное кольцо, передают
в вертикальном направлении одинаковое количество дви-
Решения задач городских туров 423

жения Dp = mv · (1 + cos 2j). Очевидно, что сумма горизон-


тальных составляющих импульсов, передаваемых шару все-
ми градинами, ударяющими в кольцо, равна нулю в силу
симметрии системы. Полный импульс, передаваемый одно-
му кольцу за рассматриваемый промежуток времени, равен
произведению передаваемого одной частицей импульса на
число ударов, DP(j) = Dp · DN = pR2 nv2 m · sin 2j · (1 + cos 2j) · Dt · Dj.
Для нахождения силы, действующей на шар со сторо-
ны всех ударяющих о него градин, необходимо выпол-
нить суммирование по всем кольцам. Вычисление такой
суммы можно существенно упростить, если осуществлять
суммирование по парам колец, расположенных под оди-
наковыми углами по отношению к направлению p/4. Для
каждой из таких пар колец выполняются соотношения
sin 2fi = sin(p − 2ji ) = sin 2ji ; cos 2fi = − cos 2ji , в силу кото-
рых некоторые слагаемые в суммируемом выражении по-
парно компенсируются, и выражение для силы существен-
но упрощается:

Fice = pR2 nv2 m sin 2ji Dj = mnv2 (2pR2 sin ji Dj) cos ji =
i i

= mnv2 DSi cos ji = mnv2 pR2 .
i

В результате элементарных тождественных преобразова-


ний находим, что каждое из слагаемых оставшейся суммы
пропорционально площади проекции соответствующего ему
кольца на экваториальную плоскость шара. Последнее свой-
ство позволяет легко выполнить суммирование. Используя
предыдущие соотношения, находим отсюда объём воздуш-
ного шара и его радиус.
О т в е т. Объём и радиус неподвижно висящего под гра-
дом воздушного шара равны:
 3
4 np · R3 g 16 ng(p · rice · r)3
V= p · rice · r , R= · 2 .
3 a2 (r − rH ) 9 a (r − rH )

337. Звуковые волны, распространяющиеся в узком


горле графина, представляют собой продольные колебания
424 Решения задач

К решению задачи 336

К решению задачи 336

плотности воздуха. В рамках школьной программы изуча-


ется один из сюжетов, связанных с продольными колебани-
ями — это гармонические колебания груза, закреплённого
на пружине. Поэтому, для объяснения наблюдаемого явле-
ния на языке школьной физики, будет естественным обра-
титься именно к нему. Роль невесомой пружины при этом
может играть газ в широкой части сосуда (очевидно, что
скорость его движения при акустических колебаниях ма-
ла, и он играет роль «упругого буфера», обладающего, глав-
ным образом, потенциальной, а не кинетической энергией).
Тогда быстро перемещающийся в узком горлышке сосуда
газ будет «резервуаром» кинетической энергии — анало-
гом движущегося массивного груза. Поясним намеченную
аналогию подробнее. Уравнение гармонических колебаний
связывает упругую силу, действующую на груз со стороны
пружины, с его продольным перемещением. Его аналогом
здесь будет уравнение состояния звуковой волны, связыва-
ющее периодические возмущения давления с сжатием или
разряжением воздуха. Запишем изменение массы Dm возду-
ха в элементе длины горлышка графина сечения S, связан-
ное с изменением плотности воздуха Dr при прохождении
в горле звуковой волны со скоростью c, Dm = Dr · ScDt. Со-
ответствующее ей изменение давления тогда запишется как
Решения задач городских туров 425

нормированное на единицу площади сечения горлышка от-


ношение переносимого импульса ко времени:
Dm · c
Dp = = c2 Dr.
S · Dt

Перепишем полученное уравнение состояния в виде урав-


нения гармонических колебаний. Для этого напомним, что
изменение давления в основной части графина обусловле-
но воздействием на находящийся в ней воздух движущейся
«воздушной пробки» в горлышке,
F m · x
Dp = = = r0 · l · x .
S S

Поэтому уравнение состояния звуковой волны эквивалент-


но уравнению гармонических колебаний,
S
x + c2 x = 0.
Vl

Откуда легко находится циклическая частота таких колеба-


ний, 2p · v = w = c · (S/V · l)1/2 , а также искомый объём широ-
кой части графина.
О т в е т. Объём широкой части графина, издающего звук
частоты v, равен V = S/l · (c/2pv)2 .
338. Состояние невесомости бусинка испытывает каж-
дый раз, когда она не давит на проволоку, или, что по тре-
тьему закону Ньютона — то же самое, когда сила реакции
проволоки обращается в нуль. В процессе спуска по прово-
локе бусинка будет испытывать состояния невесомости двух
типов. Во-первых, в точках, где центростремительное уско-
рение полностью обеспечивается силой тяжести, и, следо-
вательно, сила реакции проволоки обращается в нуль. Так,
например, самое первое состояние невесомости бусинка ис-
пытает, скатываясь с первого полукружья, в тот момент,
когда направление на бусинку b удовлетворяет соотноше-
нию mg cos b = m · V 2 /R. Скорость же в момент первой неве-
сомости можно найти, воспользовавшись законом сохране-
ния энергии, учитывая, что первоначально бусинка имела
нулевую скорость: mV 2 /2 = mg · R(1 − cos b). Первое состоя-
ние невесомости бусинка испытает при cos b = 2/3. В ходе
426 Решения задач

К решению задачи 338

дальнейшего спуска бусинка будет приходить в аналогично


расположенные на других полуокружностях точки, посте-
пенно (и очень медленно, так как угол a мал) разгоняясь.
Поскольку в первом уравнении на b, при котором имеет ме-
сто невесомость, правая часть, пропорциональная mV 2 , бу-
дет расти, то cos b тоже будет расти, а угол b — уменьшать-
ся. Кроме того, очевидно, что при подъёме на полуокруж-
ность, бусинка испытает аналогичное состояние невесомо-
сти в симметричной точке. На рисунке изображены точки,
где бусинка испытывает положения невесомости, описанно-
го типа. Итак, положения невесомости встречаются по два
на каждой обращённой вверх полуокружности и постепен-
но сливаются, при этом угол, характеризующий положение
невесомости, приближается к нулю. После его обращения в
ноль положения невесомости этого типа исчезают. Найдём,
сколько состояний невесомости описанного типа опоры на
противоположное. При любом конечном размере бусинки
это приводит в какой-то момент к обращению силы реак-
ции в ноль. (Здесь можно применить вспомогательное рас-
суждение. Представим себе «наблюдателя», находящегося в
«кабине» внутри бусинки. До прохождения точки перегиба
наблюдателя прижимает к левой стенке кабины, после про-
хождения — к правой. В момент, когда наблюдатель падает
с левой стенки на правую, он, конечно, испытывает неве-
сомость). Всего бусинка испытает 150 − 29 = 121 положение
невесомости такого рода.
О т в е т. Проволока состоит из 122 полуокружностей, и
её длина равна 122pR.
339. Задачу удобно решать в системе отсчёта, связан-
ной с землей. Величина приращения скорости платфор-
мы после прыжка каждого из солдат зависит от количе-
ства остающихся на ней солдат. Устанавливающаяся ско-
Решения задач городских туров 427

К решению задачи 338

рость платформы в этом случае выражается в виде сум-


мы большого числа слагаемых, вычисление которой про-
блематично. Однако, ответ на качественный вопрос зада-
чи может быть найден. В случае одновременного прыж-
ка всех солдат использование закона сохранения импульса,
M · V = N · m(v − V ) = 0, приводит к выражению для скорости
платформы,

N
V = Nmv/(M + Nm) = mv/(M + Nm).
n=1

Аналогичное применение закона сохранения импульса,


[M + (N − n)m]Vn = [M + (N − n − 1)m]Vn+1 − m(v − Vn+1 ),
для случая поочерёдного движения позволяет найти при-
ращение скорости платформы после начала движения сол-
дата с номером n, Vn+1 − Vn = mv/[M + (N − n)m]. Конечная
скорость платформы во втором случае выражается суммой
ряда,

N
VS = V0 + V1 + . . . + V N = mv/[M + (N − n)m].
n=1

Сравнение слагаемых сумм m · v/(M + N · m) = m · v/[M + (N − n)m],


n = 0, 1, 2, позволяет заключить, что в случае поочерёдных
прыжков солдат скорость платформы окажется выше, чем
при их одновременном движении V < VS .
428 Решения задач

О т в е т. Платформа поедет быстрее в случае поочерёд-


ных прыжков солдат.
340. Безразличный характер равновесия означает, что
стержень может оставаться в состоянии покоя при любом
его положении относительно шарнира. Задача может быть
решена двумя способами. Можно использовать условие рав-
новесия твёрдого тела с закреплённой осью вращения: сум-
ма вращающих моментов относительно точки закрепления
должна равняться нулю (независимо от ориентации стерж-
ня в пространстве). Другой способ состоит в расчёте по-
тенциальной энергии стержня при требовании независимо-
сти этой энергии от его ориентации в пространстве (усло-
вия безразличного равновесия). Поскольку поворот стерж-
ня вокруг горизонтальной оси не должен выводить его из
состояния равновесия, точка закрепления пружины должна
находиться на вертикали, проходящей через шарнир X = 0,
Y = h. Для любого угла наклона стержня относительно вер-
тикали момент силы тяжести должен быть скомпенсирован
моментом силы, действующей на стержень со стороны пру-
жины,
Mg · L/2 · sin a = Fx l · cos a + Fy l · sin a.

Подстановка сюда явного выражения для силы упруго-


сти, создаваемой линейной пружиной с малой длиной
в недеформированном состоянии, приводит к равенству
Fx = k(l − l0 ) · sin a ∼ kl · sin a, Fy = k(l − l0 ) · cos a ∼ kl · cos a, Mg · L/2 · sin a = k · lh · s
При таком условии равновесие будет безразличным (ра-
венство моментов выполняется при любых углах наклона
стержня).
О т в е т. Пружина должна быть закреплена на пер-
пендикуляре, проходящей через точку подвеса, на высоте
Mg · L/2kl над ним.
341. На рисунке изображены силы, действующие на
клин с номером k. Здесь и далее a = 29 ◦ C + 15 · 1 ◦ C = 44 ◦ C —
угол между вертикалью и верхней плоскостью верхнего
клина, b — угол при вершине клина. Клинья будем нумеро-
вать, начиная с верхнего — на наклонной плоскости лежит
клин с номером k = 15. Начнём с записи условия равенства
нулю суммы проекций сил на ось симметрии клина с номе-
Решения задач городских туров 429

ром k,

Nk · sin(b/2) + Nk−1 · sin(b/2) = mk g · cos[a + b(k − 1)/2].

Условие равенства нулю суммы проекций сил на перпенди-


кулярную ось тогда запишется как

Nk · cos(b/2) + Nk−1 · cos(b/2) = mk g · sin[a + b(k − 1)/2].

Решение системы этих уравнений есть

Nk = mk g · cos(a + k · b)/ sin b, Nk−1 = mk g · cos(a + (k + 1)b)/ sin b,

оно позволяет выписать рекуррентные соотношения для сил


реакции опоры:

Nk /Nk−1 = cos(a + k · b)/ cos[a + (k + 1)b].

Сила реакции опоры, действующая со стороны шайбы на


верхний клин, совпадает по модулю с проекцией силы тя-
жести N0 = f = mg · sin a. Результат последовательного при-
менения рекуррентного соотношения, после простых преоб-
разований, приобретает вид:

cos(a + k · b) cos(a + (k − 1)b) cos(a + k · b)


Nk =Nk−1 =Nk−2 · =
cos(a + (k + 1)b) cos(a + k · bb) cos(a + (k + 1)b)
cos(a + (k − 1)b) cos(a + (k − 2)b) cos(a + (k − 1)b)
= Nk−2 = Nk−3 · =
cos(a + (k + 1)b) cos(a + (k − 1)b) cos(a + (k + 1)b)
cos(a + (k − 2)b) cos(a + b)
= Nk−3 = . . . = N0 .
cos(a + (k + 1)b) cos(a + (k + 1)b)

Результат подстановки численных значений в последнюю


формулу даёт ответ. √
О т в е т. Искомая результирующая сила равна N1 = 2mg · sin 44 ◦ C.
342. Основные понятия классической теории поля и её
теоремы с равным успехом применимы как для электро-
статики, так и для гравитации. Известная из курса элек-
тростатики теорема Гаусса позволяет обойтись без интегри-
рования при вычислении напряжённостей электростатиче-
ских полей многих простых заряженных геометрических
430 Решения задач

К решению задачи 341

тел. Точно так же, теорема Гаусса позволяет легко, не при-


бегая к интегрированию, находить напряжённости гравита-
ционных полей. Воспользуемся аналогией между законом
всемирного тяготения и законом Кулона. Используя заме-
ну переменных, приведённую ниже, можно превращать со-
отношения для гравитационных сил в аналогичные соотно-
шения электростатики и наоборот:

⎨ F = G Mm M↔Q
R2
⇒ 1
⎩ F = 1 Qq G↔
2
4pε0 R 4pε0

Теорема Гаусса для электростатического поля и её аналог


Q
для гравитационных сил даются формулами NE = ⇒ Ng = 4pGM.
ε0
Развивая эту аналогию, перенесём хорошо известный в кур-
се школьной электростатики расчёт электростатического по-
ля на поверхности бесконечного равномерно заряженно-
го по объёму цилиндра (2p · R · H · E = rQ · p · R2 · H/ε0 , отку-
да E = rQ · R/2ε0 ) на вычисление ускорения свободного па-
дения (напряжённости гравитационного поля) на поверх-
ности планеты: g = 2p · Gr · R. Невесомость на поверхности
вращающегося цилиндра случается, когда гравитационная
и центробежная силы уравновешиваются: mg = mw2 · R. От-
сюда легко находится искомая плотность вещества плане-
ты.
О т в е т. Плотность вещества планеты равна r = w2 /2pG.
343. Поскольку все соударения между шарами упру-
гие, и шарики просто обмениваются скоростями, то если
их не различать между собой, то картина колебаний оста-
Решения задач городских туров 431

нется такой же как и в отсутствие столкновений: макси-


мальной амплитудой будет обладать последний шарик. Ес-
ли бы между маятниками не происходили столкновения,
амплитуды их колебаний удовлетворяли бы неравенствам
A1 < A2 < . . . < AN . При упругом столкновении двух маятни-
ков одинаковой массы между ними происходит обмен ско-
ростями, как это следует из закона сохранения энергии:

mv1 + mv2 = mu1 + mu2 u1 = v2
mv21 mv22 mu21 mu22 ⇒
+ = + u2 = v1 .
2 2 2 2

Поскольку амплитуда колебаний последнего маятника мак-


симальна, он вернётся в точку старта лишь спустя промежу-
ток времени, равный полному периоду колебаний системы.
О т в е т. Последний маятник будет находится в точке
своего начального положения в моменты t1 + t2 + . . . + tN−1 + 2pk(L/g)1/2 .
344. Для нахождения вектора ускорения в данный мо-
мент времени достаточно будет найти лишь угол наклона
траектории в начальный момент времени. Действительно
(см. рисунок), если этот угол равен x, то искомое ускорение,
связано с ускорением свободного падения соотношением
a = g cos(x). Найдём теперь x. Пусть V — скорость бусинки
в какой-то момент. Тогда сумма проекций этой скорости на
нити 1, 2 и 3 равна нулю. Выпишем условие нерастяжимо-
сти нити, V cos(a + x) + V cos(b + x) − V sin x = 0, из которого
можно выразить x: tg x = (cos a + cos b)/(1 + sin a + sin b).
О т в е т. Ускорение бусины в начальный момент време-
ни равно: 
(1 + sin a + sin b)2
a=g .
(1 + cos a + cos b)2

345. До соударения с землей контейнер и молекулы газа


в среднем двигались со скоростью u = (2gH)1/2 . После мгно-
венного соударения контейнер изменил направление дви-
жения на противоположное, а газ продолжает двигаться с
прежней средней скоростью. При этом скорость центра масс
системы равна vЦМ = u(M − m)/(M + m). Остаток энергии пе-
реходит во внутреннюю в результате множественных соуда-
рений молекул газа со стенками и между собой, а движе-
432 Решения задач

К решению задачи 344

ние центра масс тормозится гравитационным полем. В ре-


зультате, из закона сохранения энергии можно написать:
(M + m) · v2ЦМ /2 = (M + m) · gh.
О т в е т. После удара о землю контейнер подскочит на
высоту
 
(M − m)2 4Mm
h=H 2 =H 1− 2 .
(M + m) (M + m)
346. По условию задачи колесо вначале вращалось быст-
ро, и бусинка была прижата к ободу колеса. После того, как
колесо затормозили до угловой скорости w, при некотором
угле наклона стержня, бусинка уже сможет оторваться от
обода. Найдём, при каком угле a0 это произойдёт. В момент
отрыва от обода на бусинку действуют силы тяжести, упру-
гости и реакции стержня, направленная перпендикулярно
стержню. Сила реакции обода в этот момент равна, очевид-
но, нулю. В проекции на стержень уравнение движения бу-
сины имеет вид mg · cos a0 + k · R = mw2 · R. Выразим из него
cos a0 = (m · w2 − k)R/mg. Числовые данные в условии подо-
браны так, что mw2 = k, т. е. a0 = p/2. Угловая частота вра-
щения колеса при этом совпадает с собственной частотой
колебаний бусинки на пружинке. Итак, после торможения
колеса до угловой скорости w, бусинка сразу же отрывается
от обода. Пусть в момент t, когда стержень образует угол
a = −p/2 + w · t с вертикалью, бусинка находится на рассто-
янии r от центра колеса. Уравнение движения бусинки в
проекции на направление стержня выглядит особенно про-
сто (поскольку центробежная и упругая силы компенсиру-
ются): g cos a = a = g sin(wt). Под действием переменной си-
лы бусинка совершает гармонические колебания с частотой
Решения задач городских туров 433

К решению задачи 346

w. Скорость меняется со временем также по гармоническо-


му закону, с частотой w и амплитудой g/w, оставаясь поло-
жительной (бусина не меняет направления движения). До
момента времени t = p/w бусинка разгоняется, затем столь-
ко же времени замедляется и к моменту t1 = p/w проходит
вдоль стержня путь S = 2p · g/w2 = 0,615 м, и именно на та-
кое расстояние сдвинется по стержню бусинка за один обо-
рот колеса. Длина стержня по условию равна 1,5 м. За пер-
вые два оборота колеса бусинка сдвинется на x = 1,23 м. На
третьем обороте колеса бусинка столкнётся с ободом. За тре-
тий оборот колеса бусинка сдвинулась бы на L − x = 0,27 м.
Эта величина меньше S/2, значит бусинка столкнётся с обо-
дом на первой половине третьего оборота. Точка закрепле-
ния пружины B первую половину оборота находится ниже
центра колеса, поэтому после столкновения бусинка будет
просто лежать на ободе до тех пор, пока стержень снова не
окажется горизонтальным (это произойдёт через два с поло-
виной оборота после начала движения бусинки). Затем бу-
синка начнёт двигаться вдоль стержня в другую сторону,
так же — «шагами» по 0,615 м. Ещё через два с половиной
оборота бусинка окажется в исходном положении.
О т в е т. Через 5 оборотов колеса система вернётся в ис-
ходное положение.
347. Введём оси координат OX, OY и спроектируем на-
чальную скорость снаряда u на эти оси. В проекции на го-
ризонтальную ось и снаряд машина двигаются равномерно,
434 Решения задач

поэтому, чтобы снаряд попал в машину, их горизонтальные


скорости должны быть равны, т. е. ux = V0 . В проекции на
ось OY зависимость координаты снаряда от времени име-
ет вид Y(t) = H + uy t − gt2 /2 (здесь H — высота обрыва). В
момент попадания снаряда в машину координата Y(t1 ) = 0.
Отсюда, решая квадратное уравнение H + uy t1 − gt21 /2 = 0 и,
отбрасывая отрицательный
p корень, получаем время полё-
uy + u2y + 2gH
та снаряда t1 = . Так как существует некото-
g
рая максимальная скорость вылета снаряда umax , ей соот-
ветствует максимальная возможная вертикальная проекция
начальной скорости снаряда umax = u2max − V02 . Дуло пушки
может быть наклонено и выше и ниже по отношению к гори-
зонту, так что вертикальная проекция начальной скорости
снаряда может меняться от −umax y до umax y . Время полё-
та снарядаpпри этом меняется от минимального значения
−umax y + u2max y + 2gH
t= (когда дуло направлено ниже гори-
g p
umax y + u2max y + 2gH
зонта) до максимального значения T = .С
g
другой стороны, по условию, за это время машины прохо-
дят расстояние от минимального x = tV0 до максимального
y = TV0 . Выражая отсюда t и T и подставляя в предшеству-
ющие равенства, получаем систему уравнений:
p p
−umax y + u2max y + 2gH x umax y + u2max y + 2gH y
= = ,
g V0 g V0

(y − x)g
решая которую относительно umax y и H, получаем umax y = ,
2V0
xyg
H = 2 . Зная umax y , нетрудно восстановить и максималь-
2V0
ную скорость вылета снарядов:


(y − x)2 g2
umax = u2max y + u2max x = + V02 .
4V02

xyg
О т в е т. Пушка расположена на высоте H = над вы-
2V02
ездом из туннеля. Максимальная скорость, с которой могут
Решения задач городских туров 435

К решению задачи 347

вылетать снаряды равна



(y − x)2 g2
umax = + V02 .
4V02

348. В начальный момент (см. точку 0 на графике


зависимости давления газа от объёма) температура газа
равна T, а давление газа лежит в промежутке от p0 до
p0 + Mg/S (если бы давление было меньше, чем p0 нижний
поршень ехал бы вверх, если бы давление было больше, чем
p0 + Mg/S вверх ехал бы верхний поршень). На начальном
этапе, до тех пор, пока давление газа между поршнями не
сравнялось с p0 + Mg/S, увеличение температуры происхо-
дит изохорически — нижний поршень не может двигать-
ся вниз по условию, а давления газа ещё не хватает для
того, чтобы поднять верхний поршень (см. рис.). На гра-
фике этому этапу соответствует участок 0 − 1. Затем про-
цесс становится изобарическим — верхний поршень под-
нимается (см. участок 1 − 2). В точке 2 температура га-
за по условию равна T  . После газ начинает изохорически
остывать — верхний поршень опуститься вниз не может, а
нижний ещё не поднимется, так как давление газа между
поршнями остаётся больше чем p0 (см. участок 2 − 3). Да-
лее газ охлаждается до исходной температуры изобариче-
ски (см. участок 3 − 4, температура в точке 4 равна темпе-
ратуре в точке 0: T4 = T), при этом нижний поршень под-
нимается. Затем газ начинают снова нагревать, чему соот-
ветствует участок 4 − 5. Наконец, процесс развивается по
436 Решения задач

циклу 5 − 2 − 3 − 4 − 5. Таким образом, мы видим, что при


любом начальном давлении газа p0 ≤ p ≤ p0 + Mg/S в си-
стеме установится газовый цикл, состоящий из двух изо-
бар и двух изохор 4 − 5 − 2 − 3 − 4. Газу сообщается тепло-
та в процессах 4 − 5 − 2. Эта теплота идёт на совершение
газом работы (p0 + Mg/S)(V2 − V1 ) и на увеличение внут-
ренней энергии одноатомного газа 3v · R(T − T)/2. Объёмы
V1 и V2 находим из уравнения состояния V1 = vRT/p0 и
V2 = v · RT  /(p0 + Mg/S), а затем выражаем теплоту Q, сооб-
щённую газу в процессах 4 − 5 − 2. Из-за тепловых потерь,
затраченная в подъёмнике энергия E = 100/(100 − l) · Q ока-
зывается в раз больше, чем Q. Работа, совершённая двига-
телем за один цикл, равна изменению потенциальной энер-
гии массивного поршня: A = Mg · (V2 − V1 )/S, а КПД подъ-
ёмника равен h = A/E. Отметим, что КПД подъёмника не
совпадает с КПД цикла 4 − 5 − 2 − 3 − 4, так как часть рабо-
ты совершается «бесплатно» атмосферным давлением. Кро-
ме того, если в начальный момент температура газа была
больше, чем температура газа в точке 3, то изотерма, про-
ведённая через точку 0 пересечёт отрезок 2 − 3, и в цикле
пропадут изобарические участки. В этом случае подъёмник
работать не будет (он поднимет груз только в первый раз),
и его КПД равно нулю.
О т в е т. КПД подъёмника равен
    
100 − l T T 5(T − T) Mg −1
h= · Mg · − · −T· .
100 S · p0 + Mg S · p0 2 S · p0

349. Возникающая в материале шарика сила упругости


F задаётся соотношением s = F/S = E · Dl/l0 , где s — напря-
жение материала, S — площадь поперечного сечения образ-
ца, E — модуль Юнга, Dl/l0 — относительное удлинение
образца. Обозначим через p давление газа, которое уста-
новится в шарике радиуса r. Давление будет растягивать
шарик до тех пор, пока упругая сила резиновой оболочки
2pr · d · s (d — толщина растянутой оболочки) не скомпен-
сирует давление газа p · pr2 . Из равенства сил следует, что
s = pr/2d. Относительное удлинение любого участка шари-
ка составляет (r − r0 )/r0 , поэтому s = E · (r − r0 )/r0 = r · p/2d.
Объём резины сохраняется, поэтому d0 r02 = dr2 . Выражая
Решения задач городских туров 437

К решению задачи 348

отсюда толщину оболочки растянутого шарика d, получим


E(r − r0 )/r0 = pr3 /2d0 r02 . Состояние идеального газа в шари-
ке описывается уравнением 4p · pr3 /3 = mRT/m. Найдём из
него давление и подставим в написанное раньше равенство
сил, E(r − r0 )/r0 = 3md0 r02 · RT/8pm. Последнее равенство вы-
ражает искомые зависимости радиуса шарика r от количе-
ства закачанного внутрь газа и зависимость давления газа
от его массы.
О т в е т. Радиус шара линейно зависит от массы зака-
ченного внутрь газа,

3mRT
r(m) = r0 + .
8Epmd0 r02

Давление внутри шара зависит от массы закаченного газа


как
 −3
3mRT 3mRT
p(m) = r0 + 2 .
4pm 8Epmd0 r0

Ниже, на графиках показан качественный вид обеих зави-


симостей.
350. При решении этой задачи необходимо учесть на-
личие дополнительного давления, создаваемого под коло-
колом насыщенными парами воды. Если зимой вклад это-
го давления пренебрежимо мал, то летом (при температуре
воды 100 ◦ C) он оказывается равным атмосферному давле-
нию на Земле. Пузыри начнут выходить из-под колокола
в тот момент, когда смесь газа и насыщенных паров воды
438 Решения задач

К решению задачи 349

займёт всё пространство под ним. При этом его суммар-


ное давление должно равняться давлению воды на глубине
h, p0 + p2 = r · gh + p A . Перед погружением колокола весь его
внутренний объём был занят атмосферным газом, поэтому
справедливо будет соотношение p A = p1 . Поскольку объём
под колоколом практически не изменяется, для парциаль-
ного давления находящегося под ним газом Олимптурии
выполняется закон Шарля, p2 /T2 = p1 /T1 .
О т в е т. Атмосферное давление на планете равно p A = (rgh − p0 ) · T1 /(T2 − T1 )
351. В условии задачи требуется проанализировать тер-
модинамический процесс происходящий при разгерметиза-
ции баллона со сжатым воздухом, накрытым тяжёлым коло-
колом. Основная техническая сложность состоит в вычисле-
нии количества газа, покинувшее колокол. Помочь в реше-
нии может условие на предельно допустимое давление газа,
который может находиться под колоколом. В остальном, ре-
шение задачи сводиться к записи уравнения состояния для
идеального газа. Исходное число молей воздуха под коло-
колом равно v0 = P0 (V − V  )/RT, здесь V  — объём баллона,
а количество молей газа в баллоне равно v = P V  /RT. Чис-
ло молей воздуха вне баллона под колоколом в момент, ко-
гда из-под него начал выходить воздух (по условию в этот
момент давление в баллоне упало в 2 раза) очевидно рав-
но v = v0 + v /2. Давление под колоколом в момент начала
выхода воздуха таково, что сжатый воздух приподнимает
поршень P = 4Mg/pD2 + P0 . Анализируя последнее равен-
ство, приходим к выводу, что все параметры, необходимые
Решения задач городских туров 439

для записи уравнений состояния известны. Уравнение со-


стояния газа в момент начала его выхода из-под колокола
есть v = P (V − V  )/RT. Число молей газа, оставшихся под
колоколом после окончания всех процессов, по условию за-
дачи, есть v = 1,01 · P0 V /RT. Число молей газа, вышедших
из-под колокола тогда даётся формулой v0 + v − v = m/m.
Подстановка в последнее равенство уравнений состояния
идеального газа и решение этой системы приводят к ис-
комой величине давления. Дополнительно, следует учесть
естественное ограничение на параметры колокола, допуска-
ющие нахождение под ним газа с заданным давлением.
О т в е т. Начальное давление в баллоне было равно:
 
b(V − V  ) V (b + P0 ) mRT 8Mg
P =  = b − a − 1 , a= + 0,001 · P0 V , b = 2 .
V bV m pD

352. После того, как сосуд повернули в первый раз,


поршень начал совершать затухающие колебания и в кон-
це концов остановился на расстоянии Dx от своего перво-
начального положения. При этом потенциальная энергия
поршня перешла во внутреннюю энергию газа: Mg · Dx = 3vR · DT/2,
где R — универсальная газовая постоянная; DT — изме-
нение температуры после первого поворота сосуда. Нетруд-
но понять, что в результате второго поворота сосуда темпе-
ратура газа не изменится: работа, совершённая газом над
поршнем против силы трения перейдёт за счёт трения об-
ратно во внутреннюю энергию газа. Таким образом, DT из
предыдущей формулы совпадает с T − T0 . Несложно выра-
зить интересующее нас X. Наличие в системе силы тре-
ния затрудняет решение задачи непосредственно с помо-
щью уравнения состояния идеального газа. Для поршня в
системе существует целая «зона застоя», и определить, где
конкретно остановится в этой зоне поршень, когда его ко-
лебания затухнут, сложно. Но это уравнение существенно
для проверки согласованности заданных в задаче условий.
Можно проверить, что полученное X соответствует равно-
весному положению поршня в сосуде. Действительно, поло-
жение равновесия поршня (после первого переворачивания
сосуда) определяется условием (p1 − p2 ) · S = Mg ± fТР , где p1
и p2 — равновесные давления под поршнем и над поршнем
соответственно, S — площадь сечения сосуда, fТР < FТР —
440 Решения задач

сила трения, действующая на поршень в равновесии (оче-


видно, она может быть направлена в любую сторону и не
превышает по модулю силу трения скольжения FТР ). Для
газа под поршнем в равновесии справедливо уравнение со-
стояния p1 · (L − X) · S = v · RT, откуда получаем p1 . Анало-
гично для газа над поршнем p2 · XS = v · RT, откуда выража-
ем p2 . Подставляя p1 и p2 в уравнение, выражающее равно-
весие поршня, с учётом вычисленного значения X, выража-
ем fТР . Тогда условие fТР < FТР , когда задача имеет решение
есть:
12 · v2 R2 (T − T0 )T FТР
2 2 2
L M g − 9v R (T − T0 )
2 2 2 − 1 ≤
Mg
.
L L 3v · R(T − T0 )
О т в е т. Равновесное положение поршня есть X = + Dx = + .
2 2 2Mg
353. Обозначим через V = 3 · 10−3 м3 объём контейнера,
n = 9/18 = 0,5 моль — количество вещества воды в контейне-
ре, TH = 273 К — начальную температуру, T — конечную
температуру системы, p — парциальное давление пара по-
сле того, как вся вода превратилась в пар, p1 = 1 атм — пар-
циальное давление гелия при температуре TH , p2 — пар-
циальное давление гелия при температуре T. Вода в кон-
тейнере испаряется, пока парциальное давление пара над
жидкостью меньше давления насыщенных паров при дан-
ной температуре. График в условии задачи одновременно
представляет зависимость температуры насыщенных паров
воды от давления насыщенных паров, так как именно усло-
вие равенства внешнего давления давлению насыщенных
паров воды при данной температуре является условием ки-
пения. Когда вся вода испарилась, парциальное давление
паров воды определяется уравнением pV = nRT. Подставляя
численные значения, получаем уравнение прямой, выходя-
щей из точки с координатами p = 0, T = −273 ◦ C и пересе-
кающей график температуры от давления насыщенных па-
ров в точке A (см. рис.). Точка A определяет минимальную
температуру, при которой вся вода системы может суще-
ствовать в виде паров, что составляет примерно 158 ◦ C. Ки-
пение происходит, если давление насыщенных паров воды
при данной температуре равно давлению газа над жидко-
стью. Это давление складывается из парциального давления
воды и парциального давления гелия p2 , который нагрева-
Решения задач городских туров 441

К решению задачи 353

ется изохорически от температуры TH до конечной темпера-


туры T. Для гелия в контейнере справедлив закон Шарля:
p1 /TH = p2 /T, и полное давление газа системы, когда вся
вода испарилась, равно pn = nRT/V + p1 T/TH . Подставляя
сюда все численные значения, получим прямую, выходя-
щую из точки с координатами p = 0, T = −273 ◦ C и пересека-
ющей график температуры от давления насыщенных паров
в точке B. Точка B определяет минимальную температуру,
при которой вся вода системы выкипит — примерно 168 ◦ C.
Примечание. Чтобы вода именно выкипела, система
должна быть доведена до 168 ◦ C достаточно быстро, чтобы
вода не успела вся испариться при более низкой температу-
ре, так и не закипев.
О т в е т. Чтобы вода испарилась, контейнер надо на-
греть примерно до 158 ◦ C, а чтобы выкипела — до 168 ◦ C.
354. Обозначим количество вещества газа в отсеке с пру-
жиной n1 , а в оставшейся части сосуда n 2 . Напишем урав-
нения состояния для каждого отсека с газом в начале про-
цесса, paV = n1 RT и p(1 − a)V = n2 RT. После того, как отсек
с пружиной нагрелся до искомой температуры T  , давление
в отсеках изменилось. Обозначим давление в отсеке с пру-
жиной p1 , а в оставшейся части сосуда p2 . Уравнения состо-
яния примут вид: p2aV = n1 RT и p(1 − 2a)V = n2 RT. Кроме
того, разность давлений в отсеках обеспечивается силой на-
тяжения пружины: p1 = p2 + k · Dx/S. С другой стороны, рас-
тяжение пружины Dx связано с увеличением объёма отсека
с пружиной на величину aV , Dx = aV /S. Дальше требуется
442 Решения задач

только решить полученную систему уравнений, чтобы най-


ти требуемую температуру.
О т в е т. Чтобы объём отсека увеличился вдвое, его надо
нагреть до температуры
 
 1−a kaV
T = 2T + 2 .
1 − 2a pS

355. Когда труба разгоняется, проводящий контур, рас-


положенный на ней, всё быстрее раскручивается во внеш-
нем магнитном поле. При этом возникает ЭДС индукции,
пропорциональная скорости изменения магнитного потока
через контур. Через контур течёт ток, вследствие чего в про-
воднике выделяется тепло. Чем быстрее крутится контур,
тем быстрее меняется поток магнитного поля через кон-
тур, тем большее ЭДС возникает в системе и тем большее
тепло выделяется в контуре. Всё меньшая часть потенци-
альной энергии гравитационного поля переходит в кине-
тическую энергию, так как всё больше энергии переходит
в тепло. В некоторый момент времени труба вообще пе-
рестаёт разгоняться — устанавливается некоторая средняя
скорость V движения трубы. Рассмотрим этот «установив-
шийся» режим. Пусть в некоторый момент контур образует
угол f с горизонтом (см. рис.). Поток внешнего магнитно-
го поля через контур при этом равен Ф = BLD cos f. Пред-
положим, изменение скорости движения трубы по сравне-
нию со средней мало. Тогда можно ввести угловую ско-
рость трубы w = 2V /D и выразить через неё зависимость уг-
ла f от времени: f = wt. Возникающая ЭДС индукции при
этом будет равна, ε = −dФ/dt = BLDw sin(tw), т. е. в кон-
туре возникает переменное синусоидальное напряжение.
Джоулево тепло, выделяющееся в системе в единицу вре-
мени будет при этом равно W (t) = ε2 /R. За один оборот
трубы при этом выделится тепло Q = DtB2 L2 D2 w2 /2R, где
Dt = pD/V — период обращения трубы. Этот ответ можно
получить, вспомнив определение эффективного напряже-
ния синусоидального источника и выделяющейся на нём
мощности, либо непосредственно найдя среднее значение
квадрата синуса за период, воспользовавшись представле-
1 − cos 2wt
нием sin2 wt = , и сообразив, что первое слагаемое
2
Решения задач городских туров 443

(1/2) не меняется со временем, а среднее значение косинуса


за период равно нулю. За один оборот труба опустится на
расстояние pD sin a. Соответствующая этому потенциаль-
ная энергия mgpD sin a переходит в джоулево тепло. Отсю-
pB2 L2 D3 w2
да следует равенство mgpD sin a = Q = , из которо-
2RV
Rmg sin a
го и выражаем установившуюся скорость V = . За-
2B2 L2
дача, допускает более аккуратное решение, требующее, од-
нако, хорошего владения математическим аппаратом. Дей-
ствительно, откажемся от предположения о малости изме-
нений скорости трубы в сравнении со средней величиной.
Тогда, считая теперь функцию f(t) нелинейной, а величи-
ну угловой скорости w и скорости V зависящими от вре-
мени. Выражение для ЭДС индукции примет при этом вид
dФ d(BLD cos f)
ε=− =− = BLDw(t) sin f(t), а джоулево тепло,
dt dt
ε2 B2 L2 D2 w2 sin2 f(t)
выделяющееся в единицу времени, W (t) = = .
R R
Потенциальная энергия трубы P = mgh переходит в ки-
нетическую энергию трубы K = mV 2 , а также в тепло,
dP dK dP dK dV
+ + W = 0. С учётом = −mgV sin a и = 2mV
dt dt dt dt dt
dV B2 L2 D2 w2 sin2 f
получим уравнение −mgV sin a + 2mV + = 0,
dt R
которое легко переписать, выразив V через w (с учётом
Dw/2), и разделив на wD, в виде

mg sin a mD dw B2 L2 D2 w2 sin2 f
− + + = 0.
2 2 dt R

Проинтегрируем это уравнение на временном интервале


[0, nDt] (за время, когда труба, двигаясь в стационарном
режиме, совершила большое число n оборотов). При этом
в установившемся режиме второе слагаемое можно отбро-
 dw
nDt
сить (так как оно пропорционально dt = w(nDt) − w(0),
dt
0
и при большом n становится мало по сравнению с остальны-
ми вкладами, пропорциональными, как мы увидим, числу
444 Решения задач

К решению задачи 355

оборотов n). Это приводит к уравнению

2 2
nDt 2 2
2pn
mg sin anDt B L D df B L D B2 L2 Dpn
= sin2 f dt= sin2 f df= ,
2 R dt R R
0 0

в котором мы для правой части выполнили уже рассмотрен-


ное усреднение по углу f величины sin2 f. Отсюда период
2B2 L2 Dp
оборота колеса Dt = , что после использования оче-
mgR sin a
pD
видной формулы Vср = , даёт приведённый в предыдущем
Dt
пункте ответ.
Rmg sin a
О т в е т. Установившаяся скорость трубы равна V = .
2B2 L2
356. Обозначим через E1 , E2 напряжённости, которые
квадрат создаёт в точках, где расположены точечные заря-
ды. Тогда на заряды действует силы q0 E1 , q0 E2 . Очевидно,
искомое ускорение при этом будет зависеть только от про-
екций сил, действующих на рычаг на направление OY, пер-
пендикулярное рычагу. Поскольку эти проекции раскручи-
вают рычаг в противоположные стороны, ускорение заря-
дов равно a1 = q0 (E1y − E2y )/(2m). Задача свелась к нахож-
дению величины (E1y − E2y ). Вместо того, чтобы искать про-
екцию напряжённости E1y в точке 1, можно сдвинуть заря-
женный квадрат на r/2 вправо и искать напряжённость в
центре рычага, в точке 3 (рис. 1). Аналогично, вместо то-
го, чтобы искать величину −E2y в точке 2, можно сдвинуть
заряженный квадрат на r/2 влево, изменить знак его заря-
да на противоположный и искать напряжённость в центре
рычага, в точке 3. Таким образом, поле системы из двух
Решения задач городских туров 445

противоположно заряженных квадратов, сдвинутых относи-


тельно исходного квадрата вправо и влево на r/2, создаёт
→ −
− → − →
в точке 3 напряжённость E = ( E 1 − E 2 ), проекцию которой
на ось OY мы и будем теперь искать. Итак, требуется найти
напряжённость, создаваемую системой из двух противопо-
ложно заряженных квадратов, сдвинутых друг относитель-
но друга на расстояние r. Понятно, что там, где квадраты
перекрываются, они компенсируют заряды друг друга, по-
этому достаточно найти в точке 3 напряжённость системы
двух узких (r  a) полосок (см. заштрихованные полоски на
рис. 2). Сдвинемся от точки 3 вверх и вниз на малое рассто-
яние x/2. Обозначим потенциал электрического поля в этих
точках fВВЕРХ и fНИЗ . Поскольку расстояние между полу-
чившимися точками мало по сравнению с размерами систе-
мы, электрическое поле между этими точками можно счи-
тать однородным, и его проекцию на ось OY можно выра-
зить через потенциалы по формуле Ey = (fВВЕРХ − fНИЗ )/x.
С другой стороны, рассуждая как в предыдущем пункте,
можно показать, что разность потенциалов (fВВЕРХ − fНИЗ )
можно найти, рассмотрев систему, состоящую из двух по-
лосок, сдвинутых относительно исходных вверх на x/2, и
двух полосок, имеющих заряды противоположные исход-
ным и сдвинутых относительно исходных полосок вниз на
x/2. Вычисление потенциала такой системы в точке 3 даёт
в точности разность (fВВЕРХ − fНИЗ ). Сдвинутые друг отно-
сительно друга противоположно заряженные полоски так-
же компенсируют потенциалы друг друга в области, где они
пересекаются, так что достаточно считать потенциал систе-
мы, состоящей из 4 маленьких прямоугольников шириной
r и высотой x (см. рис. 3). При этом плотность заряда пря-
моугольников такая же, как у исходного квадрата s = Q/a2 ,
левый верхний и правый нижний прямоугольники на рис. 3
заряжены положительно, а остальные — отрицательно. По-
скольку и r и x малы относительно a (r по условию задачи, x
был выбран малым нами в ходе решения), прямоугольнич-
ки можно заменить на точечный заряды, равные по модулю
q = srx = Qrx/a2 . Потенциал в точке 3, который эти заряды
446 Решения задач

К решению задачи 356

К решению задачи 356

будут создавать, равен, очевидно


kq kq kq
(fВВЕРХ − fНИЗ ) = 2 √ − √ −√ .
5a 2 2a 2a

Отсюда находим проекцию напряжённости поля вдоль оси


OY,  
(fВВЕРХ − fНИЗ ) kQr 2 3
Ey = = 3 √ − √ ,
x a 5 2 2
и ускорение в первый момент времени.
О т в е т. Ускорение зарядов в момент, когда рычаг отпу-
стили, равно
 
q0 E y kq0 Qr 1 3
a= = 3
√ − √ .
2m ma 5 4 2

357. Катушка покатится вправо. Мгновенная ось вра-


щения катушки — точка соприкосновения со столом. Вид-
но, что сила натяжения провода создаёт момент враще-
ния, поворачивающий катушку относительно этой точ-
ки по часовой стрелке. В результате движения катушки
площадь проводящего контура, образованного проводом и
Решения задач городских туров 447

К решению задачи 356

вольтметром, будет меняться. Поэтому в контуре появит-


ся ЭДС индукции, которую и покажет вольтметр. За поло-
жительное направление контура выберем направление об-
хода по часовой стрелке. Рассмотрим катушку через время
Dt = 2p(r1 − r2 )/V , т. е. когда она совершит один оборот во-
круг своей оси. При движении провод на внешнем радиусе
катушки будет сматываться, а на внутреннем наматывать-
ся, что приведёт к изменению площади всего контура на
DS1 = p(r12 − r22 ). Так как катушка намотана против часовой
стрелки (а за положительное направление обхода контура
мы выбрали направление по часовой стрелке), такое умень-
шение площади приведёт к увеличению магнитного потока
через контур на DФ1 = BDS1 . За счёт того, что катушка сдви-
нулась на 2pr1 вправо, площадь всего контура уменьшится
на величину площади прямоугольника со сторонами 2pr1 и
r1 − r2 , что приведёт к уменьшению площади всего конту-
ра на DS2 = 2p(r12 − r1 r2 ). Соответствующее уменьшение маг-
нитного потока через контур DФ2 = BDS2 . ЭДС индукции
равна −(DФ1 − DФ2 )/Dt.
О т в е т. Вольтметр показывает ЭДС индукции BV (r1 − r2 )/2.
358. Пусть в некоторый момент один из концов труб-
ки располагается в области с полем на расстоянии x от
границы области, а второй конец — в области без поля.
Когда 0 < x < l, на трубку действует выталкивающая сила
F = −kx, где мы ввели коэффициент k = QE/l. Если бы та-
кая сила действовала бы на трубку всегда (например, при
L = l и если колебания не слишком сильные), то она дви-
галась бы словно под действием пружинки жёсткостью k,
т. е. совершала бы гармонические колебания с периодом, не
448 Решения задач

зависящим от амплитуды. Введём максимальную длину X,


на которую трубка заходит в область с полем. Она связа-
на с амплитудой колебаний A соотношением 2A + l = L + 2X
(см. рис). Предположим, x всегда меньше l. Это верно, ес-
ли X < l, т. е. если амплитуда колебаний удовлетворяет со-
отношению A < (L + l)/2. Пусть трубка находится в крайнем
правом положении. Тогда, в течение времени T0 /4, она бу-
дет двигаться под действием возвращающей силы F, а за-
тем, когда x обратится в 0, попадёт в область без поля. Обо-
значим V скорость, которую она наберёт к этому моменту.
В течение времени (L − l)/V трубка будет двигаться равно-
мерно, а затем снова попадёт в область с полем. Ещё че-
рез время T0 /4 трубка окажется в крайнем левом положе-
нии и начнёт движение вправо. Пробыв всего в левой обла-
сти с полем время T0 /2, трубка снова вылетит в среднюю
область. Таким образом, её период колебаний будет равен
T = T0 + 2(L − l)/V . Скорость V легко найти из закона сохра-
нения энергии, так как это максимальная скорость движе-
ния трубки, а значит вся энергия колебаний kX 2 /2 сосре-
доточена в момент её достижения в кинетической энергии
трубки mV 2 /2. Отсюда получаем V = X(k/m)1/2 . Тогда пери-
од колебаний трубки равен

 
m L−l L+l
T= 2p + 2 , A< .
k A + l/2 − L/2 2

Пусть теперь трубка иногда целиком влетает в область с по-


лем (x иногда больше l). Пусть снова X — максимальное
смещение трубки в область с полем (теперь X > l). Движение
трубки теперь надо разбивать на 3 этапа: равноускоренное
движение, когда трубка целиком расположена в области с
полем; движение, когда 0 < x < l и работает возвращающая
сила; равномерное движение внутри области без поля. Рас-
смотрим начальное положение трубки в крайнем правом по-
ложении (она полностью располагается в области с полем) и
найдём момент t, когда x = l (когда трубка левым концом до-
стигнет края области поля). Всё это время трубка движется
равноускоренно под действием силы kl, так что путь X − l
Решения задач городских туров 449

она проходит за время



2m(X − l)
t= ,
kl

приобретая к этому моменту скорость u = kl/mt. За это же


время работа сил поля равна kl(X − l). На втором этапе дви-
жения, пока трубка не покинула области поля, она ещё бо-
лее разгоняется — силы поля совершают при этом работу
kl2 /2. Вся работа сил поля превращается в кинетическую
энергию трубки к моменту, когда она покидает пределы об-
ласти с полем. В этот момент её скорость принимает своё
максимальное значение

2kl(X − l/2)
U= .
m

Движение трубки гармоническое, значит должно существо-


вать такое время t1 , что U cos(t1 k/m) = u. В области, где
поля нет, трубка движется с постоянной скоростью U, пре-
одолевая её за время (L − l)/U. Общий период колебаний
трубки складывается из 4 промежутков времени t, 4 проме-
жутков t1 и удвоенного времени (L − l)/U. Для получения
окончательного ответа просуммируем эти выражения и ис-
пользуем связь между X и A.
О т в е т. Период колебаний трубки вдоль нити равен
⎧ m  L−l

L+l

⎪ 2p + 2 , A < ,

⎪ k + l/2 − L/2
A 2

⎪  

⎪  A− l − L √

arccos 
m m
4 2 2
+ 2(L − 1) “ L +

T= k A −
L
kl A −



⎪  “ ”
2 2

⎪ l L

⎪ √ m A− − L+l
⎩+4 2 2 2
, A> .
kl 2

Качественный характер зависимости периода (обезразме-


ренного) колебаний трубки от амплитуды показан на графи-
ке. При маленьких амплитудах зависимость периода от ам-
плитуды гиперболическая. При очень больших амплитудах
450 Решения задач

К решению задачи 358

К решению задачи 358


Решения задач городских туров 451

период растёт как корень из амплитуды. Абсолютный ми-


нимум периода колебаний достигается, когда A = (L + l)/2.
359. Условно можно считать, что металл состоит из
положительно заряженных ионов, расположенных в узлах
неподвижной кристаллической решётки, и обобществлён-
ных электронов, которые практически свободно перемеща-
ются внутри металла. Поскольку первоначально проволоч-
ный каркас не был заряжен, то если поместить его в элек-
трическое поле, сумма зарядов на всех ребрах останется
равной нулю: электроны перераспределятся в металле, так
что в некоторых местах будет избыток электронов (там ин-
дуцируется отрицательный заряд), а в некоторых местах
каркаса будет недостаток электронов по сравнению с коли-
чеством положительно заряженных ионов (там индуцирует-
ся положительный заряд). Электроны будут перемещаться в
металле до тех пор, пока на каждый электрон не прекратит
действовать внешняя сила. Это происходит, потому что соб-
ственное поле, которое порождают индуцированные заряды
в металле, полностью компенсирует внешнее поле, в кото-
рое поместили проводник. Таким образом, напряжённость
электрического поля в металле обращается в нуль. Рассмот-
рим сначала каркас в поле, параллельном ребру AD. По-
скольку система симметрична относительно вертикальной
плоскости, проходящей через вершину O и параллельной
AD, понятно что заряды на ребрах OCC и OD равны между
собой (и равны q2 ). Аналогично равны заряды рёбер OA и
OB. Докажем, что заряд ребра AB q AB = −q1 . Рассмотрим
ситуацию, когда каркас поместили в поле E, направленное
вдоль DA (т. е. направленное противоположно исходному).
Понятно, что на ребре AB теперь индуцируется заряд q1 .
Теперь рассмотрим систему, когда одновременно включено
поле E, направленное вдоль AD, и поле E, направленное
вдоль DA, так что полное поле в системе отсутствует. Со-
гласно принципу суперпозиции полный заряд на ребре AB
в этом случае равен q AB + q1 . С другой стороны, так как на
самом деле в такой системе поля нет, этот заряд должен
быть равен нулю, откуда и следует, что q AB = −q1 . Анало-
гично доказывается, что заряды рёбер OB и OA равны −q2 .
Так как полный заряд каркаса равен нулю, а заряды рёбер
OA, OB, OC, OD, AB, DC в сумме дают ноль, заряды рёбер
452 Решения задач

AD и BC также должны в сумме давать нуль. С другой сто-


роны, заряды в металле перераспределяются только вдоль
направления AD (чтобы компенсировать внешнее поле, па-
раллельное AD). Вдоль направления AB заряды в каркасе
не перераспределятся — это создало бы внутри металла по-
ле вдоль AB, которому не чем было бы компенсироваться.
Значит, заряды и каждого из рёбер AD и BC равны нулю.
Итак, в случае поля, направленного вдоль AD, распределе-
ние зарядов по каркасу изображено на рисунке. Рассмотрим
теперь поле E, направленное вдоль AC. Его можно рассмат-

ривать, как сумму двух полей напряжённостью E1 = E/ 2
направленных вдоль AD и AB. Так как величина индуциро-
ванного заряда прямо пропорциональна силе внешнего по-
ля q ∼ E в случае когда включено только поле E1 вдоль AD,
на ребрах индуцируются следующие заряды:
ребро OA√ OB√ OC √ OD √ AB √ BC CD √ DA
заряд −q2 / 2 −q2 / 2 q2 / 2 q2 / 2 q1 / 2 0 q1 / 2 0
В случае когда включено только поле E1 вдоль AB, на
ребрах индуцируются следующие заряды:
ребро OA√ OB √ OC √ OD√ AB BC √ CD DA√
заряд −q2 / 2 q2 / 2 q2 / 2 −q2 / 2 0 q1 / 2 0 −q1 / 2
Когда включена сумма этих полей, индуцированные за-
ряды также равны сумме зарядов, индуцированных каж-
дым полем:
ребро OA√ OB OC √ OD AB√ BC √ CD √ DA√
заряд −q2 / 2 0 q2 / 2 0 −q1 / 2 q1 / 2 q1 / 2 −q1 / 2
360. Если поместить конденсатор в магнитное поле, пер-
пендикулярное плоскости рисунка, то траектории электро-
нов будут искривляться силой Лоренца. Если поле окажет-
ся достаточно сильным, то электрон может и не достиг-
нуть противоположной пластины конденсатора. Рассмот-
рим электрон в некоторой точке его траектории. Запишем
второй закон Ньютона в проекции на оси OX и OY:
eE − eBVY = maX , eBVX = maY .
Решения задач городских туров 453

К решению задачи 359

Обозначения стандартные, направление магнитного поля


указано на рисунке. Последние уравнения справедливы для
каждого момента времени и могут быть обобщены для ко-
нечных интервалов времени. Например, из последнего ра-
венства следует, что eB · x = mVY . Если к моменту времени,
когда электрон пролетел вдоль оси OX расстояние d, про-
екция его скорости на ось OX обратилась в ноль, то d —
максимальное удаление электрона от левой пластины, а со-
ответствующее BП — предельное значение магнитного по-
ля, при котором электрон ещё долетает до правой пластины.
Когда x = d, кинетическая энергия электрона оказывается
больше первоначальной за счёт уменьшения потенциальной
энергии электрона: mV 2 /2 = eU. Поскольку в крайней точке
траектории V = VY , это позволяет оценить предельное зна-
чение магнитного поля.
О т в е т. Минимальное однородное магнитное поле, за-
пирающее ток в плоском конденсаторе равно

1 2Um
BП = .
d e

361. Если в системе установилось равновесие, когда


заряды образовали плоскую конфигурацию, изображённую
на рис. 1, то их взаимное расположение однозначно задаёт-
ся углом a. Пронумеруем заряды как показано на рисунке.
Обозначим силу электростатического взаимодействия меж-
ду i-м и j-м зарядами Fij . Тогда условие неподвижности вто-
рого заряда запишется в виде:

F12 · cos a = F23 + F24 .


454 Решения задач

К решению задачи 360

Выпишем выражения для сил:

k · Qq k · q2 k · q2 1
F12 = , F23 = , F24 = , k≡ ,
(H/ sin a)2 (H · ctg a) (2H · ctg a)2 4p · ε0

и найдём cos a = (5q/4Q)1/3 . Такая конфигурация равновес-


на, если 5q ≤ 4Q. Однако, возможна и другая — тетраэд-
ральная конфигурация зарядов, изображённая на рис. 2.
Она однозначно задаётся углом b.
Условие равновесия электростатических сил запишется
для неё в виде F12 · cos b = F23 · cos p/6 + F24 · cos p/6, где

k · Qq k · q2
F12 = , F23 = F24 = ,
(H/ sin b)2 3H2 · ctg2 b

что позволяет найти угол cos b = (q/Q 3)1/3 . √
Очевидно, что конфигурация устойчива, если q/Q ≤ 3.
Наконец, возможно такое равновесие, когда два из зарядов
q улетели бесконечно далеко, а третий заряд расположился
непосредственно под Q. В действительности, реализуется та
конфигурация, которая доставляет системе минимум энер-
гии. В третьем случае, когда два заряда находятся на беско-
нечности, энергия системы равна E0 = −k · qQ/H. В первом
случае (плоская конфигурация, см. рис. 1) энергия системы
E1 есть сумма энергий парных электростатических взаимо-
Решения задач городских туров 455

действий:
U U k · q2
U13 = −E0 = 12 = 14 , U23 = U34 = 2 · U13 = .
sin a sin a H · ctg a

В результате суммирования вкладов получаем E1 = −E0 /2Q · (5q · tg a − 4Q · sin a


где a — угол, соответствующий равновесной конфигурации.
В случае тетраэдрической конфигурации зарядов (рис. 2)
энергии парных взаимодействий суть:

W12 = W13 = W14 = −E0 · sin b, W24 = W23 = W34 = k · q2 /( 3H · ctg b).

Их сумма — E2 = −E0 · ( 3 · tg b − 3Q · sin b), где угол b даёт
равновесную конфигурацию зарядов. На рис. 3 мы постро-
или зависимости энергий E1 и E2 (нормированных на E0 )
от отношения зарядов q/Q. Из графика видно, что при ма-
лых q/Q меньшей энергией обладает вторая конфигурация
(пунктирная линия), она и оказывается более устойчивой
(конфигурации с энергией E0 соответствовала бы горизон-
тальная прямая E = 1). Если q/Q > 0,716, первая конфигу-
рация может стать более энергетически выгодной, чем вто-
рая, однако, она не может реализоваться, если √ q/Q > 4/5
(вертикальная линия на рис. 3). Когда q/Q > 3, распада-
ется и вторая конфигурация. Устойчивой при больших q/Q
становится третья конфигурация, с двумя зарядами на бес-
конечности.
О т в е т. При Q  q заряды расположатся в√вершинах
тетраэдра с углом при основании b = arccos([q/Q 3]1/3 ). Ко-
гда 0,716 · Q < q < 0,8 · Q, заряды расположатся в плоскости,
как это показано на рис. 281. Угол √при основании тре-
√ будет равен a = arccos([5q/4 3] ). Наконец, ко-
угольника 1/3

гда q > 3 · Q, два заряда q уйдут на бесконечность, а тре-


тий — расположится непосредственно под Q.
362. Работа батареи есть A = q · e = c · ε2 . Одна полови-
на этой работы идёт на заряд конденсатора, а вторая —
выделяется в виде тепла. Тепло выделяется на резисто-
ре R, пока конденсатор заряжается Q1 = A/2 · R/(R + r),
а при разрядке — вся запасённая в конденсаторе энер-
гия уходит в тепло: Q2 = A/2. В этом случае КПД ра-
вен h1 = (Q1 + Q2 )/A = (2R + r)/2(R + r). Если напрямую за-
456 Решения задач

К решению задачи 361

К решению задачи 361

К решению задачи 361


Решения задач городских туров 457

К решению задачи 363

мкнуть батарею на сопротивление, то КПД h2 = R/(R + r).


Отношение КПД даёт ответ.
О т в е т. Отношение КПД двух схем равно h1 /h2 = 1 + r/2R > 1.
363. Изменение кинетической энергии определяется со-
вокупной работой гравитационного и электрического полей,
направленная перпендикулярно скорости сила Лоренца ра-
боты не совершает. Рассматривая траекторию движения на
её вертикальном участке (см. рисунок), легко придти к вы-
воду о том, что заряд частицы отрицателен, и электроста-
тическая сила направлена в ту же сторону, что и сила тя-
жести. В этом случае скорость частицы будет минимальна
в верхней точке траектории и максимальна — в нижней.
Закон сохранения энергии
mv2max mv2min
− = (mg + |q|E)h
2 2

позволяет найти q = −m/E · (3v20 /2h − g). Требование отрица-


тельности заряда приводит к неравенству v20 > 2g · h/3, при
нарушении которого траектория имела бы вид, качествен-
но отличающийся от изображённого на рисунке в условии
задачи.
О т в е т. Заряд частицы равен q = −m/E · (3v20 /2h − g).
364. Небольшой заряд Dq на расстоянии R от точки x со-
здаёт в ней электрический потенциал Df = Dq/4pεε0 R. При
этом индукция магнитного поля в точке x от этого же эле-
мента равна DB = mm0 · Dq · V /4pR2 , где V — скорость, с ко-
торой двигается рассматриваемый элемент. Сравнивая эти
выражения, с учётом V = R · w, получаем: DB = mm0 εε0 w · Df.
При этом, по правилу буравчика, вектор DB направлен па-
раллельно оси вращения. Последнее соотношение справед-
ливо для любого элемента, составляющего наше тело. Пол-
ный потенциал, создаваемый телом в точке x, равен сум-
458 Решения задач

ме потенциалов, создаваемых всеми кусочками. То же спра-


ведливо для модуля напряжённости магнитного поля (каж-
дый из элементов, образующих тело, создаёт магнитное по-
ле, направленное вдоль одной и той же оси). Мы можем за-
писать аналогичное соотношение для каждого кусочка рас-
сматриваемого тела и сложить полученные уравнения.
О т в е т. Индукция магнитного поля, создаваемого те-
лом в точке x, равна B = mm0 εε0 w · f.
365. Обозначим, s = q/S — плотность заряда пластин,
d — расстояние между пластинами, x — сжатие пружи-
ны. Найдём, насколько первоначально сжата пружина. Вос-
пользуемся для этого условием, что первоначально пла-
стина конденсатора находится в равновесии под действи-
ем кулоновской силы притяжения к другой пластине и си-
лы отталкивания пружины: k · x = E · q. Напряжённость по-
ля одной пластины E = s/(2ε0 ). Поэтому в положении рав-
новесия x = q2 /2ε0 Sk. Энергия, запасённая первоначально
в конденсаторе равна Wk = q2 /(2C). С учётом выражения
для ёмкости C = ε0 S/d (где d = L − x), несложно получить:
Wk = q2 · L/2ε0 S − q4 /4ε20 S2 k. Запасённая в пружине энергия
суть Wпр = k · x2 /2 = q4 /8ε20 S2 k. При медленном разряде ме-
ханические колебания не возбуждаются, поэтому на сопро-
тивлении выделится как энергия, запасённая в конденса-
торе, так и энергия пружины W1 = Wk + Wпр . При быст-
ром разряде в виде тепла выделится только электрическая
энергия, а упругая перейдёт в колебания обкладок. Поэто-
му, в данном случае, на сопротивлении выделится энергия
W2 = W k .
О т в е т. При медленном разряде выделится
теплота q L/2ε0 S − q /8ε0 S k, а при быстром —
2 4 2 2

q2 L/2ε0 S − q4 /4ε20 S2 k.
Примечание. Скорость разряда определяется соотноше-
нием параметров задачи. Динамика процесса описывается
системой уравнений: I = U/R = E · d/R, ma = q · E − k · x, кото-
рую с учётом выражений для E, d, x можно переписать в
следующем виде:

dq L−x d2 x q2
I= = q; m 2 = −kx + .
dt Rε0 S dt 2ε0 S
Решения задач городских туров 459

Поэтому характерное время колебательного процесса tk ∼ (m/k)1/2 ,


а характерное время электрического разряда конденсатора
tk ∼ R · C. Случай tпр  tk соответствует медленному разряду,
а tпр  tk — быстрому.
366. Принцип действия приборов магнитоэлектриче-
ской системы основан на взаимодействии проводника с то-
ком и магнитного поля. Вращающий момент, действующий
на рамку, благодаря специально сконструированному маг-
ниту, не зависит от угла поворота рамки. Под действием
этого момента рамка начинает поворачиваться. Нить или
спиральная пружина, на которой подвешена рамка, при
этом закручивается, и возникает противодействующий мо-
мент, пропорциональный углу поворота рамки. Когда про-
тиводействующий момент становится равным вращающему
моменту, рамка останавливается. Угол поворота рамки при
этом оказывается прямо пропорционален току, текущему в
ней. Это обеспечивает равномерность шкалы прибора. Заме-
тим, что магнитоэлектрические приборы пригодны только
для измерения постоянного тока. В состоянии равновесия
рамки вращающий момент, |B|I · NS = Mвр , уравновешива-
ется противодействующим моментом k · a = Mвр = a · M0 /a0 .
О т в е т. Искомый угол поворота рамки равен a = a0 |B|I · NS/M0 .
367. Предлагаемая задача, в принципе, может быть ре-
шена устно, без выписывания и решения дифференциаль-
ных уравнений, определяющих малые колебания маятника.
Помочь в этом может задача о движении заряженного ша-
рика в вертикальном магнитном поле, предлагавшаяся на
одном из районных туров олимпиады. Если бы речь шла не
о маятнике, а о заряженном шарике, летающем в плоско-
сти, перпендикулярной направлению силовых линий маг-
нитного поля, ответ был бы хорошо известен: t0 = 2p · |wB |−1 ,
где wB = |q · B|/m — циклотронная частота. В чём заключа-
ется основное отличие ситуации с маятником? Прежде все-
го, колебательное движение маятника приводит к тому, что
заряженный шарик движется в поступательном направле-
нии только половину периода. Шарику, подвешенному на
нити, потребуется, таким образом, ровно в два раза больше
времени, чтобы совершить полный оборот в плоскости пер-
пендикулярной магнитному полю. Отсюда сразу получаем
правильный ответ: 2t0 = t. Однако, корректное решение за-
460 Решения задач

дачи должно включать в себя также анализ условий, при


которых колебания маятника можно действительно считать
малыми. Ниже мы приводим полное решение задачи. Нач-
нём с качественной оценки влияния магнитного поля на ко-
лебания маятника. В присутствии магнитного поля на заря-
женный шарик действует сила Лоренца, которая искривля-
ет траекторию шарика. Если сила Лоренца мала по срав-
нению с возвращающей силой, то в течение полупериода
магнитное поле будет лишь слегка искривлять траекторию
шарика. Тогда движение шарика можно представить как ко-
лебания в вертикальной плоскости, которая сама медленно
поворачивается. Выполняется ли условие малости силы Ло-
ренца? В ходе решения мы убедимся, что необходимое усло-
вие выполняется на практике с огромным запасом. Другой
интересной особенностью предлагаемой задачи является то,
что рассматриваемый колебательный процесс двумерен и
сводится к круговым колебаниям. Его динамическое описа-
ние требует двух координат. Далее, вследствие того, что ин-
дукция магнитного поля является псевдовекторной величи-
ной (её направление зависит от выбора ориентации системы
координат), вращение плоскости колебаний маятника по ча-
совой стрелке и против неё не равноправны. На примере
этой задачи мы встретимся с уникальной ситуацией, когда
круговые частоты колебательных движений маятника, соот-
ветствующие различным направлениям вращения плоско-
сти колебаний, не совпадают! Наконец, отметим глубокую
физическую аналогию рассматриваемой задачи с задачей о
маятнике Фуко. В обоих случаях плоскость качаний повора-
чивается, а вращение по и против часовой стрелки происхо-
дит с разными угловыми скоростями. Это происходит вслед-
ствие действия псевдовекторной силы: в первом случае —
это сила Лоренца, а во втором — сила Кориолиса — си-
ла инерции, связанная с переходом наблюдателя в неинер-
циальную (вращающуюся) систему отсчёта. Разъясним воз-
никновение силы Кориолиса на простом примере с маят-
ником. Представим себе, что вначале маятник расположен
точно над северным полюсом нашей планеты. Наблюдатель,
который не вращается вместе с Землёй, а сохраняет посто-
янную ориентацию относительно звёздного неба увидит, что
маятник колеблется в фиксированной плоскости с постоян-
Решения задач городских туров 461

ным периодом
 колебаний, формула которого хорошо извест-
на: T = 2p l/g. Наблюдатель, стоящий на Земле и вращаю-
щийся вместе с ней, увидит, однако, что плоскость кача-
ний маятника поворачивается по часовой стрелке со ско-
ростью 1 оборот в сутки (но, относительно звёздного неба,
плоскость остаётся неподвижна). С точки зрения последне-
го наблюдателя (поскольку он вращается вместе с Землёй)
круговые частоты маятников не совпадают: при вращении
против часовой стрелки w1 = w0 − w3 , а по часовой стрелке —
w2 = w0 + w3 . Прежде всего проверим, что сила Лоренца дей-
ствительно мала по сравнению с возвращающей силой. Вы-
полнение этого условия необходимо, чтобы задачу можно
было считать линейной. Ниже мы предъявили отношение
максимальных величин силы Лоренца и возвращающей си-
лы, 
FЛ,max qBvmax qB l
≈ =  1.
FВ,max mga m g
Величина отношения на практике очень мала из-за то-
го, что заряд шарика в реальных условиях не превосходит

= 10−8 Кл. В противном случае, поскольку радиус шари-
ка по условию очень маленький («математический маят-
ник»), потенциал шарика будет величиной порядка 104 В
(при оценке предположили, что радиус шарика равен 1 см),
после чего наступает электрический пробой воздуха. Оцен-
ка величины отношения сил, таким образом, даёт ∼
= 10−5 , и
условие малости силы Лоренца заведомо выполняется. Вве-
дём систему координат, начало которой находится в точке
подвеса (см. рис. 287). Тогда уравнение движения шарика
имеет вид:
ma = mg + T + FЛ .
С учётом малости угла отклонения нити, в линейном при-
ближении (это обеспечивается условием малости отноше-
ния сил), проекции этого уравнения на оси координат:

x = −w02 x + wВ y qB l
, где w В = и w0 = .
y = −w02 y − wВ x m g

Здесь мы ввели две круговых частоты — частоту враще-


ния свободной заряженной частицы по круговой орбите в
462 Решения задач

магнитном поле и частоту колебаний маятника в отсут-


ствии магнитного поля. Условие малости силы Лоренца, та-
ким образом, эквивалентно условию wВ  w0 . Только в этом
случае проекция векторного уравнения движения сводит-
ся к системе линейных дифференциальных уравнений. По-
следнее особенно важно, поскольку вследствие линейности
уравнений их общее решение может быть представлено в
виде суммы простых гармонических движений и, значит,
целиком находится в рамках программы средней школы.
Интересуясь гармоническими решениями системы уравне-
ний движения, подставим в них формальные решения вида
x = A · cos wt, y = A · sin wt. На классе гармонических функ-
ций процесс решения линейных дифференциальных урав-
нений второго порядка, сведётся к решению квадратного
«характеристического» уравнения w2 = w02 − wВ · w. Его кор-
ни — собственные круговые частоты вращательных колеба-
ний маятника в магнитном поле:

w wВ2 wВ
w1,2 = − 0 ± w0 +
2 ≈− ± w0 , так как wВ  w 0 .
2 4 2

Теперь, подставим найденные собственные частоты в фор-


мальные гармонические решения и выпишем их в явном
виде:

x = A · cos w1 t + A · cos w2 t = 2A · cos w0 t · cos t,
2
w
y = A · sin w1 t − A · sin |w2 t| = −2A · cos w0 t · sin В t.
2
Рассмотрим правые части последних равенств подробнее.
Очевидно, что полученные решения представляют собой
«круговые биения»: колебания маятника с частотой w0 «мо-
дулируются» процессом с более низкой частотой wВ — мед-
ленным поворотом плоскости качаний маятника вследствие
его взаимодействия с вертикально ориентированным маг-
нитным полем. Форма траекторий, которые описывает при
этом маятник зависят от отношения круговых частот в ар-
гументах гармонических функций. Если эти частоты крат-
ны друг другу, за достаточное время траектория маятни-
ка заметает некоторый симметричный многоугольник. Ес-
ли отношение частот — рациональное число, то маятник
Решения задач городских туров 463

К решению задачи 367

К решению задачи 367

заметает со временем почти весь круг радиуса 2A. Если от-


ношение иррационально — маятник, со временем, заметет
весь круг. Если частоты равны, маятник описывает окруж-
ность. Круговая частота в рассматриваемом процессе равна
wВ /2 = q · B/2m. Отсюда непосредственно следует правиль-
ный ответ.
О т в е т. Плоскость качаний маятника сделает полный
оборот за время t = 4p · m/|qB|.
368. При решении этой задачи следует догадаться о том,
что часто используемое в школьном курсе приближение ма-
териальной точки в данном случае оказывается заведомо
неприменимым (об этом свидетельствует отсутствие прямо
пропорциональной зависимости между длиной удерживаю-
щего тело каната и силой его натяжения). Стандартная в
подобной ситуации «школьная идея» — считать центр масс
тела точкой приложения всех действующих на него сил —
464 Решения задач

К решению задачи 367

не выглядит очень убедительной (какова связь между си-


лой Лоренца и центром масс) и, более того, кажется прак-
тически бесполезной (вопрос о месте расположения центра
масс половины шара лежит далеко за рамками стандартной
школьной программы). В связанной со вращающимся телом
неинерциальной системе отсчёта сумма всех приложенных
к нему сил (сила натяжения каната, приложенные к каждой
i точке тела сила инерции и сила Лоренца) должна равнять-
ся нулю:

T1 + Dmi · w2 Ri + Dqi · BwRi = 0.
i i

Проектирование этого равенства на направление, задавае-


мое канатом, и введение в явном виде его длины приводит
к выражению

T1 = Mw2 L + Dmi w2 xi + QBw · L + Dqi Bw · xi .
i i

Здесь xi — расстояние от точки крепления полусферы на


канате до текущего элемента объёма полусферы DVi , от-
считываемое по оси OX. Учёт равномерного распределения
массы и заряда по объёму полусферы позволяет упростить
полученное выражение и привести его к виду
 
1
T1 = (Mw2 + QBw) L + xi · DVi .
V
i
Решения задач городских туров 465

Выражение для силы натяжения каната после увеличения


его длины получается отсюда очевидным образом:
 
1
kT1 = (Mw2 + QBw) nL + xi · DVi .
V
i

Сопоставление двух равенств позволяет найти величину


входящей в оба равенства и обозначаемой далее через S
суммы по элементам объёма сферы:
nL + S n−k
k= ⇒S= L.
L+S k−1
Условие равенства начальной силы натяжения силе, возни-
кающей после выключения магнитного поля, проводит к
равенству:
(Mw2 + QBw)(L + S) = Mw2 (nL + S),
из которого легко определяется величина искомого вектора.
О т в е т. Величина вектора индукции магнитного поля
равна B = Mw/Q.
369. Равномерно заряженный по объёму прямоуголь-
ный параллелепипед может быть представлен как сумма
вложенных друг в друга коробок с равномерно заряженны-
ми тонкими стенками. При этом электрическое поле такой
коробки оказывается не зависящим от её размеров. Элек-
трическое поле, создаваемое коробкой с равномерно заря-
женными стенками, не зависит от её линейных размеров:
их увеличение в n раз не изменяет величины и направле-
ния элементарных вкладов в суммарное поле от бесконечно
малых элементов поверхностей стенок (обусловленный за-
коном Кулона квадратичный спад с расстоянием компенси-
руется ростом зарядов элементов поверхности):
s · DSi −

r
Ex = k 2 · i = f(b).
ri ri
i

Сумму здесь можно вычислить, не прибегая к интегрирова-


нию, исходя из данной в условии задачи напряжённости E:
DSi − →r E
W≡ k 2 · i = x.
ri ri ks
i
466 Решения задач

Создаваемое равномерно заряженным по объёму параллеле-


пипедом поле можно вычислить как сумму полей, создава-
емых составляющими этот параллелепипед коробками:
DSi − →
r E
W≡ k 2 · i = x.
ri ri ks
i

О т в е т. Создаваемое коробкой электрическое поле не


зависит от её размеров, Ex = 2s · E/ra.
370. При малой амплитуде колебаний заряд q притяги-
вается к своему отражению −q с силой F = k · q2 /4r2 . Поэто-
му колебания происходят так, словно ускорение свободного
падения увеличилось, и стало равно g + k · q2 /4mr2 . Соответ-
ственно, такие колебания будут иметь период 4pr · (m · l/4gmr2 + k · q2 )1/2 .
Заряд q и его отражение −q, передвигаясь со скоростью
V = Aw · cos(wt), создают разнонаправленные токи q · V , па-
раллельные оси x.
О т в е т. Напряжённость поля над горизонтальной ме-
таллической плоскостью в точке подвеса ортогональна к
плоскости качаний маятника и равна H = q · Aw · cos(wt)[l−2 + (l + 2r)−2 ]/4p.
371. Рассмотрим характерный элемент схемы, пред-
ставленный на рисунке. Воспользуемся формулой, свя-
зывающей напряжение и ток в катушке индуктивности:
U = L · dI/dt. Если напряжение, поданное на этот блок, —
отрицательное (слева минус, справа — плюс), ток через ди-
од не течёт. При этом ток через катушку течёт справа нале-
во (такое направление считаем отрицательным), увеличива-
ясь по модулю. Если бы в дальнейшем ток через катушку
стал бы увеличиваться, напряжение на ней стало бы поло-
жительным, диод бы открылся и закоротил катушку. Па-
дение напряжения на закороченной катушке равно нулю,
и, следовательно, ток закороченной катушки постоянен. Та-
ким образом, мы показали, что ток катушки не может уве-
личиваться. Следовательно, установившемуся режиму соот-
ветствует наименьший ток через катушку (наименьший, и,
следовательно, постоянный). При этом падение напряже-
ния на рассматриваемом блоке схемы окажется равным ну-
лю. Нулевое падение напряжения гарантирует эквипотен-
циальность точек вход-выход рассматриваемого блока, что
позволяет мысленно закоротить его в стационарном режи-
Решения задач городских туров 467

К решению задачи 371

ме. Теперь рассмотрим расположенный ниже элемент схе-


мы, состоящий из индуктивности и следующего диода. Этот
случай сводится к предыдущему (разве что ток через катуш-
ку будет теперь достигать своего максимального значения),
и для него оказывается справедливым приведённое выше
рассуждение. В рассматриваемой схеме через каждую ка-
тушку течёт (может быть свой) постоянный ток в направле-
нии, противоположном расположенному выше данной ка-
тушки диоду. Таким образом, через нижнюю катушку L1
устанавливается наибольший возможный ток (понятно, что
это значение равно I0 ), её ток складывается с переменным
и «заряжает» L2 до тока 2I0 и т. д.
О т в е т. Через амперметр течёт постоянный ток N · I0 .
372. В рассматриваемой системе неоднородность давле-
ния жидкости вызывается не только силой тяжести, но и
кулоновским взаимодействием. Очевидно, давление в жид-
кости увеличивается с глубиной благодаря силе тяжести.
Вдобавок жидкость, будучи заряженной, расталкивает са-
ма себя, в результате чего в слоях, расположенных ближе
к центру сосуда, давление меньше, чем в слоях, располо-
женных снаружи. Это приводит к наличию в жидкости до-
полнительной силы, действующей аналогично силе Архи-
меда на плавающие в ней тела и направленной к центру
сосуда. Кроме того, тела в такой жидкости будут взаимо-
действовать друг с другом (даже не будучи заряженными).
Представим область внутри шариков в виде суперпозиции
положительно заряженной жидкости (как бы проникающей
внутрь шариков) отрицательно заряженных шариков, ком-
пенсирующих этот заряд. Рассмотрим взаимодействие цело-
го заряженного большого шара радиуса R с плотностью за-
ряда s и маленьких шариков радиуса r с плотностью заряда
−s. Найдём напряжённость, создаваемую большим шаром
внутри себя на расстоянии x от своего центра. Воспользу-
емся теоремой Гаусса. Выделим мысленно сферу радиуса
468 Решения задач

x < R c центром, совпадающим с центром большого шара.


По теореме Гаусса для напряжённости поля справедливо:
E(x) · S = Q/εε0 , где Q = 4px3 s/3 — заряд, который содержит
эта сфера, S = 4px2 — площадь этой сферы. Таким образом,
если маленький шарик (с зарядом q = −4pr3 s/3) находится
на расстоянии x от центра большого шара, на него действу-
ет направленная к центру сила

4pr3 s 2
F(x) = qE(x) = x,
9εε0

что эквивалентно действию пружины с жёсткостью K = 4pr3 s 2 /9εε0 .


Итак, задача свелась к нахождению расположения шариков
с зарядами q, подвешенными на пружинках жёсткостью K
в жидкости плотностью r (см. рисунок). На каждый шарик
действует сила тяжести mg = 4pr3 r0 g/3, кулоновская сила
отталкивания от другого шарика — FKL = q2 /4pεε0 d2 , где
d — расстояние между маленькими шариками в положении
равновесия, сила Архимеда F A = 4pr3 rg/3 и сила притяже-
ния к центру большого шара Kx. Векторная сумма этих сил
должна быть равна нулю, что в проекции на вертикальную
и горизонтальную оси даёт соответственно:

4 4p · r3 s 2 q2 4p · r3 s 2
p · r3 (r0 − r)g = · x · cos j, = · x · sin j.
3 9ε · ε0 4pε · ε0 d2 9ε · ε0

Выражая x из первого уравнения, и подставляя его во вто-


рое, с учётом d = 2x cos j, имеем x = 3εε0 (r0 − r)g/s 2 cos j.
О т в е т. В зависимости от знака разности плотностей
жидкости и шариков, шарики могут расположиться как ни-
же, так и выше центра большого шара.
373. Выберем начало отсчёта времени: пусть в момент
времени t кольцо находится на расстоянии x = Vt от точ-
ки O. Рассмотрим некоторый момент времени t. При дви-
жении кольца поток магнитного поля Ф через него меня-
ется. Это приводит к возникновению в кольце ЭДС индук-
ции E = DФ/Dt и тока I = E/R. Поскольку виток такого тока
расположен на расстоянии (a2 + x2 )1/2 от точки O, он нахо-
дится в магнитном поле B = A0 /(a2 + x2 ), и на кольцо дей-
ствует сила Ампера, проекция которой на ось OO равна
Решения задач городских туров 469

К решению задачи 372

F = LBI · cos b, где L = 2pa — длина проводника. Направле-


ние силы F противоположно направлению скорости. Таким
образом,
A cos b DФ
F = LBI cos b = 2pa 20 2 2 .
(V t + a ) DtR
Осталось выяснить, как меняется ЭДС индукции в кольце со
временем, т. е. найти скорость изменения магнитного пото-
ка. Для этого рассмотрим кольцо через малый промежуток
времени Dt после момента t. За Dt кольцо сдвинулось на Dx
и изменение магнитного потока DФ равно магнитному пото-
ку через боковую поверхность цилиндра, которую «замело»
кольцо: DФ = SB · cos b, где S — площадь боковой поверхно-
сти цилиндра. Так как высота цилиндра мала, угол между
вектором магнитного поля и поверхностью цилиндра можно
считать постоянным, cos b = a/(a2 + x2 )1/2 . Окончательно,
A0 a
DФ = 2paDx √ .
x + a2
2
x2 + a2

Подставляя выражение для DФ в формулу для силы F и


учитывая, что Dx/Dt = V получим, что в момент времени на
кольцо действует сила
 2
A0 Dx A0 a 4p2 a4 A20 V
F = 2pa 2 2 2 2pa √ = 2 3.
V t +a Dt (x + a )R
2 2 2 2 x +a 2 2 R(V t + a )

О т в е т. Чтобы кольцо в каждый момент времени двига-


лось равномерно, к нему надо прикладывать силу F, предъ-
явленную выше.
470 Решения задач

374. Сферическую полость можно разбить да большое


количество маленьких, практически плоских кусочков, при-
чём каждый кусочек будет создавать своё изображение ис-
точника света. Полезно рассмотреть например пару таких
«плоских зеркал», касающихся окружности в двух проти-
воположных точках. Например, рассмотрим кусочки, каса-
тельные к которым — прямые LM и NK (см. рис. 1). Пред-
положим, источник находится в точке A. Тогда его изобра-
жения в рассматриваемых кусочках попадут в точки B и C
соответственно. Точки P и S — изображения центра зерка-
ла O в рассматриваемых кусочках. Из построения очевид-
но, что каждая из трапеций ABPO и CSOA симметрична,
и, следовательно, CSPB — параллелограмм. Поэтому, спра-
ведливо равенство: CB = SP = 4R, где R — радиус полости.
Итак, изображения источника, построенные в противопо-
ложных кусочках зеркала, всегда находятся на расстоянии
4R и лежат на одной прямой с источником. Отсюда выте-
кает метод нахождения положения источника по исходной
картинке. Необходимо взять какое-нибудь изображение X,
найти изображение, находящееся на расстоянии 4R от него
и соединить их (см. рис. 2). Так же следует поступить с дру-
гим изображением Z. На пересечении полученных прямых
находится источник света. Конечно, для каждого изобра-
жения существует два других, находящихся на расстоянии
4R от него. В результате, применение описанного метода
даёт, на самом деле, и другие положения «источника», но
все они, как легко убедиться, лежат вне полости, «за зерка-
лом», и являются мнимыми.
О т в е т. Источник света расположен на пересечении от-
резков прямых длиной 4R, соединяющих пары изображе-
ний.
375. Для построения изображения какой-нибудь точ-
ки X, принадлежащей полупрямой AB, необходимо постро-
ить в данной оптической системе ход двух лучей, вышед-
ших из точки X. В точке пересечения Y этих лучей бу-
дет находиться изображение точки X. Для любой точки X
в качестве одного из таких лучей может быть выбран луч
XAF1 ZP1 M1 N1 : луч XA, параллельный главной оптиче-
ской оси, проходит через фокус линзы F1 , попадает на зер-
кало в точке Z, отражается, попадает на линзу в точке P1 и
Решения задач городских туров 471

К решению задачи 374

К решению задачи 374


472 Решения задач

преломляется в точку M1 . Положение точки M1 определя-


ется пересечением OM1 (параллельного ZP1 ) с фокальной
плоскостью F2 M1 линзы. Так как треугольник M1 F2 O равен
треугольнику AOF1 (по трём углам и стороне F2 O = OF1 ; ра-
венство отмеченных углов см. на рис. 1), то F2 M1 = H, т. е.
точка M1 лежит ниже точки F2 на H. Итак, изображение
точки X является точкой пересечения луча XAF1 ZP1 M1 N1
и ещё какого-нибудь луча, вышедшего из X. Значит, все
действительные изображения полупрямой ABВ лежат где--
то на луче XAF1 ZP1 M1 N1 . Все изображения полупрямой
AB, возникающие в данной оптической системе, можно раз-
бить на два типа. Первый тип — изображения, возник-
шие после однократного прохождения светом линзы (уча-
сток AF1 ZP1 ). Второй тип — изображения, возникающие
в результате того, что свет, прошедший через линзу, отра-
зился от зеркала и вторично преломился в линзе (участок
P1 M1 N1 ).Обсудим изображения первого типа. Понятно, что
отрезок AF1 не содержит изображений ни одной из точек
полупрямой AB. Действительно, в качестве второго луча,
вышедшего из X, возьмём луч XO, не преломляющийся
линзой. Понятно, что он может пересечься с лучом AF1 ZP1
только ниже главной оптической оси. С другой стороны, для
любой точки Y на F1 ZP1 несложно восстановить её полу-
прямой AB. Если Y принадлежит F1 Z, то соответствующий
прообраз лежит на продолжении отрезка YO (см. рис. 1).
Если же точка Y принадлежит ZP1 , то для построения её
прообраза нужно отразить точку Y в зеркале (см. точку Y1
на рис. 2) и соединить Y1 и O. На пересечении Y1 O с AB
лежит X — прообраз точки Y. Итак, любая точка F1 ZP1
является действительным изображением какой-нибудь точ-
ки полупрямой AB. Рассмотрим теперь изображения вто-
рого типа. В качестве второго луча, вышедшего из точки
X возьмём луч проходящий через фокус F2 . Луч XF2 после
прохождения через линзу оказывается параллельным глав-
ной оптической оси, затем отражается от зеркала обратно
и возвращается «по своим следам». Значит, изображение
точки X второго типа лежит на пересечении луча P1 M1 N1
и прямой XF2 (см. рис. 3). Пользуясь указанным выше пра-
вилом построим рис. 4, на котором покажем, куда попали
изображения точек A и B (B — бесконечно удалённая точка
Решения задач городских туров 473

К решению задачи 374

полупрямой AB) — получим точки A1 , B1 соответственно.


Также восстановим прообразы точек P1 и N1 (N1 — беско-
нечно удалённая точка луча P1 M1 ), — получим точки P и
N. Комментарии к построению: OO — главная оптическая
ось, B1 лежит на главной оптической оси, F2 N параллельна
P1 N1 , изображение точки C совпадает с ней самой. Из ри-
сунка видно, что отрезок AP изображается отрезком A1 P1 ,
точки луча NB — лучом N1 B1 . Точки отрезка A1 B1 не яв-
ляются изображениями никакой из точек полупрямой AB.
Точки отрезка PN на AB не дают изображений второго ти-
па.
О т в е т. Действительные изображения полупрямой AB,
расположенной перед линзой на расстоянии H от главной
оптической оси, даётся ломаной F1 ZP1 A1 и полупрямой
B1 N1 на рис. 5.
376. Поскольку вершина A заданного треугольника рас-
положена ближе первого фокусного расстояния, собираю-
щая линза сформирует её мнимое изображение A , лежащее
от неё слева. Точки B и C расположены на двойном фокус-
ном расстоянии, их изображения будут действительными и
лежать справа от линзы на расстоянии 2F от неё по глав-
ной оптической оси. Следовательно, изображение треуголь-
ника будет разорванным и состоять из двух частей, уходя-
щих на бесконечность: действительной и мнимой. Гипоте-
нуза треугольника AB лежит на прямой, проходящей через
474 Решения задач

К решению задачи 374

К решению задачи 374


Решения задач городских туров 475

К решению задачи 374

К решению задачи 374


476 Решения задач

К решению задачи 376

центр линзы. Это означает, что изображения всех точек ги-


потенузы будут лежать на той же прямой. Поскольку катет
AC расположен параллельно главной оптической оси, все
его точки лежат на прямой, проходящей через первый фо-
кус линзы и точку пересечения его продолжения с прямой,
изображающей линзу. Таким образом, строится точка A .
Для построения оставшихся вершин заметим, что в соответ-
ствии с формулой тонкой линзы, они находятся на двойном
фокусном расстоянии.
О т в е т. Изображение треугольника будет разорванным
и состоять из двух частей, уходящих на бесконечность: дей-
ствительной и мнимой (см. рисунок).
377. Луч начинает отклоняться в сторону с момента,
как его касается пузырек (x = r). Выберем этот момент за
начало отсчёта времени. Сперва происходит полное внут-
реннее отражение.При этом угол падения равен (см. рис. 1)
a = arcsin(x/r), где x = r − Vt. Угол отклонения луча состав-
ляет j = p − 2a, что приведёт к отклонению луча на экране
вверх на расстояние
p
2L · x(t) r2 − x(t)2
X(t) = L tg j=
r2 − 2x(t)2

(здесь мы пренебрегли x и R по сравнению с L). Последнее


выражение
√ обращается
√ в бесконечность в момент времени
t1 = r( 2 − 1)/(V 2), когда угол
ПРОПУСК ТЕКСТА
Решения задач городских туров 477

К решению задачи 377

световое пятно пропадает с экрана. Когда угол падения


оказывается достаточно маленьким (n · sin a < 1) луч начи-
нает проникать внутрь пузырька и приведённое решение
перестаёт быть справедливым. Соответствующий этому мо-
мент времени: tC = (n − 1) · r/Vn. Таким образом, луч дей-
ствительно исчезает√с экрана, только если t1 < tC . Это имеет
место лишь при n > 2. В противном случае луч остаётся на
экране и в момент времени tC отклонён максимально. Начи-
ная с момента tC становятся справедливыми новые соотно-
шения для√ углов (см. рис. 2): j = p − 2(b − a), sin b = n sin a.
При n > 2 луч вернётся на экран в момент времени t2 ,
когда угол j = p/2. В результате луч на экране отклонится
вверх на расстояние X(t), которое убывает с приближени-
ем луча к центру пузырька и обращается в ноль в момент
времени t0 = r/V . После прохождения луча через центр пу-
зырька, луч отклоняется вниз и временная зависимость его
отклонения симметрична относительно момента t0 .
О т в е т. Положение световой точки на экране даётся
формулой

X(t) = L tg j = 2L · x(t)×
p p
r2 − x(t)2 (r2 − 2n2 x(t)2 ) + n r2 − n2 x(t)2 (r2 − 2x(t)2 )
× p p ,
(r2 − 2x(t)2 )(r2 − 2n2 x(t)2 ) + 4nx(t)2 r2 − n2 x(t)2 r2 − x(t)2

оно симметрично относительно момента времени t0 = r/V .


378. Из симметрии картинки очевидно, что квадрат
расположен симметрично относительно главной оптической
оси линзы OO . Так как две стороны изображения парал-
478 Решения задач

К решению задачи 377

лельны друг другу, ясно, что две стороны квадрата перпен-


дикулярны OO . Поскольку изображение по условию «рва-
ное», понятно, что фокус линзы расположен внутри квад-
рата. Построим изображение квадрата ABCD в линзе HH
с фокусами в точках F и F (см. рис. 1). Чтобы построить
изображение точки A, построим луч ABHFA (идущий до
линзы параллельно главной оптической оси, а после лин-
зы проходящий через фокус) и луч AA (проходящий че-
рез центр линзы и, следовательно, не преломляющийся в
ней). На пересечении этих лучей лежит A — изображение
точки A. Аналогично строим лучи DCH FD и DD и нахо-
дим положение D — изображения точки D. Чтобы постро-
ить изображение точки B, построим луч BHF (идущий до
линзы параллельно главной оптической оси, а после лин-
зы проходящий через фокус) и луч BB (проходящий через
центр линзы и, следовательно, не преломляющийся в ней).
На пересечении продолжения этих лучей лежит B — изоб-
ражение точки B. Аналогично строим лучи CH F и CC и
находим положение C — изображения точки C. Из постро-
ения изображения видно, что фокус линзы лежит на пересе-
чении линий B A и C D . Значит, соединив на предложен-
ной картинке аналогичные точки получим положение фо-
куса линзы. Проведём через него главную оптическую ось
перпендикулярно A D и C B (см. рис. 2). Обозначим че-
рез R = 18 и r = 9 размеры половин параллельных сторон
изображения, через Y = 27 — расстояние между ними, че-
рез d = 9 — расстояние от действительного (уменьшенного)
изображения до фокуса. Все эти размеры можно снять с
рис. 208. Обозначим неизвестные: a — половина стороны
Решения задач городских туров 479

К решению задачи 378

К решению задачи 378

квадрата, f — фокусное расстояние линзы, b — расстояние


от линзы до ближайшей стороны квадрата (b = |BH|). Тре-
угольники FD E и FH U (U — центр линзы), UD E и UDE,
ULB и UL B — подобны, поэтому

FE FU d f UE UE d+f


= ⇒ = ·   = ⇒ =
E D UH r a ED ED r
b + 2a UL UL b Y −d−f
= · =   ⇒ = .
a LB LB a R

О т в е т. Фокусное расстояние линзы равно f = (2R + Y)r/(R + r) − d = 12


единиц, половина стороны квадрата равна a = fr/d = 12 еди-
ниц, расстояние от линзы до ближайшей стороны квадрата
равно b = a(Y − d − f)/R = 4 единицы.
480 Решения задач

379. Рассмотрим, при каком условии четырёхугольник


станет параллелограммом, т. е. изображения его противо-
положных сторон окажутся параллельными. Лучи, идущие
вдоль сторон четырёхугольника окажутся после прохожде-
ния линзы параллельными, если их пересечение попадает
в фокальную плоскость линзы (см. рис. 1). Построим эти
пересечения — точки A и B (см. рис. 2) и проведём пря-
мую AB. Любая линза, для которой AB — фокальная плос-
кость, отобразит четырёхугольник в параллелограмм. Что-
бы параллелограмм оказался прямоугольником, необходи-
мо, чтобы лучи, идущие вдоль смежных сторон четырёх-
угольника, после прохождения линзы стали взаимно пер-
пендикулярными. Рассмотрим такие лучи — A1 A и B1 B
(AB по-прежнему фокальная плоскость линзы). Пусть эти
лучи, встретившись в некоторых точках с линзой, прело-
мились, и оказались перпендикулярными друг другу. Рас-
смотрим лучи, которые проходят через центр линзы, парал-
лельные этим двум преломленным (лучи A2 A и B2 B). Эти
лучи не преломляются и пересекаются в точке O (центре
линзы), и, с другой стороны, в фокальной плоскости по-
падают в точки A и B соответственно (все параллельные
друг другу до линзы лучи после линзы собираются в од-
ну точку, лежащую в фокальной плоскости), поэтому тре-
угольник AOB является прямоугольным. Геометрическое
место точек O, образующих вместе с отрезком AB прямо-
угольный треугольник — полуокружность, построенная на
AB как на диаметре. Итак, любая линза, у которой AB —
фокальная плоскость, а центр линзы лежит на построен-
ной полуокружности (см. рис. 3) переводит наш четырёх-
угольник в прямоугольник. Чтобы прямоугольник оказал-
ся квадратом, его диагонали должны быть взаимно перпен-
дикулярны. Таким образом, лучи, идущие вдоль диагона-
лей исходного четырёхугольника, после прохождения лин-
зы должны стать взаимно перпендикулярными. Построим
точки C и D — точки пересечения этих лучей с фокальной
плоскостью. Рассуждая как в предыдущем пункте, неслож-
но показать, что образы диагоналей взаимно перпендику-
лярны, если центр линзы лежит на полуокружности, по-
строенной на CD как на диаметре. Поскольку теперь центр
линзы принадлежит двум полуокружностям, мы однознач-
Решения задач городских туров 481

К решению задачи 379

К решению задачи 379

но определили его положение — точку X. Итак, линза, с


центром в точке X, расположенная параллельно отрезку AB
и имеющая фокальную плоскость AB, переводит данный по
условию четырёхугольник в квадрат.
380. Поскольку изображение окружности уходит на
бесконечность, исходная окружность касается фокальной
плоскости линзы. Чтобы исследовать свойства изображения
окружности рассмотрим некоторое расположение окружно-
сти и линзы, обозначим крайнюю левую, верхнюю, правую
и нижнюю точки окружности через A, B, C и D (см. рис. 1).
1) Касательная к точке A вертикальна, поэтому и касатель-
ная к изображению этой точки должна быть вертикальна
(при отображении в линзе линия, перпендикулярная глав-
ной оптической оси, отображается в также перпендикуляр-
ную линию). Отсюда следует, что точка A на рис. 2 (ка-
сательная к которой — пунктир — вертикальна) является
изображением в линзе точки A. 2) Все лучи, проходящие
через точку C после линзы станут параллельными. Посколь-
482 Решения задач

К решению задачи 379

К решению задачи 379


Решения задач городских туров 483

ку изображение точки C уходит на бесконечность, эти лучи


параллельны асимптотам изображения (прямым к которым
приближается изображение на бесконечности). Луч AC по-
сле линзы также станет им параллелен. С другой стороны,
AC параллельна главной оптической оси, и значит после
линзы идёт в фокус. Отсюда следует построение фокуса лин-
зы: строим асимптоты изображения, проводим через точку
A прямую, параллельную асимптотам. Точка пересечения
её с главной оптической осью фокус (см. рис. 2). 3) Лучи,
выходящие из точек B и D, параллельные главной опти-
ческой оси, после линзы пройдут через фокус и попадут в
точки B и D изображения соответственно. С другой сто-
роны, эти лучи являются касательными к исходной окруж-
ности, значит и после преломления они станут касательны-
ми к изображению (касательные переходят в касательные).
Отсюда — построение точек B и D : строим касательные
к изображению, проходящие через F. Внимание, проверка
правильности построения: точки B и D лежали на одной
вертикали, значит и точки B , D должны лежать на одной
вертикали (см. рис. 3, отрезок B D ). 4) Луч AB, идущий
под углом 45 ◦ C попадёт в фокальной плоскости (ФП) в точ-
ку пересечения ФП с лучом, проходящим через центр лин-
зы под углом 45 ◦ C к главной оптической оси. Затем, так как
этот луч несёт информацию и о точке A, и о точке B, он дол-
жен пройти и через точку A , и через точку B изображения.
Отсюда построение центра линзы: проводим луч A B до пе-
ресечения с ФП и затем из полученной точки пересечения
строим отрезок под углом 45 ◦ C к главной оптической оси.
Точка O — центр линзы (см. рис. 4, точка A не подписа-
на, чтобы не загромождать рисунок). Внимание, проверка
правильности построения: если аналогично рассматривать
ход луча AD (проводим луч A D до пересечения с ФП и за-
тем из полученной точки пересечения строим отрезок под
углом 45 ◦ C к главной оптической оси), должна получиться
та же точка. 5) Теперь не составит труда восстановить по-
ложение окружности. Воспользуемся обратимостью лучей.
Продолжим A B до линзы. После линзы луч пойдёт под уг-
лом 45 ◦ C (так как дальше этот луч должен пойти и в A, и в
B). Он должен где-то попасть в точку A. Продолжим A D до
линзы. После линзы луч также пойдёт под углом 45 ◦ C (так
484 Решения задач

К решению задачи 379

К решению задачи 379

как дальше этот луч должен пойти и в A, и в D). Он тоже


должен где-то попасть в точку A. Точка A есть пересечение
полученных за линзой лучей (см. рис. 5). Точка C окружно-
сти лежит на одной горизонтали с точкой A и принадлежит
ФП. Поскольку мы построили диаметр окружности AC, за-
дача решена.
Решения задач городских туров 485

К решению задачи 379

К решению задачи 379

К решению задачи 379


Решения задач экспериментальных туров
381. Качественное объяснение явления. Разберёмся, по-
чему вода в бутылке так себя ведёт. Очевидно, что давление
на стенку бутылки равно атмосферному. Если бутылка от-
крыта, то внутри её к атмосферному давлению прибавляет-
ся давление столба воды и вода из бутылки вытекает. Ес-
ли бутылка закрыта, то при вытекании воды давление над
ней уменьшается до тех пор, пока на уровне отверстия оно
не сравняется с атмосферным и вода перестанет течь. Ес-
ли в воду погружена трубочка, то, как только на уровне её
нижнего конца давление станет меньше атмосферного, из
трубочки начнут выходить пузырьки воздуха. Если нижний
конец трубки опущен ниже отверстия в стенке бутылки, то в
трубке останется столбик воды до уровня отверстия в стен-
ке бутылки, и вода вытекать не будет, так как давление с
обоих сторон отверстия одинаковое.
382. Качественное объяснение явления. Прежде всего,
бросается в глаза различие пятен: тушь даёт большое, рав-
номерно окрашенное пятно, а гуашь — центральное пят-
но со слабо окрашенным влажным ореолом вокруг него.
Причина этого различия — в размерах частиц гуаши и ту-
ши: частицы туши маленькие и свободно проникают вме-
сте с водой сквозь поры фильтровальной бумаги. Напротив,
частицы гуаши — крупные, они оседают в порах, а вода
распространяется по порам — фильтруется. Все участники
олимпиады правильно описали наблюдаемое явление, по-
этому преимущество было отдано тем, кто правильно объяс-
нил также «тонкие» эффекты: кайму по периметру влажно-
го пятна (по краям вода испаряется, а поступление частиц
краски продолжается, что приводит к повышению их кон-
центрации), «усы» по краям пятна (диффузия происходит
вдоль волокон бумаги).
383. Магнит с гирькой прикрепляются под пластинкой.
Гирька отклоняется от вертикали на некоторый угол и от-
пускается. Величина угла легко определяется с помощью
линейки. Когда гирька находится в нижнем положении, на
магнит, кроме силы тяжести гирьки, действует центробеж-
ная сила. Ищется минимальный угол отклонения, при ко-
тором движущаяся гирька отрывает магнит от пластинки.
Решения задач экспериментальных туров 487

Если даже при угле отклонения 90 ◦ C магнит не отрывает-


ся от пластинки, на пути нитки, под магнитом, нужно по-
ставить линейку. Радиус кривизны траектории гирьки при
этом уменьшится, а центробежная сила — возрастёт. Чем
ближе линейка находится к гирьке, тем больше будет цен-
тробежная сила.
384. Идея эксперимента. Назовём единичной силу
f0 , необходимую для растяжения резинки вдвое. Выберем
некоторую длину резинки за единичную. Сложим резинку
следующим образом: одна её часть (единичной длины) бу-
дет одинарная, а другая — такой же длины — двойная. При
растяжении одинарной части вдвое, к каждой из резинок
второй части будет приложена половинная сила f1 = 2−1 f0 .
Удлинение этих резинок можно измерить и отметить на бу-
маге. Если единичную силу приложить к каждой из рези-
нок двойной части (они растянутся вдвое), то на одинар-
ную резинку будет действовать удвоенная сила f1 = 2f0 . Так
можно измерить удлинение резинки под действием силы
f = 2n f0 , где n — целое число (как положительное, так и от-
рицательное). Если вторую часть резинки сложить втрое,
то можно измерить удлинение резинки под действием си-
лы f = 3m f0 . Комбинируя измерения, получим удлинения
резинки под действием силы вида f = 2n 3m f0 .
385. Идея эксперимента. Подвесим металлический ша-
рик на нитке так, чтобы он касался цилиндра, отклоним
на некоторый угол от вертикали и отпустим. Предположим,
что энергия, переданная цилиндру при ударе, не зависит от
того, пришёлся ли удар в бок или в торец цилиндра. Такое
предположение легко проверить измерив угол, на который
отскакивает шарик в обоих случаях. После этого, для опре-
деления соотношения сил трения и качения останется лишь
измерить путь, который пройдёт цилиндр в обоих случаях,
так как A = Fтр S. Путь скольжения цилиндра очень мал, по-
этому его разумно измерять после нескольких ударов. Тре-
ние качения, наоборот, очень мало, и его необходимо изме-
рять два раза, при движении цилиндра в противоположных
направлениях, чтобы учесть наклон стола.
386. Идея эксперимента. Опустим трубочку в стакан-
чик с водой так, чтобы она коснулась дна. Закроем второй
конец трубочки пальцем и, вынув её из воды, измерим ли-
488 Решения задач

нейкой длину столбика воды h1 . Она равна уровню воды в


стаканчике плюс высота подъёма воды в капилляре. Набе-
рём в трубочку некоторое количество раствора. Высота стол-
ба раствора h2 должна быть меньше h1 . Наклоним трубоч-
ку так, чтобы столбик раствора сдвинулся на её середину,
закроем конец пальцем и снова опустим в стаканчик с во-
дой до дна. Пусть вода поднимется в трубочке на высоту h3 .
Между водой и раствором будет прослойка воздуха. Легко
видеть, что rВ · (h1 − h3 ) = rР · h2 , где rВ и rР — плотности во-
ды и раствора соответственно. Без прослойки воздуха вода
и раствор перемешиваются. Растворы подкрашены для то-
го, чтобы перемешивание можно было заметить. Для про-
верки точности метода, целесообразно таким же способом
измерить плотность воды.
387. Примечание. Измерения нужно провести двумя
способами, полагая, что вам дана либо линейка, либо стек-
лянная трубка.
Идея эксперимента. 1) Измерения с помощью линей-
ки проще, но менее точны. Нальём в стаканчик воду при
нулевой температуре и поместим туда исследуемое тело.
Отметим показания термометра при комнатной температу-
ре, при нуле градусов и при погружении тела в воду. Те-
перь легко составить пропорцию, связывающую теплоём-
кость воды и исследуемого тела. Массу воды определяем
измеряя линейкой размеры стаканчика. Измерения с помо-
щью линейки дают основной вклад в погрешность опыта.
Трудно учесть и теплоёмкость стаканчика и термометра, хо-
тя они невелики и не должны сильно повлиять на резуль-
тат. 2) Измерения с помощью трубки значительно точнее.
Вот как выглядит оптимальный эксперимент. С помощью
трубки нальём в стаканчик такое количество тёплой воды,
которое едва покрывает исследуемое тело. Чем меньше во-
ды, тем сильнее изменится её температура и тем точнее бу-
дут измерения. Отметим положение капельки воды в термо-
метре, например, завяжем на трубке термометра нитку. Эта
метка нам нужна только для контроля. Опустим исследуе-
мое тело в холодную воду и подождём, пока оно остынет,
после чего опустим его в стаканчик. Как можно точнее от-
метим ниткой положение воды в термометре. Снова нальём
в стаканчик такое же количество тёплой воды, как и в пер-
Решения задач экспериментальных туров 489

вый раз. С помощью трубки будем добавлять в стаканчик


холодную воду до тех пор, пока капелька воды в термомет-
ре не дойдёт до нашей отметки. Не забудем охладить трубку
прежде чем набирать ею холодную воду. Теплоёмкость до-
бавленной воды равна теплоёмкости исследуемого тела.
388. Идея эксперимента. Из алюминиевой фольги и
нитки изготовляем электроскоп. Для расчёта его чувстви-
тельности фольгу и нитку необходимо взвесить. Весом нит-
ки, скорее всего, можно пренебречь. После причесывания
на расчёске остаётся заряд, равный заряду девушки. Если
коснуться расчёской электроскопа, его листочки разойдут-
ся. Зная чувствительность электроскопа, можно найти пере-
данный ему заряд. Часть заряда, которая осталась на рас-
чёске, можно найти, если снова поднести расчёску к элек-
троскопу. Взаимодействие одноимённых зарядов приведёт к
отталкиванию электроскопа от расчёски. Для определения
заряда расчёски нужно знать вес электроскопа, угол его от-
клонения от вертикали величину заряда на нём и расстоя-
ние от расчёски до электроскопа.
389. Идея эксперимента. Резинка и шарик подобраны
таким образом, что их веса практически равны. Если их
подвесить на нитках, шарик отклонить и отпустить, то по-
сле удара шарик почти не отклонится. Если же резинку од-
ним концом упереть в массивное тело (см. рисунок), то по-
сле удара шарик отскочит на прежнюю высоту. Это означа-
ет, что волна деформации отразилась от массивного тела и
вернулась обратно. Путь волны равен удвоенной длине ре-
зинки. Для определения скорости распространения дефор-
мации необходимо только измерить время контакта шари-
ка с резинкой. Для этого можно использовать зарядку кон-
денсатора через известное сопротивление. Примерная схема
экспериментальной установки показана на рисунке.
390. Идея эксперимента. Сопротивление вольфрама лег-
ко измерить, сравнивая падение напряжения на известном
сопротивлении и на вольфрамовой нитке при протекании
последовательно через них тока (см. рисунок). Если изме-
рения провести с проволокой помещённой в воду с таю-
щим льдом, т. е. при температуре 0 ◦ C, а потом в воздухе —
при комнатной температуре, то можно найти температур-
ный коэффициент сопротивления. Трудность заключается
490 Решения задач

К решению задачи 389

К решению задачи 390

в том, что при протекании тока тонкая вольфрамовая про-


волока нагревается. Величина нагрева в воде и в воздухе
различна. Измерения в воде и в воздухе необходимо про-
водить при различных значениях тока и экстраполировать
полученные значения сопротивления на нулевой ток, когда
проволока не должна нагреваться.
391. Идея эксперимента. Диэлектрическую постоян-
ную стекла можно определить, измерив электрическую ём-
кость конденсатора, образованного металлическими обклад-
ками стеклянной пластинки. С помощью известных источ-
ника напряжения и конденсатора калибруем баллистиче-
ский гальванометр (см. рис.), то есть, измеряем величи-
ну отброса светового зайчика гальванометра при протека-
Решения задач экспериментальных туров 491

К решению задачи 391

нии известного заряда. После этого легко определить и


ёмкость, образованную металлическими обкладками стек-
лянной пластинки. Экспериментальная трудность состоит
в том, что ёмкость исследуемого конденсатора значитель-
но меньше, чем эталонного. На нём накапливается очень
маленький заряд и, следовательно, отброс зайчика гальва-
нометра мал, что приводит к очень большой погрешности.
Для увеличения точности можно использовать следующий
приём: после зарядки исследуемого конденсатора, его под-
соединяем параллельно к эталонному. При этом практиче-
ски весь заряд перейдёт на конденсатор большой ёмкости.
Такую процедуру можно повторить несколько раз и толь-
ко после этого разрядить эталонный конденсатор через бал-
листический гальванометр. Отброс зайчика гальванометра
увеличится почти во столько раз, сколько переключений ис-
следуемого конденсатора было совершено, и точность изме-
рений возрастёт.
392. Идея эксперимента. Величину сопротивления мож-
но определить, измерив, сколько тепла выделяется в нём
за определённое время, когда сопротивление подсоединяем
к известному источнику напряжения. Можно сначала най-
ти в отдельном опыте теплоёмкость стаканчика, сопротив-
ления и термометра. Для этого нужно измерить темпера-
туру, которая устанавливается в стаканчике, когда в него
наливают воду при температуре 0 ◦ C. Работу можно выпол-
нить проще и точнее. В стаканчик, в котором при комнат-
ной температуре находятся сопротивление и термометр, на-
льём воду при температуре 0oС и включим ток (см. рису-
нок). Измерим время, за которое вся система опять нагре-
ется до комнатной температуры. Всё выделившееся тепло
492 Решения задач

К решению задачи 392

при этом пошло только на нагрев воды, теплоёмкость кото-


рой известна. Останется только учесть небольшие поправ-
ки, такие, как нагрев стаканчика окружающим воздухом во
время опыта и продолжение нагрева воды в течение некото-
рого времени после выключения тока.
393. Идея эксперимента. Намотаем на гвоздь две одина-
ковые обмотки из медной изолированной проволоки. В од-
ну из них включим магазин сопротивлений, а в другую —
неизвестное сопротивление. Подсоединим обмотки к источ-
нику напряжения таким образом, чтобы ток в них шёл
в противоположных направлениях. Когда сопротивления в
обеих обмотках будут равны, гвоздь не будет намагничи-
ваться и не будет притягивать скрепку.
394. Идея эксперимента. Бросим несколько кубиков
льда в воду комнатной температуры и определим сопротив-
ление медной проволоки при 0 ◦ C, при комнатной темпера-
туре и при температуре воды когда весь лёд растает. С помо-
щью линейки определим массу воды и льда. Теперь из урав-
нения теплового баланса можно найти значение комнатной
температуры. При этом необходимо учесть теплоёмкость со-
суда и нагрев воды во время таяния льда окружающим воз-
духом. Для этого нужно поставить дополнительные опыты.
Теперь наш термометр отградуирован и можно наблюдать
за изменением температуры при растворении соли.
Решения задач экспериментальных туров 493

395. Идея эксперимента. Натяжение проволоки опреде-


лим с помощью закона Гука. Взяв рукой за середину прово-
локи, оттянем её. Измерив линейкой высоту получившего-
ся треугольника, найдём длину проволоки. Удлинение про-
волоки пропорционально натяжению. Учтём, что первона-
чально проволока была не натянута, и её удлинение начи-
нается не сразу. Два источника и два магазина сопротивле-
ний даны для повышения точности измерения сопротивле-
ния. Нужно выбрать такую схему измерения сопротивления
проволоки, чтобы использовалась наиболее чувствительная
шкала вольтметра.
396. Идея эксперимента. 1) Если в бутылку налить
немного воды и сразу её закрыть, то через некоторое вре-
мя пары воды станут насыщенными и давление в бутылке
повысится. Если это давление измерить, то задача будет ре-
шена, так как мы узнаем, насколько давление паров воды в
комнате было меньше чем насыщенных. Лучше всего нали-
вать воду в бутылку уже после её закрытия. Для этого на-
берём воду в трубку, закроем крышку, дадим небольшому
количеству воды стечь в бутылку и закроем верхний конец
трубки. Через некоторое время, когда пар в бутылке станет
насыщенным, снова откроем верхний конец трубки. Вода
будет вытекать до тех пор, пока повышенное давление в бу-
тылке не уравновесит высоту столба воды в трубке. Нуж-
но ещё учесть капиллярные явления в трубке и вытеснение
воздуха из бутылки водой. Для этого снова наберём воду в
трубку, закроем бутылку и дадим воде стекать в неё. Остав-
шийся в трубке столбик воды и будет искомой поправкой.
397. Идея эксперимента. Если бы длина трубки была
около одного метра, то задача решалась бы очень просто.
Достаточно было бы налить в бутылку воду при нулевой
температуре и закрыть её крышкой так, чтобы нижний ко-
нец трубки был под водой. Когда вода нагреется до комнат-
ной температуры, уровень воды в трубке повысится и урав-
новесит повышение давления в бутылке. Часть этого повы-
шения связана с изменением температуры и легко рассчи-
тывается. Остаток равен изменению давления паров воды.
Поскольку трубка короткая, то часть воды при нагреве вы-
льется. Измерив это количество и объём воздуха в бутыл-
ке, можем рассчитать изменение давления. Наиболее точ-
494 Решения задач

но количество вытекшей воды можно определить взвешива-


нием (попросив в качестве дополнительного оборудования
весы). Решение задачи можно обернуть: налить в бутылку
воду комнатной температуры и измерять количество воды,
которое всосётся в бутылку при охлаждении.
398. Идея эксперимента. Иголка намагничивается с по-
мощью катушки, через которую пропускается ток, втыкает-
ся в поролон и кладется на воду. По её ориентации опреде-
ляем северный полюс катушки. Помещаем тонкую проволо-
ку над одним из полюсов катушки и, по её отклонению при
пропускании тока находим полярность источника. Одного
знания полюсов катушки недостаточно, так как она может
быть намотана как по часовой стрелке, так и против часовой
стрелки. Заметить отклонение проволоки в магнитном поле
Земли практически невозможно: поле слишком слабое.
399. Идея эксперимента. Для градуировки термометра
необходимо найти ещё одну температурную точку. Нальём
в стаканчики одинаковое (взвесим) количество холодной во-
ды. В один из них добавим известное количество льда (взве-
сим). Закроем стаканчики бумагой и подождём, пока лёд
растает. В первом приближении оба стаканчика получили
одинаковое количество тепла, равное скрытой теплоте плав-
ления льда. Теперь легко рассчитать температуру второго
стаканчика с водой. Во втором приближении нужно учесть,
что второй стаканчик получил меньшее количество тепла от
окружающей среды, так как его температура была немного
выше. Нужно учесть также, что температура первого ста-
канчика будет немного выше нуля, когда весь лёд растает.
Нужно подумать, как избавиться от влияния теплоёмкости
термометра. Положение капельки воды в термометре нели-
нейно связано с температурой. Внутри термометра есть па-
ры воды и их давление нелинейно зависит от температуры.
Это может сказаться при измерении температуры ладони.
Зависимость давления насыщенного пара воды от темпера-
туры участники могли узнать у членов жюри.
400. Предварительное замечание. Эффект Пелтье состо-
ит в выделении или поглощении тепла на контакте двух раз-
нородных проводников в зависимости от направления тока,
текущего через контакт. Причина эффекта заключается в
том, что средняя энергия носителей заряда, участвующих в
Решения задач экспериментальных туров 495

электропроводности, разная в различных проводниках, так


как зависит от их энергетического спектра, концентрации и
механизма рассеяния. При переходе из одного проводника в
другой электроны либо передают избыточную энергию ато-
мам вещества, либо пополняют свою энергию за счёт энер-
гии атомов проводника. В результате чего проводник нагре-
вается или остывает.
Идея эксперимента. Сначала разберёмся, как работает
элемент Пелтье. При протекании тока через него, на одном
из контактов выделяется, а на другом поглощается тепло.
Оно пропорционально прошедшему заряду и абсолютной
температуре контактов. В первый момент ток через элемент
определяется его омическим сопротивлением. По мере на-
грева контактов, на них возникает термо-ЭДС, направлен-
ная против внешнего источника , и ток уменьшается. Если
бы отсутствовала теплопроводность, то через некоторое вре-
мя термо-ЭДС стала бы равной ЭДС внешнего источника , и
ток полностью бы прекратился. В реальном элементе тепло
постоянно уходит от горячего контакта к холодному, и че-
рез элемент продолжает течь такой ток, что выделяющееся
тепло Пелтье компенсирует этот уход. Джоулево тепло рас-
пределяется поровну между обоими контактами. На одном
контакте оно складывается с теплом Пелтье, а на другом —
вычитается. Перейдём к рассмотрению первого способа из-
мерений. Заманчиво решить задачу одним махом: выклю-
чим ток, и вольтметр покажет величину оставшейся ЭДС. К
сожалению, сразу после выключения тока, начинается из-
менение температуры внутри элемента. Термо-ЭДС опреде-
ляется температурой очень тонкого слоя в контактах и так-
же мгновенно изменится. Воспользуемся тем, что тепло Пе-
лтье имеет разные знаки на разных контактах. Поместим
элемент Пелтье на массивное металлическое тело, прикро-
ем его поролоном и закрепим скотчем. В электрической схе-
ме предусмотрим возможность изменения направления то-
ка (шестиполюсный ключ дан). Включим ток. Температу-
ра нижнего контакта останется равной комнатной, услов-
но примем её равной нулю. Температура верхнего контак-
та после установления равновесия, будет зависеть от на-
правления тока и обуславливает возникновение термо-ЭДС,
пропорциональной температуре. Эта разность потенциалов
496 Решения задач

может быть измерена вольтметром при прямом и обратном


направлениях тока. Остаётся неизвестным только коэффи-
циент Пелтье. Как было сказано выше, после выключения
тока, вольтметр покажет напряжение не равное, но про-
порциональное термо-ЭДС. Есть такой ток, когда Джоуле-
во тепло равно теплу Пелтье, тогда температуры контактов
равны, и разность потенциалов U = 0. Другой способ изме-
рения основан на определении количества тепла, выделяю-
щегося в элементе. Это тепло состоит из двух частей: джо-
улева тепла и тепла, связанного с эффектом Пелтье. Дело
в том, что коэффициент Пелтье зависит от абсолютной тем-
пературы, и на горячем контакте выделяется больше теп-
ла, чем поглощается на холодном. Необходимо провести из-
мерения при нескольких значениях силы тока и экстрапо-
лировать полученные значения сопротивления к нулевому
току. При измерении тепла сам элемент Пелтье можно ис-
пользовать как очень чувствительный термометр. Его толь-
ко нужно предварительно проградуировать. Поместим эле-
мент на массивное тело, согреем в ладони алюминиевый ци-
линдр и положим на второй контакт. Температура ладони
нам известна (термометр в лаборатории есть), а показания
вольтметра, подсоединённого к элементу Пелтье, пропорци-
ональны разности температур контактов. Дальнейшие из-
мерения просты. Масса цилиндра известна, теплоёмкость
алюминия можно узнать у жюри. Элемент Пелтье с при-
жатым к нему цилиндром теплоизолируем поролоном и в
течение известного времени будем пропускать через него
известный ток. Подождём некоторое время, пока темпера-
туры контактов сравняются, и положим элемент, вместе с
алюминиевым цилиндром , на массивное тело. Показания
вольтметра позволят нам найти температуру цилиндра, а
тем самым и поглощённое им количество тепла. В отдель-
ном опыте можно измерить теплоёмкость самого элемента
Пелтье и вычислить его омическое сопротивление.
Приложение
Плотности некоторых веществ кг/м3

свинец 11350 ртуть 1360


медь 8900 пластмасса 1350
сталь 8700 зерно гречи (сухое) 1300
чугун 7140 зерно гречи (варёное) 1100
хрусталь 3000 пластилин 1100
кристаллы соли 3000 вода 1000
алюминий 2700 лёд 800
гири бытовых весов 2000 груши свежие 800
магний 1740 масло растительное 850
воск 1400 пробка 240
воздух 0,293
гелий 0,178

Другие величины

ускорение свободного падения на Земле 10 м/c2


ускорение свободного падения на Марсе 3,8 м/c2
ускорение свободного падения на Луне 1,6 м/c2
нормальное атмосферное давление 105 Па
удельная теплота таяния снега 340 кДж/кг
удельная теплота таяния льда 330 кДж/кг
температура кипения ацетона 56,2 ◦ C
температура плавления нафталина 80,3 ◦ C
удельная теплоёмкость свинца 130 кДж/кг · град
удельная теплоёмкость воды 4,2 кДж/кг · град
удельная теплоёмкость пробки 500 кДж/кг · град
удельная теплоёмкость соли 850 кДж/кг · град
радиус Солнца 7 · 108 м
среднее расстояние от Земли до Солнца 1,5 · 1011 м
молярная масса воды 18 г/моль

Вам также может понравиться